Teaching Skills 9788131759547, 9789332506756, 8131759547

The Pearson Complete Guide to the SAT offers a methodical, thoroughly class-tested approach designed to specifically mee

393 22 30MB

English Pages 536 [491]

Report DMCA / Copyright

DOWNLOAD FILE

Polecaj historie

Teaching Skills
 9788131759547, 9789332506756, 8131759547

Table of contents :
Cover......Page 1
Contents......Page 6
Preface......Page 14
Acknowledgements......Page 15
Introduction......Page 16
Part 1: The SAT and American Universities......Page 18
1.1. General Information......Page 20
1.2.2. Advantages: Variety......Page 21
1.2.6. Perception......Page 22
1.2.8. Disadvantages: Competition......Page 23
2.1. History......Page 24
2.2. Structure......Page 25
2.3. Pedagogical Features......Page 26
2.4. Logistics......Page 27
3.1. General Information......Page 29
3.2.1. Academics......Page 30
3.2.5. Essays......Page 31
3.3.1. Criteria #1: Ranking......Page 32
3.3.6. Reach-Match-Safety......Page 33
3.4.1. Early......Page 34
3.4.3. Rolling......Page 35
3.5.1. The Waitlist......Page 36
3.6.1. Athletics......Page 37
3.6.2. Development......Page 38
Chapter 4: Thirty Top American Universities......Page 39
5.1. Long Stanford Essay......Page 48
5.2. Long UVA Essay......Page 49
Part 2: SAT Preparation......Page 51
6.1. Vocabulary: High-Frequency Vocabulary List......Page 53
6.2. Vocabulary: Grouped Hard Word List......Page 80
6.3. Vocabulary: Long Word Part List......Page 89
7.1.1. Average time per question per section......Page 97
7.3.2. General Strategy #5: The Process of Elimination......Page 98
7.3.8. Order of Difficulty......Page 99
7.4.2. General Strategy #12: Knowingly not Knowing......Page 100
7.5. Quiz #1: General......Page 101
8.2. Directions......Page 102
8.4. Frequency Distribution......Page 103
8.6. Reading Comprehension: Skills......Page 104
8.6.3. Exercise #1: Saying the Same thing a Different Way......Page 105
8.7.2. Strategy #2: Long Comparative Passages......Page 107
8.7.3. Exercise #2: Long Comparative Passages......Page 108
8.7.5. Strategy #4: Short Comparative Passages......Page 109
8.7.6. Exercise #3: Summarizing Short Comparative Passages......Page 110
8.8. Quiz #2: Reading Comprehension I......Page 112
9.1.1. Secondary Strategy #1: The Process of Elimination......Page 113
9.1.2. Exercise #4: Choosing between Hard Answer Choices......Page 114
9.1.3. Secondary Strategy #2: Two Questions......Page 115
9.1.5. Exercise #6: What is the Passage Saying?......Page 117
9.1.7. Secondary Strategy #4: Identifying Disqualifying Information: Top-Down Method......Page 118
9.1.8. Exercise #7: Finding the Trigger......Page 120
9.1.9. Secondary Strategy #5: Identifying Disqualifying Information......Page 121
9.1.10. Exercise #8: Finding Inaccuracy......Page 123
9.1.15. Secondary Strategy #10: Approach II......Page 124
9.2. Quiz #3: Reading Comprehension II......Page 126
10.1. More Secondary Strategies......Page 127
10.1.4. Exercise #9: Saying Something Saying Nothing......Page 128
10.1.6. Secondary Strategy #15: Non-Specified Nouns......Page 129
10.1.7. Secondary Strategy #16: Neutral Verbs and Adjectives......Page 130
10.1.10. Exercise #10: Identifying Disqualifying Language......Page 131
10.2.2. Secondary Strategy #19: Critical, not Resentful......Page 133
10.3. Secondary Strategy #21: Ultimate Approach......Page 134
10.4. Quiz #4: Reading Comprehension III......Page 135
11.2. Directions......Page 136
11.5.2. Double-Blank Questions......Page 137
11.6.2. Exercise #12: Finding the Clue......Page 138
11.6.3. Skill #2: Identifying Direction-Cues......Page 139
11.6.5. Exercise #13: Generating Own Solution......Page 140
11.6.6. Skill #4: Using the Word-Parts......Page 141
11.6.7. Exercise #14: Guessing Meaning......Page 142
11.7.1. Strategy #1: Clues before Direction-Cues before Word Parts......Page 143
11.7.4. Strategy #4: Approach for Double-Blank questions......Page 144
11.7.7. Strategy #7: The Deal-Breaker......Page 145
11.8. Quiz #5: Sentence Completions......Page 146
12.2. Directions......Page 147
12.4. Agreement: Frequency Distribution......Page 148
12.6. Agreement: Function......Page 149
12.6.1. Subject-Verb Agreement......Page 150
12.6.3. Noun-Pronoun Agreement......Page 151
12.6.5. Noun-Number Agreement......Page 154
12.7.3. Agreement Strategy #3: Destroy the Distracters......Page 155
12.7.4. Exercise #17: Destroy the Distracters......Page 156
12.8.3. Strategy #7: What to Expect......Page 157
12.9. Quiz #6: Writing I......Page 158
13.2. Parallelism: Sectional Frequency......Page 159
13.3.1. Parallelism: Verb Use......Page 160
13.3.3. Exercise #18: Identifying Verb Tense......Page 162
13.3.5. Parallelism: Clauses and Conjunctions......Page 163
13.3.6. Exercise #20: Clauses......Page 166
13.4.2. Strategy #2: Use What You’ve Got......Page 167
13.4.4. Strategy #4: Try it on for Size......Page 168
13.5. Quiz #7: Writing II......Page 169
14.1. Modifiers: FrequencyDistribution......Page 170
14.3.1. Modifiers: Subject Placement......Page 171
14.3.4. Modifiers: Quantifier Use......Page 172
14.4. Wordiness: Frequency Distribution......Page 173
14.6.2. Expansion......Page 174
14.9.1. Word Pairs......Page 175
14.9.6. Exercise #22: Modifiers, Wordiness and Idiomatic Expression......Page 176
14.10.2. Strategy #2: Ask the Question......Page 177
14.12. Idiomatic Expression: Strategies......Page 178
14.12.2. Strategy #2: 23-29......Page 179
14.13. Quiz #8: Writing III......Page 180
15.3. Difficulty Distribution......Page 181
15.4.1. Improving Paragraphs: Word-level......Page 182
15.4.3. Improving Paragraphs: Sentence-Level......Page 183
15.4.5. Improving Paragraphs: Paragraph-Level......Page 185
15.5.4. Strategy #4: Eight Minutes......Page 186
15.6. Quiz #9: Improving Paragraphs......Page 187
16.1. Grading Rubric......Page 188
16.3. The Essay: Function......Page 189
16.3.2. The Answer......Page 190
16.3.3. Exercise #25: Answering Questions......Page 192
16.3.7. The Support......Page 193
16.3.8. Primer: Essay Examples......Page 194
16.3.9. The Conclusion......Page 195
16.4.3. Strategy #3: Don’t Worry about Transitions......Page 196
16.4.8. Strategy #8: Danger......Page 197
16.5. Quiz #10: The Essay......Page 198
17.1. Formal Distribution......Page 199
17.2. Directions......Page 200
17.4. Frequency Distribution......Page 201
17.5.5. Strategy #5: Seeming vs. Being......Page 202
17.5.7. Strategy #7: Three Options for Hard Questions......Page 203
17.5.8. Exercise #26: Hard Maths Questions......Page 204
17.6.5. Strategy #12: Parameters Matter......Page 205
17.7. Quiz #11: Maths I......Page 206
18.2. DASP: Function......Page 207
18.2.1. Data Interpretation......Page 208
18.2.2. Descriptive Statistics......Page 209
18.2.3. Exercise #27: Descriptive Statistics......Page 210
18.3.1. Strategy #1: Know What You’re Looking For......Page 211
18.3.4. Strategy #4: Probability......Page 212
18.4. Quiz #12: Math II......Page 213
19.1. Algebra: Frequency Distribution......Page 214
19.2.1. Equations......Page 215
19.2.3. Functions......Page 216
19.2.4. Exercise #30: Functions......Page 219
19.2.5. Word Problems......Page 220
19.2.6. Properties of Exponents......Page 221
19.2.7. Algebraic Expressions......Page 222
19.2.10. Absolute Value......Page 223
19.2.12. Proportion......Page 224
19.3.3. Strategy #3: Simplification I......Page 225
19.3.11. Strategy #11: Plugging In I......Page 226
19.3.13. Strategy #13: Plugging In III......Page 227
19.4. Quiz #13: Math III......Page 228
20.1. Geometry: Reference Information......Page 229
20.3.1. Coordinate Geometry......Page 230
20.3.3. Area and Perimeter......Page 235
20.3.4. Triangles......Page 236
20.3.7. Errata and Vocabulary......Page 238
20.5. Geometry: Strategies......Page 239
20.5.6. Strategy #6: Use the Basic Concepts......Page 240
20.6. Quiz #14: Math IV......Page 242
21.2. Numbers and Operations: Function......Page 243
21.2.1. Word Problems......Page 244
21.2.3. Properties of Integers......Page 246
21.2.4. Sequences and Series......Page 248
21.2.6. Counting Techniques......Page 250
21.3.8. Strategy #8: Sequences: Find the Difference......Page 252
21.3.13. Strategy #13: Intervals vs Interval Markers......Page 253
21.4. Quiz #15: Math V......Page 254
8.7.3. Exercise #2: Long Comparative Passages......Page 255
9.1.4. Exercise #5: What the Question is Asking......Page 256
9.1.10. Exercise #8: Finding Inaccuracy......Page 257
10.1.10. Exercise #10: Identifying Disqualifying Language......Page 258
11.8.5. Exercise #13: Generating Own-Solution......Page 259
12.6.2. Exercise #15: Subject-Verb Agreement......Page 260
13.3.4. Exercise #19: Identifying Verb Lists......Page 261
14.3.5. Exercise #21: Manipulating Modifiers......Page 262
16.3.3. Exercise #25: Answering Questions......Page 263
21.2.2. Exercise #33: Word Problems......Page 264
9.2. Quiz #3: Reading Comprehension II......Page 265
12.9. Quiz #6: Writing I......Page 266
15.6. Quiz #9: Improving Paragraphs......Page 267
17.7. Quiz #11: Maths I......Page 268
19.4. Quiz #13: Maths III......Page 269
21.4. Quiz #15: Maths V......Page 270
Part 3: Four Full Practice Tests......Page 271
Scoring......Page 273
Student Produced Responses......Page 274
Student Produced Responses......Page 276
Essay......Page 278
Section 1......Page 279
Section 2......Page 281
Section 3......Page 287
Section 4......Page 293
Section 5......Page 299
Section 7......Page 304
Section 8......Page 309
Section 9......Page 313
Section 10......Page 317
How to Score Practice Test #1......Page 321
SAT Practice Test #1 Scoring Sheet......Page 323
SAT Reasoning Test — General Directions......Page 328
Student Produced Responses......Page 330
Student Produced Responses......Page 331
Essay......Page 333
Section 2......Page 336
Section 3......Page 341
Section 4......Page 347
Section 5......Page 353
Section 7......Page 357
Section 8......Page 363
Section 9......Page 367
Section 10......Page 372
How to Score Practice Test #2......Page 376
SAT Practice Test #2 Scoring Sheet......Page 378
SAT Reasoning Test—General Directions......Page 383
Student Produced Responses......Page 384
Student Produced Responses......Page 385
Student Produced Responses......Page 386
Essay......Page 388
Section 2......Page 391
Section 3......Page 396
Section 4......Page 402
Section 5......Page 408
Section 7......Page 413
Section 8......Page 419
Section 9......Page 423
Section 10......Page 427
How to Score Practice Test #3......Page 431
SAT Practice Test #3 Scoring Sheet......Page 433
SAT Reasoning Test—General Directions......Page 438
Student Produced Responses......Page 439
Student Produced Responses......Page 440
Student Produced Responses......Page 441
Essay......Page 443
Section 2......Page 446
Section 3......Page 450
Section 4......Page 456
Section 5......Page 462
Section 7......Page 467
Section 8......Page 472
Section 9......Page 476
Section 10......Page 481
How to Score Practice Test #4......Page 485
SAT Practice Test #4 Scoring Sheet......Page 487

Citation preview

The Pearson Complete Guide to the SAT Nicholas Henderson Morgan Henderson

Chandigarh • Delhi • Chennai

Intro.indd 1

5/23/2011 5:38:46 PM

The aim of this publication is to supply information taken from sources believed to be valid and reliable. This is not an attempt to render any type of professional advice or analysis, nor is it to be treated as such. While much care has been taken to ensure the veracity and currency of the information presented within, neither the publisher nor its authors bear any responsibility for any damage arising from inadvertent omissions, negligence or inaccuracies (typographical or factual) that may have found their way into this book. Copyright © 2011 Dorling Kindersley (India) Pvt. Ltd Licensees of Pearson Education in South Asia No part of this eBook may be used or reproduced in any manner whatsoever without the publisher’s prior written consent. This eBook may or may not include all assets that were part of the print version. The publisher reserves the right to remove any material present in this eBook at any time. ISBN 9788131759547 eISBN 9789332506756 Head Office: A-8(A), Sector 62, Knowledge Boulevard, 7th Floor, NOIDA 201 309, India Registered Office: 11 Local Shopping Centre, Panchsheel Park, New Delhi 110 017, India

Intro.indd 2

5/23/2011 5:38:46 PM

To our good friends and colleagues: Dinanath Sonkar, Ganesh Kumar Sonkar, and Rakesh Sonkar

Intro.indd 3

5/23/2011 5:38:46 PM

Epigraph

The one thing that matters is the effort.  It continues, whereas the end to be attained is but an illusion of the climber, as he fares on and on from crest to crest; and once the goal is reached it has no meaning.  — Antoine de Saint-Exupéry, The Wisdom of the Sands

Intro.indd 4

5/23/2011 5:38:46 PM

Contents    Preface

xiii

   Acknowledgement

xiv

   INTRODUCTION

xv

  Part One: The SAT and    American Universities 1. Chapter One: Why American    Universities?

1.1 1.3

1.1 General Information

1.3

1.2 Advantages and Disadvantages

1.4

1.2.1 Advantages: Flexibility

1.4

1.2.2 Advantages: Variety

1.4

1.2.3 Advantages: Rankings

1.5

1.2.4 Advantages: Endowment

1.5

1.2.5 Advantages: Post-Graduation

1.5

1.2.6 Advantages: Perception

1.5

1.2.7 Disadvantages: Fees

1.6

1.2.8 Disadvantages: Competition

1.6

2.  Chapter Two: The SAT 2.1 History

2.1

2.1

2.2 Structure 2.2 2.3 Pedagogical Features 2.4 Logistics

2.3

2.4

3.  Chapter Three: Application Process 3.1

Intro.indd 5

3.1 General Information

3.1

3.2 Elements of Competitive Application

3.2

3.2.1 Academics

3.2

3.2.2 Trend

3.3

3.2.3 SAT Scores

3.3

3.2.4 Extracurricular Activities

3.3

3.2.5 Essays

3.3

3.2.6 Recommendations

3.4

3.2.7 Extenuating Circumstances

3.4

3.3 Where to Apply?

3.4

3.3.1 Criteria #1: Ranking

3.4

3.3.2 Criteria #2: Setting

3.4

3.3.3 Criteria #3: Size

3.5

3.3.4 Criteria #4: Programs

3.5

3.3.5 Criteria #5: Cost

3.5

3.3.6 Reach-Match-Safety

3.5

3.4 When to Apply?

3.6

3.4.1 Early

3.6

3.4.2 Regular

3.7

3.4.3 Rolling

3.7

3.4.4 Timeline

3.8

3.5 Post-January

3.8

3.5.1 The Waitlist

3.8

3.6 Special Exceptions

3.9

3.6.1 Athletics

3.9

3.6.2 Development

3.10

4. Chapter Four: Thirty Top American Universities

4.1

5. Chapter Five: Successful Application Essays

5.1

5/23/2011 5:38:46 PM

vi

  Contents

5.1

Long Stanford Essay

5.1

8.3

Difficulty Distribution

8.2

5.2

Long UVA Essay

5.2

8.4

Frequency Distribution

8.2

8.5

Reading Comprehension: Function

8.3

   Part Two: SAT Preparation

6.1

8.5.1

Reading Comprehension: Specific Interpretation

8.3

6. Chapter Six: Vocabulary

6.3

8.5.2 Reading Comprehension: General Interpretation

8.3

8.5.3 Reading Comprehension: Vocabulary-in-Context

8.3

8.5.4 Reading Comprehension: Tone, Style, and Attitude

8.3

Reading Comprehension: Skills

8.3

7.1

8.6.1

8.4

8.6.2 Skill #2: Saying the Same Thing a Different Way

8.4

8.6.3 Exercise #1: Saying the Same Thing a Different Way

8.4

Reading Comprehension: Strategies

8.6 8.6

6.1

High-Frequency Vocabulary List

6.2

Grouped Hard Word List

6.30

6.3

Long Root List

6.39

7. Chapter Seven: General Strategies 7.1

7.2

7.3

Time

7.5

Average time per question per section

7.1

7.1.2

Strategy #1: Skip-and-Return

7.1

7.1.3

Strategy #2: Use Every Minute

7.2

Space

7.2

8.6

8.7

Skill #1: Locating Information

7.2.1

Strategy #3: Write in the Test Book

7.2

7.2.2

Strategy #4: Bubble Seriously

7.2

8.7.1 Strategy #1: Long Non-Comparative Passages

7.2

Logic

8.7.2

Strategy #2: Long Comparative Passages 8.6

7.3.1

Multiple-Choice

7.2

8.7.3

Exercise #2: Long Comparative Passages 8.7

7.3.2

Strategy #5: Process of Elimination

7.2

7.3.3

Strategy #6: Rule of 1

7.3

8.7.4 Strategy #3: Short Non-Comparative Passages

7.3.4

Guessing Penalty

7.3

8.7.5

7.3.5

Strategy #7: More with Less

7.3

7.3.6

Difficulty-Rating

7.3

8.7.6 Exercise #3: Summarizing Short Comparative Passages

8.9

Quiz #2: Reading Comprehension I

8.11

7.3.7

Strategy #8: Skip the Hard Questions

7.3

7.3.8

Order of Difficulty

7.3

7.3.9

Strategy #9: Seeming Hard vs. Being Hard

7.4 7.4 7.4

Question

7.4

7.4.1

Strategy #11: Best Fit

7.4

7.4.2

Strategy #12: Knowingly not Knowing 7.4

Quiz #1: General Strategies

8.8

7.5

8.1

8.8

Strategy #4: Short Comparative Passages 8.8

9. Chapter Nine: Critical Reading II: Reading Comprehension 9.1

7.3.11 Strategy #10: Make Time Early, Take Time Late

8. Chapter Eight: Critical Reading I: Reading Comprehension

Intro.indd 6

7.1

7.1.1

7.3.10 Equality of Worth

7.4

6.3

9.1

Reading Comprehension: Secondary Strategies

9.1

9.1.1 Secondary Strategy #1: The Process of Elimination

9.1

9.1.2

Exercise #4: Hard Question Primer

9.2

9.1.3

Secondary Strategy #2: Two Questions

9.3

9.1.4

Exercise #5: What is the Question Asking?

9.4

8.1

Formal Distribution

8.1

9.1.5

Exercise #6: What is the Passage Saying?

9.5

8.2

Directions

8.1

9.1.6

Secondary Strategy #3: Approach I

9.6

5/23/2011 5:38:46 PM

vii

  Contents

9.1.7 Secondary Strategy #4: Identifying Disqualifying Information: Top-Down Method

9.6

9.1.8

9.7

Exercise #7: Finding the Trigger

9.1.9 Secondary Strategy #5: Identifying Disqualifying Information: Bottom-up Method 9.1.10 Exercise #8: Finding Inaccuracy

9.9 9.11

9.1.11 Secondary Strategy #6: Find the Trigger before you Find Inaccuracy 9.12 9.1.12 Secondary Strategy #7: Short Story Better than Long Story 9.12

10.2.3 Secondary Strategy #19: The Appreciation Effect

10.7

10.3

Secondary Strategy #20: Ultimate Approach

10.8

10.4

Quiz #4: Reading Comprehension III

10.9

11.­ Chapter Eleven: Critical Reading IV: Sentence Completions 11.1 11.1 Formal Distribution 11.2 Directions

11.1 11.1

11.3 Difficulty Distribution

11.2

9.1.14 Secondary Strategy #9: Literal vs. Liberal

11.4 Frequency Distribution

11.2

9.12

11.5 Sentence Completions: Function

11.2

9.1.15 Secondary Strategy #10: Approach II

9.12

10. Chapter Ten: Critical Reading III: Reading Comprehension 10.1 Reading Comprehension: More Secondary Strategies 10.1.1 Secondary Strategy #11: Disqualifying Information before Disqualifying Language 10.1.2

9.14

11.5.1 Sentence Completions: Single-Blank

10.1 10.1

11.2

11.5.2 Sentence Completions: Double Blank 11.2 11.6 Sentence Completions: Skills

11.3

11.6.1 Skill #1: Finding the Clue

11.3

11.6.2 Exercise #12: Finding the Clue

11.3

11.6.3 Skill #2: Identifying Direction-Cues

11.4

11.6.4 Skill #3: Generating Own-Solution

11.5

11.6.5 Exercise #13: Generating Own-Solution 11.5 10.2

Secondary Strategy #12: “In order to…” 10.2

11.6.6 Skill #4: Guessing Meaning

11.6

11.6.7 Exercise #14: Guessing Meaning

11.7

11.7 Sentence Completions: Strategies

11.8

10.1.3 Secondary Strategy #13: Saying Something Saying Nothing

10.2

10.1.4 Exercise #9: Saying Something Saying Nothing

10.2

10.1.5 Secondary Strategy #14: The Laser Effect

10.3

10.1.6 Secondary Strategy #15: Non-Specified Nouns

10.3

10.1.7 Secondary Strategy #16: Neutral Verbs and Adjectives

10.4

10.1.8 Secondary Strategy #17: Deliberately Weak Language

10.5

11.8 Quiz #5: Sentence Completions

11.11

10.1.9 Secondary Strategy #18: The Indefinite Article Effect

10.5

10.1.10 Exercise #10: Identifying Disqualifying Language

10.5

12.  Chapter Twelve: Writing I: Agreement

12.1

10.2 Reading Comprehension: TSA 10.2.1 Exercise #11: Tone

Intro.indd 7

10.7

9.1.13 Secondary Strategy #8: (A) → (B); (A) 9.12

9.2 Quiz #3: Reading Comprehension II



10.2.2 Secondary Strategy #18: Critical, not Resentful

11.7.1 Strategy #1: Clues before Direction-Cues before Word Parts

11.8

11.7.2 Strategy #2: +/- Polarity

11.8

11.7.3 Strategy #3: Approach for Single-Blank Questions

11.9

11.7.4 Strategy #4: Approach for Double-Blank Questions

11.9

11.7.5 Strategy #5: Save the Worst for Last

11.10

11.7.6 Strategy #6: Cut your Losses

11.10

11.7.7 Strategy #7: the Deal-Breaker

11.10

10.7

12.1 Formal Distribution

12.1

10.7

12.2 Directions

12.1

5/23/2011 5:38:46 PM

viii

  Contents

12.3 Difficult Distribution

12.2

12.4 Agreement: Frequency Distribution

12.2

12.5 Agreement: Sectional Frequency

12.3

12.6 Agreement: Function

12.3

12.6.1 Subject-Verb Agreement

12.4

12.6.2 Exercise #15: Subject-Verb Agreement 12.5 12.6.3 Noun-Pronoun Agreement 12.6.4

12.5

Exercise #16: Identifying Pronoun Error 12.8

12.6.5 Noun-Number Agreement 12.7 Agreement: Strategies

12.8 12.9

12.7.1 Strategy #1: The Check-List

12.9

12.7.2 Strategy #2: Identify the Subject

12.9

12.7.3 Strategy #3: Destroy the Distracters

12.9

12.7.4 Exercise #17: Identifying Distracters 12.10 12.7.5 Strategy #4: Cut the Object 12.8 General Strategies

12.11 12.11

14. Chapter Fourteen: Writing III: Modifiers, Wordiness, and Idiomatic Expression

14.1

14.1 Modifiers: Frequency Distribution

14.1

14.2 Modifiers: Sectional Frequency

14.2

14.3 Writing: Modifiers: Function

14.2

14.3.1 Modifiers: Subject Placement

14.2

14.3.2 Modifiers: Adverb Use

14.3

14.3.3 Modifiers: Modifier Placement

14.3

14.3.4 Modifiers: Quantifier Use

14.3

14.3.5 Exercise #21: Manipulating Modifiers 14.4 14.4 Wordiness: Frequency Distribution

14.4

14.5 Wordiness: Sectional Frequency

14.5

14.6 Writing: Wordiness: Function

14.5

14.6.1 Writing: Wordiness: Redundancy

14.5

14.6.2 Writing: Wordiness: Expansion

14.5

14.7 Idiomatic Expression: Frequency Distribution

14.6

12.8.1 Strategy #5: Short is Good

12.11

12.8.2 Strategy #6: No Error for Hard Questions

12.11

14.8 Idiomatic Expression: Sectional Frequency

14.6

12.8.3 Strategy #7: What to Expect

12.11

14.9 Idiomatic Expression: Function

14.6

12.9 Quiz #6: Writing I

12.12

14.9.1 Idiomatic Expression: Word Pairs

14.6

14.9.2 Idiomatic Expression:

13. Chapter Thirteen: Writing II: Parallelism

13.1

13.1 Parallelism: Frequency Distribution

13.1

13.2 Parallelism: Sectional Frequency

13.1

13.3 Parallelism: Function

13.2

13.3.1 Parallelism: Verb Use

13.2

13.3.2 Primer: Verb Tense

13.4

13.3.3 Exercise #18: Identifying Verb Tense

13.4

13.3.4 Exercise #19: Identifying Verb Lists

13.5

13.3.5 Parallelism: Clauses & Conjunctions

13.5

13.3.6 Exercise #20: Clauses

13.8

13.3.7 Writing: Parallelism: Lists

13.9

13.3.8 Writing: Parallelism: Comparison

13.9

13.4 Parallelism: Strategies

13.5

Intro.indd 8



13.9

13.4.1 Strategy #1: The Check-List

13.9

13.4.2 Strategy #2: Use What You’ve Got

13.9

Idiomatic Redundancy

14.7

14.9.3 Idiomatic Expression: The To-be Effect 14.7 14.9.4 Idiomatic Expression: Meaning

14.7

14.9.5 Idiomatic Expression:

The Between Effect

14.7

14.9.6 Exercise #22: Modifiers,

Wordiness, and Idiomatic Expression

14.10 Writing: Modifiers: Strategies

14.7 14.8

14.10.1 Strategy #1: The Check List

14.8

14.10.2 Strategy #2: Ask the Question

14.8

14.10.3 Strategy #3: Choose a Primary Clause 14.9 14.10.4 Strategy #4: Identify the Verb 14.11 Writing: Wordiness: Strategies

14.9 14.9

14.11.1 Strategy #1: The Check List

14.9

14.11.2 Strategy #2: Recognizing Wordiness

14.9

14.12 Writing: Idiomatic Expression: Strategies

14.9

13.4.3 Strategy #3: Compare Answer Choices 13.10

14.12.1 Strategy #1: The Check List

14.10

13.4.4 Strategy #4: Try it on for Size

13.10

14.12.2 Strategy #2: 23-29

14.10

Quiz #7: Writing II

13.11

14.13 Quiz #8: Writing III

14.11

5/23/2011 5:38:46 PM

ix

  Contents

15. Chapter Fifteen: Writing IV: Improving Paragraphs

15.1

15.1 Frequency Distribution 15.2 Directions

15.1 15.1

15.3 Difficulty Distribution

15.1

15.4 Improving Paragraphs: Function

15.2

15.4.1 Improving Paragraphs: Word-level

15.2

15.4.2 Exercise #23: Error List

15.3

16.10

16.4.7 Strategy #7: Counter-Example

16.10

16.4.8 Strategy #8: Danger

16.10

16.5 Quiz #10: Writing V

16.11

17.  Chapter Seventeen: Math I: General

17.1

17.1 Formal Distribution

17.1

15.4.3 Improving Paragraphs: Sentence-Level 15.3

17.2 Directions

17.2

15.4.4 Exercise #24: Assembling Paragraphs

15.5

17.3 Difficulty Distribution

17.3

15.4.5 Improving Paragraphs: Paragraph-Level 15.5

17.4 Frequency Distribution

17.3

17.5 Math: Strategies:

17.4

15.5 Improving Paragraphs: Strategies 15.5.1 Strategy #1: The Approach

15.6 15.6

15.5.2 Strategy #2: Inserting/Deleting

Sentences

15.6

15.5.3 Strategy #3: Keep it Local

15.6

15.5.4 Strategy #4: Eight Minutes

15.6

15.6 Quiz #9: Improving Paragraphs

16. Chapter Sixteen: Writing V: The Essay

15.7

16.1

16.1 Grading Rubric

16.1

16.2 Directions

16.2

16.3 The Essay: Function174

16.2

16.3.1 Cosmetic Elements

16.3

16.3.2 The Answer

16.3

16.3.3 Exercise #25: Answering Questions

16.5

16.3.4 The Reason

16.5

16.3.5 The Analysis

16.6

16.3.6 The Examples

16.6

16.3.7 The Support

16.6

16.3.8 Primer: Essay Examples

16.7

16.3.9 The Conclusion

16.8

16.4 The Essay: Strategies

Intro.indd 9

16.4.6 Strategy #6: Time Management

16.9

17.5.1 Grid-In: Strategy #1: Guess Freely

17.4

17.5.2 Grid-In: Strategy #2: Double Order of Difficulty

17.4

17.5.3 Strategy #3: Easy Process for Easy Questions

17.4

17.5.4 Strategy #4: Hard Process for Hard Questions

17.4

17.5.5 Strategy #5: Seeming vs. Being

17.4

17.5.6 Strategy #6: The Turning Point

17.4

17.5.7 Strategy #7: Three Options for Hard Questions

17.5

17.5.8 Exercise #26: Hard Math Questions

17.6

17.6 80 and 80

17.7

17.6.1 Strategy #8: Plug In

17.7

17.6.2 Strategy #9: Drawing

17.7

17.6.3 Strategy #10: Go Slowly

17.7

17.6.4 Strategy #11: Go Quickly

17.7

17.6.5 Strategy #12: Parameters Matter 17.7 Quiz #11: Math I

18. Chapter Eighteen: Math II: Data Analysis, Statistics & Probability

16.4.1 Strategy #1: The Template

16.9

18.1 Data Analysis, Statistics & Probability

16.4.2 Strategy #2: Memorize Examples

16.9



(DASP): Frequency Distribution

18.2 DASP: Function

17.7 17.8

18.1 18.1 18.1

16.4.3 Strategy #3: Don’t Worry about Transitions

16.9

18.2.1 Data Interpretation

18.2

16.4.4 Strategy #4: 2 is more than 3

16.9

18.2.2 Descriptive Statistics

18.3

16.4.5 Strategy #5: Vary Word Choice

16.9

18.2.3 Exercise #27: Descriptive Statistics

18.4

5/23/2011 5:38:46 PM



  Contents

18.3

18.4

18.2.4 DASP: Probability

18.5

19.3.13 Strategy #13: Plugging In III

19.14

18.2.5 Exercise #28: Probability

18.5

19.4 Quiz #13: Summarizing Main Points

19.15

Math: DASP: Strategies

18.5

18.3.1 Strategy #1: Know What You’re Looking For

18.5

18.3.2 Strategy #2: Know What You’re Looking At

18.6

20.1 Geometry: Reference Information

20.1

18.3.3 Strategy #3: List the Whole Set

18.6

20.2 Geometry: Frequency Distribution

20.2

18.3.4 Strategy #4: Probability

18.6

20.3 Geometry: Function

20.2

Quiz #12: Math II

18.7

19. Chapter Nineteen: Math III: Algebra

Intro.indd 10

20. Chapter Twenty: Math IV: Geometry

19.1

20.1

20.3.1 Coordinate Geometry

20.2

20.3.2 Exercise #32: Coordinate Geometry 20.3.3 Area and Perimeter

20.7 20.7

20.3.4 Triangles

20.8

19.1 Algebra: Frequency Distribution

19.1

20.3.5 Properties of Lines

20.10

19.2 Algebra: Function

19.2

20.3.6 Volume and Surface Area

20.10

19.2.1 Equations

19.2

20.3.7 Errata & Vocabulary

20.10

19.2.2 Exercise #29: Equations

19.3

20.4 Necessary Information

20.11

19.2.3 Functions

19.3

20.5 Geometry: Strategies

20.11

19.2.4 Exercise #30: Functions

19.6

20.5.1 Strategy #1: Use What You’ve Got

20.12

19.2.5 Word Problems

19.7

20.5.2 Strategy #2: Use the Figures

20.12

19.2.6 Properties of Exponents

19.8

19.2.7 Algebraic Expressions

19.9

20.5.3 Strategy #3: Use the Test Booklet Space 20.5.4 Strategy #4: Know the Formulae

20.12 20.12 20.12

19.2.8 Exercise #31: Substitution & Simplification

19.10

19.2.9 Inequalities

19.10

20.5.5 Strategy #5: Find the Necessary Information

19.2.10 Absolute Value

19.10

20.5.6 Strategy #6: Use the Basic Concepts 20.12

19.2.11 Newly Defined Operations

19.11

19.2.12 Proportion

19.11

19.3 Math: Algebra: Strategies

19.12

19.3.1 Strategy #1: Solve in Isolation

19.12

19.3.2 Strategy #2: Substitution

19.12

19.3.3 Strategy #3: Simplification I

19.12

19.3.4 Strategy #4: Simplification II

19.13

19.3.5 Strategy #5: Follow the Number

19.13

20.6 Quiz #14: Math IV

21. Chapter Twenty One: Math V: Numbers & Operations

20.14

21.1

21.1 Numbers & Operations: Frequency Distribution

21.1

21.2 Numbers & Operations: Function

21.1

21.2.1 Word Problems

21.2

19.3.6 Strategy #6: Graphs and Functions I 19.13

21.2.2 Exercise #33: Word Problems

21.4

19.3.7 Strategy #7: Graphs and Functions II 19.13

21.2.3 Properties of Integers

21.4

19.3.8 Strategy #8: Word Problems I

19.13

21.2.4 Sequences and Series

21.6

19.3.9 Strategy #9: Word Problems II

19.13

21.2.5 Number Theory

21.8

19.3.10 Strategy #10: (n-1)

19.13

19.3.11 Strategy #11: Plugging In I

19.13

19.3.12 Strategy #12: Plugging In II

19.14

21.2.6 Counting Techniques 21.3 Numbers & Operations: Strategies 21.3.1 Strategy #1: Word Problems

21.8 21.10 21.10

5/23/2011 5:38:46 PM

xi

  Contents

21.3.2 Strategy #2: Percent

Of vs. Off vs. More Than

21.10

21.3.3 Strategy #3: Ratio vs. Proportion/Probability

21.10

21.3.4 Strategy #4: Ratio + Ratio = Proportion

21.10

21.3.5 Strategy #5: Rate

21.10

21.3.6 Strategy #6: Obvious vs. Oblivious

21.10

21.3.7 Strategy #7: Sequences: Make a Table 21.10 21.3.8 Strategy #8: Sequences: Find the Difference.

21.10

21.3.9 Strategy #9: Series: Summing Sets

21.11

21.3.10 Strategy #10: Series: Counting Elements

21.11

21.3.11 Strategy #11: Fundamental Counting Principle

21.11

21.3.12 Strategy #12:

Permutations vs. Combinations

21.3.13 Strategy #13: Intervals vs. Interval Markers 21.4 Quiz #15: Math V

22. Chapter Twenty-Two: Answer Keys

21.11 21.12

22.1

22.1 Answer Key: Exercises

22.1

22.2 Answer Key: Quizzes

22.11

Part Three: Four Full Practice Tests 23. Chapter Twenty-Three: Test One

23.1

23.3

23.41

23.11 Section 10

23.45

23.12 Answer Key

23.48

23.13 Scoring Sheet

23.49

23.14 CR and Math Tables

23.53

23.15 Writing Conversion Table

23.54

23.16 Writing MC Table

23.55

24. Chapter Twenty-Four: Test Two

24.1

24.1 Intro Sheet

24.1

24.2 Bubble Sheets

24.2

24.3 Essay

24.6

24.4 Section 2

24.9

24.5 Section 3

24.14

24.6 Section 4

24.20

24.7 Section 5

24.26

24.8 Section 7

24.30

24.9 Section 8

24.36

24.10 Section 9

24.40

24.11 Section 10

24.45

24.12 Answer Key

24.48

24.13 Scoring Sheet

24.49

24.14 CR and Math Tables

24.53

24.15 Writing Conversion Table

24.54

24.16 Writing MC Table

24.55

25. Chapter Twenty-Five: Test Three

25.1

25.1 Intro Sheet

25.1

25.2 Bubble Sheets

25.2

23.1

25.3 Essay

25.6

23.2 Bubble Sheets

23.2

25.4 Section 2

25.9

23.3 Essay

23.6

25.5 Section 3

25.14

23.4 Section 2

23.9

25.6 Section 4

25.20

23.5 Section 3

23.15

25.7 Section 5

25.26

23.6 Section 4

23.21

25.8 Section 7

25.31

23.7 Section 5

23.27

25.9 Section 8

25.37

23.8 Section 7

23.32

25.10 Section 9

25.41

23.9 Section 8

23.37

25.11 Section 10

25.45

23.1 Intro Sheet

Intro.indd 11

21.11

23.10 Section 9

5/23/2011 5:38:46 PM

xii

  Contents

25.12 Answer Key

25.48

26.6

Section 4

26.19

25.13 Scoring Sheet

25.49

26.7

Section 5

26.25

25.14 CR and Math Tables

25.53

26.8

Section 7

26.30

25.15 Writing Conversion Table

25.54

26.9

Section 8

26.35

25.16 Writing MC Table

25.55

26.10 Section 9

26.39

26.  Chapter 26: Test Four

26.1

26.11 Section 10

26.44

26.12 Answer Key

26.47

26.13 Scoring Sheet

26.48

26.14 CR and Math Tables

26.52

26.15 Writing Conversion Table

26.53

26.16 Writing MC Table

26.54

Intro.indd 12

26.1 Intro Sheet

26.1

26.2 Bubble Sheets

26.2

26.3 Essay

26.6

26.4 Section 2

26.9

26.5  Section 3

26.13

5/23/2011 5:38:46 PM

Preface

Higher education is changing. Never before we had so many students attending college. Never before have there been so many places to choose from. And never before have so many international students thought of America as an option for a meaningful and productive university experience. As the times change, the brave change with them. The SAT is an essentially daunting test: long, challenging, complicated. But it is also the great equalizer. Properly prepared for, the SAT is a chance for any student to improve his application profile, show his true potential, and even learn some interesting things. And that is exactly what this book is intended for. The Pearson Guide to the SAT is unique in having been written specifically for the Indian students. We developed this program over the course of several years, including a full year teaching in New Delhi with the Ivy Exam Group, during which we learned everything we could about the Indian high school system. We came to believe the Indian student almost unusually well-positioned for the SAT success. Our students have repeatedly proven us right. Anything worthwhile is never easy. You will find in this book a series of self-contained chapters, on everything from the American university admissions process, to algebra. There is no dead weight; there is no wasted time. The teaching material —lectures, exercises, sample problems—is based entirely on analyses of the SAT tests. The vocabulary list is based exclusively on words which have shown up in past SATs. The sample SAT tests are designed and extensively tested to reflect SAT ability in exam conditions. It is good to work hard; it is better to work intelligently. The American college education is the very best in the world. The process through which you get there might seem complex and intimidating. It isn’t. Step by step, month by month, a good plan takes you to your goal without you even noticing. All you have to do is follow it. This book is a plan, a guide. A way to help you do something extraordinary, using every trick and tip we’ve ever learned. Take heart, and work well—the goal is very much worth it.

Luck (not that you’ll need it), Nicholas Henderson Morgan Henderson

Intro.indd 13

5/23/2011 5:38:46 PM

Acknowledgements

The writing of this textbook could never have happened were it not for the efforts of many individuals. Without belief, an enterprise means nothing. Therefore, to those who believed in Ivy Exam Group from the start— Sameer Arora of Springdale School, Oren Ridenour of the American Embassy School, and Priyamvada Taneja of the British School—we thank you. Likewise, without our dedicated colleagues at Ivy Exam Group—especially Vijay Rishi and P. Bhuvaneshwari – we would never have been able to develop our program so carefully. Indeed, we would have been lost. Their professionalism and friendship made our work a pleasure. So, thank you. Without family, a man has nothing. This mammoth undertaking would surely have overwhelmed were it not for the support and patience of Mum, and E, and Andre, and Susanna. A Henderson never forgets. Thus, thank you. Without students, a teacher is nothing. Therefore, a special acknowledgment must be awarded our pupils, each different, each delightful, every one of whom remains absolutely clear in thought and memory. Sometimes it seems they taught us much more than we could have ever taught them; their input makes this book special. Thank you. Nicholas Henderson Morgan Henderson

Intro.indd 14

5/23/2011 5:38:46 PM

Introduction

Congratulations! By reading this sentence, you are starting an intellectual journey that will, eventually, culminate with an actual journey: you heading to America for your university education. We will give you fair warning, right from the start: if you study for the SAT as hard as you possibly can and if you do your applications as well and as thoroughly as you possibly can, then the process of applying can be quite exhausting. It takes about a year from start-to-finish and it demands quite a lot of you. But be ye men and women of valour! Every American college-bound student goes through the same thing and although the process is a challenge, it doesn’t have to be a misery. The end result will be admission into a university of your choice and the prospect of four excellent years of learning and growth. So throughout this book, we will do our best to remind you that you are working with an end in sight: the completion of competitive application to the universities of your dreams. Once you get the applications in the mail (or press the “submit” button), your part is done… This text is primarily designed to help you, the international student, with your SAT preparation and college applications, but the focus will be on SAT preparation. We, the authors, both attended Harvard College and have taught the SAT in New Delhi for several years. We know the test and more importantly we know how international (that is, nonAmerican) students approach the test; there are certain mistakes that non-American students tend to make and we’ll do our best to help you avoid those! To that end, this book will NOT be a replica of the myriad (SAT word!) SAT preparation books in the market… it is written and designed specifically for you. The book will start with general information about the American Higher Education system: how it’s structured, what it’s like and how it’s different from India’s Higher Education system (since every country has its own ideas about what a

Intro.indd 15

university education should be like!). There are advantages and disadvantages to an American college education and we will discuss both. An American education isn’t right for all people and there are significant costs associated with it (most notably, the tuition fees), but there are also some very significant advantages. Theoretically, by reading this book you have already decided to go to an American college or university, but I t never hurts to complement your thinking: we will provide you with unbiased, neutral information on the “lay of the land” of American universities and we will present it to you in such a way that, hopefully, helps clarify your thinking. Next, we will discuss the SAT in general terms. There is an interesting history to the test (it was initially started as a tool to equalize the college admissions process!) and there are several unique features to it. In our experience, being familiar with the SAT this way has helped our students with the actual test and we think that it will help you too. This section will also include general information on SAT registration dates and fees and testing center locations. Then we get to the good stuff—college applications. There’s no real “secret” to the admissions process, but it does hinge upon a distinctly American skill: the ability to market and sell yourself to the universities of your choice. This process isn’t very complicated but there are a few common mistakes that non American students tend to make in their college applications and we’ll make sure that you don’t fall prey to those. To cap off the applications section, we list what we consider to be the top 30 colleges and universities in America. This is not a strictly academic ranking, nor is it strictly based on quality of student life; in fact, the list isn’t even ranked! The institutions on the list are those that, in our opinion, balance very strong academics with all the elements for a happy undergraduate experience and we’d be glad to send our stu-

5/23/2011 5:38:46 PM

xvi dents to any of these. We’ll expand a little bit on why each university has “made the cut” and what makes it special. Note that this list is far from comprehensive… there are plenty of universities that are excellent and where you could be very happy. In addition to the top — thirty list, we also offer you four successful application essays to top-ranked American universities. These will give you some idea of the variety of topics that can serve as application essay material, as well as the level of writing required for attention from the best colleges and universities. Now, to the meat-and-potatoes of the book: SAT preparation. The SAT is a test of developed abilities, so there isn’t too much that you need to know, information-wise. What you do need to know, however, you need to know so well that you can apply it in a variety of different contexts to many different kinds of questions! We will tell you two kinds of things: information you need to know and ways to expect to apply that material on the test. We can’t tell you exactly what to expect for any given test date—if we could, we’d be mindreaders—but we can and will tell you how often certain kinds of problems are expected to show up and how to deal with those questions. The SAT preparation material is the same as what we teach our students: it begins with a big vocabulary list, because often the thing preventing success on the SAT is words! Students must toil diligently to augment their comprehension of challenging lexemes (translation: our students work hard to increase their vocabularies), because very often on the test unfamiliarity with a hard word can make the difference

Intro.indd 16

  Introduction

between a correct and incorrect answer and those incorrect answers add up! So, our students learn a big list of words and you will too. Other SAT books don’t stress this as much as we do and that’s because those books are intended for students with well-developed vocabularies. We mean no offense: we, the authors, speak only English and some very poor French, whereas you know at least two languages fluently and possibly more. The SAT is in English, though and in order to be competitive with native speakers of English, you’ll need to work extra-hard on learning the necessary words. After the vocabulary list, you will learn about general testtaking strategies, then Critical Reading, then Writing and then Math. We won’t discuss these sections here because they will be covered at length later in the book, but suffice to say that there is much to learn and there are plenty of exercises to reinforce that material, so you will not be bored. Finally, we have included for your consideration four full-length tests, with bubble sheets and answer keys, so you can test yourself in proper SAT conditions and get a good idea of your competency. This will inform your revision schedule and strategies: you’ll know if you need extra work on Math or Writing or—if you’re like most students—Critical Reading. Please enjoy this book. In the words of Thomas Jefferson, the third president of the United States of America, “I’m a great believer in luck and I find the harder I work the more I have of it.” Work hard, reader and you will find out exactly how lucky you can be. We promise you: if you put in the work, you will be satisfied with your results.

5/23/2011 5:38:46 PM

PA R T

1

chap01.indd 1

The SAT and American Universities

5/23/2011 5:03:03 PM

This page is intentionally left blank.

chap01.indd 2

5/23/2011 5:03:03 PM

C h apt e r

1

Why American Universities?

I

n reading this book, you have most likely already made up your mind to apply to an American university; we think this is a very good idea. In that spirit, this chapter contains general no-nonsense information on the American college system and expands on the reasons why an international student might and might not, want to go to college in America.

1.1. General Information We’ll start with the most obvious place to start: the names! Often, Americans use the phrases “college” and “university” interchangeably—as we did in the introduction—but the two don’t necessarily mean the same thing. There is actually no clear-cut way to discern the two, since different institutions can be organized into “schools” or “colleges”, or both, but the technical difference is that a university provides both undergraduate and graduate education and has at least one doctoral program, while a “college” is primarily for undergraduates. More generally, a university is the umbrella organization encompassing all the various schools (or, colleges), within it: so, for instance, within Harvard University there is an undergraduate college, a graduate school of arts and sciences, a medical school, a divinity school, a business school and more. As a result, the undergraduate population at Harvard is about 7000, while the total number of all students is over 20,000! At some universities, you apply to a certain “school” (like the Stanford School of Engineering, or the Georgetown School of Nursing), while at others you apply to a certain “college” (like the College of Arts and Sciences at the University of Pennsylvania). This can be confusing, but rest assured: if you’re confused, everyone else probably is, too! Check out the universities’ websites and you will almost certainly find all the information you need to know. Almost all undergraduate degrees in America require four years of study; it is possible to use A.P. and I.B. test scores to

chap01.indd 3

obtain credit for introductory classes and so “skip” enough classes so four years is compressed to three years, but the programs are designed to be four years long and the vast majority of students use all four years (and sometimes more!). You might have heard of a “community college”: they are two-year institutions and offer an Associate’s Degree, which is sort of a stepping-stone to an undergraduate degree. All undergraduate programs grant Bachelor’s degree—usually either a BA (Bachelor of Arts) or B.Sc. (Bachelor of Science). Again, most Bachelor’s degrees require four years, but some (such as the B.Arch, or Bachelor of Architecture) require five and some (such as the Bachelor of Nursing) require only three. Indeed, due to the growing cost of American universities, there is a growing trend of three-year undergraduate courses, but that is as yet insignificant. If you do well in your studies and receive good marks, then you might graduate with “honors”; note, though, that this is not the same as an Indian Honors program. A student studies a subject (for example, economics) and then, if she graduates, receives a BA in economics. If she receives the highest possible honors, then she graduates summa cum laude with a BA in economics. All accredited universities have requirements that students must satisfy to earn a diploma (beware the “diploma mills”, where you can simply pay for a diploma… it shouldn’t be quite that easy!) and most often these requirements are in the form of “credit hours” and necessary classes. It varies subjectby-subject and university-by-university, but most often the university will require that a student takes a certain number of “credit hours” (where each class is usually three or four credit hours, depending on how often it meets) overall and a certain number of those credit hours in a certain subject. So, for instance, if you attend the University of Maryland and major in mathematics, you will be required to take a certain number of “credit hours” in order to graduate and within that, you will need to study a certain amount of “credit hours” in math classes. Generally, students take four classes per semester and

5/23/2011 5:03:03 PM

1.  take two of the four classes each semester in their area of specialization. Classes usually meet two or three times per week. You might be thinking—I have to be there for four years, but only half the classes I take each semester will be in my field of study?! That’s right. It’s an interesting quirk of the American higher-education system that there is an emphasis on “well-roundedness”, so even if you want to study mathematics or engineering, you will still be required to take literature and language classes. Other countries’ systems don’t have such requirements: in England, for example, if you study Economics, then every class you take will be economics-oriented. Not so in America! There are valid arguments for and against this system: on the one hand, it broadens the students’ academic horizons and potentially helps them grow in ways that wouldn’t have happened had the students not taken the extra classes; on the other hand, the extra classes don’t impart any knowledge specific to the degree and extend the time at university by at least one year. Love it or hate it, if you attend a university in America, you’re stuck with it. So, on the one hand, four years is a long time, each semester at university costs money (we’ll get to that!) and you’ll have to take classes that don’t contribute to your knowledge in your field of choice; on the other hand, in most cases you don’t have to know what you want to study. Of course, if you apply to and are accepted at a particular school—say, a school of engineering—then, you’ll have to study whatever it is that that school specializes in. But, if you don’t attend a specific “school”, then you have a wonderful, liberating situation: you can show up at college and have no idea what subject you want to specialize in! Most universities allow two years for you to choose, so you can spend your first year (and maybe your second) exploring various subjects that you think you might want to study seriously. A perfect example is our roommate from Harvard: he showed up on campus dead-set on studying biology and ended up with a religion major. That’s one of the best part of an American education—the freedom to discover subjects and to study what you truly want to study. Of course, like everything, there is a potential down-side: it is distinctly possible for a person to be so busy “exploring” various topics that once he has chosen a subject, there isn’t much time left in which to actually learn the subject! It’s a common critique of the American system and a valid one. But there’s little unfairness to it: it’s up to you to get the most out of your university career and you live with the choices you make. You are not bound to your choice of the Commerce, Humanities, or Science stream and we relish the fact that in the American system, choices you make at age 15 do not still bind by age 20. People change and tastes change and your academic interests should be allowed to change too.

chap01.indd 4

Part 1    The SAT and American Universities So, you have a year or two to explore and then it’s time for you to declare a “major”, as fields of specialization are usually called (although at Harvard it’s called a “concentration”). Some universities offer a “minor”, too, which you can tack on to your degree provided you take enough classes in the field. A “minor” can be a good way to distinguish yourself from other applicants for jobs, graduate school, etc., especially if it’s in a field that complements your major (e.g., a major in business with a minor in mathematics).

1.2. Advantages and Disadvantages of an American Education We were educated in America, but many of our friends and family were not and some of our students don’t eventually end up at an American university. We like to think of ourselves as impartial (SAT word!)—that is, we don’t have any inherent bias for or against American universities. Different students have different circumstances and attending an American university may or may not make sense depending on your circumstances. Here, we’ll discuss what we consider to be the advantages and disadvantages of an American higher education.

1.2.1. Advantages: Flexibility We mentioned this before and we’ll mention it again because it’s one of the distinguishing features of the American highereducation system: flexibility! Unless you attend a specific school (say, of business, or engineering, or nursing), then you are almost completely free to study whatever you want and you have at least a year in which to make up your mind. If you take four classes per semester then that’s eight potential explorations into a variety of subjects that you might not have even known existed! Philosophy, linguistics, classics, anthropology, literature, romance languages, history… no matter how good your high school score how hard you’ve studied, you have had very limited exposure to the vast scope of academic subjects in the world and you might just have an affinity (SAT vocab!) for a subject you never knew existed. An American college education will give you the chance to figure that out.

1.2.2. Advantages: Variety There are over six thousand—yes, thousand!—degree-granting institutions in America (note: this includes community colleges, trade schools and other non-university institutions). That means that there is a huge variety in the sort of undergraduate experience that can be had: urban, suburban, rural, huge, large, small, tiny, religious, secular and so forth. The

5/23/2011 5:03:03 PM

1. 

Chapter 1    Why American Universities? downside to this is that while there are many, many colleges and universities, only a subset of the population of degreegranting institutions are accredited (that is, able to award degrees that other institutions recognize) and even among the accredited universities, only a subset of these are well-known and well-respected. A good rule of thumb we use in advising our students is that of the 4,000 institutions, 400 are respectable and 40 are excellent. Among those top 40, though, one is almost as good as another in terms of the quality of education that they will provide.

1.2.3. Advantages: Rankings We don’t put too much faith in college rankings—since the order can change significantly depending on the criteria used—but measured along almost any lines (for example, student satisfaction, professional reputation, research output, endowment levels), American universities will not only top the list, but comprise around 15 of the top 20. Take, for instance, two prominent rankings: the Times Higher Education Ranking of World Universities (English) and the Academic Ranking of World Universities (Chinese). Of the two rankings, the first lists 13 American universities among the top 20 and the second lists 17 American universities in the top 20.

1.2.4. Endowment Forget the subjective rankings—what about a ranking that does not change according to the standard of measurement being used? You could use a metric like graduation rate, or job placement, or Nobel Laureates on faculty, but we think there’s a more convenient measure that encompasses all others: endowment levels. Money is the universal lubricant that makes endowed professorships, research institutions and sports teams possible and endowments play a big part of that. Universities get their money from a variety of sources—primarily tuition, government appropriations, donations and endowment income—and for private universities and colleges in 2006 endowment income (or, simply, the interest a university earns on its endowment) accounted for over onethird of the money universities spent per student. Endowment matters! Despite the recent economic turmoil, the American higher education system still dominates the world in terms of dollars and cents. While there is no comprehensive world ranking of universities by endowment level, Harvard University’s endowment tops the list of American universities in 2009, at about 25 billion USD (down from about 36 billion USD the year before), whereas Oxford and Cambridge Universities, as

chap01.indd 5

of 2009, were the best-endowed of all British universities and each had endowments of about 5 billion USD; the University of Toronto is the best-endowed of Canadian universities and has an endowment of about 1 billion USD. As of 2009, there were 56 American universities with endowments greater than 1 billion USD; by contrast, in India as of 2008, there wasn’t one university with an endowment of 100 million USD. In fact, the endowment levels for the IITs vary from 10 to 20 million USD! High endowments, alone, don’t mean much—after all, it’s just a number that universities officials take great pleasure in seeing increase. However, the endowment fund things like higher salaries for professors (resulting, theoretically, in better professors), more better dorms, more and better research centres, sports teams, student clubs, and, last but certainly not least, scholarships and financial aid. But, more on that later.

1.2.5. Post-Graduation America has not only an excellent under-graduate education system, but also an excellent post-graduate education system. In fact, more international students go to America for graduate education (276,000 in 2008) than for undergraduate education (1,77,000 in 2008)! Depending on what you want to do after your under-graduate career—whether to work, or to pursue a professional degree, or to pursue another academic degree—it can behoove (SAT vocab!) you in many ways to have studied in America. Most American employers aren’t familiar with the quality of non-American universities and so will hesitate to hire even properly-credentialed internationals; and while the top graduate programs will know very well the nuances of the Indian-higher education system, the same cannot be said for all graduate programs in America. And let’s not forget the little things like acculturation and language. if you have already spent a few years in America for your undergraduate degree, doing a graduate degree will not require a significant transition. However, coming straight from an IIT to a graduate program in a major American city might for some be challenging.

1.2.6. Perception The last “advantage” we list applies only to those students who plan to return to India after doing their under-graduate education in America and is based less on fact than on our experience. It is this: graduates of American universities generally enjoy a positive perception abroad. This, in turn, is most likely based on a perception that America reigns supreme in higher education (which, arguably, it does) based on the competitiveness of the best American colleges, the endowment

5/23/2011 5:03:03 PM

1.  levels, the expectations of increased earning potential, etc. It might not be fair, but the general fact remains that a degree from an American college or university confers personal and professional benefit to the degree-holder.

1.2.7. Disadvantages: Fees No education system is perfect and America’s is no exception. The main disadvantage is, without doubt, cost: as of 2010, the yearly fees (tuition and living expenses) at top universities were over 50,000 USD. That’s the bad news. The good news is that, because of their huge endowments, the richest universities can afford to be very generous with financial aid; so generous, in fact, that eight colleges and universities (Harvard, Yale, Princeton, MIT, Dartmouth, Williams, Middlebury and Amherst) offer need-blind admissions to international students and meet the full need of all students. This is an important topic to most students, so we’ll break it down. Financial aid: Generally, the term “financial aid” means grants or gifts from the university, not loans. Colleges and universities vary in how generous they are with their aid and they determine how much aid they think you need, based on how much money your parents have. Some universities are generous, some not at all. Universities that offer “full-need” aid will, for instance, give you every dollar they think you need, based on the income and assets your parents report to them. Need-blind admissions: Just as some universities are more generous than others in terms of how much aid they give, some universities make it harder for students seeking aid to be admitted than those who are not seeking aid. Sounds unfair, right? Well, it is and it isn’t. Universities need money to operate and it isn’t feasible for all but the very well-endowed to be extremely generous with financial aid. However, the fact is that it’s easier for students who are not seeking aid (that is, wealthier student) to be admitted to most universities; all universities, in fact, except for those offering need-blind admission. “Need-blind admission” means that the university does not consider whether or not an applicant is requesting financial aid, or how much financial aid, when making a decision. About 50 American colleges and universities claim to have need-blind admissions for domestic applicants, but, again, only eight have need-blind admissions for international students. The not-so-generous: The other end of the spectrum from “full-need, need-blind” universities are those that either do

chap01.indd 6

Part 1    The SAT and American Universities not admit international students who request finance aid, or those that make it very tough for international students requesting aid to get in. Notable not-so-generous universities include George Washington University (9% of international students awarded aid), the University of Chicago (8%), Northwestern University (3%), Cornell University (2%), Emory University (1%) and Carnegie-Mellon University, which doesn’t admit any international students who needs financial aid! Progressive pricing: Two-third of all university students in America receive financial aid and the average aid award in 2008 was $9,100 per year. At the more generous universities, the aid award is distinctly higher: at Harvard, the average distributed aid in 2009 was $41,000! In fact, as of 2009 at Harvard, any admitted student whose parents earn less than $60,000 year doesn’t have to pay a cent. That $50,000 annual price tag isn’t so bad with such generous aid: those who can afford to pay it, do and those who can’t afford to pay it, don’t. Of course, most universities aren’t as generous as Harvard, or the rest of the “elite eight” that offer full-need aid and need-blind admissions to international students. But be not discouraged: if you work hard enough in school and get good enough marks and do well enough on your SATs, it’s possible to be admitted to and receive aid from, the less-generous universities.

1.2.8. Disadvantages: Competition We mentioned this in the second point in the advantages section and we’ll mention it again but this time as a disadvantage: there are many, many colleges and universities in America, but not all of them are good. In fact, relatively few of them are good. And among the good universities, competition for entry is absolutely fierce. In 2010, Harvard received over 30,000 applications and accepted only 6.9%! Getting into a top university has never been as hard as it is right now and students (and their parents) spend huge amounts of time and money on tutors, coaches and consultants of all sorts. We, the authors, didn’t use anything like that—and didn’t go to a fee-paying school, either! —and we don’t think you need to, either. But being a non-American student means that you will benefit from SAT tuition and that’s what this book is all about. Work hard in school, study hard for your SATs and follow the advice in the Applications chapter (in chapter 3) and you’ll have as good a shot as possible.

5/23/2011 5:03:03 PM

Chapter

2

The SAT

I

f you’ve ever seen a SAT preparation book published within the past 15 years refer to the test as the “Scholastic Aptitude Test”, immediately put down the book, walk away and telephone the authors to inform them of their mistake: the “SAT” doesn’t stand for “Scholastic Aptitude Test” any more and hasn’t since 1993. Moreover, if you’ve ever seen an SAT preparation book within the past five years refer to the SAT tests as either SAT I or SAT II, run to your computer and writing a scathing review of the book on your blog: since 2004, the big, long, Math-Critical Reading-Writing SAT has been known as the “SAT Reasoning Test” and the one-hour exams in a variety of different topics have been known as “SAT Subject Tests”. The point is pedantic (SAT vocab!), but not insignificant: how can you trust a book that doesn’t even get the tests’ names right? We won’t make those mistakes. We’ve been taking and teaching this test for years and know it all too well. There are patterns in question-types and answer-choices that show up from test-to-test and we’ll share those with you in the second part of the book. For now, we’ll discuss the origins, intent, structure and registration details of the SAT. This isn’t the stuff you’ll need to know on the test, but it certainly won’t hurt you to know it now.

2.1. History The SAT is not intended to be an intelligence test and it is not intended to be a subject test; it incorporates elements of both into what’s called “developed abilities”: the SAT tests skills you’ve learnt, not facts you’ve learnt. The SAT represents one element of an application to the vast majority of colleges and universities in the US (more on that later) and while a good SAT score cannot get you in to a great university, it’s certain that a poor SAT score can keep you out of a great university; it is used as more of a screening tool than a sign

chap02.indd 1

of intelligence and that means that it’s important to do well! Moreover, you can significantly improve your SAT score with a few months of work, whereas it takes years to build a good high school transcript. The SAT is technically intended to predict first-year grades in college. So, theoretically, if you score well on the SAT, then you have the ability to do well in college and so the college should want you. Of course, it’s possible to have a bad test, or a great test, or a bad year, or to get lazy, or to gain motivation, so the correlation is far from perfect. Upon this writing, the SAT accurately predicts 18% of the variation in first-year college grades. It does a slightly worse job of predicting freshman-year grades than high-school grades, but not by much. The Scholastic Aptitude Test was first given in 1926 and was much more “speeded” than the test today. Today, you have 225 minutes to complete about 190 questions; back then, test-takers had 97 minutes to complete 315 questions! The material that the test covered was quite different from what it covers today; in that version, the topics tested were definition, classification, artificial language, antonyms, analogies, logical inference, paragraph reading, number series and arithmetic, while today’s test covers definitions; passage-based reading comprehension; application of the rules of grammar; geometry; algebra; data analysis and arithmetical operations. There are some similarities, but some significant differences! Ever since the first administration of the test, the SAT has been changed gradually in various ways and since 1970 all changes have allegedly been made with the goals of:

• •

Ensuring that test content is balanced and appropriate for test-takers with different backgrounds Ensuring test performance is reliably measured and effectively differentiates among test-takers across the full range of scores (so it can tell really smart kids from pretty smart kids, etc.), reducing the influence of speed on test performance

5/23/2011 5:04:03 PM

2. 



Reducing the effects of special preparation on test performance • Ensuring that scores can be meaningfully compared across time There were big changes to the test in 1994, which included an increase in the number of passage-based reading comprehension questions; a removal of “antonym” questions; the introduction of “grid-in” math questions, (where you have to fill in the correct answer); and the acceptance of calculators as an aid on the math portion of the test. The College Board also “recentered” the scores in 1994, so if you ever talk to anyone who took the test before 1994 and who claims that the test was harder “back in their day”, they are actually correct! The SAT is designed to have an average score of 500 for each section, but by 1994 the average score had dropped to about 900 (out of a possible 1600). The recentering simply increased the average score up to 1000 (500 for Maths and 500 for Verbal, as it was called back then), so a score of 1100 post-1994 is equivalent to a score of 1000 pre-1994. The most recent change to the SAT and the change that put the SAT into the format it has today, happened in 2005. Most significantly, a “Writing” section was added, making the test longer than it used to be and involving one 25 minute essay and two additional sections. Analogies were removed completely from the test, replaced by Short Reading Comprehension and the name “Verbal” was changed to “Critical Reading”. This test, the test you take, is scored out of 2400 points and is taken by about 2 million people world-wide every year.

2.2. Structure The test you will take has 10 sections: seven are 25 minutes long, two are 20 minutes long and 1 is 10 minutes long. There are three Maths sections, three Writing sections and three Critical Reading sections… but wait! That adds up to nine and there are supposed to be 10! That’s because there’s an “experimental section” on each test, or a section that you take just like every other section, but which does not count toward your overall score. Theoretically, if you knew which one was the experimental section, you could completely skip it, take a 25-minute nap and your score wouldn’t be hurt at all. The problem is, the SAT test-makers take great pains to ensure that you don’t know for sure where the experimental section will be. This means that to be safe you have to try as hard as you can on all the sections. The SAT test-makers (or, ETS, the Educational Testing Service) use the experimental section to test out new questions and to equate the scores between testing administrations. If everyone does terribly on the experimental section,

chap02.indd 2

Part 1    The SAT and American Universities for example, then the test administrators (the College Board) know that this test was harder than the previous test, so they raise the scores on the harder test to account for this. This raises an interesting question: at this point, if we’re explaining this clearly, you ought to be wondering how the test administrators can simply raise the average score from one test to another. How can they? The reason is that the test is scored by scaling: the questions you answer correctly in each section (Maths, Writing, Critical Reading) contribute to your Raw Score for that section, which is then scaled up to a Scaled Score out of 800 points. The scaling can and does, change from test to test, so for instance 50 Raw Points in Critical Reading might for one test date mean a score of 630 and on another a score of 610. There are all sorts of theories as to which test dates have “harder” or “easier” scoring, but there’s really no way of knowing. All you can do is try your best on every section, no matter when you take it. All but 10 questions on the test are Multiple Choice and so lend themselves well to things like the Process of Elimination and Plugging In answer choices. We won’t get into this here, but we will think carefully about it later, in terms of strategies and approaches. The main question that concerns the SAT is this: can it be prepared for? You will hear very different answers from very different sources, but we think that, emphatically, yes: it can be prepared for. There wouldn’t be a multi-billion dollar SAT preparation industry if this wasn’t the case! While there is some scope for “competitive herding”, where parents sign kids up for SAT classes just because they don’t want them to fall behind their peers (even if the classes actually do nothing), but so many of our students have improved on the test that we’re convinced that there is a significant positive effect of preparation. The College Board will have you believe that, at most, preparation confers upon the test-taker 20-30 points per section of improvement, or about 80 points overall. We think that this is a gross underestimate of the true effect of coaching (that, or the preparation the College Board uses in its studies isn’t very good!). In an ideal world, the SAT wouldn’t be “coachable”. That way, students with coaching would have no advantages over students without coaching and everything would be much fairer. Coaching does work, though and in America (and India!) people pay a lot of money for it. Companies offering high-end private SAT tuition in America typically charge $200 per hour, with young Ivy League graduates doing the tutoring. This, actually, is what motivates our writing this book: we think that top-notch, high-end instruction should not be available only for those who can afford it. All students should have access to high-end preparation and with this

5/23/2011 5:04:03 PM

Chapter 2    The SAT text, they will: our program of preparation has netted for our students an average gain of just over 250 points between their first exam-conditions practice test and their official SAT result and you’re holding that program in your hands. On the other hand, no SAT tutor is a miracle-worker and so you, the test-taker, will not be able to improve on the test without serious and rigorous preparation. Simply showing up to lessons, or flipping through the pages of this book, won’t suffice (SAT vocab!). You have to try, really try and to some students that’s a foreign concept. We’ll say to you what we say to our students: we are not psychologists, nor motivational speakers, nor guidance counselors. We can’t make you try and we don’t want to have to make you try. If you are willing to work as hard as you can, we will work as hard as we can to get you the highest possible SAT score. But if you are not willing to work as hard as you can, then perhaps you’d be better off reading another book, or not even taking the SAT.

2.3. Pedagogical Features We think that the SAT can be prepared for, but some sections are more easily prepared for than others. The consensus (SAT vocab!) of our students is that the passage-based Reading Comprehension questions are toughest and that matches up with our perspective: of all sections of the test, students take the longest time to see improvement on their Reading Comprehension scores. This isn’t just an international issue… students in America bemoan and bewail the Reading Comprehension questions, with the common complaints that they either don’t understand the passage, don’t understand the question, or don’t know the meaning of a word that they suspect holds the key the question. The most frequently heard complaint is the most justified: that reading comprehension is just… so… long! There are 48 reading comprehension questions on the test (assuming that the experimental section is not Critical Reading) and that’s more than triple any other sub-section of the SAT. Unfortunately for the test-takers, there is no magical solution to this. Of the hundreds of students we’ve taught, we have only seen two “naturals” in critical reading and both had been serious bookworms from a young age. There are strategies for dealing with Reading Comprehension and improvement is possible, but it takes time and effort. But, you already know that… The other part of the Critical Reading portion is “Sentence Completions”, which ask the test-taker to choose the word or set of words missing from a particular sentence. There are 19 of these per test and they range from very easy to very tough and the only way to prepare for these is to memorize, memorize, memorize. We have compiled a list of High-Frequency Vocabulary Words that have appeared multiple times

chap02.indd 3

2.  on multiple real SATs in the past and we’ll share that with you in Chapter Six. There’s an obvious trade-off to learning SAT words: they take time and energy to learn, but, unlike for example the Pythagorean Theorem, the probability that any particular word will appear on the next test is quite low, while the probability that you will need to use some form of the Pythagorean Theorem is 100%. So, you shouldn’t learn words willy-nilly; you should keep your word-learning focused. Enter the High-Frequency Vocabulary list. We have scoured years and years of SATs to make a big list of words that have appeared multiple times and that are therefore more likely to appear in the future. Some lists you can find online are too hard, some too easy… we have had great success with our High-Frequency list and have complete confidence in it. Learn it and then have fun recognizing the number of words you recall from it on SAT test day. The Writing Multiple Choice section has 48 questions and asks you three different kinds of questions: you have to pick the best re-phrasing of part of a bad sentence (Improving Sentences); pick the part of a sentence with an error in it (Error Identification); and pick the best way to re-write part of a paragraph (Improving Paragraphs). We teach our students and will teach you, every single rule of grammar you need to know and we find that non-American students improve quite readily on this section. Our theory is that since English is not most of our students’ first language and they learn English in school by the rules, rather than at home by listening to their parents, they already know the rules of grammar very well and so are well-positioned to apply them once they’ve had a “refresher”. We’ll give you that refresher and you’ll have all the tools you need in order to crack the Writing questions. There are 54 Math questions and the Math sections have a little wrinkle: the formulas for areas, volumes and special right triangles are provided for you at the beginning of the section. Are you thinking, “wow, how very generous of the kind and noble SAT test-makers”? If you are, think again! The SAT test-makers are not generous and have never claimed to be; it is their job to separate the kids who “get it” from the kids who “don’t get it” and they’ll use every tool they can to do so. So, in this case that means that they won’t ask you questions like “what is the formula for calculating the area of a triangle”. Instead, they’ll give you some information from which you have to derive the dimensions of the triangle to which you will then have to apply the area formula in order to find the solution. Very tricky! Tricky, but interesting. To do you well on the SAT Math, you’ll need to know the rules—the rules they give you and the rules they don’t—and you’ll need to know them so well that you can apply them in creative ways that maybe you had

5/23/2011 5:04:03 PM

2.  never thought of before. Success on Math requires creativity, flexibility and attention to detail: so many people do so well on SAT Maths that if you miss only five questions (out of 54!), your top score will be only about 700. In the Chapters 17-21, you will have lots of practice with the Math concepts and hard, multi-step math problems requiring you to find the necessary information you need to solve the problem. Take heart: practice and you will improve. You don’t have to be a “math person” to do well on SAT Math. You just have to learn the rules and learn how to apply them. The level of math on the test isn’t even very high—it covers material that you will have learned in the IXth or Xth standards. We think that, on the whole, the SAT is a fair test. It serves a useful purpose; without it and its cousin the ACT, there would be no way for universities to compare students from California with students from Massachusetts with students from India and so colleges would have to base their decisions on “school quality”, which quickly becomes a question of school resources and then you’re back in the bad-old-days when kids who went to expensive private schools got into the good universities and kids who went to public schools did not. The SAT is equalizing in principle and, we think, in practice. It’s true that there are big differences on how certain groups perform on the test—boys do better in math than girls, etc.—but we don’t think that’s the fault of the test. More likely, there are other things going on interfering with the quality of education received by the low-scoring groups. The biggest threat to the validity and fairness of the test is the large and expensive test-prep industry… but we’re here to change that.

2.4. Logistics The SAT is offered in America seven times per year (October, November, December, January, March, May and June) and in the rest of the world six times per year (no March). The test is written and graded by an American company called Educational Testing Services and it is administered by the College Board, which is a non-profit organization consisting of almost 6,000 colleges, universities and educational institutions. The test is used as one element of an application to an American college or university, in addition to essays, high school grades and letters of recommendation. Not all colleges and universities require the SAT and there is a small but growing movement of colleges and universities shunning the SAT in favor of increased emphasis on high school grades. (Interesting aside: this movement is somewhat controversial because it generally has the effect of causing “reported” SAT scores of incoming students at these non-SAT institutions to rise, since the students who get in but who also have bad SAT scores

chap02.indd 4

Part 1    The SAT and American Universities choose not to report them. The universities then trumpet this reported rise in incoming SAT scores as a sign of the improving quality of the university, when it is nothing more than a statistical artifact of going SAT-less.) There are some good colleges and universities that don’t require the SAT, so if you have real problems with the test then you might want to look into that option; however, the top institutions still require the SAT, so it’s better to stick with the test than to give it up. Moreover, at some colleges and universities SAT scores are used to determine how many scholarship dollars a student can receive. The ACT can be a viable alternative to the SAT for the courageous applicant. We don’t teach the ACT and don’t know as much about it as the SAT, but it’s generally thought to test a wider range of knowledge than the SAT and all four-year colleges and universities in America accept it in lieu of SAT scores. The correlation between SAT and ACT scores is quite high—if you score well on the SAT, you’ll probably score well on the ACT. The one potential down-side to the ACT is that there are fewer testing dates per year (4) and fewer testing centers in India (8) than for the SAT (which has six dates and 17 Indian testing centers). For Indian test-takers, the ACT costs $73 and the SAT costs $94. It’s possible to obtain fee waivers for each—check their respective websites for more information. There is no limit to how many times you can take the SAT and it’s a very good idea to take it more than once. Our experience agrees with recent research, which shows that students who take the SAT more than once see a natural improvement in their scores, above-and-beyond whatever bump tuition gives them. It’s up to you, your parents and your school counselor to make a timeline for yourself, but most of our students prepare most heavily for the SAT in January of their junior year (or, coming to the end of the XIth standard). They take the January test and can then re-test in May or June if they aren’t happy with their score and then again in October, November, or even December. You’ll probably have to take at least two SAT Subject Tests (since most American colleges and universities require applicants to submit two or three scores) and it’s a good idea to reserve at least two testing dates for those. Some students begin early—at the end of Xth standard, even—and that’s fine. You’re limited by only two things: if you start too early, then you won’t know enough Math to be competitive and if you start too late, then you won’t have time to re-test if you aren’t happy with your scores. Not only is there no limit to the number of times you can take the test, there is no penalty to taking it multiple times! In fact, there is a significant benefit to taking the test multiple times, since almost all colleges accept your best combined score. That is, if you take the SAT twice and do very well on Math the first time and very well on Critical Reading the second time, then the score that the university will consider

5/23/2011 5:04:03 PM

Chapter 2    The SAT includes both the top Math and the top Critical Reading. It’s not a universal policy—it varies university by university—but all of our Top Thirty (Chapter 4) accept the highest combined score and many, many other good colleges and universities do, too. More information can be found on the College Board website. You can register for the SAT on the College Board website and the registration deadline is usually about a month before the

chap02.indd 5

2.  actual test. “Standby Registration” is theoretically possible, but it isn’t advisable: it costs more than regular registration and there’s no guarantee that you’ll actually get a spot at the test! Moreover, some SAT testing centers are stricter with enforcing test-day rules than others (in terms of noise levels, minimum seating distances and so forth). All testing centers are the same in the eyes of the College Board, so check with your fellow SAT test-takers to get the real story on the SAT testing centers near you.

5/23/2011 5:04:03 PM

Chapter

3

Application Process

Taking and doing well on, the SAT is a very important part of the college application process. But the SAT is not an “entrance test”—far from it. It’s only one part of a college application and not even the most important part (most people agree that high school grades are more important than anything else). In this chapter, we’ll discuss the elements of a competitive application, where to apply, when to apply, what to do after the applications are in and what might make you a special exception at your university of choice.

3.1. General Information There are essentially two kinds of college applications in America: the algorithmic and the complete. The “algorithmic” kind of application is used mostly by very large, usually public universities and it consists of you reporting your grades and your SAT scores and then you receive a yes/no decision based on some internal formula the university uses. Think about it for a second—this is the easy way to run a university admissions department! There’s no reading of essays, no weighing of applicants, no dealing with last-minute supplemental material. And it can be quite comforting for the applicant: as long as your numbers are above the cut-off points, then you are almost guaranteed admission. The “complete” application is well approximated by the Common Application, which is an application used by 414 colleges and universities (47 of them public universities) and which is used only in conjunction with a holistic (SAT word!) reading of the application. There has been a trend in recent years for admissions departments to consider not just the applicant’s two numbers—SAT score and high school grade point average—but also the applicant’s interests, extracurricular activities, writing skills, academic trajectory and references in order to better assess whether or not the application is a “good fit” for a particular institution. We think that this is

chap03.indd 1

a much better way for colleges and universities to consider applications, because applicants are so much more than just their grades and SAT scores and there can be good reasons why those numbers aren’t as high as they could have been otherwise (which applicants get a chance to explain in the Common Application). The top universities all accept the Common Application and that’s very convenient for you: think of how it used to be when applicants had to write different essays for every single university they applied to! If all of the universities you’re applying to use the Common Application, then you only need to fill out the Common Application and it gets sent to all the schools you apply to. Very easy. The main downside to the trend of holistic application reading, aside from its expense for the universities, is the time applicants must take to fill it out. The universities want to know all about you, so you are doing yourself a disservice if you don’t make the application as complete as possible! It’s best not to consider this a chore, but a treat: there are very few times in your life when you will be encouraged to brag about yourself as much as possible. Take this opportunity to do so. The actual application part of the Common Application is quite unprepossessing (SAT vocab!): it is a five-page form and asks for, among other things, your future plans, your nationality, your mother’s and father’s names and occupations, high school information, your AP, IB, SAT Subject Test and SAT/ ACT scores, any other academic honors and extracurricular activities, a short essay asking you to relate information on one of your extracurricular activities in 150 words or fewer and a personal statement of no fewer than 250 words on one of the five generic topics or a topic of your choice. Finally, there’s a space for “additional information”, where you can relate any information you feel that you’d like the admissions boards to know, whether about special circumstances, additional qualifications, or anything else. Essentially, it’s space

5/23/2011 5:04:15 PM

3.  for more bragging, or for explaining why your high school record doesn’t accurately represent your potential. Seems short, right? Don’t be fooled! This is not—repeat, not—something you can do in an afternoon. Or a week. Or even a month. You get one shot to fill out this application and it’s very important that you do it to the best of your abilities; therefore, it’s advisable for you to start writing your essays at least a month before the application due date, in order to periodically come back to them with “fresh eyes”. In terms of the things you need to write, there is also an “arts supplement” and an “athletic supplement”, in which you can list your accomplishments in the arts or sport in more detail. Moreover, many universities have supplemental essays of their own that you must write in addition to those required for the Common Application. So, what looks like an afternoon of work actually takes at least a month! There are also forms that your school needs to fill out and these are very important. You can write the best essays in the world and have the best grades in the school, but if your school doesn’t complete and submit the necessary forms, then your chances of admission at your dream university will be greatly diminished. Different universities have different requirements, but all require a School Report, which your coordinator or counselor completes and which says what a wonderful student you are; a Mid-year Report, which is used to report any changes that have occurred since the submission of the School Report (such as a very good, or poor, grade in a class); a Final Report, to report any changes that may have taken place since the submission of the Mid-year Report. As you can see, the colleges take very seriously your results in your final year of school—so seriously that they give your school coordinator three chances to say great things about you! Additionally, most colleges and universities require you to submit an International Supplement, in which your school coordinator provides information about the exams that you take (whether CBSE, or A-Level, or IB, etc.). Some school coordinators know all about these forms and complete them without you having to ask. We wish we could say that all school coordinators do, but in our experience, that hasn’t been the case! If your school does not regularly have a large student outflow to the US for university, then this is something you have to monitor. It is no cause for concern; simply make an appointment to meet with your coordinator of your XIIth standard in September and tell him or her your plans to apply to American universities. If necessary, check up with him or her as you near the application deadline, to make sure that he or she is completing the necessary forms. Don’t be afraid to do this… it might seem a bit pushy, but this is your life and your future! A quick note about SAT Subject Tests: requirements vary

chap03.indd 2

Part 1    The SAT and American Universities university-by-university, but you will most likely have to submit either two or three SAT Subject Test scores. These are one-hour tests on any of 20 different subjects, in the general categories of languages, sciences, history, math and literature. These can be prepared for, but not in the same way that the SAT can be prepared for; for the SAT, you need to know skills, while for the SAT Subject Tests, you need to know facts. In general, it’s best to take the SAT Subject Tests at the end of the academic year in a subject for which you have just completed a class; so, if you are in the science stream, you could take the biology/chemistry/physics/math SAT Subject tests in May or June and be quite prepared. You can take up to three subject tests on each test date and remember that we recommend our students allot two testing dates for SAT Subject Tests: one re-take is generally enough. Back in our day, we had to mail in our college applications and received a response in the mail, too. Not so these days! It’s all online now and all serious action happens through the Common Application website (which has been known to crash the day before the Regular Application date, by the way…). Check the websites of the colleges and universities to which you are applying for more information. There is some variation in when the applications are due and we’ll get to that shortly.

3.2. Elements of a competitive application You now know so much about college applications that you ought to feel confident when the time comes to start your own. But we haven’t yet addressed the million dollar question: what makes an application good? An equivalent, but not identical, question is this: what do colleges want to see? It’s a simple question with a not-so-simple answer, because different colleges tend to have different missions and objectives and so want different things from their students. There are some common threads, though and we’ll go into those now.

3.2.1. Academics This can’t be stressed enough. The best thing you can do for yourself is to work hard at your high school and get very good grades in challenging classes. All good colleges and universities want students who will work hard during their under-graduate years and the best way of showing that is by getting good grades in high school. However, not all classes are created equal: AP Calculus is probably harder than Introduction to Art and colleges know that. The best case scenario is for you to get good grades in hard classes; the worst case is for you to get bad grades in easy classes. On the whole, we think colleges

5/23/2011 5:04:15 PM

3. 

Chapter 3    Application Process would prefer to see you get respectable grades in very tough classes, rather than excellent grades in easy classes… but that’s just our opinion. Note: your “transcript” is just a record of your school grades.

3.2.2. Trend Colleges care about your entire high school career, not just your XIth and XIIth standard grades (or, in American parlance, your “junior” and “senior” year grades). But they also care about your academic trend. Translation: if you can’t get top marks all throughout high school, it’s better for your application if you do really well in the last two years of high school, rather than the first two years! This way, you’re signaling to the universities that whatever was happening early in your high school career, you are now serious about academic and you’re ready to hit the ground running once you get to college. Note: this does not mean that it’s OK to fail classes early in your high school career.

3.2.3. SAT Scores This isn’t the most important part of your application, but good SAT and SAT Subject Test scores can tip the balance in your favor. More importantly, it’s possible to significantly improve your SAT scores within a relatively short period of time: start studying now and you can see a big score jump within a couple of months (whereas it can take years to undo a single failed class, in terms of your overall high school grades). The main idea is this: while good SAT scores cannot completely counterbalance poor high school grades, they can be used as evidence to support the argument that you will do better in college than you did in high school. Something else to note about SAT scores: since the Writing section of the SAT is relatively new—it was only instituted in 2004—some universities don’t consider Writing scores as a part of a student’s application. There are a couple of main reasons for this: some universities aren’t yet sure how to incorporate it into their decision-making process and some universities think that the Writing section isn’t a reliable predictor of a student’s ability. There aren’t many of these non-Writing institutions, but notable ones are MIT and Georgetown University. Finally, while SAT scores in general can’t “cancel out” poor high school grades, in our experience high SAT Subject Test scores can, to a significant extent, make up for relatively low SAT Reasoning Test scores.

3.2.4. Extracurricular activities Remember when we told you that most top colleges and universities read applications holistically? That means that they

chap03.indd 3

care about you-the-person, not just you-the-test score. But note: that doesn’t mean that they want to know about your favorite films and bands; it means that they want to make sure that you’ll be a “good fit” for the university community in addition to studying hard and getting good grades. The way they do this is to look at your extra-curricular activities, or what you do in addition to studying. The popular term is “well-rounded”: most people now think that colleges want to see applicants with many interests outside of school. We think that that is close to being true, but not entirely true. Colleges and universities want to see the “multi-pointed” applicant, who has pursued one or two interests seriously and thoroughly outside of school for a number of years. Almost every ambitious American high school student will join a flurry of clubs and organizations in his junior year in high school to beef up his college application and the colleges are wise to that trick! They know that it’s a ploy to seem “well-rounded” and they don’t give it much credit. But if you can demonstrate to the colleges and universities that you have stuck with an activity—like debate, sports, volunteering, etc.—for years, that will make you very attractive in their eyes. See “Special Exceptions”, below, for more information.

3.2.5. Essays As you’ve seen, the Common Application involved three essays: one short, one long (the “Personal Statement”) and one optional. Each supplemental application will involve more essays and if you fill in the Arts or Athletic supplements you’ll be responsible for even more essays. If you ask any writer, we guarantee you that they’ll tell you that re-writing is just as important as writing and the same principle applies for your essays. Write them and then put them away for a few days and then look at them again. You’ll think that some lines are great and some are awkward and some ideas don’t flow together very well and maybe the colon ought to be a semi-colon and maybe those two paragraphs should be combined… you get the idea. Writing anything meaningful will require at least a two or three serious edits. Remember that for when you write your papers in college. There used to be little to prevent an unscrupulous applicant from asking someone else to write her essays for her. This became so prevalent in the US that colleges and universities started looking very closely at essays that looked “too good”, checking them not just for plagiarism but also for consistency with the SAT essay. Their reasoning is that even though the SAT essay is written in only 25 minutes, there ought to be some similarities between it and your college essays if you truly are the author of both. We don’t know if this is just a threat, but why test it? It’s okay if you get input from your fam-

5/23/2011 5:04:15 PM

3. 

Part 1    The SAT and American Universities

ily and teachers, but you must write your own essays. Moral issues aside, the risk of detection and punishment (through the rejection of the application) is proportionally higher than the benefit that a good essay can confer. Just like SAT scores, a good essay will form part of a competitive application, but it won’t get you into a university. The ideal personal statement says something about you and says it in clear, correct English. Don’t use the statement to write what you think the admissions committee wants to hear and don’t use it to try to explain any underperformances on your part: that’s for the extra space at the end of the Common Application. If you manage to make the personal statement honest, you’ll have achieved something special. The universities want to see you write about you, so don’t give them your thoughts on the recent presidential election, or your experiences volunteering in a slum, or an excuse for that low SAT Math score. Give them something real.

invitation to whine—nothing will look worse to a university than that. But if there is a serious and valid reason why you didn’t do as well as you could have—family trouble, or personal issues, or something like that—then, colleges do take that into account. Don’t think you can explain the poor mark in chemistry by saying that the teacher was mean to you, but know that if a death in the family made your grades drop for a semester, colleges won’t overly penalize you for it.

3.2.6. Recommendations

3.3. Where to apply?

Teacher and coordinator recommendations are an important part of the college application process, but we think that they don’t matter very much. After all, there is a finite list of things that teachers can write in a letter of reference: “hardworking”, “smart”, “pleasure to work with” and so forth. The trend in recent years is for “reference inflation”, in which coordinators and teachers feel that they have to say better and better things about students in order to help with their applications. That isn’t the case! Think of letters of recommendation like SAT scores: a bad letter (like a bad score) can keep a student out of a university, but a good letter won’t get a student into a university. You might not have any influence over what your teachers write about you, but in case you do, know this: universities want to know that you work hard, that you are intellectually curious, that you work well with classmates, that you will be a good fit into the university and that the teacher—or who ever is writing the letters—unconditionally recommends you for the university. If you can get a reference saying something like that, then you have nothing to worry about!

Before you start writing your college essays or even seriously start planning your applications and maybe even before you take the SATs, you have to figure out which university you’d like to attend. This seems like an easy question, but it actually isn’t: there are so many colleges and universities in America that the options seem nearly endless. Where do you even begin? There is a small subset of American colleges and universities that are actually good (remember that one of the disadvantages of American universities is the scope for variety in the quality of education); still, it helps to have a few criteria with which you can eliminate colleges and universities from your potential list. There are five main points that we emphasize when we counsel our students on their college decisionmaking, in no particular order: Ranking, Setting, Size, Programs and Cost. After you make these choices, you’ll have a Big List, which you then need to trim down again based on the likelihood of successfully applying. More on that follows.

3.2.7. Extenuating circumstances

How important to you are ranking and reputation? Would you be just as happy at a less-well-known college as at a very-wellknown college, provided the quality of education is the same? Would you much rather be at Princeton than at Swarthmore?

Remember how the “algorithmic” application takes your grades and SAT scores and spits out a yes/no response, with no heed to any perfectly good reasons why your high school grades might not be representative of how well you could have done? The “complete” application does pay heed to these sorts of explanations and recall that there’s a space on the Common Application for just such a thing. This is not an

chap03.indd 4

Final thought: There are no guarantees in the college application process, no matter how stellar your academics (and beware anyone who tells you differently). Case in point: we had a student with terrific CBSE scores and near-perfect SATs and she didn’t get in to Harvard, Stanford, or Yale. You shouldn’t worry too much about this, because with a smart application strategy, you’ll maximize your chances of admission to a university that is well-matched to your preferences. We turn to that now.

3.3.1. Criteria #1: Ranking

3.3.2. Criteria #2: Setting Would you prefer an urban setting, or a more closed-off campus? Are there any towns or cities in which you have relatives

5/23/2011 5:04:15 PM

3. 

Chapter 3    Application Process and which you would like to be close to? Are you concerned about surviving a harsh winter? Do you have any medical conditions that would benefit from a certain climate?

3.3.3. Criteria #3: Size Do you want to go to a small, medium, large, or huge university? Some people enjoy the closeness and support of a small university and some enjoy the freedom of a huge university. Do you think you would attend classes and work hard if you had the option of slacking off? If you would stay motivated, then you have nothing to fear attending a large or huge university. If you think you would slack off, then perhaps you need a small university where advisors and professors keep you on top of things. Would you like to know the name of every single person in your college class, or would you feel bored? Do you want to have the option to join lots of different clubs and activities, or do you know what you want to do with your free time?

3.3.4. Criteria #4: Programs Do you know what you want to study? If you know that you want to study business, then consider universities with undergraduate business programs (MIT, UVA, etc.). If you know you want to study engineering, then consider universities with great engineering programs (Stanford, MIT, etc.). If you have a general idea that you might want to do something in the social sciences—like economics, anthropology, or psychology—then you ought to consider universities with good colleges of social science. And if you know that you have no idea what you want to study, seriously consider a liberal-arts college! At a liberal-arts college you have all the possibilities of the university before you: if you want to major in physics, you can major in physics. If you want to be pre-med, you can be pre-med. If you want to study history, you can study history. The one downside to this flexibility is that there is a dilution of quality: necessarily, a liberal-arts colleges offering every subject in the academic spectrum will offer less specialization in, say, chemistry, than a College of Natural Sciences (or something like that). If you know what you want to study, great. But if you don’t know, or if you think your tastes might change, seriously consider a liberal-arts college.

3.3.5. Criteria #5: Cost The average price of a year of undergraduate education has risen significantly over the past two decades, easily outpacing inflation (the natural level of price increases each year) and many people think that the situation isn’t sustainable—that

chap03.indd 5

is, prices are so high and some students have to take on so much debt and the current labor market for recent college graduates is so competitive, that things can’t continue in this way. That may be true. But there won’t be a significant change within at least the next five years and by the time change does come you’ll have graduated years ago! So, don’t waste any time wishing things were different… you’re stuck with the situation at hand and it’s up to you to figure out your best path. The cost of a year of college at one of the top universities is about $50,000; however, financial aid is widely available—although some universities are more generous with it than others—and there are excellent universities that cost much less than $50,000 (more on that in our Top Thirty list in Chapter 4). Scholarships are available, although anecdotal evidence suggests that there are approximately 10 international scholarship-seekers for every scholarship available and taking on student loans is something that almost every student has to do. Moreover, campus jobs (which international students can hold, up to 20 hours per week) can help to defray some of the cost. A college education is expensive and there’s a lot of unintentional unfairness built into the system: unless the colleges or university is very rich and very generous, applicants who can afford it have a better chance of getting in than applicants who need financial aid. That means that if you think you’ll need financial aid, you have to be so good—in terms of grades, SAT scores, essays, activities and all the other elements listed above—that no admissions committee in its right mind could turn you down. We’re here to help with that.

3.3.6. Reach-Match-Safety Once you’ve answered all the questions about ranking, setting, size, programs and cost, you should hit the books (or the ‘net) to find places that match your preferences! There are so many colleges and universities in the US that you ought to have a long list of colleges and universities that meet—or, roughly meet—your criteria. This is your Big List and now, it’s time to whittle the list and at the same time, to diversify your college portfolio! Imagine that you had a limited sum of money and were investing in stocks. Would you put all your money into risky stocks that have a small chance of yielding very high returns? Or would you put your money into very safe stocks that will give you a low rate of return but with a very high probability? The ideal solution is to do a little bit of both: to buy some risky stocks and some safe stocks. That way, you’ve protected yourself to a certain extent against risk and you’ve maximized the chances of getting a good return. The same principle applies to your college applications.

5/23/2011 5:04:16 PM

3.  Your impulse is probably to apply to a huge number of colleges and universities, since the more places you apply, the more likely you are to get into at least one of them. However, each application has a cost! Two costs, actually: a monetary cost and a time-cost. Money-wise, the average application fee is about $45—for each application! Not all applications require fees, though and there are fee waivers if your family’s income is low enough (check the College Board website for more information). But even if the applications are free, you still have to complete the supplemental applications, fill out the supplemental essays, have a family member read these essays, re-write them… it adds up. Moreover, just like an admissions department can tell if an application essay is “too good”, an admissions department can also tell if an essay on the supplemental application has been used for many other applications! If you can’t come up with good, real reasons for why you want to apply to a particular college or university, don’t bother. We generally advise our students to apply to no more than10 colleges or universities from the Big List and fewer if possible. (It didn’t used to be this way: most people used to apply to maximum of eight colleges or universities and usually only five or six. The trend of submitting 10+ applications is definitely new.) If you feel like you need to apply to more than 10, you aren’t choosing your colleges properly. Let’s assume you apply to 10 places: which 10 from your big list should you pick? Remember that you have to diversify! It isn’t smart to pick the 10 most selective colleges and universities on your list, because if the worst happens and you don’t get in to any of them, you have a difficult situation on your hands. No, you ought to choose two “reach” schools. These are your dream universities, the ones on the top of the big list you made when you went through the five selection criteria. Next, you ought to choose five or six “match” schools: that is, colleges or universities on your Big List that you match in terms of grades and SAT scores. To find these, look online to find the average grades and SAT scores of the incoming freshman class (or the 25–75% range, which amounts to the same thing), or look in the College Board’s College Handbook: your grades and SAT score ought to be close to the average of the incoming freshman class for your match schools. Be honest with yourself! These match colleges and universities are supposed to be the ones where you stand a good chance of admission and the best way to approximate this is to use your grades and SAT scores as a standard. Finally, the remaining two or three places ought to be your “safety schools”, or the schools for which your grades and SAT scores significantly exceed the average grades and SAT scores of the incoming students. You apply to these colleges and universities just to be “on the safe side” (like the name indicates!). There are no promises with the college admissions pro-

chap03.indd 6

Part 1    The SAT and American Universities cess. But, using the reach-match-safety school method, you will maximize the probability that you get into a college or university from your Big List. There are a few things you can do to increase the probability that you get into a “dream” school, though and we’ll talk about those next.

3.4. When to apply? As if the application process weren’t complicated enough, there are actually two application deadlines—Early and Regular—and four types of application: Early Decision, Early Action, Regular Decision and Rolling. We’ll take these one at a time.

3.4.1. Early The early application date is generally in mid-November, but can be in early-November or early-December: each college sets its own policy. There are over 400 colleges and universities offering Early Action or Early Decision (see below for information on the difference between the two), including almost all of the institutions that accept the Common Application. Generally, people apply both Early and Regular to different places and there are advantages and disadvantages to both. Applying Early means you have to be quite satisfied with your grades and SAT scores up to that point; so, if you think you’re capable of a terrific SAT score in November or December, or you hold out hopes that you might win that big science fair in mid-December, you might want to consider applying only Regular Decision. Moreover, if cost is an issue and you need to attend the college or university that offers you the most money, you won’t know what other universities would have offered you under the strictures of Early Decision (see below). On the other hand, applying Early does confer upon the applicants a slightly increased chance of admission! The admissions standards are no different between Early and Regular Decisions, but it’s a fact that a higher proportion of Early applicants are accepted than through the Regular round, especially for the selective universities. We think this is for three reasons: (1) the Early applicants are generally the well-qualified ones, (2) by applying Early you show the university that it is your top choice and universities like to see that and (3) by admitting some students Early, colleges and universities maintain more control over the admissions process (consider a college that has only regular admissions and finds that a very low number of students apply… it would panic!) If those aren’t good enough reasons for you, look at the historical record: Harvard, Princeton and the University of Virginia all discontinued their Early application process in 2007 because they felt that it gave an unfair advantage to the students applying Early. Maybe so,

5/23/2011 5:04:16 PM

Chapter 3    Application Process but if there’s an advantage to be had, you want to make sure that you’re getting it! There are two different kinds of early applications: Early Decision and Early Action. Early Decision: If you apply to a college or university via Early Decision, then it means that if you get in you have to attend if the financial aid package is adequate. The only way “out” of an Early Decision is if you’re applying for financial aid and the college’s aid gift isn’t sufficiently generous; nothing other than that can get you out of it. But, hopefully, if you’ve thought long and hard about your college choices, you won’t want to get out of it! You can’t apply to more than one college or university Early Decision—for the obvious reason that if you get in to both you can’t attend both—but you can apply to one college or university Early Decision and others Early Action or for Regular Decision (the latter two being non-binding). There are about an even number of colleges and universities offering Early Decision and Early Action admissions. Early Action: This is like Early Decision, but non-binding. That is, you can apply to multiple colleges and universities Early Action and you aren’t bound to attend if you get in. Some places have “single-choice Early Action”, where you can’t apply early to any other college or university, but you can still apply to other schools Regular Decision. Note that under Early Action, you can still “bargain shop” and compare financial aid offers from competing universities.

3.4.2. Regular If you aren’t sure where you want to go, or if you expect that you will get very good grades in the fall of your XIIth standard that don’t arrive in time to include with the Early application, it’s better for you to wait until the Regular round. The deadline for this varies from place to place, but it’s generally in the beginning of January for the most selective colleges and universities. Most of our students apply both Early and Regular, but some apply only Regular and we have no reason to think this puts them at a disadvantage. It’s a good thing for most, because it gives them more time to prepare for their SATs! Note: all the material presented in this chapter pertains to Regular application unless otherwise indicated (Early, Rolling, etc.). The Reach-Match-Safety application strategy applies only to colleges to which you are Applying Regular Decision.

3.4.3. Rolling Put yourself in the shoes of the dean of admissions of a university. If you are lucky enough to be dean at Harvard, then you have tens of thousands of students applying and you have

chap03.indd 7

3.  the luxury of being able to accept a small fraction of those who apply knowing that, if history is any guide, about 2/3 of the applicants you admit will eventually enroll (yes, there are some people who turn down Harvard!). That’s right—Harvard admits about 2,100 applicants even though it only has about 1,700 beds in the freshman dorms! It is betting that some of the accepted students won’t enroll and it would look very silly if every single admitted student did enroll. Something close to that happened recently and as a result some of the freshmen class had to live in upper-class dorms! At a place as steeped in history as Harvard, we assure you that this is highly unusual. And consider the opposite situation: not enough people decide to enroll and so even though you, the dean, budgeted for 1,700 students, only 1,500 will attend. Again, bad news. Consider Dartmouth College, another Ivy League institution—about 2,200 applicants are admitted and 1,100 enroll, for a 50% yield rate! And consider the University of Michigan, an excellent institution: 14,000 applicants are admitted and 6,000 enroll, for a 43% yield rate. Most colleges and universities have yield rates around 30-40%. There are three possible ways around this: you, the dean, could let in a huge amount of people in hopes that enough would enroll so there wouldn’t be a shortfall. But, there’s a significant probability that too many students would enroll and, more importantly, you might have to lower your admissions standards to let in enough students! You want to avoid that at all costs. The second way is to admit a small number of students through the Early and Regular rounds and then, if you need more students, to let in applicant one-at-a-time from the “waitlist” (more on that below). The third way is to open the admissions process early and let people apply until all the spots are full, no matter how long it takes. That’s the principle of Rolling Admissions. “Rolling Admissions” means that there is no set deadline for applications: if a university uses Rolling Admissions, it accepts applications within a very large timeframe and continues accepting applications until all the spots are filled. Most universities using Rolling Admissions start accepting applications as early as August of your XIIth standard year! And, even better, if you apply through Rolling Admissions, you’ll get a decision within a few weeks. The obvious strategy for the Rolling applicant is to apply as early as possible, since the fewer spots there are left, the higher the admissions standard becomes. This sounds great, but there is a downside: the most selective colleges and universities don’t use Rolling Admissions. But some very good colleges and universities do use Rolling Admissions and it can significantly ease the burden of the college admissions process. We’ve had students who applied through

5/23/2011 5:04:16 PM

3.  Rolling Admissions and were accepted in September of their XIIth standard and who were very happy to avoid all the stress and hassle of the Early and Regular application processes.

3.4.4. Timeline You should work with your parents and school counselor to make a timeline for yourself as you go into this process. There isn’t too much to remember, but there are moving parts and making a good plan can simplify and streamline this process. We help our students plan their application timelines; find a sample plan below. January of your junior year/XIth standard: Prepare for and take the SAT Reasoning Test for the first time. In our experience, it’s best for students to prepare the most for the first test; that way, when they retest, they already know the strategies (or, at least, have the notes so they can re-learn the strategies!) they should apply. So work hard to establish a solid Baseline Score against which you can target specific sections to focus on for your next SAT test. May of your junior year/XIIth standard: Prepare for and take three SAT Subject Tests. These can be in any subjects you like, unless the colleges or universities you’re applying to have specific subjects they want you to take. June of your junior year/XIIth standard: Retake the SAT Reasoning Test. You’ll thank yourself, even if you don’t beat your Baseline Score—remember how most colleges accept your best Combined Score? June, July and August after your junior year/of XIIth standard: Compile your Big List of colleges and universities based on the criteria we mentioned in the section above. This list can and should, be long. Don’t cut this corner! September of senior year/XIIth standard: Decide if you want to apply to any colleges or universities via the Rolling, Early Action, or Early Decision processes. If so, start working on your application essays and meet with your school coordinator to discuss your application(s). October of senior year/XIIth standard: Retake whichever has the lower score: the SAT Reasoning Test or the SAT Subject tests. November of senior year/XIIth standard: If you are submitting an Early application, submit! Choose your Reach, Match and Safety schools from your Big List based on the overall strength of the application you have compiled and start working on your Regular Decision application essays. Finally, retake whichever still have the lower score: the SAT Reasoning Test or the SAT Subject tests.

chap03.indd 8

Part 1    The SAT and American Universities December of your senior year/XIIth standard: Last-minute, emergency SAT re-takes. Generally, colleges and universities don’t consider January SAT scores in their admissions decisions. Continue to work on your regular applications. January of your senior year/XIIth standard: Submit Regular applications and heave a great sigh of relief!

3.5. Post-January Our timeline isn’t totally accurate: the work doesn’t completely stop after you send in your Regular Decision college applications. You’ll receive your decisions in March based on your Regular applications and there are still things you can do between January and the Decision Day (“D Day”, to most) to improve your chances. The most important thing: keep working hard in school. A lot of students think that they can “slack off ” once their applications have been submitted, since it’s too late for colleges to consider your grades. While colleges won’t ask the school for any grades or scores after you submit your application, you can and should let the colleges know if anything good happens: maybe you decide to take the January SAT and you finally get that Dream Score; maybe you win a poetry prize; maybe you have an editorial published in your local newspaper. If you accomplish something exceptional after you submit the application but before you hear a decision, definitely let the college or university know. One caveat: don’t overdo this. The admissions department doesn’t want to know that you got an 88% on your most recent physics test and if you are truly annoying they might reject you as not being a “good fit” for the university. You’ll receive your admissions decisions in March: you will either be accepted, or rejected, or put on the Waitlist (sometimes called “Deferred”). Even if you’ve been accepted, don’t stop working in school! We have heard of cases when an admitted student lets his grades slide so much that the offer is rescinded (SAT vocab!). You will have until early May to make your decision about which college or university to attend. If you’ve been rejected from your Dream college, don’t worry about it: there are many good colleges and universities in the US and how you do in college matters much more than where you go to college. And if you’re put on the Waitlist, it’s even more important for you to keep your grades up in the spring of your senior year/ XIIth standard! Why is that? We’re glad you asked…

3.5.1. The Waitlist Remember when you pretended you were the dean of admissions for Harvard and how you were confronted with the

5/23/2011 5:04:16 PM

3. 

Chapter 3    Application Process problem of admitting the right number of students? Well, since you’re the dean of Harvard, you’ll probably have thousands and thousands of applicants, so you don’t need to bother with Rolling Admissions; you can set a firm admissions date and be reasonably sure that you will get enough applications so that you’ll be able to fill all your spots. But, how best to go about filling those spots? Remember the three potential solutions to your problem: letting in everyone; letting in a small amount of people and then once you know how many spots you have left, letting people in one-at-a-time from the Waitlist until the college is full; and using Rolling Admissions. Well, option 1 is no good and you don’t need to use Rolling Admissions, so you’ll go with option 2: the Waitlist. The principle of the Waitlist is that it’s a “holding pen” for applicants who, for whatever reason, the college or university doesn’t immediately want, but who would still be a good fit for the institution. Back to our scenario: you’re the dean (but not of Harvard anymore—sorry!). Say you want to have an incoming class of 2,000, you have 10,000 applicants and you use Early Action and Regular Decision admissions. Let’s say that 3,000 people apply Early Action and you admit 600 of them and 7,000 people apply Regular Decision and you admit 1000 and put 3000 on the “Waitlist”. So, you’ve admitted 1,600 students total. Let’s say that half of them decide to attend, so you have 800 spots filled and need to fill 1,200 more. Enter the waitlist! Now, you can start contacting kids on the Waitlist and asking them if they want to attend and that way increase your enrollment to 2000. It would be bad for you, the dean, if you ran out of kids on the Waitlist who you can offer admission to, so you’d better make the Waitlist big! Back to the real world. There’s no official data that we can find on the size of college Waitlists, but anecdotal evidence suggests that they are substantial (which is the way any smart dean would have it). And, given that applicants now apply to so many colleges and universities, the colleges and universities have to use the Waitlist even more. Think about it: when students only applied to five colleges or universities and in the best-case scenario were only accepted to five, each university had a 1/5 chance of “getting” the applicant, all else equal. But now, with students applying to as many as 20 colleges and universities, even though most applicants wouldn’t be admitted to all 20, each institution has a reduced chance of “getting” the admitted student and so has a lower yield rate and has to rely more heavily on the Waitlist. So, it’s not the end of the world if you are put on the Waitlist! It’s not an outright rejection and there is a real and distinct possibility that you may be extended an offer—although this can come at any time, right up until

chap03.indd 9

September, when the academic year starts. If you get good grades in the spring of your senior year/XII standard, you’re only helping yourself get the university’s attention as someone who deserves a spot. Officially, there is nothing you can do to get yourself “off ” the Waitlist, but letting the college or university know that you will definitely attend if admitted can’t hurt your chances and neither can letting the admissions department know if you have any new academic accolades. We tell our students not to count on being made an offer from a college at which they’re on the Waitlist, but not to rule it out.

3.6. Special Exceptions We mentioned above that colleges actually want applicants to be “multi-pointed” rather than “well-rounded”, because they want to see both that the applicant has interests outside of school and also that the applicant can stick with an activity for a number of years. Top colleges and universities won’t care that you joined twelve clubs in your junior year/XIth standard, because they’ll realize it for what is it: a trick to seem well-rounded. Top colleges and universities will care, however, if you’ve been playing the violin since aged 10, or if you play squash very seriously, or if you are the best painter in your school, because that will show them that you are an interesting, hard-working person. And if you are really good at an outside activity, then that can significantly influence your chances of admission. There are two main channels for this: athletics and “development”.

3.6.1. Athletics Most universities have “varsity” sports teams that are funded by the university and that compete against other university teams (as compared to “club” teams, which are not funded by the university and “intramural” teams, which compete against other teams within the same university). Colleges and universities usually want their varsity sports teams to do well against other teams, since the teams can generate money for the university (rarely) or, through their success, attract more applicants (often). And since they want their teams to do well, most colleges and universities have recruiting spots, where high school athletes are “recruited” by the university to play a particular sport and so face lower academic standards for admission. Translation: if the university cares about the varsity sport and you are very good at the sport, then you could be allowed in with worse grades and SAT scores than the average admitted student. Most college coaches keep an eye out for very good high school athletes and so if you’re good enough to be

5/23/2011 5:04:16 PM

3. 10 recruited you’ll probably know it: the coaches will be calling and emailing to tell you so! But coaches aren’t perfectly efficient and it’s easy for them to miss out on very good athletes who, for whatever reason, are “under the radar”. If you think you are very good—as in, nationally ranked—at a particular sport (like tennis, squash, or soccer) and if you want to play that sport in college and if you want to try to use your skill that sport to help your college admissions, start doing some research! In the spring of your junior year/XIth standard, find out which American colleges and universities are good at your sport and then start contacting coaches via email. The worst that can happen is that they’ll tell you they aren’t interested and the best that can happen is that you’ll have help getting in to the university and possibly a scholarship! So, it’s worth a try. You can talk to you high school coach about it if you think it would help, but you are also completely free to contact coaches on your own.

chap03.indd 10

Part 1    The SAT and American Universities

3.6.2. Development We have seen that money, or more particularly the dearth (SAT vocab!) of money, can make it harder for the less-wealthy applicant to gain admission to certain colleges and universities. The opposite is also true: if your family can pay full tuition with no need for financial aid, then you stand a better chance of admission at all universities whose admissions departments are not “need-blind”. And what’s more, if your family has considerable means, then you might be able to qualify as a “development” case. What is a development case? It is when a large sum of money is given as a donation in tacit exchange for an admissions spot. Colleges and universities really don’t like disclosing information about this, but we can say for certain that it happens and that a donation—or the promise of a donation—above $100,000 could possibly make things easier for the associated applicant. Have your parents contact the university admissions department if you think you might qualify.

5/23/2011 5:04:16 PM

Chapter

4

E

Thirty Top American Universities

arlier, we mentioned two college rankings that exist for international universities (Academic Ranking of World Universities and Times Higher Education Ranking of World Universities), and there are even more for just American colleges and universities. The problem with sticking to the rankings is that each organization uses different criteria for its ordering; for instance, the ARWU thinks that research output is more important than does the THERWU, so it puts at the top of its list colleges and universities with the highest research output, rate of publication, etc. Other lists might weigh the quality of student living as most important, or the accessibility of professors, or the diversity of the student body. Each ranking does, or should, have a “methodology” section in which it specifies the details of its computations and orderings. But that doesn’t mean those are tailor-made to your exact preferences. Still, rankings can be useful. Think about it: the organizations that make the rankings have gone to the trouble of making an algorithm, collecting data, and crunching the numbers (although there are legitimate issues that can be raised against all of those steps, but we’ll assume for now that they’re all valid)—do you have the kind of time to do that? Most people don’t. So, as long as you understand the limitations of each ranking, and the relative weights placed on the various criteria, you can learn a great deal. One caveat about rankings: please don’t think a university ranked #8 on any particular list is “better” than the university ranked #9. Remember how there are over 6,000 colleges and universities in America? We assure you that it is possible to be equally happy and successful at both #8 and #9. Indeed, on

chap04.indd 1

any given list, #8 will not be much different than #18, and not even much different from #38; in our experience, as you move down the list on any particular ranking, the smartest students are all bright, motivated, hard-working, and ambitious, and professors everywhere are smart (that’s their job!). The main difference we’ve found between, say, a #8 and #38 lies in the attitude of the lower-achieving students: at a #8, even the lazy students will work hard, whereas at a #38, the lazy students will be truly indolent (SAT vocab!). One caveat to our caveat: while a #38 won’t be much different from a #8, a #238 will be different, and a #738 very different, in terms of resources, quality of teaching, quality of student body, and so forth. Remember, there are about 400 good colleges and universities in America, and 40 great ones. Finally, choosing a college is a very personal thing. You have to go through the process of compiling your Big List on the basis of a personal weighting of the five recommended criteria (Chapter 3), so it’s possible for two students with similar application profiles to generate very, very different lists. We can’t possibly tell you where to go, but in the spirit of offering guidance, we list below 30 colleges and universities that we would unconditionally recommend to our students. It isn’t a “Top Thirty” list—it’s a “Thirty On Top” list! What follows is an unranked selection, in alphabetical order, of universities with high research output, excellent teaching, selective admissions, generous financial aid, and high levels of student happiness. These things are important to us, and we think they’ll be important to you. This list is far from exhaustive, but can and should serve as a starting point in any serious college search: these are what we think are the best.

5/23/2011 5:04:25 PM

4. 

Part 1    The SAT and American Universities

BROWN UNIVERSITY

AMHERST COLLEGE Location:

Amherst, MA

Location:

Providence, RI

Type

Private — Liberal Arts College

Type:

Private — University

Date Founded:

1821

Date Founded:

1764

Undergrad Population: 1,680

Undergrad Population:

5,800

% Non-American:

7

% Non-American:

7

% Admitted

18

% Admitted

14

Median SAT score:

2150

Median SAT score:

2150

Early Application?

Yes (Early Decision)

Early Application?

Yes (Early Decision)

Common Application?

Yes

Common Application?

Yes

Average Global Ranking

Very High (Private Universities)

Near Big City?

Yes: Boston

Type of Weather

Very cold during the winter.

The Reputation:

Ivy League. Relaxed environment. Very flexible curriculum. Not very generous with financial aid to int’l students.

Average Global Ranking Very High (Liberal Arts Colleges) Near Big City?

Yes: Boston

Type of Weather

Very cold during the winter.

The Reputation:

Superior liberal-arts college. Excellent academics. Very generous financial aid. Happy students.

BOWDOIN COLLEGE Location:

Brunswick, ME

Type:

Private — Liberal Arts College

Date Founded:

1794

Undergrad Population:

1,710

% Non-American:

3

% Admitted

19

Median SAT score:

2080

Early Application?

Yes (Early Decision)

Common Application?

Yes

Average Global Ranking Very High (Liberal Arts Colleges)

BUCKNELL UNIVERSITY Location:

Lewisburg, PA

Type:

Private — University

Date Founded:

1846

Undergrad Population:

3,500

% Non-American:

3

% Admitted

30

Median SAT score:

1950

Early Application?

Yes (Early Decision)

Common Application?

Yes

Average Global Ranking

High (Private Universities)

Near Big City?

No

Near Big City?

No

Type of Weather

Very cold during the winter.

Type of Weather

Cold during the winter.

The Reputation:

Top liberal-arts college. Very good academics, accessible professors. Very generous financial aid. Happy students. Outdoorsy.

The Reputation:

Very respectable university, with good liberal arts programs. Excellent engineering program. Generous financial aid to int’l students.

chap04.indd 2

5/23/2011 5:04:25 PM

4. 

Chapter 4    Thirty Top American Universities

CLEMSON UNIVERSITY

CALIFORNIA INSTITUTE OF TECHNOLOGY Location:

Pasadena, CA

Location:

Clemson, SC

Type:

Private — University

Type:

Public — University

Date Founded:

1891

Date Founded:

1889

Undergrad Population:

913

Undergrad Population:

14,173

% Non-American:

9

% Non-American:

% Admitted

17

% Admitted

50

Median SAT score:

2250

Median SAT score:

1230 (No Writing)

Early Application?

Yes (Early Action)

Early Application?

Rolling

Common Application?

Yes

Common Application?

No

Average Global Ranking

Very High (Private Universities)

Average Global Ranking

Moderate-High (Public Universities)

Near Big City?

Yes: Pasadena, Los Angeles

Near Big City?

No

Type of Weather

Warm and mild all year

Type of Weather

Short, mild winters

The Reputation:

Best scientific university in the country. Very, very motivated student population.

The Reputation:

Significant improvement in academic quality over the past five years. Very happy student body. Affordable tuition, but no financial aid to int’l students.

COLUMBIA UNIVERSITY

CLAREMONT MCKENNA COLLEGE

chap04.indd 3

Location:

Claremont, CA

Location:

New York, NY

Type:

Private — Liberal Arts College

Type:

Private — University

Date Founded:

1946

Date Founded:

1754

Undergrad Population:

1,135

Undergrad Population:

5,600

% Non-American:

5

% Non-American:

9

% Admitted

18

% Admitted

11

Median SAT score:

1390 (No Writing)

Median SAT score:

2140

Early Application?

Yes (Early Decision)

Early Application?

Yes (Early Decision)

Common Application?

Yes

Common Application?

Yes (in 2011)

Average Global Ranking

High (Liberal Arts Colleges)

Average Global Ranking

Near Big City?

Yes: Los Angeles

Very High (Private Universities)

Type of Weather

Warm and mild all year

Near Big City?

Yes: New York City

The Reputation:

Very good liberal arts college. Very accessible professors, students happy with financial aid packages

Type of Weather

Moderately cold winters

The Reputation:

Excellent university. Ivy League. Has colleges of liberal arts and engineering. Not very generous financial aid for int’l students.

5/23/2011 5:04:25 PM

4. 

Part 1    The SAT and American Universities

DARTMOUTH COLLEGE

HARVARD COLLEGE

Location:

Hanover, NH

Location:

Cambridge, MA

Type:

Private — University

Type:

Private — University

Date Founded:

1769

Date Founded:

1636

Undergrad Population:

4,100

Undergrad Population:

6,700

% Non-American:

6

% Non-American:

10

% Admitted

15

% Admitted

7

Median SAT score:

2160

Median SAT score:

2250

Early Application?

Yes (Early Decision)

Early Application?

No

Common Application?

Yes

Common Application?

Yes

Average Global Ranking

Very High (Private Universities)

Average Global Ranking

Very High (Private Universities)

Near Big City?

No

Near Big City?

Yes: Boston

Type of Weather

Extremely long and cold winters

Type of Weather

Long, cold winters.

The Reputation:

The Reputation:

Excellent university. Ivy League. “Alternative” feel. Very generous financial aid for int’l students.

Simply the best. Very generous with financial aid. Outstanding student body. Excellent academics.

DAVIDSON COLLEGE

JOHNS HOPKINS UNIVERSITY

Location:

Davidson, NC

Location:

Baltimore, MD

Type:

Private — Liberal Arts College

Type:

Private — University

Date Founded:

1876

Date Founded:

1837

Undergrad Population:

4,600

Undergrad Population:

1,670

% Non-American:

5

% Non-American:

3

% Admitted

24

% Admitted

28

Median SAT score:

2080

Median SAT score:

1350 (No Writing)

Early Application?

Yes (Early Decision)

Early Application?

Yes (Early Decision)

Common Application?

Yes

Common Application?

Yes

Average Global Ranking

Average Global Ranking

High (Liberal Arts Colleges)

Very High (Private Universities)

Near Big City?

No

Near Big City?

Type of Weather

Short, mild winters.

Yes: Baltimore, Washington DC

The Reputation:

Good reputation. Small classes. Moderately generous financial aid to int’l students.

Type of Weather

Moderate winters.

The Reputation:

Excellent sciences. Very high research output. Quite expensive.

chap04.indd 4

5/23/2011 5:04:26 PM

4. 

Chapter 4    Thirty Top American Universities

MASSACHUSETTS INSTITUTE OF TECHNOLOGY

POMONA COLLEGE

Location:

Cambridge, MA

Location:

Claremont, CA

Type:

Private — University

Type:

Date Founded:

1861

Private — Liberal Arts College

Undergrad Population:

5,900

Date Founded:

1887

% Non-American:

8

Undergrad Population:

1,521

% Admitted

12

% Non-American:

3

Median SAT score:

2170

% Admitted

16

Early Application?

Yes (Early Action)

Median SAT score:

2190

Common Application?

No

Early Application?

Yes (Early Decision)

Average Global Ranking

Very High (Private Universities)

Common Application?

Yes

Average Global Ranking

Very High (Liberal Arts Colleges)

Near Big City?

Yes: Boston

Type of Weather

Long, cold winters.

Near Big City?

Yes: Los Angeles

The Reputation:

Excellent sciences. Very hard-working student body. Very generous financial aid to int’l students.

Type of Weather

Warm, mild climate

The Reputation:

Small classes, excellent dorms. Happy students.

MIDDLEBURY COLLEGE

chap04.indd 5

PRINCETON UNIVERSITY

Location:

Middlebury, VT

Location:

Princeton, NJ

Type:

Private — Liberal Arts College

Type:

Private — University

Date Founded:

1746

Date Founded:

1800

Undergrad Population:

4,833

Undergrad Population:

2,500

% Non-American:

9

% Non-American:

10

% Admitted

10

% Admitted

21

Median SAT score:

2230

Median SAT score:

2090

Early Application?

No

Early Application?

Yes (Early Decision)

Common Application?

Yes

Common Application?

Yes

Average Global Ranking

Average Global Ranking

Very High (Liberal Arts Colleges)

Very High (Private Universities)

Near Big City?

Yes: New York City

Near Big City?

No

Type of Weather

Moderately cold winters

Type of Weather

Long, harsh winters.

The Reputation:

The Reputation:

Excellent in languages. Very high quality of life for students. Very generous financial aid to int’l students.

Excellent university. Ivy League. Students are thought to be quite “preppy”. Generous financial aid to int’l students.

5/23/2011 5:04:26 PM

4. 

Part 1    The SAT and American Universities

RICE UNIVERSITY

SWARTHMORE COLLEGE

Location:

Houston, TX

Location:

Swarthmore, PA

Type:

Private — University

Type:

Date Founded:

1891

Private — Liberal Arts College

Undergrad Population:

3,000

Date Founded:

1864

% Non-American:

5

Undergrad Population:

1,500

% Admitted

25

% Non-American:

7

Median SAT score:

2100

% Admitted

18

Early Application?

Yes (Early Decision)

Median SAT score:

2170

Common Application?

Yes

Early Application?

Yes (Early Decision)

Average Global Ranking

High (Private Universities)

Common Application?

Yes

Near Big City?

Yes: Houston

Average Global Ranking

Type of Weather

Mild to hot weather.

Very High (Liberal Arts Colleges)

The Reputation:

Small classes. Very high quality of life for students. Generous aid to int’l students.

Near Big City?

Yes: Philadelphia

Type of Weather

Cold winters

The Reputation:

Excellent academics. Diverse, tolerant student body. Small, closed campus. Very generous financial aid to int’l students.

STANFORD UNIVERSITY Location:

Stanford, CA

Type:

Private — University

Date Founded:

1885

Undergrad Population:

TUFTS UNIVERSITY Location:

Medford, MA

Type:

Private — University

6,600

Date Founded:

1852

Undergrad Population:

5,000

% Admitted

10

% Non-American:

6

Median SAT score:

2160

% Admitted

28

Early Application?

No

Median SAT score:

2150

Common Application?

Yes

Early Application?

Yes (Early Decision)

Average Global Ranking

Very High (Private Universities)

Common Application?

Yes

Average Global Ranking

High (Private Universities)

Near Big City?

Yes: San Francisco

Near Big City?

Yes: Boston

Type of Weather

Mild and cool winters

Type of Weather

Very long and cold winters

The Reputation:

Excellent academics. Happy students. Very generous financial aid to int’l students.

The Reputation:

Good academics. Excellent study-abroad program. Little financial aid for int’l students.

% Non-American:

chap04.indd 6

5/23/2011 5:04:26 PM

4. 

Chapter 4    Thirty Top American Universities

UNIVERSITY OF CALIFORNIA — BERKELEY

UNIVERSITY OF MICHIGAN — ANN ARBOR

Location:

Berkeley, CA

Location:

Ann Arbor, MI

Type:

Public — University

Type:

Public — University

Date Founded:

1868

Date Founded:

1817

Undergrad Population:

25,000

Undergrad Population:

25,900

% Non-American:

3

% Non-American:

5

% Admitted

23

% Admitted

50

Median SAT score:

1990

Median SAT score:

1320 (No Writing)

Early Application?

No

Early Application?

Yes (Early Action)

Common Application?

No

Common Application?

No

Average Global Ranking

Very High (Public Universities)

Average Global Ranking

High (Public Universities)

Near Big City?

Yes: San Francisco

Near Big City?

Yes: Detroit

Type of Weather

Mild and warm all year

Type of Weather

Long, cold winters

The Reputation:

Good academics. Very high research output. Excellent post-graduate programs. Open and tolerant student body. Affordable tuition, but little financial aid for int’l students.

The Reputation:

Excellent academics. Considered a “Public Ivy”. Big sports culture. Affordable.

UNIVERSITY OF PENNSYLVANIA UNIVERSITY OF MARYLAND — COLLEGE PARK

chap04.indd 7

Location:

Philadelphia, PA

Type:

Private — University

Date Founded:

1740

Undergrad Population:

9,700

% Non-American:

10

% Admitted

16

Median SAT score:

2130

Early Application?

Yes (Early Decision)

Common Application?

Yes

Average Global Ranking

Very High (Private Universities)

Location:

College Park, MD

Type:

Public — University

Date Founded:

1856

Undergrad Population:

25,300

% Non-American:

2

% Admitted

47

Median SAT score:

1280 (No Writing)

Early Application?

Yes (Early Action)

Common Application?

No

Average Global Ranking

Moderate-High (Public Universities)

Near Big City?

Yes: Philadelphia, New York City

Near Big City?

Yes: Washington DC

Type of Weather

Moderately cold winters

Type of Weather

Short, cold winters

The Reputation:

The Reputation:

Some good academic programs, high research output, considered a “Public Ivy”. Very affordable tuition, but little financial aid for int’l students.

Excellent academics. Ivy League. Large sorority/fraternity scene. Urban setting. Little financial aid for int’l students.

5/23/2011 5:04:26 PM

4. 

Part 1    The SAT and American Universities

UNIVERSITY OF VIRGINA

WASHINGTON UNIVERSITY IN ST. LOUIS

Location:

Charlottesville, VA

Location:

St. Louis, MO

Type:

Public — University

Type:

Private — University

Date Founded:

1819

Date Founded:

1853

Undergrad Population:

13,700

Undergrad Population:

6,500

% Non-American:

5

% Non-American:

4

% Admitted

35

% Admitted

17

Median SAT score:

1960

Median SAT score:

1450 (No Writing)

Early Application?

No

Early Application?

Yes (Early Decision)

Common Application?

Yes

Common Application?

Yes

Average Global Ranking

Very High (Public Universities)

Average Global Ranking

Very High (Private Universities)

Near Big City?

No

Near Big City?

Yes: St. Louis

Type of Weather

Short, cool winters

Type of Weather

Cold winters

The Reputation:

Excellent undergraduate business program. Considered a “Public Ivy”. Happy students. Minimal financial aid for int’l students.

The Reputation:

Very good academics, especially sciences. Excellent medical school. Happy students, nice campus.

UNIVERSITY OF WISCONSIN — MADISON

WHITMAN COLLEGE

Location:

Madison, WI

Location:

Walla Walla, WA

Type:

Public — University

Type:

Date Founded:

1849

Private — Liberal Arts College

Undergrad Population:

29,000

Date Founded:

1883

% Non-American:

4

Undergrad Population:

1,500

% Admitted

56

% Non-American:

3

Median SAT score:

1900

% Admitted

48

Early Application?

Rolling

Median SAT score:

1990

Common Application?

No

Early Application?

Yes (Early Decision)

Average Global Ranking

High (Public Universities)

Common Application?

Yes

Near Big City?

No

Average Global Ranking

High (Liberal Arts Colleges)

Type of Weather

Very long, cold winters

Near Big City?

No

The Reputation:

Some very good undergraduate programs. Considered a “Public Ivy”. Affordable tuition, but minimal financial aid for int’l students.

Type of Weather

Rainy and mild

The Reputation:

Very good academics. Happy students. Generous financial aid to international students.

chap04.indd 8

5/23/2011 5:04:26 PM

4. 

Chapter 4    Thirty Top American Universities

WILLIAMS COLLEGE

chap04.indd 9

YALE UNIVERSITY

Location:

Williamstown, MA

Location:

New Haven, CT

Type:

Private — Liberal Arts College

Type:

Private — University

Date Founded:

1701

Date Founded:

1793

Undergrad Population:

5,300

Undergrad Population:

2,000

% Non-American:

8

% Non-American:

7

% Admitted

10

% Admitted

18

Median SAT score:

2240

Median SAT score:

2140

Early Application?

Yes (Early Action)

Early Application?

Yes (Early Decision)

Common Application?

Yes

Common Application?

Yes

Average Global Ranking

Average Global Ranking

Very High (Liberal Arts Colleges)

Very High (Private Universities)

Near Big City?

Yes: New York City

Near Big City?

No

Type of Weather

Cold winters.

Type of Weather

Long, cold winters.

The Reputation:

The Reputation:

Superb academics. Many small-class tutorials. Very generous financial aid to int’l students.

Excellent academics. Ivy League. Very generous financial aid to int’l students.

5/23/2011 5:04:26 PM

Chapter

5

Successful Application Essays

W

riting a good application essay is both an art and a science. It’s your opportunity to show the college what a good addition you’d be to the community while subtly playing up your strengths and minimizing your weaknesses. Think: the most competitive universities will be seeing 30,000 applications and picking 10% for admission. You’ve got to make yourself stand out. It’s been reviewed already: a great essay won’t get you into the college of your choice, but a bad essay—and, worse, a plagiarized essay—can certainly keep you out. Think of it as your chance to let your hair down for a few paragraphs and be yourself: the you that is more than the sum of your high school grades, SAT scores, extracurricular activities, and references. Show (don’t tell) the reader that you’re a special student with a lot to offer. Although it might not need mentioning, you must follow the rules of standard written expression on your college essay. Creativity with sentence length and punctuation is one thing; but errors of grammar and usage will not make you look good. To this end, you will want to have many, many people read your essay before you submit it, to effectively zero the probability of any little mistakes occurring. It doesn’t have to be perfect, you but want it as good as you can make it. The material contained within Chapters 12–16 will be of use, here. What follows in this chapter are two successful application essays used by our students for the 2009 application season. The essays are preceded by a brief description of each student. Take careful note as to what you think effective or useful within each piece.

5.1. Long Stanford Essay The following essay was submitted by R.S., who applied successfully to Stanford via Early Action. He had an excellent

chap05.indd 1

SAT Reasoning Score and a well-developed extracurricular activity list, in every respect a strong candidate: but no stronger than many that get rejected by the very best universities each year. But his essay made him stand out: it is poignant, funny and intelligent, without being didactic (SAT vocab!) or self-consciously flashy. There’s a sense of the person within this essay—a confident, articulate student. A lot of work went into this essay, that much is clear. But that doesn’t matter. R.S. takes a very unusual and off-beat theme (not knowing his name) and develops, through various digressions, a series of points that are both philosophical and personal, with a very neat circular structure that ends up where it started.

My Second First Name Perhaps because one wasn’t enough, my parents decided to paradoxically give me two first names. Although Raj does come first in my name, we never really got around to deciding which one was my first name. Because saying ‘Raj Angad’ or, worse, discussing the reason behind having two first names isn’t the most practical way of introducing yourself, I now find myself alternating between ‘Raj’ and ‘Angad’. Luckily, Indians don’t address others by their last names as often as people in some other cultures do. In India, names are much more than just words used to address people: Indians give their children names which have profound philosophical meanings and believe that the child will personify his name when he grows up. Angad comes from the word for ‘limb’, and denotes someone who is such a good follower that he almost becomes a limb (integral part) of his master. In a land of strong religious beliefs, the name is often used to describe a devout follower of God. Raj, a common suffix for names, means ‘king’.

5/23/2011 5:04:35 PM

5.  My parents always wanted to name me Angad. To everyone in my family, that was the name I was known by for the five months when I didn’t have an official name. But one morning, things changed. My parents possibly thought that having just one positive name was too little and I must have two. Or maybe my farsighted grandfather saw the man that the child was going to become, and how much people would love him, and persisted that there must also be a Raj in the name. Perhaps my aunt, a believer in Numerology, insisted (like any eldest sibling of a family would) that her nephew’s name must begin with an R, and since my parents saw that people were too used to calling me Angad to call me anything else, they decided to convert a suffix into a prefix and keep Angad as a part of my name. Whatever it was, life was going to be very different for the five-month-old child from then on, who barely knew what was going on at that time. To the world, he was now Raj Angad Singh. But to the family who named him Raj Angad Singh, he would always be Angad. In India, barely anyone has a second first name, or even a middle name. What’s worse is that my second first name was actually my first name. A person from nearly any part of the world would address a certain Raj Angad Singh as Raj by default and I learnt that there’s a limit to how much you can correct people against the convention. Eventually, I also got used to being called by my other first name. If you think things couldn’t get any worse, you probably missed the fact that it is way more common for people to have two surnames rather than two first names. My original first name was now becoming my surname. Now, that led to some strange situations. For instance, when I changed school after Grade 10, someone in the admissions office erroneously noted Angad as my last name and letters addressed to my parents are now labeled ‘Mr and Mrs Angad’ (thinking that Angad is the family’s surname). When those letters are sent to conservative parents who still know their son as Angad, matters get awkward because they think that there was a marriage they didn’t hear about: mine. People are a bit like computer programs, I guess. As long as the input complies with the recognized format, the system works perfectly. But as soon as there is an exception, no one can predict what the result will be. I am a Syntax Error. I had two last names too, by the way: my full name is supposed to be Raj Angad Singh Rishi. That was until my parents got fed up of either the caste system in India or of having four words in their son’s name, and they dropped the caste from the family’s names. Either way, I thankfully now have only two first names to deal with. But I guess the names are more than just coincidence. Sometimes I think I might have been a different person if I had just one first name. Frankly, I can’t imagine not having

chap05.indd 2

Part 1    The SAT and American Universities either of my names now. Raj to me represents the attachment I quickly develop for people and things around me. Raj also represents my belief in the power of strength, which can overcome all evils. Angad represents my passionately following every goal that I set for myself and my faith in God. As for what my first name is: as I said, I don’t know. And until I do, I guess I could just live with two first names and give people something of a choice, along with one more thing to talk about.

5.2. Long UVA Essay P.S. gained admission to the University of Virginia in the Regular round of applications. She had a strong SAT score and a very good high school transcript: a very good candidate overall. Her essay presents itself as a well-structured, well-informed piece that takes the opportunity to mention the seriousness with which she has pursed dance: seven years is a long time, and universities love to see that kind of commitment and dedication. In writing about a subject that is at once specific as well as highly personal, P.S. is leapfrogging thousands of cookie-cutter essays by exploring a topic (Bharatanatyam) that is probably brand new for the reader, but about which she is passionate about. Note the dual structure of the essay—at once the recollection of a significant event (dance festival in Germany) interspersed with a series of personal reflections—that draws you in and makes you feel like you’re there with her.

Dance and Me I peered out at a crowd of strange faces—all alike in their genuine, honest welcome. I raised my arms and waited for the conductor’s sign. Cue the music. There are instances in life that stay with a person and one that has stuck with me is the day I performed classical Indian dance—Bharatanatyam—at the Gothenburg Book Fair, Sweden, to inaugurate the International Scene. I learnt a lot about myself that day. I realised that I wanted to be part of this future—a place where cultures and traditions amalgamate in a complex and beautiful way. I also learnt the importance of culture in expressing and defining the person one is: that day, I understood myself to be Indian in a way that was not limiting, but the reverse, a kind of sharing. Most importantly I learnt that any art form practiced over a period of years becomes an expression of identity: that day I was a dancer, I was an Indian dancer far from home… I was. The dance has started and I become lost in music but in no danger of becoming lost in my steps: this is inside me, this interplay of war and geometry, of force and delicacy.

5/23/2011 5:04:35 PM

Chapter 5    Successful Application Essays Having learnt this dance form for over seven years, I realise that my dance lessons have been my place of happiness and hard work, of learning and executing, of grace and power. The hard experiences have taught me that life can be difficult and that to achieve anything of value, one must be prepared to struggle. The beauty has taught me about freedom. When I dance, I dance for myself, free to express myself uniquely and just be absorbed in a different world, a modern world, a brand new world, as a transcendental citizen. In this world where everyone tries to conform to the stereotypes of life, dance has taught me to be different. The climax of the fire-dance draws near. Transcendence approaches. My breathing matches the transition between Karanas, which matches the music. I can no longer see the faces of the crowd. I now understand what Abraham Maslow meant when he stated in one of his theorems: “A satisfied need cannot motivate a person.” For me, life has always been about taking the “next step”. I love to challenge myself; to find that “need” that drives me towards accomplishment. Above all, curiosity

chap05.indd 3

5.  has always been my guiding force. As a dancer, student or director of my school’s business club, I have always tried to dig deeper beneath the surface around me. I have come to the realisation that life, more or less, is like a work of art; a symmetrical dance of desire and accomplishment set against the backdrop of hard work. I am in a state of inner peace. Each part of my body is synchronised with the other in a beautiful symphony. The present is all that matters. Those moments are mine, I am those moments. I perceive my world in terms of my community. From this, I understand my identity and my future: as an Indian living in a global society. The values and customs of India are intrinsic to my character; I cannot be the same person without them. Energy, passion, strength and faith: to me these are Indian characteristics, but not solely so; they are the tenets of success in the modern world and keys to the good life, as well. The dance ends. The sound of applause brings me back to my surroundings. I peer out at the crowd of strange faces once again; their pleasure echoes my own.

5/23/2011 5:04:35 PM

Part

2

chap06.indd 1

SAT Preparation

5/23/2011 5:04:56 PM

This page is intentionally left blank.

chap06.indd 2

5/23/2011 5:04:56 PM

Chapter

6

Vocabulary

T

he SAT does not use hard words in all of its passages and questions, but the hard words do exist and it is important that you are able to understand them. After all, if you don’t understand a word in the question, then you won’t fully understand the question; and if you don’t fully understand the question, there’s a greater chance you will not choose the correct answer. There are roughly three stages of SAT required vocabulary: basic, intermediate and advanced. Basic words are what you’d find on Easy Sentence Completion questions; intermediate vocabulary will be very important in cracking questions of Reading Comprehension; the most advanced level will be the key to handling Medium and Hard Sentence Completions. It is important you make a serious effort to identify your areas of weakness and fix those problems.

chap06.indd 3

6.1. Vocabulary: High-Frequency Vocabulary List

hensive, but it is a start to building an advantageous lexicon! (See list) Note also that some of these words are provided with multiple definitions. The purpose here is not to memorize every variation and definition of every word, but to familiarize yourself with the meanings of the words so that on the SAT you’re not fooled when you see an intermediate-level vocabulary word being used in its secondary sense. Your immediate task is to go through this list and cross off every word you already know. Be honest with yourself when doing this: it doesn’t matter if you only cross off a few, but it does matter if you cut corners. Identify the unfamiliar words on this list so you can plan a schedule of revision based on learning the words you don’t know that finished just before your targeted SAT test date. Cramming words the week before the SAT can be an OK strategy, but you’d be better off learning them in a measured fashion so you can apply this increased knowledge to the other aspects of your SAT preparation.

Find below a list of “high-frequency” SAT words. These are challenging words that were used on SATs in the past and that are likely to pop up again. Note that this list is not compre-

Final point: If you are intending to score 600+ on the Critical Reading portion of the SAT, then you must be familiar with all of these words.

1

Abate

(v) to decrease in strength; to diminish

2

Abstract

(adj) thought of apart from concrete realities, specific objects or actual instances; theoretical; not applied or practical

3

Abstruse

(adj) hard to understand; esoteric

4

Abundance

(n) an extremely plentiful supply; wealth

5

Accentuate

(v) to give emphasis or prominence to; to mark or pronounce with an accent

6

Accessible

(adj) easy to approach, reach, enter, speak with or use; obtainable; attainable

7

Accommodate

(v) to do a kindness or favour to; to oblige; to provide suitably

5/23/2011 5:04:56 PM

6. 

Part 2    SAT Preparation

8

Acknowledge

(v) to admit to be real or true; to recognize the existence, truth or fact of

9

Acquisitive

(adj) the quality of wanting to acquire and own, often greedily; eager to get wealth or possessions

10

Acumen

(n) keen insight; shrewdness

11

Acute

(adj) sharp or severe in effect; intense; extremely great or serious; crucial; critical

12

Adaptation

(n) an adjustment or alteration of something to more properly fit its environment

13

Adept

(adj) very skilled; proficient; expert

14

Adhere

(v) to stay attached; to stick fast; to cling to

15

Admonish

(v) to caution, advise or counsel against something; to reprove or scold in a mild manner

16

Adroit

(adj) expert and nimble in the use of the hands or body; cleverly skilful, resourceful or ingenious

17

Advantageous

(adj) providing convenience or opportunity; favorable; profitable; useful

18

Adversary

(n) a person, group or force that opposes or attacks; opponent; enemy; foe

19

Advocate

(v) to speak or write in favor of; to support or urge by argument; to argue or labour on someone’s behalf

20

Aesthetic

(adj) pertaining to a sense of the beautiful; pertaining to emotion or sensation as opposed to pure intellectuality

21

Affable

(adj) pleasantly easy to approach and to talk to; friendly; cordial; warmly polite

22

Affect

(v) to act on; to produce an effect or change in; to impress the mind or move the feelings of

23

Affliction

(n) a state of pain, distress or grief; an unfortunate or unlucky occurrence, e.g. an illness

24

Affluence

(n) abundance of money, property and other material goods; riches; wealth

25

Ailment

(n) a physical disorder or illness, especially of a minor or chronic nature

26

Alacrity

(n) cheerful readiness, promptness or willingness; briskness

27

Alert

(adj) fully aware and attentive; wide-awake; keen; swift; agile

28

Alienate

(v) to cause to become unfriendly or hostile; to drive away one’s peers

29

Allege

(v) to assert without proof; to claim that something is true

30

Alleviate

(v) to make easier to endure; to lessen; to mitigate

31

Alliance

(n) a formal agreement between two or more persons, families, organizations or nations to cooperate for specific purposes; a merging of efforts or interests

32

Altruism

(n) the principle of unselfish concern for the welfare of others

33

Amalgam

(n) a composition or mixture of different elements

34

Amass

(v) to gather for oneself; to collect as one’s own

35

Ambiguous

(adj) open to or having several possible meanings or interpretations; of uncertain nature; lacking clarity or definition, obscure, indistinct

36

Ambivalence

(n) uncertainty or fluctuation, especially when caused by inability to make a choice

37

Ameliorate

(v) to make better, more bearable or more satisfactory; to cause to improve in condition

38

Amiable

(adj) having or showing pleasant, good-natured personal qualities; affable; friendly; sociable

chap06.indd 4

5/23/2011 5:04:56 PM

6. 

Chapter 6    Vocabulary

chap06.indd 5

39

Amorphous

(adj) lacking definite form; having no specific shape; formless

40

Amputate

(v) to cut off (all or part of a limb or digit of the body), as by surgery; to prune or remove

41

Anachronism

(n) an historical or chronological inaccuracy; an error in chronology in which a person, object, event, etc., is assigned a date or period other than the correct one

42

Analogous

(adj) corresponding to something in some function; similar or alike in such a way as to permit the drawing of an analogy

43

Analysis

(n) the separating of any material or abstract entity into its constituent parts for individual study

44

Anecdotal

(adj) based on personal observation rather than systematic scientific evaluation

45

Anguish

(n) excruciating or acute distress, suffering or pain; torment

46

Animosity

(n) an active feeling of strong dislike, ill-will or hatred; bitter hostility

47

Annotated

(adj) supplied with or containing explanatory notes, textual comments or notes, as applied to written work

48

Anonymous

(adj) without any name acknowledged; of unknown name; lacking individuality or distinction

49

Antagonistic

(adj) acting in opposition; hostile; unfriendly

50

Antecedent

(adj) preceding; prior; coming earlier

51

Antiquity

(n) the quality of being ancient; ancient-ness; ancient times; former ages; the period of history before the Middle Ages

52

Anxiety

(n) distress or uneasiness of mind caused by fear of danger or misfortune

53

Apathetic

(adj) having or showing little or no emotion; not interested or concerned; indifferent or unresponsive

54

Aplomb

(n) imperturbable self-possession, poise or assurance; self-confident assurance

55

Apocalyptic

(adj) pertaining to the end of the world; involving or portending widespread devastation or ultimate doom

56

Apparition

(n) a supernatural appearance of a person or thing, especially a ghost; a spectre or phantom; anything that appears in a remarkable or startling way

57

Appealing

(adj) evoking or attracting interest, desire, curiosity; attractive

58

Appease

(v) to bring to a state of peace, quiet, ease, calm or contentment; to pacify; to soothe; to satisfy

59

Appreciation

(n) gratitude; thankful recognition; an increase or rise in the value of goods or property

60

Apprehend

(v) to take into custody; to grasp the meaning of

61

Appropriate

(adj) suitable or fitting for a particular purpose, person or occasion

62

Apropos

(adv) fitting; at the right time; to the purpose; opportunely

63

Apt

(adj) suited to a purpose or occasion; inclined or disposed to; unusually intelligent or quick to learn

64

Aptitude

(n) capability; ability; talent; readiness or quickness in learning; intelligence

65

Arbitrary

(adj) subject to individual will or judgment without restriction; determined by chance, will or impulse and not by necessity, reason or principle

66

Arbitrate

(v) to judge or moderate an argument between two parties; to help in communication between two or more parties

67

Arcane

(adj) known or understood by very few; mysterious; secret

5/23/2011 5:04:57 PM

6. 

Part 2    SAT Preparation

68

Archaic

(adj) marked in the manner of an earlier period; antiquated; outdated

69

Arduous

(adj) requiring great exertion; laborious; difficult; strenuous; hard to endure

70

Aroma

(n) an odour or fragrance, especially an agreeable one

71

Arouse

(v) to stir to action or strong response; to excite; to awaken

72

Arrogance

(n) an offensive display of superiority or self-importance; overbearing pride

73

Articulate

(v) to utter clearly and distinctly; to give clarity or distinction to; to reveal or make distinct

74

Artefact

(n) any object made by human beings characteristic of an earlier time; the remains of a manmade object found at archaeological sites

75

Artifice

(n) a clever trick or stratagem; a cunning, crafty device or trick; ingenuity; inventiveness

76

Ascertain

(v) to find out definitely; to learn with certainty or assurance

77

Aspirations

(n) strong desire, longing or aim; ambition; a goal or objective

78

Assertion

(n) a positive statement or declaration, often made without support

79

Assimilation

(n) the act or process of bringing to a resemblance, likeness or identity; the process whereby a minority group gradually adopts the customs and attitudes of the prevailing culture

80

Astonish

(v) to fill with sudden overpowering surprise or wonder; to amaze

81

Atrocity

(n) an appalling condition, quality or behaviour; monstrousness; an act of unusual cruelty

82

Atypical

(adj) not conforming to the type; irregular; abnormal

83

Audacious

(adj) extremely bold or daring; recklessly brave; fearless; insolent or brazen; lively or unrestrained

84

Audibly

(adj) able to be heard; loud enough to be heard

85

Augment

(v) to increase in size, number, strength or extent; to increase

86

Auspicious

(adj) promising success; opportune; favorable; lucky

87

Austere

(adj) severe in manner or appearance; uncompromising; strict; forbidding; grave; solemn

88

Authentic

(adj) real, true, of documented origin; not false or copied

89

Autonomy

(n) independence of freedom of the will or action; self-government

90

Axiom

(n) a self-evident truth that requires no proof; a universally accepted principle or rule

91

Banal

(adj) lacking originality; commonplace; predictable

92

Bane

(n) a person or thing that ruins or spoils; death, destruction or ruin

93

Bankrupt

(adj) at the end of one’s resources; empty

94

Belie

(v) to show to be false; to contradict

95

Belittle

(v) to regard or portray as less impressive or important than appearances indicate; to disparage

96

Belligerent

(adj) war-like; given to waging war; aggressively hostile

97

Beneficial

(adj) advantageous; helpful

98

Benevolent

(adj) expressing goodwill or kindly feelings; desiring to help others; charitable

99

Beseech

(v) to implore urgently; to beg eagerly for

100

Bewilder

(v) to confuse or puzzle completely; to baffle or perplex

chap06.indd 6

5/23/2011 5:04:57 PM

6. 

Chapter 6    Vocabulary

chap06.indd 7

101

Botch

(v) to spoil by poor work; to ruin through clumsiness; to bungle

102

Breadth

(n) the measurement of the width of a figure; size in general; extent

103

Brevity

(n) shortness of time or duration; briefness

104

Buoyancy

(n) the power to float or rise in a fluid; lightness or resilience of spirit; cheerfulness

105

Byzantine

(adj) intricate; complex; characterized by elaborate scheming and intrigue

106

Calamity

(n) a great misfortune or disaster; grievous affliction; adversity; misery

107

Calculable

(adj) determinable by calculation; knowable; reliable

108

Callous

(adj) made hard; hardened; insensitive; indifferent; unsympathetic

109

Capricious

(adj) erratic; subject to or led by whim

110

Cataclysm

(n) a violent upheaval that causes great destruction or brings about a fundamental change

111

Catalyst

(n) an element that sparks or instigates change

112

Catastrophic

(adj) having the qualities of a widespread disaster or emergency

113

Censor

(v) to examine media for the purpose of suppressing parts deemed objectionable

114

Censure

(n) a strong expression of disapproval; an official reprimand

115

Cerebral

(adj) characterized by the use of intellect rather than intuition or instinct

116

Challenge

(v) to summon to a contest of skill, strength, etc; to take exception to; to demand as something due or rightful

117

Characterize

(v) to mark or distinguish as an individual quality of; to describe

118

Charisma

(n) a spiritual power or personal quality that gives an individual influence or authority over large numbers of people; personal magnetism or charm

119

Charitable

(adj) generous in donations or gifts to help the needy; kindly; lenient in judging people or acts

120

Cherish

(v) to hold as dear or precious; to care for tenderly; to nurture

121

Chronic

(adj) constant; habitual; continuing a long time or recurring frequently

122

Chronological

(adj) arranged in the order of time of occurrence

123

Circumlocution

(n) a roundabout or indirect way of speaking; the use of more words than necessary to express an idea

124

Circumstance

(n) a condition, part or detail that modifies a fact or event

125

Circumvent

(v) to go around or to bypass; to avoid by artfulness or deception

126

Clarify

(v) to make (a statement, idea, etc) clear or intelligible; to ease understanding; to free from confusion

127

Coerce

(v) to compel by force, intimidation or authority; to bring about through use of force; to dominate or control

128

Cogent

(adj) convincing by virtue of clear presentation; relevant; pertinent

129

Cognitive

(adj) of or pertaining to the mental processes of perception, memory, judgment and reasoning (as contrasted with emotional processes)

130

Cognizant

(adj) aware (of ); fully informed; conscious

131

Coherent

(adj) logically connected; consistent

5/23/2011 5:04:57 PM

6. 

Part 2    SAT Preparation

132

Cohesive

(adj) well-integrated; unified; capable of sticking together, as in a mass

133

Collaborate

(v) to work together toward a final goal, especially in an intellectual effort

134

Commendable

(adj) worthy of being commended or praised; laudable

135

Commonplace

(adj) ordinary; undistinguished or uninteresting; without individuality

136

Compassionate

(adj) having or showing pity or sympathy; sympathetic

137

Compensate

(v) to counterbalance; to offset

138

Competent

(adj) having suitable or sufficient skill, knowledge, experience, etc., for some purpose; properly qualified

139

Complacent

(adj) pleased, especially with oneself; self-satisfied and unconcerned; contended to a fault

140

Complex

(adj) composed of many interconnected parts; composite; so complicated as to be hard to understand or deal with

141

Compound

(v) to put together into a whole; to combine; to increase or add to

142

Comprehensive

(adj) of large scope; inclusive; covering or involving much

143

Compromise

(n) a settlement of differences by mutual concessions

144

Compunction

(n) a feeling of uneasiness or anxiety caused by regret for doing wrong or causing pain; remorse

145

Conceal

(v) to hide from view; to keep secret

146

Conceit

(n) an excessively favorable opinion of oneself; a thought or idea

147

Concession

(n) a thing or point yielded in an argument

148

Conciliatory

(adj) tending to appease or to regain friendship or goodwill by pleasant behavior

149

Concise

(adj) brief; condensed; expressing much in few words

150

Concord

(n) agreement between persons, groups, nations, etc

151

Condemn

(v) to express an unfavorable or adverse judgment on; to pronounce to be guilty; to sentence to punishment

152

Condescending

(adj) showing a patronizing descent from superiority; displaying a patronizingly superior attitude

153

Condone

(v) to approve; to allow to happen; to pardon or forgive

154

Conflagration

(n) a destructive fire, usually an extensive one

155

Conform

(v) to fit something (appearance, actions, etc,.) to a certain standard

156

Congenial

(adj) agreeable, suitable or pleasing in nature

157

Conglomerates

(n) anything composed of different or unique parts; usually, a corporation consisting of a number of divisions in a variety of unrelated industries

158

Conjecture

(v) to conclude from grounds or evidence insufficient to ensure reliability; to infer on slight evidence

159

Connoisseur

(n) a person who is fit to pass judgments in matters of taste

160

Conscientious

(adj) guided by principles; scrupulous; thorough and meticulous

161

Consecrate

(v) to make or declare sacred; to make (something) an object of honor or veneration

162

Consequences

(n) the results of a particular action occurring earlier

chap06.indd 8

5/23/2011 5:04:57 PM

6. 

Chapter 6    Vocabulary

chap06.indd 9

163

Conspicuous

(adj) easily seen or noticed; visible or observable

164

Consumption

(n) the act of using up (a supply of something)

165

Contagious

(adj) the quality of being easily spread or catchable

166

Contamination

(n) pollution; defilement

167

Contemplate

(v) to look at or view with continued attention; to observe or study thoughtfully; to think studiously

168

Contempt

(adj) the feeling or attitude of regarding someone or something as inferior or worthless; disdain, scorn; the state of being despised

169

Contentious

(adj) tending to argument or strife; quarrelsome

170

Context

(n) the part of a text or statement that surrounds a particular word or passage and thus determines its meaning; a setting

171

Contradiction

(n) an assertion of the contrary or opposite; denial

172

Contrast

(v) to compare in order to show unlikeness or differences

173

Contrite

(adj) the quality of being sorry or apologetic for something

174

Controversial

(adj) the quality of being not universally agreed upon; causing disputes

175

Conventional

(adj) ordinary, rather than different or original

176

Convey

(v) to carry from one place to another; to transport; to communicate; to impart

177

Convivial

(adj) friendly, agreeable; fond of feasting and merriment

178

Convoluted

(adj) twisted; complicated; needlessly complex

179

Cooperation

(n) the act of working together for a shared reason or goal

180

Corollary

(n) a natural consequence or easily-drawn conclusion

181

Corpulence

(n) fatness; largeness of body

182

Corroborate

(v) to agree with another’s account of something; to give evidence to

183

Cosmopolitan

(adj) free from local, provincial or national ideas, prejudices or attachments; at home all over the world

184

Courteous

(adj) having or showing good manners; polite

185

Covenant

(n) an agreement between two or more parties to do or not to do something specified; a promise

186

Coy

(adj) artfully or affectedly or flirtatiously shy or modest; slyly hesitant

187

Craven

(adj) cowardly; contemptibly timid

188

Credential

(n) evidence of being qualified for a position or role

189

Critical

(adj) inclined to find fault or to judge with severity; characterized by careful evaluation and judgment

190

Crux

(n) a vital, decisive or pivotal point; the basic or central feature or something

191

Cryptic

(adj) mysterious in meaning; puzzling

192

Culminated

(adj) having reached the highest point or final stage

193

Culpable

(adj) deserving of blame; blameworthy

5/23/2011 5:04:57 PM

6. 10

Part 2    SAT Preparation

194

Cultivate

(v) to promote the growth of (something) by labor and attention

195

Curtail

(v) to stop a process or action before its natural conclusion; to reduce

196

Cynical

(adj) distrusting or disparaging the motives of others; bitterly or sneeringly distrustful

197

Dalliance

(n) a trifling away of time; dawdling

198

Dearth

(n) an inadequate supply; a lack of something

199

Debacle

(n) a sudden disastrous collapse, downfall or defeat; a total, often ludicrous, failure

200

Debilitated

(v) injured; unable to perform as before; showing impairment of energy or strength

201

Deceit

(n) concealment or distortion of the truth for the purpose of misleading; fraud; an act or device intended to trick or cheat

202

Decimate

(v) to destroy a great number or proportion of (a group)

203

Decisive

(adj) putting an end to controversy; crucial; indisputable

204

Decorous

(adj) characterized by dignified propriety in conduct, manners, appearance, character, etc; proper

205

Deficit

(n) the amount by which a sum of money falls short of the required amount; a lack or shortage

206

Deference

(n) respectful submission to the judgment, opinion or will of another; courteous respect

207

Defiant

(adj) boldly resistant or challenging

208

Deft

(adj) dextrous; nimble; skilful; clever

209

Deleterious

(adj) harmful, injurious to health

210

Deliberate

(adj) carefully weighed or considered; studied; intentional

211

Deliberation

(n) careful consideration before decision

212

Delineate

(v) to trace the outline of; to describe or outline with precision

213

Delude

(v) to mislead the mind or judgment of; to deceive

214

Deluge

(n) a great flood of water; anything that overwhelms like a flood

215

Demagogue

(n) a person who gains power by arousing the passions, emotions and prejudice of other people

216

Demeanour

(n) conduct, behavior, deportment

217

Demonstrate

(v) to show, to display, to make evident, to prove

218

Demonstrative

(adj) the quality of being open with one’s thoughts or emotions

219

Denial

(n) the state of refusing to accept something as true

220

Denotation

(n) a word that names or signifies something specific

221

Denounce

(v) to condemn openly; to make a formal accusation against; to proclaim as false

222

Deplorable

(adj) causing or being a subject for grief or regret

223

Derelict

(adj) left or deserted; abandoned; neglectful of duty

224

Derision

(n) ridicule; mockery

225

Detachment

(n) aloofness; freedom from prejudice or partiality

226

Deteriorate

(v) to disintegrate; to wear away

chap06.indd 10

5/23/2011 5:04:57 PM

Chapter 6    Vocabulary

chap06.indd 11

6. 11

227

Determined

(adj) set on a course of action; decided; resolute

228

Deterministic

(adj) the notion that all events, including human choices and decisions, have sufficient causes

229

Devastate

(v) to lay waste; to render desolate; to overwhelm; to destroy

230

Devious

(adj) not straight-forward; departing from the proper or accepted way; shifty or crooked

231

Dextrous

(adj) skilful or adroit in the use of hands or body; clever

232

Diagnose

(v) to ascertain the cause or nature of (a disorder, malfunction, problem, etc) from the symptoms

233

Dictator

(n) a ruler with complete power

234

Differentiate

(v) to make singular; to induce to stand out from the norm; to distinguish

235

Diffident

(adj) lacking confidence in one’s own ability, worth or fitness; timid; shy

236

Dilemma

(n) a situation requiring a choice between equally undesirable alternatives

237

Diminished

(adj) made smaller in size or quantity

238

Diplomatic

(adj) skilled in dealing with sensitive matters or people; tactful

239

Discernment

(n) acuteness or accuracy of judgement and understanding

240

Disclaim

(v) to deny association with; to disavow

241

Disclosure

(n) the act of revealing; something uncovered; a revelation

242

Discord

(n) strife; conflict; lack of harmony; a state in which parties do not agree

243

Discourse

(v) to communicate thoughts orally; to talk; to converse

244

Discredit

(v) to injure the reputation of; to defame; to destroy confidence in

245

Discreet

(adj) prudent in one’s conduct or speech, especially with regard to maintaining silence about something of a delicate nature; marked by wise self-restraint in speech and behavior

246

Discrepancy

(n) the state or quality of being different; an inconsistency

247

Disdain

(n) a feeling of haughty contempt; scorn

248

Disillusion

(v) to free from illusion, belief or idealism; to disenchant

249

Disingenuous

(adj) lacking in frankness or sincerity; insincere; not candid

250

Disintegrated

(adj) dissolved; fell apart; decayed

251

Dismay

(n) sudden loss of courage; perturbation; alarm

252

Dismiss

(v) to release; to let go; to free

253

Disparage

(v) to speak or treat slightingly; to belittle

254

Dispense

(v) to give out; to distribute; to administer

255

Disposition

(n) the predominant tendency of one’s spirits; one’s natural mood

256

Disregard

(v) to ignore; to pay no attention to

257

Dissect

(v) to take apart with the intention of examining the insides

258

Disseminate

(v) to scatter or spread widely; to broadcast

259

Dissolute

(adj) indifferent to moral constraints; given to immoral conduct

260

Dissonant

(adj) disagreeing or harsh in sound; out of harmony

5/23/2011 5:04:57 PM

6. 12

Part 2    SAT Preparation

261

Distort

(v) to twist out of shape; to make crooked or deformed

262

Distress

(n) great pain, anxiety or sorrow; acute suffering; trouble; a state of extreme misfortune

263

Diverge

(v) to move in different directions from a common point; to differ in opinion, character, form, etc; to deviate

264

Diverse

(adj) varied; not uniform

265

Doctrinaire

(adj) fanatical or inflexible about others’ acceptance of one’s ideas, without regard to practicality

266

Dogmatic

(adj) asserting opinions in an arrogant matter; opinionated

267

Domineering

(adj) inclined to rule arbitrarily or despotically

268

Dramatize

(v) to express oneself in an exaggerated way

269

Dubious

(adj) of questionable authenticity; causing doubts

270

Duped

(adj) easily deceived or fooled

271

Duration

(n) the length of time something continues or exists

272

Dynamic

(n) pertaining to or characterized by energy or effective action; vigorously active or forceful; energetic

273

Ebullient

(n) overflowing with enthusiasm; high-spirited

274

Eccentric

(adj) out of the ordinary; irregular; erratic; peculiar

275

Eclectic

(adj) diverse; not uniform; consisting of a variety of sources, systems or styles

276

Economical

(adj) sparing; not wasteful

277

Edify

(v) to instruct or benefit, especially morally or spiritually; to inform or enlighten

278

Egotism

(n) excessive and objectionable reference to oneself in conversation or writing; boastfulness; self-centeredness

279

Elaborate

(v) to work out carefully or minutely; to develop to perfection

280

Elicit

(v) to draw out; to bring forth; to evoke

281

Eliminate

(v) to remove; to get rid of; to leave out or omit from consideration

282

Elitism

(n) practice of or belief in rule by an elite

283

Eloquent

(adj) having the power of fluent, forceful and appropriate speech; well-spoken; movingly expressive

284

Elucidate

(v) to make lucid or clear; to explain

285

Elusive

(adj) cleverly evasive; hard to express or define

286

Emancipation

(n) the act of setting free an enslaved group or class of people

287

Embellish

(v) to beautify by adornment; to adorn; to enhance (a statement or narrative) with fictitious additions

288

Embody

(v) to give a concrete form to

289

Emigrate

(v) to leave one country with the intention of settling in another

290

Empathy

(n) identification with and understanding of another’s thoughts or feelings

291

Emphasize

(v) to stress, to highlight, to accentuate

chap06.indd 12

5/23/2011 5:04:57 PM

Chapter 6    Vocabulary

chap06.indd 13

6. 13

292

Emphatic

(adj) spoken or uttered with force or insistence

293

Empirical

(adj) derived from or guided by experience or experiment

294

Encompass

(v) to enclose; to include comprehensively; to encircle or surround

295

Encroach

(v) to advance beyond proper or usual limits

296

Endeavor

(n) a strenuous effort; an attempt

297

Endemic

(adj) natural to or characteristic of a specific people or place; native; indigenous

298

Endorsed

(adj) recommended; approved; supported

299

Endow

(v) to provide (with a permanent fund or source of income); to furnish with some talent, faculty or quality; to equip

300

Endure

(v) to bear without resistance; to tolerate; to continue to exist; to last

301

Enervate

(v) to deprive of force or strength; to weaken (someone)

302

Enigma

(n) a puzzling or inexplicable occurrence or situation; a riddle

303

Enjoin

(v) to order a course of action with authority or emphasis; to prohibit or forbid

304

Enthusiastic

(adj) ardent; showing great interest

305

Entrepreneur

(n) a person who organizes and manages an enterprise, usually a business

306

Enveloped

(adj) covered completely; surrounded

307

Epitomize

(v) to serve as a typical example of

308

Equanimity

(n) mental or emotional stability or composure; calmness

309

Equitable

(adj) fair; equal; just

310

Equivocal

(adj) open to two or more interpretations and often intended to mislead; deliberately ambiguous; of doubtful nature

311

Eradication

(n) the act of utter destruction; the act of plucking up by the roots

312

Erode

(v) to eat into or away; to destroy by slow disintegration

313

Erratic

(adj) out of the ordinary; having no certain or definite course; not fixed

314

Esoteric

(adj) understood only by the select few who have the knowledge; secret

315

Essential

(adj) absolutely necessary; crucial; very important

316

Estrangement

(n) a separation resulting from hostility

317

Euphoria

(n) elation; a feeling of great happiness or well-being

318

Evaluate

(v) to judge or determine the significance, worth or quality of; to assess

319

Evocative

(adj) serving to call to mind; calling forth; serving as a reminder

320

Evoke

(v) to call up or to produce (in response to a stimulation); to summon; to cause to appear

321

Exacerbated

(adj) made worse in degree; aggravated

322

Exalt

(v) to raise high, to elevate; to praise

323

Excavate

(v) to make a hole in; to hollow out; to unearth; to discover

324

Excessive

(adj) going beyond the usual or proper limit or degree

5/23/2011 5:04:57 PM

6. 14

Part 2    SAT Preparation

325

Excised

(adj) removed; expunged

326

Exclusive

(adj) intended only for use by some; not for all

327

Exemplar

(n) a model or pattern to be copied or imitated

328

Exemplify

(v) to show or illustrate by example; to serve as an example of

329

Exhaustive

(adj) treating all parts or aspects without omission; thorough

330

Exonerate

(v) to free from guilt or blame; to discharge from duty or obligation

331

Expanded

(adj) increased in size; spread out

332

Expansive

(adj) having a wide range or extent; comprehensive; effusive, unrestrained speech; talkative

333

Expedient

(adj) tending to promote some desired object; suitable for the purpose; convenient

334

Experiment

(n) a test, trial or tentative procedure for the purpose of discovering something unknown or testing a supposition

335

Expertise

(n) expert skill or knowledge; considerable skill in a particular area

336

Explicit

(adj) fully and clearly expressed or demonstrated

337

Exploit

(v) to take advantage of for one’s own gain

338

Expurgate

(v) to amend by removing words, passages, etc., deemed offensive or objectionable; to censor

339

Exquisite

(adj) of special beauty or charm; extraordinarily fine or admirable

340

Extant

(n) still in existence; not destroyed or lost

341

Extol

(v) to praise highly; to laud; to eulogize

342

Extravagant

(adj) wasteful; going beyond what is deserved or justifiable; exceeding the bounds of reason

343

Extroverted

(adj) marked by interest in and behavior directed to others; outgoing; gregarious

344

Exuberant

(adj) almost uninhibitedly enthusiastic; extremely joyful and vigorous

345

Exude

(v) to emit or discharge gradually

346

Fabricated

(adj) created by man; forged; faked

347

Facetious

(adj) not meant to be taken seriously or literally; amusing; humorous

348

Facile

(adj) moving, working or doing with ease; easily done; arrived at without due care or effort

349

Facsimile

(n) an exact copy or reproduction, as of a book, painting or manuscript

350

Faction

(n) a sub-group within a larger group; a clique

351

Fad

(n) a temporary fashion, notion, manner of conduct, etc., that is taken up with great enthusiasm for a brief period of time; a craze

352

Fallacy

(n) a deceptive, misleading or false notion; a misleading or unsound argument

353

Fastidious

(adj) excessively critical or demanding; hard to please; painstaking

354

Fathom

(v) to penetrate to the truth of; to understand; to comprehend

355

Feign

(v) to represent fictitiously; to put on an appearance of; to pretend

356

Felicity

(n) the state of being very happy; bliss

357

Fervor

(n) great earnestness of feeling; intensity of emotion

chap06.indd 14

5/23/2011 5:04:57 PM

Chapter 6    Vocabulary

chap06.indd 15

6. 15

358

Fidelity

(n) strict observance of promises or duties; loyalty; faithfulness

359

Filial

(adj) pertaining to a son or daughter

360

Flabbergasted

(adj) shocked; stunned; overcome with surprise

361

Flagrant

(adj) shockingly noticeable or evident; obvious; notorious; scandalous

362

Flattery

(n) excessive, insincere praise

363

Flourish

(v) to be in a vigorous state; to thrive; to prosper; to make dramatic, sweeping gestures

364

Flux

(n) a flowing or flow; continuous change, passage or movement

365

Forbidding

(adj) grim, unfriendly, hostile, sinister, dangerous, threatening

366

Foreshadow

(v) to show or indicate beforehand; to presage

367

Fortitude

(n) mental or emotional strength that allows one to endure pain or adversity with courage

368

Foster

(v) to promote the growth or development of; to encourage; to bring up, raise or rear (a child)

369

Fraudulent

(adj) characterized by, involving or proceeding from dishonesty or deception

370

Frivolous

(adj) characterized by lack of seriousness or sense; self-indulgently carefree

371

Frugality

(n) the quality of being not wasteful; prudent economy; thrift; the careful management of anything valuable

372

Futility

(n) ineffectiveness; uselessness

373

Gaiety

(n) the state of being merry, cheerful or exuberant; merry or joyful activity

374

Gait

(n) a manner of walking, stepping or running; a particular way of moving on foot

375

Garish

(adj) crudely or tastelessly colorful; excessively ornate

376

Garner

(v) to gather; to acquire; to collect or hoard

377

Generic

(adj) descriptive of an entire group or class; general

378

Genre

(n) a class or category of artistic work having a particular form, content, technique, etc

379

Ghoulish

(adj) strangely diabolical or cruel; monstrous

380

Glean

(v) to gather slowly, bit by bit

381

Glorify

(v) to honor with praise, admiration or worship; to cause to seem more excellent than is actually the case

382

Gracious

(adj) pleasantly kind, benevolent and courteous; characterized by good taste, comfort, ease or luxury

383

Gratification

(n) that which affords pleasure, satisfaction, enjoyment or delight

384

Gratitude

(n) the quality of being or feeling thankful; thankfulness

385

Gregarious

(adj) fond of the company of others; sociable; living in flocks or herds (as animals)

386

Gullibility

(n) the quality of being easily deceived or cheated

387

Habitual

(adj) commonly used, followed, observed; customary

388

Habituate

(v) to accustom (a person, the mind, etc) as to a particular situation; to familiarize

389

Hackneyed

(adj) commonplace or trite; stale, banal; overused

5/23/2011 5:04:57 PM

6. 16

Part 2    SAT Preparation

390

Hallmark

(n) any mark or special indication of genuineness, good quality, etc.; any distinguishing feature or characteristic

391

Harangue

(n) a scolding; a long or intense verbal attack

392

Harmony

(adj) agreement in feeling or opinion; accord; a pleasing combination of elements in a whole

393

Hasty

(adj) moving or acting quickly or hurriedly; unduly quick; rash; impatient

394

Haven

(n) a safe place; a refuge

395

Havoc

(n) great destruction or devastation; ruinous damage

396

Heterogeneous

(adj) consisting of dissimilar elements or parts

397

Hierarchy

(n) a ranked social structure; any system of persons or things ranked one above another

398

Histrionics

(n) insincere or exaggerated expression of an emotion; dramatics; theatrics

399

Holistic

(adj) emphasizing the importance of the whole and the interdependence of its parts; concerned with wholes rather than analysis

400

Homogeneous

(adj) composed of parts or elements that are all of the same kind

401

Hostility

(n) the state of antagonism or enmity; animosity; unfriendliness

402

Hubris

(n) excessive pride that inevitably causes downfall

403

Humane

(adj) characterized by tenderness, compassion and sympathy for people and animals, especially the suffering or distressed

404

Humanitarian

(adj) having concern for the welfare and happiness of people

405

Hypocrisy

(n) a pretence of having a virtuous character or moral principles that one does not really possess

406

Iconoclast

(n) a person who attacks cherished beliefs, traditional institutions, etc, as being based on error or superstition

407

Idealist

(n) one who cherishes or pursues high or noble principles, purposes, goals, etc

408

Idiosyncrasy

(n) a characteristic, habit or mannerism that is peculiar to an individual; quirk

409

Ignominy

(n) disgrace; dishonour; public contempt

410

Immerse

(v) to plunge into or place under a liquid; to involve deeply

411

Immoral

(adj) not conforming to the patterns of conduct usually accepted as consistent with principles of personal and social ethics; inconsistent with purity

412

Immunize

(v) to render harmless or ineffective; to neutralize

413

Immutable

(adj) unchangeable; permanent

414

Impasse

(n) a position or situation from which there is no escape; deadlock

415

Implement

(v) to fulfil, to perform; to carry out; to put into effect according to a definite plan or procedure

416

Implicate

(v) to show to be also involved, usually in an incriminating manner; to imply as a necessary circumstance

417

Implicit

(adj) not expressly stated; unquestioning or unreserved; absolute

418

Impugn

(v) to challenge as false (another’s statements, etc); to cast doubt upon

419

Impulsive

(adj) prone to sudden action without planning; swayed by involuntary urges

chap06.indd 16

5/23/2011 5:04:57 PM

Chapter 6    Vocabulary

chap06.indd 17

6. 17

420

Inadequate

(adj) incomplete; not sufficient; inept; unsuitable

421

Inane

(adj) lacking sense, significance or ideas; silly; empty

422

Inapt

(adj) not fitting; inappropriate

423

Incarnation

(n) a living being embodying deity or spirit; a person or thing which is believed to personify a given abstract quality or idea

424

Inception

(n) beginning; start; commencement

425

Incite

(v) to stir, to encourage, to urge on; to stimulate or prompt to action

426

Incompatible

(adj) unable to exist together in harmony; contrary or opposed in character

427

Incongruous

(adj) out of place; inappropriate

428

Inconsequential

(adj) of little or no importance; insignificant; trivial

429

Inconsistency

(n) incoherence or irreconcilability of parts in a story

430

Inconspicuous

(adj) not noticeable or prominent; hardly discernible

431

Incredulous

(adj) disinclined to believe; sceptical; doubtful

432

Incubate

(v) to sit upon (eggs) for the purpose of hatching; to develop or produce as if by hatching

433

Indeterminate

(adj) not precisely fixed in extent; indefinite; uncertain

434

Indictment

(n) any charge, accusation or cause for blame

435

Indifference

(n) lack of interest or concern; unimportance

436

Indigenous

(adj) originating in and characteristic to a particular area; native (to); innate; inherent

437

Indignation

(n) righteous anger on someone else’s behalf; displeasure at something considered unjust

438

Indiscretion

(n) lack of wise self-restraint in sensitive matters; imprudence

439

Individualism

(n) the principle of belief in independent thought or action

440

Indolence

(n) habitual laziness; sloth

441

Indomitable

(adj) that which is not able to be conquered; never-ending

442

Inducement

(n) something that motivates or persuades; an incentive

443

Indulgent

(adj) lenient; permissive

444

Ineffectual

(adj) without satisfactory or decisive effect; futile; powerless; impotent

445

Inevitable

(adj) unable to be avoided, evaded or escaped; certain; sure to occur

446

Inexplicable

(adj) incapable of being accounted for or explained

447

Infatuation

(n) foolish or all-absorbing passion; extravagant, short-lived passion

448

Infer

(v) to derive by reasoning; to conclude or judge from premises or evidence

449

Infinitesimal

(adj) exceedingly small; minute; immeasurably small

450

Infirmity

(n) a physical weakness or ailment; lack of strength

451

Inflexible

(adj) unbending; unyielding

452

Ingenuity

(n) the quality of being cleverly inventive or resourceful; inventiveness

453

Ingenuous

(adj) free from reserve or restraint; candid; sincere; innocent; naïve

5/23/2011 5:04:57 PM

6. 18

Part 2    SAT Preparation

454

Inglorious

(adj) shameful; disgraceful; not famous or honored

455

Inherent

(adj) existing in someone or something as a permanent and inseparable element, quality or attribute; intrinsic

456

Inimitable

(adj) incapable of being copied; matchless

457

Innate

(adj) existing in one since birth; inborn; native; inherent in the essential character of something

458

Innocuous

(adj) not harmful or injurious; harmless; inoffensive

459

Innovation

(n) a new idea or concept; something new introduced

460

Inquiry

(n) a seeking or request for truth, information or knowledge; an investigation

461

Inscrutable

(adj) incapable of being understood; impenetrable; mysterious

462

Insidious

(adj) intended to entrap or beguile; stealthily treacherous or deceitful

463

Insight

(n) an instance of understanding the true nature of something; the capacity to understand the true nature of a situation

464

Insipid

(adj) without distinctive or interesting qualities; vapid; bland

465

Insolence

(n) contemptuously rude or impertinent behavior or speech

466

Insouciance

(n) lack of care or concern; indifference

467

Instantaneous

(adj) occurring, done or completed in an instant; immediate

468

Instrumental

(adj) serving as a means of accomplishing something; useful; helpful

469

Insular

(adj) detached; standing alone; isolated; narrow-minded or provincial; of or pertaining to islands

470

Insulation

(n) material used to prevent heat loss; a barrier

471

Integrated

(adj) combining separate elements so as to provide a harmonious, interrelated whole

472

Integrity

(n) adherence to moral or ethical principles; honesty

473

Intensity

(n) a high degree of energy or emotional excitement; exceptionally great power, concentration or force

474

Interleave

(v) to insert something alternately and regularly between the pages or parts of something else

475

Intermittent

(adj) stopping or ceasing for a time; alternately ceasing and beginning again

476

Interpersonal

(adj) existing or occurring between people

477

Intonation

(n) the pattern or melody of pitch changes in connected speech

478

Intricacy

(n) the condition or quality of being complex or complicated

479

Intrigued

(adj) interested; curious; captivated

480

Intuition

(n) direct perception of truth independent of any reasoning process; immediate apprehension or understanding; a quick insight

481

Invariable

(adj) not changing or capable of being changed; static, constant

482

Inverse

(adj) reversed in position, order, direction or tendency; turned upside down

483

Investigate

(v) to examine, study or inquire into systematically; to search or examine into the particulars of

chap06.indd 18

5/23/2011 5:04:57 PM

Chapter 6    Vocabulary

chap06.indd 19

6. 19

484

Invocation

(n) the act of calling upon a deity, spirit, etc., for aid, protection, inspiration or the like; supplication; any petitioning for aid or help

485

Irate

(adj) angry; enraged

486

Irony

(n) the use of words to convey a meaning that is the opposite of its literal meaning

487

Irrepressible

(adj) incapable of being restrained; uncontrollable

488

Irreverent

(adj) disrespectful of traditionally highly-respected ideas, people or objects

489

Jaded

(adj) dulled or satiated by overindulgence; worn out or weary by overwork or overuse

490

Jocular

(adj) given to joking or jesting; facetious

491

Joviality

(n) the state or quality of being convivial or in good cheer; merriment

492

Justifiable

(adj) that which can be shown to be just, right or warranted

493

Juxtapose

(v) to place close together or side by side, especially for comparison or contrast

494

Lackadaisical

(adj) without interest, vigor or determination; listless; lethargic; lazy

495

Languid

(adj) lacking in vigor or vitality; slack or slow; listless

496

Laud

(v) to praise; to extol

497

Lavish

(adj) expended, bestowed or occurring in profusion; using or giving in great amounts; extravagant

498

Legitimacy

(n) authenticity; undisputed credibility; the quality of being lawful

499

Levity

(n) lightness of mind, character or behavior; lack of appropriate seriousness

500

Lexicon

(n) a wordbook or dictionary; the vocabulary of a particular language, field, social class, etc

501

Licentious

(adj) morally unrestrained; lewd; lawless

502

Linguistic

(adj) of or belonging to language

503

Literary

(adj) pertaining to books and literature

504

Lithe

(adj) supple; flexible; bending readily

505

Litigious

(adj) inclined to dispute or disagree; argumentative

506

Livelihood

(n) a means of supporting one’s existence, especially financially

507

Loyalist

(n) one who remains loyal to an established government, political party or sovereign in the face of revolt

508

Lucid

(adj) easily understood; completely comprehensible; shining or bright

509

Ludicrous

(adj) causing laughter because of absurdity; provoking or deserving derision; ridiculous

510

Luminous

(adj) radiating or reflecting light; shining; bright; well-lit

511

Luxury

(n) something conducive to sumptuous living; a delicacy, elegance or refinement of living

512

Magnanimous

(adj) generous in forgiving an insult or injury; high-minded; noble

513

Malevolence

(n) ill-will; malice; hatred; malicious behavior

514

Malfeasance

(n) misconduct or wrongdoing, especially by a public official

515

Malicious

(adj) full of, characterized by or showing a desire to inflict injury or harm on another

516

Mandatory

(adj) authoritatively ordered; obligatory; compulsory

5/23/2011 5:04:58 PM

6. 20

Part 2    SAT Preparation

517

Manifestation

(n) outward or perceptible indication

518

Manipulative

(adj) having the power to manage or influence (people) with skill

519

Marginal

(adj) situated on the border or edge; at the outer limit; almost insufficient

520

Marred

(adj) somewhat damaged or disfigured; rendered less than perfect

521

Matriarch

(n) female head of a family; mother or grandmother

522

Maudlin

(adj) tearfully or weakly emotional; foolishly sentimental

523

Mawkishness

(n) the quality of being sickeningly sentimental or weakly emotional

524

Mechanism

(n) an assembly of moving parts performing a complete functional motion

525

Meddle

(v) to involve oneself in a matter without right or invitation

526

Medium

(n) a middle condition or state; environment; one of the means or channels of general communication

527

Megalomania

(n) an obsession with doing extravagant or grand things

528

Melancholy

(n) a gloomy state of mind, especially when habitual or prolonged; depression

529

Melodramatic

(adj) exaggerated and emotional; overdramatic

530

Metaphor

(n) a figure of speech in which a term or phrase is applied to something to which it is not literally applicable in order to suggest a resemblance

531

Methodology

(n) a set or system of principles and rules for regulating a given subject, as in the arts or sciences

532

Meticulous

(adj) taking or showing extreme care about minute details; precise; thorough

533

Miffed

(adj) annoyed; upset; put into an irritable mood, especially by an offending incident

534

Migrate

(v) to go from one country, reason or place to another

535

Millennia

(n) multiples of one thousand years; a long time

536

Mimetic

(adj) characterized by, exhibiting or of the nature of imitation

537

Mimic

(v) to imitate or copy in action or speech, often playfully or derisively

538

Mischievous

(adj) seeking to make good-natured trouble; playful or naughty

539

Misconception

(n) a mistaken notion

540

Miscreant

(n) a vicious or depraved person; a villain

541

Moderate

(v) to reduce the excessiveness of; to preside over (e.g. a meeting)

542

Modicum

(n) a small amount

543

Momentous

(adj) of great importance

544

Monotonous

(adj) lacking in variety; tediously unvarying; dull; wearisome

545

Monstrosity

(n) the state or character of being frightful, hideous, shocking or revolting

546

Morbid

(adj) suggesting an unhealthy mental state or attitude; unwholesomely gloomy, sensitive, extreme, etc

547

Motif

(n) a recurring subject, theme, idea, etc., in a literary, artistic or musical work; a dominate feature or idea

548

Motley

(adj) exhibiting great diversity of elements; being of many colours

chap06.indd 20

5/23/2011 5:04:58 PM

6. 21

Chapter 6    Vocabulary

chap06.indd 21

549

Mundane

(adj) common; ordinary; pertaining to earthly details

550

Myopic

(adj) nearsightedness; unable or unwilling to act prudently; lack of long-range perspective in thinking or planning

551

Myriad

(n) a very great or indefinitely great numbers of persons or things

552

Mysterious

(adj) of obscure nature, meaning, origin, etc; puzzling; inexplicable

553

Mysticism

(n) the notion of an immediate spiritual intuition of universal truths or of a direct, intimate union of the soul with God through contemplation or ecstasy

554

Mythic

(adj) without foundation in fact; imaginary; fictitious

555

Mythology

(n) a body of stories, traditions or beliefs associated with a particular group or the history of an event, arising naturally or deliberately fostered

556

Naivete

(n) natural or artless simplicity

557

Narrative

(n) a story or account of events, whether true or fictitious

558

Nebulous

(adj) hazy, vague, indistinct

559

Nefarious

(adj) extremely wicked or villainous

560

Negate

(v) to deny the existence, evidence or truth of; to nullify or cause to be ineffective

561

Neglected

(adj) left untended or alone; paid little attention

562

Negligent

(adj) guilty of a lack of due care or concern

563

Neutral

(adj) not aligned with or supporting any side or position in a controversy; of no particular kind

564

Nocturnal

(adj) of or pertaining to the night; done, occurring or coming at night; active at night

565

Nonchalant

(adj) cooly unconcerned, indifferent or unexcited; casual

566

Nostalgia

(n) a wistful desire to return to a previous time in one’s life

567

Notoriety

(n) fame or public awareness because of misdeed, crime or evil actions

568

Noxious

(adj) harmful or injurious to health or physical well-being; morally harmful or corrupting

569

Nuance

(n) a subtle difference or distinction in expression, meaning, response, etc

570

Objectivity

(n) external reality; what is, as opposed to what is perceived

571

Obligated

(adj) pledged, committed or bound to meet an obligation

572

Obscure

(adj) not clear or plain; ambiguous, vague or uncertain; not readily noticeable

573

Obsolete

(adj) no longer in use; of a discarded or outmoded type; out of date

574

Obsequious

(adj) characterized by servile deference; fawning; obedient

575

Obstinate

(adj) characterized by inflexible persistence or an unyielding attitude; stubborn

576

Odoriferous

(adj) having or giving off a scent

577

Officious

(adj) objectionably aggressive in offering one’s unrequested and unwanted services, help or advice

578

Olfactory

(adj) of or pertaining to the sense of smell

579

Ominous

(adj) menacing; foreboding; threatening; indicating the possibility of future harm

580

Onerous

(adj) burdensome, oppressive or troublesome; causing hardship

5/23/2011 5:04:58 PM

6. 22

Part 2    SAT Preparation

581

Opacity

(n) vagueness of meaning; lack of clarity

582

Opportunism

(n) the act of taking advantage of a new situation for one’s own goals, regardless of principles

583

Oppressive

(adj) burdensome; unjustly harsh; distressing

584

Opprobrium

(n) disgrace incurred by shameful conduct; infamy

585

Optimism

(n) a disposition or tendency to expect the best possible outcome

586

Opulent

(adj) wealthy, rich or affluent; abundant or plentiful

587

Ornate

(adj) elaborately decorated, often excessively or showily so; flashy, showy

588

Outlandish

(adj) freakishly or grotesquely strange or odd; bizarre; strikingly unfamiliar

589

Overlook

(v) to fail to notice, perceive or consider; to disregard or ignore indulgently

590

Overt

(adj) open to view or knowledge; not secret

591

Palliative

(n) something designed to relieve or soothe the symptoms of a disease without effecting a cure

592

Paltry

(adj) ridiculously or insultingly small; utterly worthless

593

Paradox

(n) a statement that seems self-contradictory or absurd but that in reality expresses a possible truth

594

Paramount

(adj) chief in importance or impact; supreme

595

Paranoia

(n) baseless or excessive suspicion of the motives of others

596

Pardon

(n) kind indulgence, as in the forgiveness of an offence

597

Parody

(n) a humorous imitation of a serious piece of writing or literature

598

Partisanship

(n) an inclination to favor one group or view or opinion over alternatives

599

Pathetic

(adj) causing or evoking pity, sympathetic sadness, sorrow, etc; affecting or moving the feelings

600

Pathos

(n) the quality or power that arouses pity or compassion

601

Patriotism

(n) devoted love, support and defence of one’s country; national loyalty

602

Patronage

(n) financial support or business provided to an establishment by paying customers

603

Paucity

(n) smallness of quantity; scarcity; scantiness

604

Pedagogy

(n) the work of a teacher; the art or science of teaching

605

Pedestrian

(adj) lacking in vitality, imagination, distinction, etc; commonplace; dull

606

Penchant

(n) a strong inclination, taste or liking for something

607

Penurious

(adj) extremely stingy; extremely poor; lacking in resources

608

Perception

(n) the act of understanding by means of the senses or the mind; understanding

609

Perfidy

(n) deliberate breach of faith or trust; faithlessness; treachery; betrayal

610

Perfunctory

(adj) performed merely as a routine duty; hasty and superficial; lacking interest or enthusiasm

611

Peripheral

(adj) related to, located in or constituting the outer boundary; of minor importance or relevance

612

Permutation

(n) a complete change; a transformation; the act of altering a given set of objects in a group

613

Perpetuate

(v) to preserve from extinction or oblivion; to cause to continue indefinitely

614

Perplex

(v) to cause to be puzzled; to confuse mentally; to hamper with complications or uncertainty

615

Personable

(adj) having an agreeable or pleasing personality; affable; amiable; sociable

chap06.indd 22

5/23/2011 5:04:58 PM

6. 23

Chapter 6    Vocabulary

chap06.indd 23

616

Personages

(n) a person of distinction or importance; any person

617

Pertinent

(adj) relating directly and significantly to the matter at hand; relevant

618

Pervasive

(adj) tending to spread throughout

619

Pessimism

(n) the tendency to see or anticipate only negative outcomes, results, etc; the belief that all things naturally tend to evil

620

Petulant

(adj) showing sudden, impatient irritation over some trifling annoyance

621

Phenomenon

(n) a fact, occurrence or circumstance observed or observable

622

Philanthropy

(n) selfless concern for human welfare and advancement, usually manifested by donations of money, property or labor

623

Phlegmatic

(adj) not easily excited to action and display of emotion; sluggish; apathetic

624

Physiological

(adj) consistent with the normal functioning of an organism

625

Placid

(adj) pleasantly calm or peaceful; unruffled; tranquil

626

Plaintive

(adj) expressing sorrow or melancholy; mournful

627

Platitude

(n) a flat, dull or trite remark, especially one expressed as if it were original or significant

628

Plaudits

(n) enthusiastic expression of approval

629

Plummet

(v) to fall straight down; to plunge; to decline steeply or sharply

630

Polemical

(n) a controversial argument, especially one refuting or attacking a specific opinion

631

Portend

(v) to indicate in advance; to foreshadow, as an omen does; to signify

632

Pragmatic

(adj) of or pertaining to a practical point of view or practical considerations; concerned with facts or actual occurrences

633

Precipitate

(v) to hasten the occurrence of; to bring about suddenly; to fling or hurl down

634

Preclude

(v) to prevent the presence, existence or occurrence of; to make impossible

635

Precocious

(adj) unusually advanced or mature in development, especially in mental development

636

Precursor

(n) a person or thing that precedes or comes before another; a predecessor

637

Premature

(adj) occurring, coming or done too soon; ripe before the proper time

638

Presentiment

(n) a feeling or impression that something is about to happen, especially something evil; a premonition

639

Prestige

(n) reputation or influence arising from success or achievement

640

Presumption

(n) assumption of something to be true, based on reasonable grounds or probable evidence

641

Presumptuous

(adj) impertinently bold; forward; overconfident or venturesome

642

Pretense

(n) a false show of something; an act of pretending

643

Pretentious

(adj) characterized by an assumption of dignity or importance

644

Prevailing

(adj) predominant; most frequent or common

645

Prevalence

(n) the quality of being widespread or in general use

646

Primacy

(n) the state of being first in order, rank or importance

647

Profitable

(adj) yielding gain, especially financial; advantageous or lucrative

5/23/2011 5:04:58 PM

6. 24

Part 2    SAT Preparation

648

Profound

(adj) having deep insight or understanding

649

Profundity

(n) great depth of intellect or feeling; the quality of having deep understanding

650

Prolific

(adj) producing offspring abundantly; highly fruitful or productive

651

Prominence

(n) fame; visibility

652

Propensity

(adj) natural inclination or tendency; usual behavior

653

Prophetic

(adj) predictive; foretelling events as if by divine inspiration

654

Propitiate

(v) to make favorably inclined; to appease

655

Prosaic

(adj) commonplace or dull; unimaginative; lacking in spirit

656

Prosperous

(adj) having financial success or good fortune; successful; well-to-do

657

Prostrate

(adj) lying flat face down on the ground as in a token of humility, submission or adoration

658

Provident

(adj) having or showing foresight; providing for future needs or events

659

Provocative

(adj) tending or serving to incite, stimulate, irritate or vex

660

Provoke

(v) to anger, enrage, exasperate or vex; to incite or to stimulate to action; to bring about

661

Prudent

(adj) wise in practical affairs; exercising good judgment or common sense

662

Punctilitious

(adj) strict or exact in the observance of the formalities of conduct

663

Purported

(adj) reputed or claimed; alleged

664

Peruse

(v) to read through with thoroughness or care

665

Quantify

(v) to determine, indicate or express the amount of

666

Quarrel

(v) to disagree angrily; to squabble

667

Quell

(v) to suppress; to put an end to; to subdue; to vanquish

668

Querulous

(adj) full of complaints; complaining; peevish

669

Quirky

(adj) given to peculiarities of behavior; having idiosyncrasies

670

Quizzical

(adj) odd, queer or comical; questioning or puzzled

671

Rambunctious

(adj) difficult to control or handle; active and noisy

672

Ramification

(n) a related or derived subject or problem; a consequence; an implication

673

Rationalize

(v) to devise self-satisfying but incorrect reasons for one’s behavior

674

Raze

(v) to tear down; to demolish; to level to the ground

675

Rebellious

(adj) inclined to defy or resist some established authority; insubordinate

676

Rebuke

(v) to express sharp, stern disapproval of; to reprimand

677

Rebuttal

(n) a reply, as in a debate, intended to show fault in an opponent’s argument

678

Recant

(v) to withdraw a statement, opinion, etc, especially formally; to retract

679

Reciprocate

(v) to give or take mutually; to show, feel or give in return

680

Reclusive

(adj) seeking or preferring isolation

681

Recognize

(v) to identify something as previously seen or known

682

Recrimination

(n) a counter-charge; a defence to an accusation

chap06.indd 24

5/23/2011 5:04:58 PM

Chapter 6    Vocabulary

chap06.indd 25

6. 25

683

Recuperate

(v) to recover from sickness or exhaustion; to regain health or strength

684

Redolent

(adj) odorous or smelling (usually followed by “of ”); aromatic

685

Refute

(v) to prove an opinion or charge to be false or erroneous

686

Regenerate

(v) to revive or produce anew; to bring into existence again

687

Regressive

(adj) tending to return or revert

688

Reinforce

(v) to strengthen with some added piece, support or material; to make more forcible or effective

689

Reinstate

(v) to put back or establish again, as in a former position or state

690

Relentless

(adj) unyieldingly severe, strict or harsh; steady or persistent

691

Reluctance

(n) an unwillingness or disinclination

692

Remedy

(n) something that cures or relieves a disease or bodily disorder; a healing medicine, application or treatment

693

Remission

(n) the period during which the symptoms of a disease subside; release from a duty or obligation; forgiveness

694

Remnant

(n) a fragment or scrap; a trace or vestige

695

Remorseful

(adj) characterized by or full of deep and painful regret for wrongdoing

696

Remote

(adj) far apart; far distant in space or time; situated at some distance away

697

Renown

(n) widespread fame; the quality of being widely honored

698

Reparation

(n) the making of amends for wrong or injury done; something given in compensation for loss, suffering or damage

699

Repeal

(v) to revoke or withdraw formally or officially

700

Repercussion

(n) an effect or result, often indirect, of some event or action

701

Reprehensible

(adj) deserving of rebuke or censure; blameworthy

702

Reprimand

(v) a severe rebuke, especially in a formal or official way

703

Reproach

(v) to find fault with; to blame

704

Reprove

(v) to gently criticize or correct; to disapprove of strongly; to rebuke

705

Repudiate

(v) to reject as having no authority; to cast off or disown

706

Resentment

(n) the feeling of displeasure at some act, remark or occasion regarded as causing injury or insult

707

Resigned

(adj) submissive; passively accepting

708

Resilience

(n) the ability to recover quickly from illness, adversity or misfortune

709

Resistant

(adj) characterized by unwillingness or opposition

710

Resolute

(adj) firmly determined; steadfast or unwavering in commitment

711

Respite

(n) a temporary delay of anything distressing or trying; a short interval of rest or relief

712

Retain

(v) to keep possession of; to continue to use, have, hold, etc

713

Retrospect

(n) contemplation of the past; a survey of past time, events, etc

714

Revelation

(n) something disclosed, especially in a striking way; an uncovering of that which had been previously hidden

5/23/2011 5:04:58 PM

6. 26

Part 2    SAT Preparation

715

Reverence

(n) a feeling or attitude of deep respect or awe; veneration

716

Revolutionary

(adj) of or characterized by a sudden, complete and marked change; radically new or innovative

717

Rhetoric

(n) the study of the effective use of language; in speech or language, the undue use of exaggeration or display

718

Rhetorical

(adj) used for, belonging to or concerned with mere style and effect

719

Ridicule

(n) speech or action intended to cause contemptuous laughter at a person or thing; derision

720

Rigorous

(adj) rigidly severe or harsh, as people, rules or discipline; severely exact or accurate

721

Romanticize

(v) to make fanciful, impractical or unrealistic; to imbue with idealism or a sense of adventure

722

Rousing

(adj) exciting; stirring

723

Sagacious

(adj) having or showing keen practical sense, sound judgment and farsightedness

724

Salutary

(adj) favorable to or promoting good health; healthful

725

Sanction

(v) to authorize, approve or allow; to penalize, especially for violating a moral principle or international law

726

Sanctity

(n) holiness; saintliness; something considered sacred

727

Sarcasm

(n) harsh or bitter derision or irony; sneering or cutting remark

728

Sated

(adj) satisfied; filled to excess

729

Satirical

(adj) characterized by the use of irony, sarcasm, ridicule or the like to expose vice, folly, etc

730

Savvy

(adj) shrewdly informed; experienced and well-informed

731

Scarcity

(n) insufficiency or shortness of supply; dearth

732

Scrupulous

(adj) having a strict regard for what one considers right; principled; careful; precise

733

Scrutinize

(v) to examine closely with careful or critical attention

734

Secluded

(adj) sheltered or screened from activity or view; isolated; alone

735

Sedulous

(adj) diligent in application or attention; persevering; assiduous

736

Segregated

(adj) restricted to one group, especially on the basis of racial or ethnic membership; characterized by separation along group lines

737

Sensibility

(n) capacity for feeling; keen consciousness or appreciation

738

Sentimental

(adj) weakly emotional; susceptible to tender emotions and feelings

739

Serene

(adj) calm; peaceful; tranquil

740

Servitude

(n) slavery or bondage of any kind; compulsory service or labor

741

Short-sighted

(adj) caring only about the near-future; lacking the ability to perceive future actions; unable to see far in advance

742

Shrewd

(adj) keen awareness or sharp intelligence in practice matters; clever; calculating

743

Scepticism

(n) doubt or unbelief, usually with regard to religion

744

Smug

(adj) contentedly confident in one’s ability, superiority or correctness

745

Solace

(n) comfort in sorrow, misfortune or trouble; something that gives comfort or relief

746

Solicitous

(adj) anxious or concerned; anxiously desirous

chap06.indd 26

5/23/2011 5:04:58 PM

Chapter 6    Vocabulary

chap06.indd 27

6. 27

747

Sombre

(adj) grave; silent; gloomily dark

748

Sophisticated

(adj) altered by education and experience so as to be world-wise; not naïve

749

Sophistry

(n) superficially plausible but ultimately faulty reasoning; a false argument

750

Spawn

(v) to produce in large numbers; to give birth to; to give rise to

751

Speculative

(adj) contemplative; given to thought and conjecture; theoretical; involving chance or risk

752

Splendour

(n) brilliant or gorgeous appearance; magnificence; a display of imposing grandeur

753

Spontaneous

(adj) coming or resulting from a natural impulse or tendency; unplanned; natural; not premeditated

754

Sporadic

(adj) appearing or happening at irregular intervals in time; occasional

755

Squabble

(v) to engage in a disagreeable argument over a trivial matter

756

Squander

(v) to spend or use (money, time, etc) extravagantly or wastefully

757

Stagnate

(v) to cease to flow or run (water, air, etc); to become foul or stale from standing still; to stop developing, growing or advancing

758

Stalemate

(n) any position in which no action can be taken or progress made; deadlock

759

Stalwart

(adj) strongly built; sturdy and robust; strong and brave

760

Stationary

(adj) standing still; not moving; having a fixed position

761

Status quo

(n) the existing state or condition

762

Statute

(n) a law enacted by a legislature; a decree or edict

763

Steadfast

(adj) fixed in direction; firm in purpose or resolution; unwavering

764

Stifle

(v) to quell, crush or end by force; to suppress

765

Stimulate

(v) to rouse to action or effort; to spur on

766

Stodgy

(adj) heavy, dull or uninteresting; te­diously commonplace; unduly formal

767

Stricture

(n) an adverse remark or criticism; a restraint or limit

768

Strident

(adj) making or having a harsh sound; having an irritating quality or character

769

Stupefy

(v) to put into a state of little or no sensibility; to stun; to astound or astonish

770

Subdued

(adj) quiet; inhibited; lowered in intensity or strength

771

Subjective

(adj) existing in the mind; belonging to the thinking subject, rather than the object of thought; what is perceived, rather than what is

772

Subordinate

(adj) placed in or belonging to a lower rank or order; of less importance

773

Substantial

(adj) of ample or considerable amount, quantity, size, etc; of solid character or quality

774

Substantiate

(v) to establish by proof; to give evidence to

775

Subversive

(adj) tending to advocate the overthrow or destruction of an established government

776

Succinct

(adj) expressed in few words; concise

777

Sullen

(adj) showing irritation or ill-humour by a gloomy silence or reserve

778

Superficial

(adj) being at, on or near the surface; external or outward

779

Superfluous

(adj) being more than is sufficient or required; unnecessary; needless

5/23/2011 5:04:58 PM

6. 28

Part 2    SAT Preparation

780

Supersede

(v) to replace in power, authority, effectiveness, etc; to take the place of

781

Supplant

(v) to take the place of as through force, scheming, strategy, etc

782

Surfeit

(n) overabundance; excess

783

Surplus

(n) an amount or quantity greater than needed

784

Surreptitious

(adj) obtained or done by stealth; secret or unauthorized

785

Tactful

(adj) characterized by a sense of what is right or proper

786

Tedium

(n) the quality or state of being wearisome; the feeling of being bored

787

Temperance

(n) moderation or self-restraint in action, statement, etc; self-control

788

Tenacity

(n) the quality of holding fast, of being persistent, of clinging to an object or idea

789

Tendency

(n) a natural disposition to act in a certain way

790

Tendril

(n) a twisting, thread-like structure; something that is long, slender and curling

791

Tentative

(adj) unsure; uncertain; not definite or positive; hesitant

792

Terminate

(v) to end; to cause to end; to occur at the conclusion of

793

Testimonial

(n) a letter or written statement of recommendation; something given or done as an expression of esteem, admiration or gratitude

794

Theoretical

(v) existing only in theory; speculative; hypothetical

795

Thrive

(v) to prosper; to be fortunate or successful; to flourish

796

Thwart

(v) to oppose successfully; to prevent from accomplishing a purpose

797

Timidity

(n) a lack of self-assurance, courage or bravery

798

Topical

(adj) dealing with matters or current or local interest; contemporary; relevant

799

Tractable

(adj) easily managed or controlled; docile; yielding

800

Transcend

(v) to rise above or go beyond (boundaries); to outdo or exceed in excellence, elevation, extent, degree, etc

801

Transparent

(adj) admitting the passage of light; easily seen through, recognized or detected

802

Tremulous

(adj) timid, timorous, fearful

803

Tribute

(n) a gift, compliment or the like given as acknowledgment of gratitude or esteem; a rent or tax paid by a subject to a sovereign

804

Trite

(adj) unoriginal; hackneyed; lacking in freshness because of excessive repetition

805

Trivialize

(v) to cause to appear unimportant

806

Tyranny

(n) unrestrained exercise of power; abuse of authority

807

Ubiquity

(n) the state of being everywhere, especially at the same time; omnipresence

808

Ultimatum

(n) a final, uncompromising demand or set of terms issued by a party to a dispute; a final proposal

809

Unconscious

(adj) not awake; without awareness or sensation

810

Unconscionable

(adj) not in accordance with what is just or reasonable; unscrupulous

811

Undermine

(v) to injure or destroy by indirect or secret means

chap06.indd 28

5/23/2011 5:04:58 PM

Chapter 6    Vocabulary

chap06.indd 29

6. 29

812

Unequivocal

(adj) unambiguous; clear; having only one possible meaning

813

Unnerve

(v) to deprive of courage, strength, determination or confidence; to upset

814

Unorthodox

(adj) breaking with convention or tradition

815

Unwarranted

(adj) having no justification; groundless

816

Urbane

(adj) having the polish and suavity of manner regarded as characteristic of sophisticated social life in major cities; elegance

817

Vacillate

(v) to waver in mind or opinion; to be indecisive or irresolute

818

Vacuous

(adj) without contents; empty; lacking in ideas or intelligence

819

Validity

(n) legal soundness or force

820

Valor

(n) boldness or determination in facing great danger; heroic courage

821

Vehement

(adj) ardent; impassioned; strongly emotional; marked by great energy

822

Venality

(adj) openness to bribery or corruption

823

Venerate

(v) to regard or treat with reverence, respect or heartfelt deference

824

Veracity

(n) conformity to truth or fact; accuracy, truthfulness

825

Verify

(v) to prove the truth of, as by evidence or testimony; to confirm

826

Vernacular

(n) the native speech or language of a place; the language or vocabulary peculiar to a class or profession

827

Vicarious

(adj) performed, exercised or suffered in place of another

828

Vicissitude

(n) a change or variation occurring in the course of something

829

Vindication

(n) the defence that serves to justify a claim or action

830

Vindictive

(adj) disposed or inclined to revenge; vengeful

831

Volatile

(adj) tending or threatening to break out into open violence; explosive

832

Voluptuous

(adj) characterized by indulgence in luxury, pleasure and sensuous enjoyment; pleasing or delightful to the senses

833

Vulgar

(adj) indecent; obscene; lewd; characterized by a lack of good taste

834

Vulnerable

(adj) capable of being wounded or hurt

835

Whimsical

(adj) prone to fanciful notions; erratic; unpredictable

836

Wistful

(adj) characterized by melancholy or longing

837

Wrath

(n) strong, stern or fierce anger; vengeance or punishment as a consequence of anger

838

Wreak

(v) to inflict or execute (punishment, etc); to bring about; to cause

839

Wretched

(adj) very unfortunate in condition or circumstances; pathetic; miserable

840

Zeal

(n) passion or fervour for a person, cause or desire

5/23/2011 5:04:58 PM

6. 30

Part 2    SAT Preparation

6.2. Vocabulary: Grouped Hard Word List Find below a list of 300+ hard SAT words, listed by category and roughly grouped into synonyms/antonyms. Some of these you will also find on the High-Frequency Vocabulary List.

The connection and associations you’ll be able to develop through revising this list will help enormously on Hard Sentence Completions and Medium and Hard Reading Comprehension. If you intend to score in the 700s in Critical Reading, you should learn this list in its entirety.

Hard to Understand/Easy to Understand Abstruse

(adj) hard to understand; esoteric

Arcane

(adj) known or understood by very few; mysterious; secret

Archaic

(adj) marked in the manner of an earlier period; antiquated; outdated

Esoteric

(adj) understood only by the select few who have the knowledge; secret

Byzantine

(adj) intricate; complex; characterized by elaborate scheming and intrigue

Discernment

(n) acuteness or accuracy of judgment and understanding

Lucid

(adj) easily understood; completely comprehensible; shining or bright

Opacity

(n) vagueness of meaning; lack of clarity

Elucidate

(v) to make lucid or clear; to explain

Succinct

(adj) expressed in few words; concise

Glean

(v) to gather slowly, bit by bit Intelligence/Unintelligence

Acumen

(n) keen insight; shrewdness

Savvy

(adj) shrewdly informed; experienced and well-informed

Cogent

(adj) convincing by virtue of clear presentation; relevant; pertinent

Profundity

(n) great depth of intellect or feeling; the quality of having deep understanding

Cerebral

(adj) characterized by the use of intellect rather than intuition or instinct

Sagacious

(adj) having or showing keen practical sense, sound judgment and farsightedness

Myopic

(adj) nearsightedness; unable or unwilling to act prudently; lack of long-range perspective in thinking or planning

Acuity

(n) keenness, acuteness

Perspicacious

(adj) keen understanding or penetration

Visceral

(adj) pertaining to gut-feeling; intuition. ANT: cerebral Skilled/Unskilled

Adroit

(adj) expert and nimble in the use of the hands or body; cleverly skilful resourceful or ingenious

Dextrous

(adj) skilful or adroit in the use of hands or body; clever

chap06.indd 30

5/23/2011 5:04:58 PM

6. 31

Chapter 6    Vocabulary Efficiency/Inefficiency Alacrity

(n) cheerful readiness, promptness or willingness; briskness

Expedient

(adj) tending to promote some desired object; suitable for the purpose; convenient

Efficacious

(adj) functional, efficient, producing desired effect

Utilitarian

(adj) the quality of being useful; having a purpose Moral Goodness/Badness

Altruism

(n) the principle of unselfish concern for the welfare of others

Licentious

(adj) morally unrestrained; lewd; lawless

Magnanimous

(adj) generous in forgiving an insult or injury; high-minded; noble

Notoriety

(n) fame or public awareness because of misdeed, crime or evil actions

Bane

(n) a person or thing that ruin or spoils; death, destruction or ruin

Decorous

(adj) characterized by dignified propriety in conduct, manners, appearance, character, etc; proper

Dissolute

(adj) indifferent to moral constraints; given to immoral or improper conduct

Venality

(adj) openness to bribery or corruption

Nefarious

(adj) extremely wicked or villainous

Equitable

(adj) fair; equal; just

Unconscionable

(adj) not in accordance with what is just or reasonable; unscrupulous

Culpable

(adj) deserving of blame; blameworthy

Avarice

(adj) greed

Charlatan

(n) a con-man; a trickster

Chicanery

(n) fraudulent behavior

Cupidity

(n) greed

Incorrigible

(adj) impossibility of being corrected (behaviorally)

Iniquity

(n) sin, venality Uniform/Diverse

chap06.indd 31

Amalgam

(n) a composition or mixture of different elements

Eclectic

(adj) diverse; not uniform; consisting of a variety of sources, systems or styles

Heterogeneous

(adj) consisting of dissimilar elements or parts

Motley

(adj) exhibiting great diversity of elements; being of many colors

Insular

(adj) detached; standing alone; isolated; narrow-minded or provincial

Homogeneous

(adj) composed of parts or elements that are all of the same kind

Myriad

(n) a very great or indefinitely great numbers of persons or things

Variegated

(adj) diverse, varied

5/23/2011 5:04:58 PM

6. 32

Part 2    SAT Preparation Careful/Sloppy

Punctilious

(adj) strict or exact in the observance of the formalities of conduct

Meticulous

(adj) taking or showing extreme care about minute details; precise; thorough

Scrupulous

(adj) having a strict regard for what one considers right; principled; careful; precise

Sedulous

(adj) diligent in application or attention; persevering; assiduous

Austere

(adj) severe in manner or appearance; uncompromising; strict; forbidding; grave; solemn

Indolence

(n) habitual laziness; sloth;

Fastidious

(adj) excessively critical or demanding; hard to please; painstaking

Pedantic

(adj) painstakingly exact; literal-minded

Slovenly

(adj) sloppy in appearance or demeanour

Stringent

(adj) strict, meticulous, exacting

Ascetic

(adj) self-denying. SYN: austere Clear/Unclear

Ambiguous

(adj) open to or having several possible meanings or interpretations; of doubtful or uncertain nature; lacking clearness or definition, obscure, indistinct

Ambivalence

(n) uncertainty or fluctuation, especially when caused by inability to make a choice

Equivocal

(adj) open to two or more interpretations and often intended to mislead; deliberately ambiguous; of doubtful nature

Unequivocal

(adj) unambiguous; clear; having only one possible meaning

Convoluted

(adj) twisted; complicated; needlessly complex

Circumlocution

(n) a roundabout or indirect way of speaking; the use of more words than necessary to express an idea

Nebulous

(adj) hazy, vague, indistinct

Enigma

(n) a puzzling or inexplicable occurrence or situation; a riddle To Make Better/To Make Worse

Ameliorate

(v) to make better, more bearable or more satisfactory; to cause to improve in condition

Palliative

(n) something designed to relieve or soothe the symptoms of a disease without effecting a cure

Deleterious

(adj) harmful, injurious to health

Enervate

(v) to deprive of force or strength; to weaken (someone)

Propitiate

(v) to make favourably inclined; to appease

Marred

(adj) somewhat damaged or disfigured; rendered less than perfect

Curative

(n) a substance with healing properties

Pernicious

(adj) ruinous, injurious to health

Salutary

(n) having a beneficial or healthful effect

chap06.indd 32

5/23/2011 5:04:59 PM

6. 33

Chapter 6    Vocabulary Deceptive/Honest Artifice

(n) a clever trick or stratagem; a cunning, crafty device or trick; ingenuity; inventiveness

Insidious

(adj) intended to entrap or beguile; stealthily treacherous or deceitful

Disingenuous

(adj) lacking in frankness or sincerity; insincere; not candid

Ingenuous

(adj) free from reserve or restraint; candid; sincere; innocent; naïve

Perfidy

(n) deliberate breach of faith or trust; faithlessness; treachery; betrayal

Naivete

(n) natural or artless simplicity

Surreptitious

(adj) obtained or done by stealth; secret or unauthorized

Duplicitous

(adj) sneaky, deceptive Calmness

Aplomb

(n) imperturbable self-possession, poise or assurance; self-confident assurance

Nonchalant

(adj) coolly unconcerned, indifferent, unexcited; casual

Equanimity

(n) mental or emotional stability or composure; calmness

Insouciance

(n) lack of care or concern; indifference Appropriate/Inappropriate

Apropos

(adv) fitting; at the right time; to the purpose; opportunely

Apt

(adj) suited to a purpose or occasion; inclined or disposed to; unusually intelligent or quick to learn

Inapt

(adj) not fitting; inappropriate Strong Negative Feeling

Animosity

(n) an active feeling of strong dislike, ill-will or hatred; bitter hostility

Irascible

(adj) angry, bitter

Vitriol

(n) bitter criticism Weakness/Strength

chap06.indd 33

Craven

(adj) cowardly; contemptibly timid

Tremulous

(adj) timid, timorous, fearful

Diffident

(adj) lacking confidence in one’s own ability, worth or fitness; timid; shy

Timidity

(n) a lack of self-assurance, courage or bravery

Obsequious

(adj) characterized by servile deference; fawning; obedient

Infirmity

(n) a physical weakness or ailment; lack of strength

5/23/2011 5:04:59 PM

6. 34

Part 2    SAT Preparation

Maudlin

(adj) tearfully or weakly emotional; foolishly sentimental

Mawkishness

(n) the quality of being sickeningly sentimental or weakly emotional

Indomitable

(adj) that which is not able to be conquered; never-ending

Stalwart

(adj) strongly built; sturdy and robust; strong and brave

Tractable

(adj) easily managed or controlled; docile; yielding

Emphatic

(adj) spoken or uttered with force or insistence

Vacillate

(v) to waver in mind or opinion; to be indecisive or irresolute

Vehement

(adj) ardent; impassioned; strongly emotional; marked by great energy

Doctrinaire

(adj) fanatical or inflexible about others’ acceptance of one’s ideas, without regard to practicality

Enjoin

(v) to order a course of action with authority or emphasis; to prohibit or forbid

Cloying

(adj) overly sweet; weak; insipid

Foible

(n) minor weakness or failing of character

Fulsome

(ajd) offensively flattering or insincere

Obdurate

(adj) strongly stubborn

Pugnacious

(adj) having a quarrelsome nature; inclined to fighting. SYN: truculent

Taciturn

(adj) quiet, soft-spoken, shy. SYN: reticent

Audacity

(n) boldness, fearless daring

Autonomy

(n) independence; self-government

Cavalier

(adj) carefree and jaunty. SYN: nonchalant Strong Positive Feeling

Felicity

(n) the state of being very happy; bliss

Affable

(adj) pleasantly easy to approach and to talk to; friendly; cordial; warmly polite

Ebullient

(n) overflowing with enthusiasm; high-spirited

Exuberant

(adj) almost uninhibitedly enthusiastic; extremely joyful and vigorous

Rambunctious

(adj) difficult to control or handle; active and noisy Difficult

Arduous

(adj) requiring great exertion; laborious; difficult; strenuous; hard to endure

Onerous

(adj) burdensome, oppressive or troublesome; causing hardship Boring/Exciting

Banal

(adj) lacking originality; commonplace; predictable

Monotonous

(adj) lacking in variety; tediously unvarying; dull; wearisome

chap06.indd 34

5/23/2011 5:04:59 PM

6. 35

Chapter 6    Vocabulary

Rousing

(adj) exciting; stirring

Mundane

(adj) common; ordinary; pertaining to earthly details

Inane

(adj) lacking sense, significance or ideas; silly; empty

Hackneyed

(adj) commonplace or trite; stale, banal; overused

Insipid

(adj) without distinctive or interesting qualities; vapid; bland

Lackadaisical

(adj) without interest, vigour or determination; listless; lethargic; lazy

Prosaic

(adj) commonplace or dull; unimaginative; lacking in spirit

Stodgy

(adj) heavy, dull or uninteresting; te­diously commonplace; unduly formal or traditional

Tedium

(n) the quality or state of being wearisome; the feeling of being bored

Phlegmatic

(adj) not easily excited to action and display of emotion; sluggish; apathetic

Trite

(adj) unoriginal; hackneyed; lacking in freshness because of excessive repetition

Platitude

(n) a flat, dull or trite remark, especially one expressed as if it were original or significant

Lugubrious

(adj) dull, glum, gloomy

Moribund

(adj) not progressing; stagnant

Pedestrian

(adj) commonplace, boring, ordinary

Quiescence

(n) state of inactivity, dormancy

Sedentary

(adj ) unmoving, slugging Approval/Disapproval

chap06.indd 35

Censure

(n) a strong expression of disapproval; an official reprimand

Compunction

(n) a feeling of uneasiness or anxiety caused by regret for doing wrong or causing pain; remorse

Derision

(n) ridicule; mockery

Disdain

(n) a feeling of haughty contempt; scorn

Extol

(v) to praise highly; to laud; to eulogize

Exalt

(v) to raise high, to elevate; to praise

Plaudits

(n) enthusiastic expression of approval

Impugn

(v) to challenge as false (another’s statements, motives, etc); to cast doubt upon

Laud

(v) to praise; to extol

Opprobrium

(n) disgrace incurred by shameful conduct; infamy

Repudiate

(v) to reject as having no authority; to cast off or disown

Reprove

(v) to gently criticize or correct; to disapprove of strongly; to rebuke

Reverence

(n) a feeling or attitude of deep respect or awe; veneration

Ignominy

(n) disgrace; dishonour; public contempt

Cynical

(adj) distrusting or disparaging the motives of others; bitterly or sneeringly distrustful

5/23/2011 5:04:59 PM

6. 36

Part 2    SAT Preparation

Recrimination

(n) a counter-charge; a defense to an accusation

Harangue

(n) a scolding; a long or intense verbal attack

Stricture

(n) an adverse remark or criticism; a restraint or limit

Vindictive

(adj) disposed or inclined to revenge; vengeful

Penchant

(n) a strong inclination, taste or liking for something

Vindication

(n) the defense that serves to justify a claim or action

Aspersion

(n) an insult

Denounce

(v) to call out as false; to strongly disagree

Diatribe

(n) angry speech; SYN: polemical, harangue

Effusive

(adj) unrestrained in emotional expression; gushing

Tout

(v) to advertise loudly; to publicize Agree/Disagree

Contentious

(adj) tending to argument or strife; quarrelsome

Covenant

(n) an agreement between two or more parties to do or not to do something specified; a promise

Dissonant

(adj) disagreeing or harsh in sound; out of harmony

Corroborate

(v) to agree with another’s account of something; to give evidence to

Polemical

(n) a controversial argument, especially one refuting or attacking a specific opinion

Dogmatic

(adj) asserting opinions in an arrogant matter; opinionated

Acquiesce

(n) to agree, to give in

Conciliate

(v) to overcome the distrust of; to appease Important/Unimportant

Crux

(n) a vital, decisive or pivotal point; the basic or central feature or something

Dalliance

(n) a trifling away of time; dawdling

Frivolous

(adj) characterized by lack of seriousness or sense; self-indulgently carefree

Innocuous

(adj) not harmful or injurious; harmless; inoffensive

Infinitesimal

(adj) exceedingly small; minute; immeasurably small

Modicum

(n) a small amount

Vacuous

(adj) without contents; empty; lacking in ideas or intelligence

Perfunctory

(adj) performed merely as a routine duty; hasty and superficial; lacking interest or enthusiasm

Subordinate

(adj) placed in or belonging to a lower rank or order; of less importance

Ubiquity

(n) the state of being everywhere, especially at the same time; omnipresence

Picayune

(adj) trifling or unimportant detail

Potentate

(n) VIP, powerful individual

chap06.indd 36

5/23/2011 5:04:59 PM

6. 37

Chapter 6    Vocabulary Positive Event/Negative Event Cataclysm

(n) a violent upheaval that causes great destruction or brings about a fundamental change

Catastrophic

(adj) having the qualities of a widespread disaster or emergency

Calamity

(n) a great misfortune or disaster; grievous affliction; adversity; misery

Auspicious

(adj) promising success; opportune; favorable; lucky Plenty/Not Enough

Dearth

(n) an inadequate supply; a lack of something

Surfeit

(n) overabundance; excess

Superfluous

(adj) being more than is sufficient or required; unnecessary; needless

Paucity

(n) smallness of quantity; scarcity; scantiness

Opulent

(adj) wealthy, rich or affluent; abundant or plentiful

Expurgate

(v) to amend by removing words, passages, etc., deemed offensive or objectionable; to censor

Corpulence

(n) fatness; largeness of body

Indigent

(n) poverty; lack of wealth

Penurious

(n) quality of being stingy, cheap

Prodigious

(adj) extraordinary in size, amount or degree

Prolific

(adj) producing abundant work or results; fertile

Augment

(v) to make greater in size or quantity Past/Present

Anachronism

(n) an historical or chronological inaccuracy; an error in chronology in which a person, object, event, etc., is assigned a date or period other than the correct one

Antecedent

(adj) preceding; prior; coming earlier

Chronological

(adj) arranged in the order of time of occurrence

Progenitor

(n) creator; person who came before

Filial

(adj) having to do with a son or daughter Positive Social Characteristic/Negative Social Characteristic

chap06.indd 37

Facetious

(adj) not meant to be taken seriously or literally; amusing; humorous

Eloquent

(adj) having the power of fluent, forceful and appropriate speech; well-spoken; movingly expressive

Gregarious

(adj) fond of the company of others; sociable; living in flocks or herds (as animals)

Irreverent

(adj) disrespectful of traditionally highly-respected ideas, people or objects

Jocular

(adj) given to joking or jesting; facetious

5/23/2011 5:04:59 PM

6. 38

Part 2    SAT Preparation

Levity

(n) lightness of mind, character or behavior; lack of appropriate seriousness

Satirical

(adj) characterized by the use of irony, sarcasm, ridicule or the like to expose vice, folly, etc

Capricious

(adj) erratic; subject to or led by whim

Coy

(adj) artfully or affectedly or flirtatiously shy or modest; slyly hesitant

Histrionics

(n) insincere or exaggerated expression of an emotion; dramatics; theatrics

Hubris

(n) excessive pride that inevitably causes downfall

Officious

(adj) objectionably aggressive in offering one’s unrequested and unwanted services, help or advice

Sullen

(adj) showing irritation or ill-humour by a gloomy silence or reserve

Petulant

(adj) showing sudden, impatient irritation over some trifling annoyance

Pretentious

(adj) characterized by an assumption of dignity or importance

Amicable

(adj) very friendly

Droll

(adj) amusing, witty

Glib

(adj) witty, light-hearted

Flippant

(adj) glib, quit-witted, unserious

Loquacious

(adj) chatty, outgoing; ANT: terse, succinct

Sardonic

(adj) quietly amusing; wry

Supercilious

(adj) arrogant, full of yourself, haughty

Querulous

(adj) whiny, complaining

Cosmopolitan

(adj) wordly, experienced Characteristics and Traits

Idiosyncrasy

(n) quirk, weird habit

Iconoclastic

(adj) quality of one who attacks cherished beliefs

Itinerant

(adj) prone to travelling; nomadic

Odoriferous

(adj) smelly. SYN: redolent

Ostentatious

(adj) showy. ANT: stark Literary Devices/Logic

Metaphor

(n) comparison of two unlike things to suggest resemblance

Personification

(n) a figure of speech in which an inanimate object is endowed with human characteristics

Anecdote

(n) short account of an incident

Axiom

(n) a self-evident or universally recognized truth

Corollary

(n) a practice consequence that follows naturally

Conjecture

(n) speculation; a hypothesis

chap06.indd 38

5/23/2011 5:04:59 PM

6. 39

Chapter 6    Vocabulary

Empirical

(adj) derived from experiment and observation rather than theory

Imagery

(n) the use of language to form vivid mental pictures

Juxtaposition

(n) the act of positioning close together for purposes of comparison

Pathos

(n) a quality that arouses emotions

Parody

(n) an artistic work that imitates the style of another author or work for comic effect

Rhetoric

(n) the use of language to persuade

Vernacular

(n) slang; the native language of a place

Lexicon

(n) a vocabulary; a list of words

Tenet

(n) an opinion or principle held as true without proof

Irony

(n) the use of words to convey a meaning that is the opposite of its literal meaning

Allusion

(n) an instance of indirect reference

6.3. Vocabulary: Long Word Part List Learning Word Parts is the best thing you can do for yourself to prevent ever being in a situation on the SAT where you simply do not understand any of the vocabulary in a question. If you have a good familiarity with prefixes and word roots, then you will always at least be able to identify +/- Word Polarity (is this word positive? is this word negative?) and probably more. Understand that etymology—the study of words and their component parts—is a rough science and that for our pur-

chap06.indd 39

poses, it is rougher still. Words change from their Latin and Greek origins, morphing over time so that the root might not even resemble the way it is used in a contemporary vernacular (SAT vocab!). So be prepared to use these roots in a cautious way. How helpful can it be to know some Word Parts? Say you’re stuck on “grandiloquence”. Think of the word parts: grand + loqu + ence. You know that “grand” means big. Big…what? What does “loqu” mean? If you can remember that it means “speech” or “speaking”, then you’ve just deduced the definition of a word you didn’t know. And that’s the beauty of Word Parts.

aesth-, esth

feeling, sensation, perception

aesthete, anaesthesia, synesthesia

-crat, -cracy, -cratic

govern, rule; strength, power

autocrat, democracy, bureaucrat, democrat

-latry, -latria, -latrous

worship; excessively, fanatically devoted to

idolatry

-mania

mental disorder

pyromaniac, megalomania, maniacal

-phobia, phobo-, -phobe

fear, extreme fear of

claustrophobia, arachnophobia

-poly, -pole, -polism

sale, selling, one who sells

monopoly, oligopoly

a-, an-

not, without

amoral, anesthetic, apolitical, asocial

ab-

away from

abduction, abstain, abnormal

ad-

to, toward

adjoin, adjacent

alter-

other

alternate, alter ego

ambi-, amb-

both

ambidextrous, ambivalent

ambul-, ambulat-

walk, take steps, move around

amble, ambulance, somnambulate

5/23/2011 5:04:59 PM

6. 40

Part 2    SAT Preparation

ami-, amic-

friend

amiable, amicable

ampho-, amphi-, amph-

around, about, both, both sides

amphibian, amphitheater, amphora

amour-

love, loving

enamored

ana-

up, back, again

analogy, anatomy, anagram

anima-, anim-

animal life, breath, soul, mind

animation, animate, animal

ann-, annu-, enni

year

anniversary, annual, biennial, perennial

anthrop

human, man

anthropology, anthropomorphic, misanthrope

ante-

before, in front of, prior to

antecedent, antebellum, antediluvian

anti-

against

antipathy, antiwar, antisocial

apo-

from, away from

apology, apologize

aqua-, aque-, aqui-

water

aquatic, aquarium, aqueduct

arch

chief, leader, ruler

archangel, monarch, archaic, archenemy

arthro

joint

arthritis

astro-, astra-, astr-

star, star-shaped, pertaining to outer space

astronomy, astronaut

audio-, aud-, audi-

hearing, listening, perception of sounds

auditorium, audible, audiologist, audiotape, audacious

auto-, aut-

self

automobile, autonomy, autobiography

bell-

war

belligerent, bellicose, causus belli

bene-, ben-, beni-

good, well

benediction, benevolent, benefactor

bi-, bin-, bino-

two, twice, double

binoculars, bifocals, bicycle

biblio-, bibli-, bibl-

book

bibliography, bibliophile, bible

bio-

life

biography, autobiography, biology, antibiotic

brevi-, brev-

short

brief, abbreviate, brevity

cand-, can-, cend-

glow, white

candle, incandescent, candor

capit-, capt-

head, leader, chief, first

capital, decapitate

cardio, cardi-, card-

heart, pertaining to the heart

cadiovascular, electrocardiogram

carno-, carn-, carne-

meat, flesh

carnivorous, incarnate, carnival, carnal

cata-, cat-

down, against

catastrophe, catatonic, cataclysm

celer-

fast, swift, speed, rapid

celerity, accelerate

centro-, centri-

around, center

concentric, centrifugal

chrom-

color

chromatic, monochrome, polychrome

chron-

time

chronicle, chronology, chronometer, synchronize

-cide, -cidal

kill, killer, murder, cause death

genocide, insecticide, suicide

cine-, cinem-, cinema-

move, movement, set in motion

cinematography, kinetic

circum-

around, about, surrounding on all sides

circumlocution, circumference, circumvent

chap06.indd 40

5/23/2011 5:04:59 PM

6. 41

Chapter 6    Vocabulary

chap06.indd 41

-cise, -cis

to cut

scissors, incisive, excise

cogn-

know

recognize, cognitive, incognito

com-, co-, cog-, col-, con-

with, together

communal, community, connect, confide, collect

contra-

against, opposed to, opposite

contradict, contravene

cor-, cord-, cour-

heart, pertaining to the heart

core, cordial, excoriate

corp-, corpor-, corpus-

body

corpus, corpse, corporal

cred-, credit-, creed-

believe, belief, faith, trust

credible, credulous, credibility, credit, credo, credence

cruc-, cru-

cross

crucifix, crucial, crux

crypt-

hidden

cryptogram, cryptology, cryptic

culp-

guilt

culpable, culprit

de-

down, away

descend, deject

deci-, deca-

ten

decimate (to kill one of every ten), decathlon, decile

dei-

god

deity, deify

demo-, dem-

people

demography, democracy, epidemic, pandemonium

dento-, dent-, denta-

tooth

dentist, dentifrice, dentin

derm-

skin

dermatology, epidermis, hypodermic

dia-, di-

through, across

diameter, division

dic-, dict-

talk, speak, say

dictate, predict, diction, indict

dis-

apart, not

disengage, discord, discomfort

dorm-, dormi-

sleep, sleeping

dormitory, dormant

-dox

belief, opinion

orthodoxy, paradox, heterodoxy

duc-, -duce, -duct

to lead, to draw forth

induce, deduce, seduction, conduct, abduct

duo-, du-

two

dual, duet, duplicate

dyna-, dyn-

power, strength, force, mightiness

dynamo, hydrodynamics, dynamite

dys-

ill, difficult, bad

dysfunctional, dysentery

e-

out of, from

elect, eject

eco-

house, home, dwelling; household affairs

economics, ecosystem, ecology

ego-

self

egotist, egomania

endo-

within, inside

endoscope

epi-, ep-

above, over, on; besides; in addition to

epidermal, epilogue, epicure

equ-, equi-

same, similar, even

equal, equity, equanimity, equate, equidistant

eso-

inward, within

esoteric, esophagus

eu-

well, good

euthanasia, euphoria

5/23/2011 5:04:59 PM

6. 42

Part 2    SAT Preparation

ex-

out of, from

exhume, exhale, exodus

-fac-

make, do

manufacture, factory, benefactor

fid-, fidel-

believe, belief, faith, trust

confide, infidel, fidelity

fil-

threadlike

filament, filagree

fin-

end, last, limit, boundary

finale, infinity

flagr-

fire, burn, blaze

conflagration, flagrant

fluct-, flucti-, -flux

flow, wave

fluctuate, flux, influx

frag-, frang-, fract-

to break

fragment, fragmentary, fracture, fractious

frater-

brother

fraternal, fraternize, fratricide

fug-, -fuge, -fugal

to drive away, to flee, to fly

fugitive, centrifugal

gam-

marriage

monogamy, polygamy, bigamy

gen-

born, beginning, kind

genesis, genetics, generate

geo-

earth, world

geopolitical, geology, geography, geothermal

glot-

tongue, speech, language

plolyglot, epiglottis

grad-, -grade, -gress

step, walk, take steps

gradual, progression, transgression, gradient

graph-

writing, printing

graphology, biography, telegraph, geography

grav-, griev-

heavy, weighty

gravity, gravitas, grief

gyno-, gyn- gynec-

woman, female

gynecologist, androgynous, misogyny

helio-, heli-

sun

helicopter, heliotrope

hemo-, hema-, hem-

blood

hemophilia, hematology, hemoglobin

hetero-

other, different

heterosexual, heterodoxy, heterodox, heterogeneous

holo-

whole, entire

holograph, holistic

homo-

same

homosexual, homogeneous, homogenized

hydro-, hydr-

water

dehydrate, hydraulics, hydroelectric, hydroplane

hyper-

over, excessive, more than normal

hypertension, hypersensitive, hyperactivity

hypo-

under, below, beneath; less than; too little

hypotension, hypodermic, hypoglycemic

ign-, igni-, ignis-

fire, burn

ignite, ignition, igneous

il-

not

illegitimate, illicit, illegal, illegible

im-

not

imperfect, impolite, impossible

in-

not

indiscreet, invisible

inter-

between

intervene, interstate

intra-

within

intrastate, intramural

ir-

not

irregular, irrational, irredeemable

iso

equal, identical

isolate, isoceles, isothermal

chap06.indd 42

5/23/2011 5:04:59 PM

6. 43

Chapter 6    Vocabulary

chap06.indd 43

jet-, -ject, jac-

throw

inject, reject, subject, projection, jettison, dejected

jud-, -jud-

judge

judicial, judge, adjudicate

lava-, lav-, lavat-

wash, bathe

lavatory, lavish

leg-, lect-

read, choose

legible, lectern, lecturer, election

lex-

word, speech

lexicography, dyslexia

liter-, -lit-

letter

literature, illiterate, literal

litho-, lith-

stone, rock

lithography, monolith

loc-

place

local, location, locus

log-, loq-, -logy-, logue-

talk, speech, word

monologue, epilogue, dialogue, loquacious

lumen-, lumin-, lum-

light, shine

illuminate, luminary

luna-, luni-

moon, light, shine

lunar, lunatic, lunacy

macro-

large, great

macrocosm, macroeconomics

magni-, magn-

large, big, great

magnify, magnate, magnificent

mal-, male-, mali-

bad, evil

malevolent, malnutrition

manu-, man-, mani-

hand

manufacture, manual, manuscript

mar-

sea

marine, mariner, maritime

mater-

mother

maternal, maternity, matriarchy, matricide

mega-, meg-

large, great, big, powerful

megaphone, megalopolis, megastore

meta-, met-

behind, between

metacognition, metaphysics, metafiction

metri-, meter-, metro-

measure

geometric, thermometer, odometer, symmetrical

micro-

small, tiny

microscope, microcosm, microeconomics

migr-, migrat-

wander, moving

migrate, immigrant, emigrate

mini-, minut-, minor-

small, little

minority, minuscule, minute

miso-, mis-

hate, hatred

misogyny, misanthopy

mit-, miss-, -miss, -mit

send

permit, submission, mission, emit, missile, transmit

mob-, mot-, mov-

move

mobile, automobile, motion, promote, movie

mono-

one, single

monologue, monotheism, monarchy, monogamy

mor-, mort-

death

mortal, mortician, immortality

morph-, morpho-

form, structure

metamorphosis, amorphous, morphology, ectomorph

mut-

change

mutant, mutability, mutate

naus-, nau-, naut-

ship, sailor

nausea, astronaut, nautical

neo-

new, recent, current, young

neologism, neo-liberal, neonatology

neuro-

nerve

neurology, neurosis, neurobiology

5/23/2011 5:04:59 PM

6. 44

Part 2    SAT Preparation

nomen-, nomin-, nom-

name

nominal, nominate, nomenclature

non-

nothing, not

anonymous, nondescript, nonentity

nov-

new

novel, renovate, innovation, novella

nym-, onym-

word, name

synonym, acronym, anonymous, pseudonym

ob-

against

object, obstruct

omni-, omn-

all, every

omnivore, omnipotent, omniscient

ortho-

rigid, straight, correct

orthodox, orthodontist, orthopedic

pac-, peac-, peas-

peace

pacify, pacifist, appease

palin-, pali-

back, again

palindrome, palimpsest

pan-

all, every

pantheism, Pan-Hellenic, panorama, pandemic

pater

father

paternal, paternity, patricide, patrilineal, patriotic

patho-, -path-, -pathy

feeling, sensation, suffering

sympathy, apathy, empathy, telepathy, pathology

ped, pod (Latin)

foot

pedal, pedometer, centipede, gastropod, podium

pedo-, ped- (Greek)

child

pediatrician, pedagogy

pel-, -puls-

push

pulsate, repulsive, impulse, compel, propel

pend-

hang, weigh

pendulum, pendant, suspend, pending

per-

through

percolate, perforate

peri-

around, about, near, enclosing

perimeter, periscope, peripatetic

petro-, petr-, peter-

stone, rock

petrify, petroleum, saltpeter

phile-, phil-, -phile

love, loving, affinity for

philharmonic, philosophy

phon-, phono-, -phone

sound, voice

telephone, euphony, cacophony, phonograph

photo-, phot-, -photic

light

photogenic, photosynthesis, photography

plan-

flat

planar, plantation, plane

pneum-

lung

pneumatic, pneumonia

polis-, polit-, poli-

city, method of government

metropolis, political, polity, Annapolis

poly-

many, several, too many

polygon, polygamy, polytechnic, polytheism

port-, portat-

carry, bring, bear

portable, transport, portage, report, export

post-

after

postgraduate, posthumous, postpone

poten-, pot-, poss-, -potent

power, strength, ability

potent, omnipotent, potentate

pre-

before

precede, predict, predestined

pro-

for, forward

promote, project

proto-

first

prototype, protoplasm, protobiology

pseudo-, pseud-

false, deception, lying, untrue

pseudonym, pseudoscience

psych-

soul, spirit, mind

psychology, psychic, psychobiography

pugna-, pug-

fight

pugnacious, pugilist

chap06.indd 44

5/23/2011 5:04:59 PM

6. 45

Chapter 6    Vocabulary

chap06.indd 45

pyro-, pyr-

fire, burn, heat

pyromania, pyrotechnics, pyre

quer-, quisite-, quest-

ask, seek

query, inquisition, inquisitive, quizzical, quiz

re-

again, back

repeat, recede, regress

retro-, retr-

back

retrogression, retroactive, retrieve, retrospect

rupt-, -rupting, -ruption

bear, tear, rend

rupture, erupt, interrupt

scent-, scend-

climb

ascend, ascent, descend

schizo-, schiz-

division, split

schizophrenic, schism, scissors

sci-, -science, -sciently

know, learn, knowledge

prescience, omniscience, conscience

scop-, -scope-

see, view, sight, examine

telescope, microscope

scrib-, script-, -scribe

write

manuscript, scribe, proscribe, scripture, scribble

sec-, -sect-

cut

dissect, section, secede

sed-, sedat-, -sid, -sess

calm, not moving

sedative, sedentary, sedate

seis-, seismo-

to shake

seismograph, seize, seizure

senso-, sent-, sens-

feel, be aware

sensible, sentient, sentimental

sequ-, secu-

follow

sequence, sequel, consecutive, obsequious

serv-

serve, protect

service, servile, servility, servitude, conserve

simil-

same

similar, assimilate, simile, facsimile, verisimilitude

sol-

sun

solar, solarium

son-, -son

sound

sonar, resonate, unison

sopho-, soph-, -sophy

wisdom, knowledge

philosophy, sophisticated, sophomore

spec-, spect-, spic-

look, see

spectacles, spectator, inauspicious, prospect   

spiro-, spir- spira-

breathe, spirit, courage

inspire, respiration, conspire, perspiration

spond-, spons-

promise, answer for

respond, responsible

stato-, stat-, sta-, -stasis

standing, stay, make firm, fixed

station, static, status quo, homeostasis

sub-

under

submarine, subject, subhuman subterranean

sur-, super-

over, above

superhuman, superego, superintend, surpass

syn-, sym-, syl-, sys-

with, together

symphony, synonym, system,

tang-, tact-

touch

tactile, tangible

techno-, techn, tect-

art, still, craft

technology, technique, architect, technocrat

tele-

distant, far off

telephone, telepathy, television, telegram

tele-, tel-, telo-, -telic

far away, far off

telegraph, telephone, telepathy

tempo-, temp-

time

temporary, temporize, contemporary

ten-, tent-, tin-, -tain

hold

tentative, tenable, tenuous, tenacious

terr-

earth

subterranean, terrain, terrestrial, disinter

5/23/2011 5:04:59 PM

6. 46

Part 2    SAT Preparation

theo-, the-, -theism

god, deity

theology, polytheism, atheist, monotheism

thermo-, therm-, -thermic

heat

thermal, thermos, thermometer

toxico-, toxic-, toxi-, tox-

poison

toxicology, anti-toxin

trans-, tran-, tra-

across, through, over, beyond

transient, Transatlantic, transport

ultra-, ult-

beyond, on the other side, excessive

ultimate, ultimatum

vac-

empty

vacation, vacuum, vacuous, vacant

vaga-, vag-

wander, move around

vagrant, vagaries, vagabond

ven-, vent-

come, go

intervene, convene, contravene

veri-, ver-

truth

veracity, verify, verity, verisimilutude

vita-, vit-

life

vital, revitalize, vitamin

voc-

call

revoke, invocation, vocal, evocative, convocation

zoo-

animal

zoo, zoology, zoolatry

chap06.indd 46

5/23/2011 5:04:59 PM

Chapter

7 T

General Strategies

his program of preparation is designed to acquaint you with each of the three fields on the SAT—Math, Critical Reading and Writing—and with each primary and secondary subsection of those fields. Each field has four or five primary subsections; each sub-section generally has between two and four sub-types (though some of these aren’t very significant). This adds it up to a lot of material, but that’s a good thing: the subsections and sub-types are the building blocks with which the SAT is constructed, and a careful study of these types of questions will make it so you face no surprises on test day. Because there are no good surprises on the SAT. A surprise is something you do not expect; something that sneaks up on you. The SAT is too important to allow for the possibility of the unknown. But here’s the good news: the standardised and recurring structure of the SAT, which is more or less the same every test date, with similar patterns of questions and similar distributions of sub-types, means that with an inclusive and comprehensive program of preparation, you can almost zero the possibility of surprise—unless, of course, it’s the surprise you feel at being so well-informed and well-prepared! Chances are that the SAT is a test unlike any you’ve ever taken; as such, you’ll need to learn an entirely new skill-set specifically for it. Familiarity with the subsections and sub-types is just a part of this; with every chapter you’ll also be learning specific skills and strategies designed to help you crack the Hard questions on every SAT section. These specific strategies have been honed and tested in the classroom, and have shown real results on SAT test day. You won’t be disappointed. However, there are several general test-taking skills and strategies you must first become familiar with in order to extract maximum value from each of the specific test-taking strategies. What follows in this chapter is a consideration of some of the unique qualities of the SAT and several effective and easy to use strategies designed to help you deal with these

chap07.indd 1

novelties. These strategies will show up again and again during each chapter as we explore their application to the specific components of the SAT. Consider this review your first step in a rigorous and effective program of SAT self-study; pay close attention and ultimately you will find yourself wholly prepared and totally ready on SAT test day.

7.1. Use of Time The SAT is a ‘speeded’ exam—which means that it is not designed to be finished by everyone. Most students find time management to be one of the most challenging aspects of the SAT, especially if their first language isn’t English. The good news it that everyone finds struggles with this, and if you can get familiar with the resources provided you then you provide yourself a significant edge over other SAT test-takers.

7.1.1. Average time per question per section You have the following average time per question per section: Test Section

Time

Critical Reading

1 minute 4 seconds

Math

1 minute 30 seconds

Writing

43 seconds

Total

1 minute 3 seconds

Notice that you have more twice as much time allotted for Math as Writing; the fact of this will ultimately beget several specific strategies.

7.1.2. General Strategy #1: Skip-and-Return This is a basic test-taking skill for any timed test. You have a fixed number of minutes to answer a fixed number of ques-

5/23/2011 5:05:17 PM

7.  tions. The last thing you want to do is waste time over a difficult question—time that could be spent answering easier subsequent questions! Your strategy: when confronted with a question whose answer is not immediately apparent, clearly note that question on the test paper, then skip it and attempt the questions after it. If you have time at the end of the section, revisit the skipped questions, easiest first; if you do not have time at the end of the section, then you will have left out the question you chose to skip. This will be a big advantage, especially in sections of Critical Reading.

7.1.3. General Strategy #2: Use Every Minute There’s nothing worse you can do than sitting idly at the end of an SAT section. Every minute matters! If you have time at the end, go back and re-check the problems you found difficult, or make certain you haven’t made any careless mistakes on the easier questions. By not productively using every minute, you are not only giving away the opportunity to fix potential mistakes, but you’re allowing for the possibility of getting bored and losing focus. One of your biggest enemies on the SAT is boredom: so stay sharp and don’t let yourself get sluggish in the final minutes of a section.

7.2. Use of Space Space is another finite resource on the SAT. It is important to know where to be able to write and where you must not make stray marks.

7.2.1. General Strategy #3: Write in the Test Book The primary resource of the test book (apart from the questions) is its space. You must use the test-booklet as a space to work out your answers. Many of the specific strategies you will be implementing with each section will involve writing copiously in your test book.

7.2.2. General Strategy #4: Bubble Seriously Another spatial opportunity is bubbling. You have to watch for: Incomplete Bubbling: An incomplete bubble might not be counted as answered. Double Bubbling: If you bubble two answer choices for the same question, then you will be marked wrong. Misplaced Bubbling: A misplaced bubble can skew your entire section. If you elect to leave a question—say, you skip

chap07.indd 2

Part 2    SAT Preparation question 10—but you accidentally bubble question 10 with your answer for question 11, question 11 with your answer for question 12, etc., then you’ll get to the end of the section and realise that you have incorrectly bubbled the remainder of the section. This is the potential downside of General Strategy #1: Skip-and-Return, So pay close attention that the question you are bubbling matches the question you have attempted in the test book. Slow Bubbling: On an average, you have 63 seconds per question. How long does it take you to consider your answer, bubble it, and then refer back to your test booklet? 10 seconds? That’s over 15% of your available time per question! Get used to quick bubbling to make the most of your allotted time. Non-#2 Bubbling: You will be expected to use a #2 pencil when bubbling on the SAT (#2 has to do with how dark the lead is). If you do not use a #2 pencil, then your answer sheet may not be marked properly. Consider yourself warned. Erasing Bubbles: Sometimes an erasure can be counted as a “stray mark” on your answer sheet. If you must erase, do so fully and with a clean eraser. Every aspect of the exam yields opportunities for efficiency: remember that you’re taking the test not just against yourself, but over 2 million other test-takers who will be sloppy and slow at the little things. If you can practice good Test-Economy, making the most the small opportunities for speed, then you’ll be better off.

7.3. Use of Logic The particular forms of the SAT allow for the development and deployment of several purely logical strategic devices. The following is a list of qualities of the exam, and a discussion of strategic derivations from those qualities.

7.3.1. Multiple-Choice The SAT asks questions that are largely multiple-choice—160 of the test’s 170 total questions are asked in this format. The associated strategies are:

7.3.2. General Strategy #5: The Process of Elimination The PoE is the standard test-taking strategy for all SAT testtakers, everywhere. What it means is this: for every multiplechoice question, you are given five potential answers: A, B, C,

5/23/2011 5:05:17 PM

7. 

Chapter 7    General Strategies D, or E. The correct answer is always found there. Therefore, the logical strategy when facing a problem whose answer is not apparent is to eliminate the answer choices one by one until you have determined the correct answer, made it so you feel confident about guessing, or concluded that you can’t make an educated guess and so skip the question.

ficulty-Rating of 1-5. This rating is determined by the rate of accuracy of a random sample of college-bound high school seniors on official administrations of the SAT. The sectional and overall distribution of Easy, Medium, and Hard questions is as follows: Difficulty

Critical Math Reading

Writing

Total

Easy

24.88

32.72

41.95

32.29

Medium

58.71

48.56

50.79

53.20

Hard

16.42

18.72

7.26

14.51

7.3.3. General Strategy #6: Rule of 1 If you can eliminate one answer choice via the Process of Elimination—or, better still, if you can eliminate more than one answer choice—then you ought to guess. In choosing between two, three, or four answer choices, you have a statistically advantageous chance of guessing the right answer; and therefore it is in your interest to do so.

7.3.4. Guessing Penalty One of the more interesting formal wrinkles of the SAT is that the test-makers have installed a Guessing Penalty into the scoring of the exam. Your Raw Score (from which your scaled score is derived) is calculated from a simple equation involving the (# COR) - .25*(# INCOR). The subtraction is folded into the scoring to prevent guessing: if you guessed on every multiple-choice question, you would (in theory) answer 32 of the 160 questions correctly. However, if you subtract away 1/4 of the # INCOR (128/4 = 32) you would conveniently have achieved a Raw Score of zero.

7.3.5. General Strategy #7: More with Less The Guessing Penalty can work to your advantage if used intelligently. By skipping answers that you would have gotten wrong, you save yourself from losing Raw Points and you give yourself more time to work on questions you can get right; with skipping you effectively earn More points with Less effort. Skip-and-Return is the key here. If you leave out the questions that you can’t answer right away and spend your time on the questions that seem more attainable, then if you happen to run out of time, you will be leaving out questions you had already looked at and decided were not for you (right away, at least). The worst-case scenario—running out of time—might actually end up working to your advantage by preventing you from attempting questions you might have gotten wrong.

chap07.indd 3

The fact of this rating will help inform your decision when to skip questions, and implies yet another strategy:

7.3.7. General Strategy #8: Skip the Hard Questions The Hard questions are not designed to be answered by all people—and most people would generally be better off simply Skipping the Hard Questions. You can skip between 2-5 questions per section (Critical Reading, Math, Writing) and still score 700 or better on each (though you’d have to get all the other questions correct!) If you’re shooting for a score of 1800, you can skip between 6-9 questions per section (Critical Reading, Math, Writing), and if you’re shooting for a 1500, then you should consider skipping 12-15 questions per section.

7.3.8. Order of Difficulty The SAT thoughtfully orders most of its questions according to their difficulty level: Easy early, Hard late. Consult the following chart to learn which SAT sub-sections are Ordered by Difficulty: Sub-Section

Ordered by Difficulty?

Sentence Completions

Yes

Reading Comprehension

No

Math (multiple-choice)

Yes

Math (grid-in)

Yes

7.3.6. Difficulty-Rating

Improving Sentences

Yes

All SAT questions are not created equal, in terms of difficulty. This is no secret—on the answer keys in publically available materials, the test-makers give every question a Dif-

Error Identification

Yes

Improving Paragraphs

No

5/23/2011 5:05:17 PM

7.  Nearly 70% of the questions on the SAT are Ordered by Difficulty—this means that nearly 70% of the time you know roughly where you are in terms of how challenging a question is meant to be (and therefore how complicated the answerprocess is meant to be). Knowing whether or not you can rely on the first answer that springs to mind is an enormous advantage. (Hint: on Hard questions, the first answer that springs to mind is almost never right. But much more on that later).

7.3.9. General Strategy #9: Seeming Hard vs Being Hard Apart from utilising the Order of Difficulty to deploy specific subsectional strategies, you can use it to monitor yourself when undertaking Hard questions. If you’re working on a question that you know is meant to be Hard (the final question in a Math or Writing section), but which you find surprisingly Easy, then you know you have misunderstood what’s being asked of you. There are no good surprises on the SAT! Make the Order of Difficulty work for you by using how Hard a question seems as a standard of your true understanding.

7.3.10. Equality of Worth Though all SAT questions are not created equal in terms of difficulty, they are created equal in terms of point value. This means that the Easy questions are worth as much as the Hard questions, even though they’re less challenging.

7.3.11. General Strategy #10: Make Time Early, Take Time Late The early questions are Easy because they don’t require as much process as the Hard questions. Take advantage of this simplicity! Your job is to make time by answering the Easy questions efficiently so that you can take time over the Medium/Hard questions. With Math, Sentence Completions and Writing, you really need to be moving through the early questions as quickly as you can (watching for silly mistakes, naturally), leaving minutes more time at the end to puzzle through the most challenging Hard question.

7.4. Use of Question The question is your final and most useful resource. This resource involves not just the question-prompt but the answer

chap07.indd 4

Part 2    SAT Preparation choices too; the latter can be very useful with specific sectional strategies. For the time being, remember the following:

7.4.1. General Strategy #11: Best Fit With questions of Writing and especially Reading Comprehension, there often is no clear-cut answer—if there were, the questions wouldn’t be so challenging! Your job, in these sections, is not to select the answer choice that you think provides the Best Fit; your job is to choose the answer choice that you think the SAT thinks is the best fit! Keep in mind that the ‘perfect’ answer choice—one that matches your internal solution—rarely exists on the Writing and Critical Reading sections and most of the time you have to choose whichever solution fits best according to what the SAT wants. This means that you’re going to have to get familiar what what the SAT thinks is the Best Fit so you can learn to recognize an appropriate answer choicer when you see one.

7.4.2. General Strategy #12: Knowingly not Knowing If you are anything like the typical test-taker, there will be questions on the SAT that you do not understand. That means there are three possible types of question: Questions you understand: Easy Math, Writing, or Sentence Completions. You have no difficulty grasping what you are meant to accomplish. OK. Questions you know you don’t understand: Hard Math, Hard Sentence Completions. You know that what is being asked of you is not possible (right now); your best bet is to pay no attention to them at all. OK Questions you think you understand but actually don’t: Medium Math, Hard Writing, all Reading Comprehension. These are the questions that will cause you a great deal of grief because you will think you understand them well, but it will transpire that actually you did not truly grasp what was being asked of you. One of your jobs in preparing for the SAT is to minimise the number of times you are in this situation by Writing in the Test Book, using the Order of Difficulty, and Skipping the Hard Questions. It is always better to Knowingly not Know, than to think you know (when actually you don’t). NOT OK.

5/23/2011 5:05:17 PM

7. 

Chapter 7    General Strategies

7.5. Quiz #1: General 1. True or False: an improperly marked bubble may be marked as incorrect

2. True or False: test-takers are encouraged to mark more than one bubble on one question per section

3. How many raw points are subtracted per wrong answer on the multiple-choice questions?

4. For which section do you have the most time per question?

5. Which situation would yield more points: a. Marking 18 questions correctly and 20 questions incorrectly b. Marking 14 questions correctly and zero questions incorrectly

6. Which section has the greatest proportion of Hard questions?

7. Are Reading Comprehension questions presented on the SAT in Order of Difficulty?

8. How does knowing the Order of Difficulty help you in your choice to skip or attempt a question?

9. True or False: the answer choice you consider to be the Best Fit will always be what the SAT considers the Best Fit

10.  What is Knowingly not Knowing?

chap07.indd 5

5/23/2011 5:05:17 PM

Chapter

8

Critical Reading I: Reading Comprehension

R

eading Comprehension is considered by most students to be the single most difficult element of the SAT because the questions are challenging, the passages are complex and the answer choices can be difficult to choose between. Any serious program of SAT preparation starts with Reading Comprehension and sticks with it all the way until test day. But it can be cracked. Improving your abilities in Reading Comprehension means not only working hard to increase vocabulary, but also working to extend the limits of your ability to think about the meaning of questions, passages, and the differences between answer choices. It is also going to become important to be able to retain focus and sharpness for the full 25 minutes you are allotted for each section, and to learn not to be bored with the material presented in the passages. Because Reading Comprehension is recognized as the most difficult element of the SAT, we will spend three chapters looking very closely at basic, intermediate and advanced strategies. In this chapter we will focus on your approaches to Reading Comprehension, looking carefully at how you can save time and stress by approaching the passages and questions in a maximally efficient way. In this section, we also review some basic skills you will require for the more advance strategies, and we will undertake some exercises designed to reinforced these skills.

Formal Sub-Sections

Avg. # Questions per SAT

% SAT

Reading Comprehension (CR)

48

28.24

Math (multiple-choice)

44

25.88

Improving Sentence (Writing)

25

14.71

Sentence Completion (CR)

19

11.18

Error ID (Writing)

18

10.59

Math (grid-in)

10

5.88

Improving Paragraphs (Writing)

6

3.53

There are almost as many Reading Comprehension questions as there are Writing questions; there are more Reading Comprehension questions than there are multiple-choice Math questions! The College Board has made this subsection the biggest because it is the toughest element of the exam and, therefore, the best indicator of potential success at the university level (which is what the SAT is meant to measure). If you can beat Reading Comprehension, you can beat the SAT.

8.2. Directions 8.1. Formal Distribution Reading Comprehension presents as the single most significant formal sub-section of the SAT. See chart for breakdown:

chap08.indd 1

Reading Comprehension questions are always preceded by the same set of directions. Consult the box below for the exact directions you’ll see on the SAT; familiarise yourself with these now so you don’t waste any time reading them on the actual test:

5/23/2011 5:05:26 PM

8. 

Part 2    SAT Preparation

The passages below are followed by questions based on their content; questions following a pair of related passages may also be based on the relationship between the paired passages.Answer the questions on the basis of what is stated or implied in the passages and in any introductory material that may be provided. Note that the instructions suggest that questions can only be answered based on the content of the passage. This is incorrect—certain questions can be answered correctly based only on the language of the answer choice. Note also that you are told that you will be asked questions on what is explicitly stated in the passages as well as merely implied. Understand that it is possible that your view of a potential implication differs from the SAT’s view; let this serve as another reminder of the importance of being able think in terms of what the SAT considers the Best Fit.

8.3. Difficulty Distribution The Difficult Distribution of Reading Comprehension is as follows: Difficulty

Frequency

Easy

22.69

Medium

65.51

Hard

11.81

Note the lack of Hard questions but the huge number of Medium questions and remember that Reading Comprehension questions are not Ordered by Difficulty; therefore, you are much more reliant on your impression of the difficulty of a question when trying to decide to implement General Strategy #8: Skip the Hard Questions.

8.4. Frequency Distribution Reading Comprehension breaks down two ways: into the kinds of passages you’ll see and the kinds of questions you’ll see. Consult the following chart for more information on the kinds of passages you’ll see on the SAT: Reading Comp sub-section

% of Reading Average # Comp per test

Long Non-Comparison questions constitute a strong majority of the Reading Comprehension portion of the SAT; because of this, you’ll see that the approaches and strategies evolved for questions of Reading Comprehension all shake out of those for Long Non-Comparative passages. Now consult the following chart for more information on the kinds of Reading Comprehension questions you’ll see on the SAT: Reading Comp. subsection

% of Reading Comp.

Average # per test

Specific Interpretation

70.39

33

General Interpretation

14.80

7

Vocabulary-in-Context

9.54

5

Tone, Style and Attitude

5.26

3

Each of these question-types has its own Difficulty Distribution: remember that some questions are harder than others. Consult the following chart for detailed information on the Difficulty Distribution of the Reading Comprehension sub-sections: Specific Int.

General Int.

ViC

TSA

Easy

25.71

16.67

42.86

13.04

Medium

54.29

83.33

54.29

52.17

Hard

20.00

0.00

5.71

17.39

Note that there are very rarely any Hard-rated general questions. One reason for this could be that there General questions—with the sheer amount of material to be synthesized—are considered sufficiently difficult by the testmakers without any addition wrinkles added in. A definite thing to remember is that as a result of this, you won’t usually have to deal with tricky answer choices on General questions. And remember: time is of the essence! Consult the following chart to see how much average time you have per question: Test Section

Time

Long Non-Comparative

56.25

27

Critical Reading

1 minute 4 seconds

Long Comparative

27.08

13

Math

1 minute 30 seconds

Short Non-Comparative

8.33

4

Writing

43 seconds

Short Comparative

8.33

4

Total

1 minute 3 seconds

chap08.indd 2

5/23/2011 5:05:26 PM

8. 

Chapter 8    Critical Reading I: Reading Comprehension

8.5. Reading Comprehension: Function Each of the four Reading Comprehension question-types asks you to perform a different kind of task; it is your job to learn these types so that on test day you are not only not surprised, but also able to apply your specific strategic knowledge.

8.5.1. Reading Comprehension: Specific Interpretation By far the largest material sub-category of Reading Comprehension (and, indeed, the SAT as a whole), questions of Specific Interpretation can be roughly divided into two categories:





Within: these questions generally ask directly about what happens in the passage. They might ask you to locate and re-state different elements of the text (most common), answer direct questions about events in the passage, characterize elements of the passage, or determine whether or not a device was used in the passage. Examples: • Example: “The sentence in lines 11-12 is best described as…” • Example: “The author of Passage 1 makes use of which of the following rhetorical devices?” • Example: “The central character leaves his position as a tax collector because…” Beyond: these questions generally pertain to what the author thinks; they might ask for the effect of a quote or device, the author’s assumptions or beliefs, the similarity between hypothetical or analogous situations, possible logical inferences and implications, or how an author might respond to a part or all of a comparative passage. Examples: • Example: “Based on the reasoning in lines 24-27, the author probably believes which of the following?” • Example: “The quote in line 24 serves to…” • Example: “The quote in lines 72-73 is comparable to which of the following?” • Example: “The author of Passage 2 would probably respond to the sentence at the end of Passage 1 by saying…”

8.5.2. Reading Comprehension: General Interpretation These questions may ask you to interpret material in the context of the passage as a whole; may ask you of the overall purpose of the passage; or may ask you to provide an overall summary of the passage. Occasionally, you will also be asked about the function of certain elements or devices within the context of the passage as a whole. Examples:

chap08.indd 3

• • • •

Example: “The passage suggests that contact between Native Americans and Europeans ultimately…” Example: “The primary purpose of Passage 1 is to...” Example: “The passage can be described as a…” Example: “Within the passage as a whole, the first paragraph functions to…”

It is sometimes useful when attempting a General question to consult the first and last sentences of the passage; these often contain hints as to the passage’s main purpose.

8.5.3. Reading Comprehension: Vocabulary-in-Context Among the easiest questions on the SAT, these ask the meaning of vocabulary based on its context within the passage. You should always make it a point to attempt every single one of these. Example:



Example: “in line 35, “invention” most nearly means…”

You should make it a point to attempt every single one of these (unless you Knowingly do not know the vocabulary). If you’re running out of time within the section, and have to choose which questions to attempt in, say, the final minute, always go for the Vocabulary-in-Context questions. They’re quick and more importantly local: you do not have to have read the entire passage to be able to hit the right answer.

8.5.4. Reading Comprehension: Tone, Style and Attitude These questions— usually associated with one-word answers—ask after the author/narrator’s tone/attitude; they can also ask after the material tone of an element within the passage. There is a specific strategy that applies to these, and as such, you should always attempt them. Examples:

• •

Example: “the tone of lines 37-39 is best described as…” Example: “the author’s attitude is best described as…”

8.6. Reading Comprehension: Skills Without developing a basic skill set it will be impossible to deploy the intermediate and advanced strategies for questions of Reading Comprehension. Consider the development of the following skills a priority in your mission to becoming a Reading Comprehension expert.

5/23/2011 5:05:26 PM

8.  8.6.1. Skill #1: Locating Information In order to answer specific questions, you must be able to locate information within the passage by:







Using the Line Numbers: most questions are asked in reference to a particular element of the passage; these questions usually cite line numbers. You must refer back to those lines and read approximately 4 lines above and 4 lines below the specified bit. Once in a while, the information needed to answer a question is 8 or 9 lines below, so be aware that if you do not think you have found what you need within the 8-line area, keep scanning above and below, one sentence at a time: the answer will probably lie further above or below. Use The Order: the questions are all asked in Order—meaning that the first question (assuming it isn’t a General question) will ask after the first sentence or paragraph of the passage—and each succeeding question will ask after succeeding elements of the passage, in order. Question Five might ask about material in lines 50-52; Question Seven might ask about material in lines 67-69; therefore, you know that Question Six asks about material between lines 52-67. Use Keywords: occasionally you will be asked questions that are specific but which do not provide line numbers. Determine the Keyword of the question and then scan the passage in the appropriate area—i.e., between the given line numbers of the non-General question immediately before and after—to find the relevant passagearea. If, for instance, the question asks specifically about Thomas Jefferson and you know that the relevant material will lie between lines 43 and 75, read that area quickly until you find the Keyword (Thomas Jefferson)..

This skill—Locating Information—is the backdrop against which you will be able to deploy strategies that will help you in your Approach to Reading Comprehension. If you do not practice this skill with every practice section, then you will never maximize your score on Reading Comprehension sections, because you will either be missing information or trying to absorb too much and, therefore, losing focus.

Part 2    SAT Preparation passage. This involves no analysis but requires a little creativity. See below for examples: • Question: The author uses “ubiquitous” to convey the

• •

Passage Excerpt: “The corporate logo was truly ubiquitous: it could be seen everywhere, on buses, on tshirts and hats, in the windows of homes” Correct Answer Choice: “omnipresence of a marketing device”

From the Passage

From the Correct Answer Choice

“corporate logo”

“marketing device”

“could be seen everywhere”

“omnipresence”



Question: The author argues that the “hallmark of business” is essentially • Passage Excerpt: “It can be universally agreed that the hallmark of business consists in a firm commitment to competition, quality, and service within the framework of an ever-changing modern world” • Correct Answer Choice: “the retention of certain principles with the flexibility to change without losing those core values”

From the Passage

From the Correct Answer Choice

“a firm commitment to “retention of certain principles” competition, quality, and customer service” “framework of an ever- “flexibility to change” changing modern world” There is, however, a caveat: don’t make the mistake of selecting an answer choice just because it contains repeated words or phrases from the passage. This inclusion and repetition is purposely meant to distract and confuse you. In an answer choice, repeated words from the passage rarely indicate a correct answer.

8.6.2. Skill #2: Saying the Same Thing a Different Way

Try an exercise to practice your skill in Saying the Same thing a Different Way.

More than half the Reading Comprehension questions can be cracked by simply re-stating information in the passage in a different way, using hard(er) vocabulary. This is the great secret to “Within” questions: your job can really be as easy as Locating Information and choosing the answer choice that most closely re-phrases the Located Information from the

8.6.3. Exercise #1: Saying the Same thing a Different Way

chap08.indd 4

Directions. Below you will find a series of questions with five answer choices. Select the correct answer choice based on the material in the italicised excerpt, and indicate the words

5/23/2011 5:05:26 PM

Chapter 8    Critical Reading I: Reading Comprehension in the passage that led you there using the chart provided for each question. It is never the case that government can accommodate the most modern cultures that the world of art has to offer: there’s always a dampening effect. This makes it so the concept of governmental assistance to young artists is thus rendered not only impossible, but also undesirable. 1. The author believes that the notion is governmental assistance to artists is “not only impossible, but also undesirable” because (A) most governmental official lack the insight to appreciate the most cutting-edge art (B) there are some who deride funding for the arts as unnecessary and irresponsible (C) the result of aid is in reality counter-productive with respect to the creative processes (D) the criteria to qualify for financial assistance is overly stringent (E) the proffered support had a marginalising effect on minority artists and writers From the Passage

From the Correct Answer Choice

Let us put it another way. If we consider the counterfactual, it cannot be the case that open-water long distance swimmers do not use performance enhancing drugs. Studies have suggested that even with the most robust of cardiovascular health, no mortal has the endurance to complete the Bearing Strait swim without some kind of chemical assistance. 2 The author refers to “studies” in order to (A) maintain a position even in the face of strongly contrary evidence (B) suggest a compromise between two seemingly intractable viewpoints (C) demonstrate the body types that tend to have an advantage in endurance sports (D) rebut an argument about the desirability of open water swimming (E) make a point about the implausibility of drug-free endurance athletes From the Passage

chap08.indd 5

From the Correct Answer Choice

8. 

The book was designed to raise popular awareness for those displaced and dispossessed by the ethnic cleansing, but the irony was that those who would pay money to purchase the book already had their minds firmly opened; and those who didn’t, simply wouldn’t. A book can’t alter people’s perspectives if it doesn’t get bought; and it won’t get bought unless people think they’re going to like what they find inside 3. The author emphasizes the word “would” in order to stress that (A) the publishing industry is reliant on young talent (B) the sale of books depends as much on marketing as it does on content (C) most literary awards are given on the basis of who you know (D) a book cannot cause a cause change if people will not read it (E) the cover art of a book is important because most people generally will not purchase it unless their interest is piqued From the Passage

From the Correct Answer Choice

It was a result that even the most optimistic of us would have had trouble envisaging: a full pardon for our actions undertaken in rebelling against an unjust law, and a new Parliamentary act altering the legislation once and for all. 4. The “result” mentioned by the author is best characterised as (A) a peremptory strike on the arbitrariness of the political justice system (B) the release of a group of activists for committing previously-illegal acts (C) a last-minute reprieve from a draconian punishment (D) a stay of execution for a group of wrongly-imprisoned refugees (E) a successful appeal for a prisoner who argued his own case From the Passage

From the Correct Answer Choice

5/23/2011 5:05:26 PM

8.  8.7. Reading Comprehension: Strategies Because Reading Comprehension questions constitute a significant proportion of the SAT, and because all Reading Comprehension questions share a set of unique qualities, it is possible to develop and apply a few Strategies to facilitate your task of identifying and interpreting information from within the passage. A poorly-organized approach to Reading Comprehension will leave you confused and frustrated. However, with a little management you can learn to find meaning and even enlightenment in the learned material presented in these passages. Using the backdrop of Locating Information and Saying the Same Thing a Different Way, in addition to the Process of Elimination and other general strategies, it is now possible to formulate the most general strategies for Reading Comprehension—your approaches for each of the four types of passage.

8.7.1. Strategy #1: Long Non-Comparative Passages Recall that the Long Non-Comparative Passages are not only what you see the most often but also which offer the maximum number of questions: 27 per test. A common mistake with questions of Reading Comprehension is to think that you must first read the entire passage before attempting to answer the questions. This is incorrect. Another common misconception is that you must attempt the question in the order in which they are given. This too is not correct. It is your first task to be informed and make the choices you are allowed to make: because the SAT is a paperbased test, you get to choose which questions to attempt, and when. Now, the 13-step approach: 1. Read the italicised introductory information. This can and often does include important information. 2. Skip directly to the questions. 3. Find the first Non-General question. Note that this is not always the first question in the question series; about 40% of the time the SAT test-makers purposely start with a General question, to make you think you need to read the entire passage. This trick may be part of the reason why General questions are almost never rated Hard. 4. Read the question so you have an idea of what you’re being asked to do. 5. Refer back to the passage directly using the skill of Locating Information (line numbers, order, keywords) to find what you need in order to answer the question. 6. Read from the start of the passage to the first question-area, and do so actively and intelligently. If you scan the passage with the question in mind, knowing

chap08.indd 6

Part 2    SAT Preparation what is being asked, you will be more able to extract that information when you see it. 7. Return to the question, with your own answer in mind. 8. Read every answer choice. See if you can find the answer choice that Says the Same Thing a Different Way. 9. Use the Process of Elimination to leave only one answer possible. 10. Find and read the next Non-General question, developing an idea of what is asking of you and Locating Information using Line Numbers and Keywords. 11. Read from the passage-area of the previous question to the passage-area of the next question, and repeat steps 7–9. 12. Repeat steps 10 and 11 for all remaining Non-General questions, saving the General questions for last (this may involve having to Skip-and-Return). By the time you finish all the non-General questions, you have will effectively read the entire passage—though not all at once. 13. Attempt the remaining General questions, reviewing the first and last sentences of the passage for possible hints.

8.7.2. Strategy #2: Long Comparative Passages On every SAT, you will be asked to answer 13 questions of Reading Comprehension on Long Comparative Passages. The passages will share a topic but each will approach that topic differently. There are four types of question you will see:

• • •

Specific: Ask about a specific part of either Passage 1 or Passage 2. These are just like questions on Long NonComparative Passages, and you will see 6-8 per test. General: Ask about Passage 1 or Passage 2 in general. Again, these are just like General questions on Long Non-Comparative Passages. Possibly 1-2 per test. Specific Comparison: Ask about a specific part of Passage 1 in the context of Passage 2, or vice-versa; line numbers included. Usually around 4-6 per test. Examples: • “The author of Passage 1 would most likely respond to the quote in final paragraph of Passage 2 by stating…” • “The author of Passage 2 would probably react to the situation presented in lines 4-6 by…”

Note: In order to be able to answer a Specific Comparison question, you have to have read at least one full passage.



General Comparison: Ask about Passage 1 in the con-

5/23/2011 5:05:26 PM

Chapter 8    Critical Reading I: Reading Comprehension text of Passage 2, or vice versa; line numbers not included. Usually 1-2 per test. Examples: • “The authors of both passages would probably agree that…” • “Which of the following best characterizes the relationship between Passage 1 and Passage 2?” Note: in order to be able to answer a Specific Comparison question, you have to have read both full passages. Long Comparative Passages can be tricky not only for the regular reasons—difficult language and dense writing—but also because they’re longer than normal passages and present two often-contrasting views. This is a lot more information for you to deal with: two perspectives instead of one. Questions of Long Comparative Passages are also difficult because their order will make you think that you need to have read both passages before you can begin attempting questions. This is not the best way to approach these passages! To minimize reading-time and maximize comprehension, think about using the following 16-step approach: 1. Read the italicised introductory information. This can and often does include important information. 2. Skip directly to the questions. 3. Find the first Specific question. This may feel unorthodox because quite often you will have to miss out three or more questions at the start of the series. Don’t worry if you do. 4. Read the question so you have an idea of what you’re being asked to do. 5. Refer back to the passage directly using the skill of Locating Information (line numbers, order, keywords) to find what you need in order to answer the question. 6. Read from the start of the passage to the first Specific question-area, and do so actively and intelligently. If you scan the passage with the question in mind, knowing what is being asked, you will be more able to extract that information when you see it. And: because you may have skipped several questions, it is possible that you will have to read down a lot further than you normally do on Long Non-Comparative Passages. This is acceptable: though you might be reading 20-30 lines, it is still much less than reading both passages through first 7. Return to the question, with your own answer in mind. 8. Read every answer choice. See if you can find the answer choice that Says the Same Thing a Different Way. 9. Use the Process of Elimination to leave only one answer possible. 10.  Find and read the next Specific question, developing an idea of what is asking of you and Locating Information using Line Numbers and Keywords.

chap08.indd 7

8. 

11. Read from the passage-area of the previous question to the passage-area of the next question, and repeat steps 7–9. Again, you might have to read more than you would do on Long Non-Comparative Passages, but again, this is acceptable. 12. Repeat steps 10 and 11 until you have answered every Specific question from Passage 1. 13. Attempt any General questions for Passage 1, reviewing the first and last sentences of the passage for possible hints. 14. Repeat steps 6–9 until you have answered every Specific question for Passage 2. 15.  Read the remainder of Passage 2. 16. Attempt any General questions for Passage 2, reviewing the first and last sentences of the passage for possible hints. You have now read both passages in their entirety. 17. Attempt any Specific Comparison questions by repeating steps 8 and 9. 18. Attempt any General Comparison questions, repeating steps 8 and 9. Manipulating the order of questions can take getting used to. Attempt the following exercise for practice with question order in Long Comparative Passages.

8.7.3. Exercise #2: Long Comparative Passages Directions. What follows is a series of Reading Comprehension questions from a section which features Long Comparative Passages. Identify these questions as Specific, General, Specific Comparison, or General Comparison. 1. The authors of both passages would most likely disagree on which of the following? • Question Type: 2. Passage 1 could be best described as • Question Type: 3. The chefs mentioned in the first paragraph of Passage 1 probably believe the final paragraph of Passage 2 suggests a • Question Type: 4. In lines 22-29, Van Meeger’s basic philosophy of food could best be characterized as • Question Type: 5. In line 29, “expeditious” most nearly means • Question Type: 6. In lines 30-42, the author of Passage 1 rejects the traditional method of baking because • Question Type:

5/23/2011 5:05:26 PM

8.  7. Passage 1 could be best described as • Question Type: 8. The author of Passage 2 would most probably respond to the objections raised in lines 30-32 (Passage 1) by saying • Question Type: 9. Jan-Michel Kenard “expectorated” (line 51, Passage 2) because • Question Type: 10. As indicated in Passage 2, Chef Kenard considered his choice of pastry making a form of • Question Type: 11. On the basis of the information in Passage 2, which statement most accurately describes the reaction of the “restaurant establishment” in lines 12-14 of Passage 1? • Question Type: 12. In line 89, “superfluous” most nearly means • Question Type: 13. Which aspect of the creation of Crème Brule seems to matter a great deal in Passage 1, but not in Passage 2? • Question Type:

8.7.4. Strategy #3: Short Non-Comparative Passages There are four Short Non-Comparative Reading Comprehension questions on the SAT and a lot of students have difficulty with these. This is not surprising: these questions are usually rated by the College Board as among the Hardest of any on the exam. Why is this? Because the passages are short and Reading Comprehension is meant to be hard! In order for the SAT testmakers to feel they’ve created challenging questions—without being able to bury information in 800-word essays—they make the structure of the passages complex, the meanings of the answer choices very similar, and the vocabulary challenging. However, the 8-Step Approach to these questions is simple. Your strategy: 1. Read the first passage through fully. Note that this differs from the approach to Long passages. 2. Find the first Non-General question. This may or may not be the first question of the series. 3. Read the question so you have an idea of what you’re being asked to do. 4. Refer back to the passage directly using the skill of Locating Information (line numbers, order, keywords) to find what you need in order to answer the question.

chap08.indd 8

Part 2    SAT Preparation 5. Return to the question, with your own answer in mind. 6. Read every answer choice. See if you can find the answer choice that Says the Same Thing a Different Way. 7. Use the Process of Elimination to leave only one answer possible. 8. Attempt any remaining General questions. 9. Repeat all steps for the other Short Non-Comparative Passage.

8.7.5. Strategy #4: Short Comparative Passages There are always four Short Comparative Reading Comprehension questions on the SAT and often these are among the most difficult. The passages will always compare different sides of a single issue; sometimes the passages will be in direct opposition, but most of the time they will come at the issue from different angles. For the most part you will see questions that ask you to compare passages, approaches, or authorial perspectives. Usually three of the four questions will be Comparative. Your seven-Step Approach for these questions: 1. Read the first passage fully. 2. Read the second passage fully. 3. Find the first Non-Comparative question. Note: there may not be any. 4. If there is a Non-Comparative question, read the question so you have an idea of what you’re being asked to do. 5. Refer back to the passage directly using the skill of Locating Information (line numbers, order, keywords) to find what you need in order to answer the question. 6. Return to the question, with your own answer in mind. 7. Read every answer choice. See if you can find the answer choice that Says the Same Thing a Different Way. 8. Use the Process of Elimination to leave only one answer possible. 9. Having attempted all the Non-Comparative questions and having read both passages, consider a brief (one or two sentences) summary of the first passage, in the context of the issue being discussed and its approach to the subject matter. Could be simply “pro-” or con-”. 10. Consider a brief summary of the second passage, in the context of the issue being discussed. Note how the second passage differs from the first in its opinion and perspective. 11. Using your summaries—which can be either written down on the paper or held in mind—attempt the Comparative questions in order. The ability to distill and synthesize complicated material is not instinctive. Try the following exercise for practice in this skill.

5/23/2011 5:05:26 PM

Chapter 8    Critical Reading I: Reading Comprehension

8.7.6. Exercise #3: Summarizing Short Comparative Passages Directions. Below are two sets of Short Comparative Passages. Read and write summaries for each passage, noting—in one sentence or less—the central idea and how are they different from one another. Then attempt the questions that follow. Passage 1: Harvard anthropologist Daniel Leiberman was watching a pig run on a treadmill when he experienced his life-changing, career-making epiphany: we were born to run. The evidence is us, the structures of our hips, shoulders and noses, the location of our centers of gravity, even the size of our buttocks. All these adaptation have made us the ideal long-distance runners. This philosophical corollary: a society which has gotten away from exercise and towards obesity has moved away from itself. We shouldn’t run because it’s fun or good for us or healthy. We shouldn’t run because someone tells us to. We should run because quite simply it’s what we were built to do. Passage 2: With the crisis of obesity reaching epidemic levels—nearly 1 billion adults worldwide, all of them at risk for diabetes, cancer, and heart disease—there has been a concerted reaction by the health care establishment to promote awareness of this condition. Treatment options include therapy, hypnosis, and surgery. There are food addiction clinics and the government is considering a ban on partially-hydrogenated oils. Things have gotten serious. But people seem to be forgetting the most important component of a healthy lifestyle: physical activity! If you want to be healthier, go out and run. Put down the Big Mac and consider persistence hunting your food like our ancestors did until 50,000 years ago: run behind an antelope until it falls down from exhaustion. Poke it with a stick. Then dig in. 1. The author of Passage 1 would most likely respond to the first sentence of Passage 2 by (A) demonstrating that obesity is in most cases a product of hormone imbalance (B) arguing that obesity is a product of affluence and rising levels of income (C) suggesting that obesity runs counter to our evolutionary adaptations (D) pointing out that humans must obey a biological imperative to horde food (E) indicating that obesity has always been a problem for the poorest members of society 2. The two passages differ in their views of obesity in that Passage 1 states that obesity (A) has been studied by the most distinguished professors in the world

chap08.indd 9

8. 

(B) is compensated for by various biomechanical adaptations (C) is programmed into the DNA of human beings (D) is a philosophically detestable condition (E) may be a product of a reflexive societal alienation 3. Which generalisation is supported by both passages? (A) Obesity is generally the product of a sedentary lifestyle and a lack of will-power (B) Studying barn-yard animals can tell us a lot about our own physiology (C) In most societies weight is positively correlated with professional success (D) Humans are evolutionarily adapted for long-distance running (E) Exercise can be a pleasant component of a healthy lifestyle Passage 1: How can you tell virtual reality from, well, real reality? This is a question that has interested philosophers for many centuries; Rene Descartes in his Meditations was perhaps the first to kick off the issue by theorizing that it could be the case that an “evil demon” corrupts and controls our senses to the extent that we cannot be sure if what we’re seeing, hearing, feeling, tasting and touching is an accurate reflection of the way things actually are. This topic was recently popularized in the Wachowski brothers’ Matrix trilogy and, strange as it seems, remains a legitimate philosophical question: in the event of a perfect simulation, how can we tell what is, from what isn’t? Passage 2: Second Life is a virtual reality program which allows a user to build or create an “avatar” and interface with other users in the electronic “metaverse” created by Phillip Linden. Second Life residents can own property, get married, and even make money. A recent study revealed that more than half the users of Second Life identify more strongly with the lives of their “avatars” than with their own situations. With people becoming ever more fascinated with virtual reality, and with a burgeoning technology that facilitates that interest, we may be forced to reconsider what had been until now a clear distinction between the “virtual” and the “real”. 1. Which of the following statements best captures the relationship between the two passages? (A) Passage 1 considers the validity of a problem; Passage 2 rebuts with a specific example (B) Passage 1 explores a philosophical question in depth which Passage 2 responds to (C) Passage 1 considers the history of an issue; Passage 2 offers a more modern perspective (D) Passage 1 makes an argument against a point of view, which Passage 2 responds to (E) Passage 1 summarizes a series of events, the legitimacy of which Passage 2 takes issue

5/23/2011 5:05:26 PM

8. 10 2. The author of Passage 1 would most likely regard the findings of the “recent study” in Passage 2 as (A) another example in the pantheon of experimental philosophy (B) an inevitable product of a technologically advanced civilization (C) a regrettable indication of the pervasive escapism of our culture (D) a real-world example of a thought-provoking issue (E) a substantial alteration in the perspective of popular opinion

chap08.indd 10

Part 2    SAT Preparation 3. Unlike the author of Passage 2, the author of Passage 1 makes use of (A) persuasive rhetoric designed to change opinions on a topic (B) historical facts with respects to a technological innovation (C) specific reference to relevant literary and cinematic works (D) extended explanation of a technological development (E) recent studies and news reports to buttress an argument

5/23/2011 5:05:26 PM

Chapter 8    Critical Reading I: Reading Comprehension

8. 11

8.8. Quiz #2: Reading Comprehension I 1. True or False: a viable strategy for Long Reading Comprehension is first to read very carefully the entire passage, and then to start on the questions. 2. What is a Within question? 3. What are some types of Beyond questions? 4. True or False: you should not specially try to attempt every Vocabulary-in-Context question. 5. What does it mean to Say The Same Thing a Different Way?

On the Approach to Long Non-Comparative passages: 6. When should you not answer the first question on a question-series? 7. Approximately how often does this happen?

On the Approach to Long Comparative passages: 8. True or False: you should answer the questions in the order in which they are asked. 9. What is the difference between a Specific question and a Specific Comparison question?

On the Eight-Step Approach to Short Non-Comparative passages: 10. What makes these questions difficult?

chap08.indd 11

5/23/2011 5:05:26 PM

Chapter

9

Critical Reading II: Reading Comprehension

H

aving established the basic approach to questions of Long Reading Comprehension Short Reading Comprehension and Comparative Passages, there are some intermediate skills and strategies that build on these approaches and which can help you engage with the material and eliminate wrong answer choices. These are more advanced than the material in the previous chapter and involve a close scrutiny of the question, passage and answer choices. In this chapter, we will discuss two different methods of cracking questions of Specific and General Interpretation; in doing so we will be shifting focus from the basic approaches to a much more deliberate consideration of what has to go into the thought process for answering a single Specific or General question of Reading Comprehension. We will work on developing skills designed to facilitate the identification and “translation” of material from the passage into correct answer choices, and we will consider some methods of identifying incorrect answer choices, thereby refining the Processes of Elimination as applied to Reading Comprehension. We will finally start to develop a complete approach for everything you must think about when evaluating the aptness of an answer choice, the Ultimate Reading Comprehension Approach.

9.1. Secondary Strategies The Secondary Strategies are slow, designed to unlock your basic processes of Reading Comprehension, tidy them up and put them back together with an improved focus and efficacy (SAT vocab!). These guidelines have a more limited scope than the basic approaches but an increased depth of application. The aim is for you to start to develop a better

chap09.indd 1

idea of what makes a Hard question Hard, while starting to assemble a mechanism for cracking those Hard questions by focusing on three components: the difficulty or incomprehensibility of the question itself, the synthesis and comprehension of the relevant material in the passage in terms of vocabulary, wording and structure, and the differentiation between two or three similar-sounding answer choices. The latter skill will be your primary defence tool in considering Hard questions but all three elements are important. The ultimate goal is to make it more difficult to get questions wrong.

9.1.1. Secondary Strategy #1: the Process of Elimination With questions of General and Specific Interpretation, you are always searching for the Best Fit answer choice that combines what the question is asking with what the passage is saying; Locating Information and Saying a Different Way is a good way to do this. Remember: one of the reasons Hard questions are Hard is because wrong answer choices sound right; that is the nature of Reading Comprehension. After all, just think: everything you need to know is right in the passage; this section would be the easiest on the SAT if the questions were straight-forward and the answer choices unambiguous. But they’re not! With Hard questions you can count on having a tough choice between two or three very similar-sounding answer choices. Thus, it becomes even more important to use the Process of Elimination effectively to be able to choose between those similar sounding answer choices. Example:

5/23/2011 5:05:38 PM

9. 

Part 2    SAT Preparation

As a result of this consolidation, the major fashion designers and magazine editors have effectively hijacked American consumer culture, becoming the ultimate arbiters of taste in virtually every dimension. 1. The author mentions that American consumer culture has been “effectively hijacked” in order to

1. The author uses the word “stimulating” to suggest (A) a less-successful artist can produce work that is more artistically valid than an art celebrity (B) unrecognized artists can serve as catalysts of creative development (A)

(B)

Passage

OK?

(A) make a point about a controlling body (B) suggest the nation’s mainstream preferences are being skewed (A), (C) and (E) are all off base; (A) because there is no mention of consumer advocacy groups, (C) because while the excerpt does suggest a conspiracy, it specifically does not state that the conspiracy might have an entrepreneurial motivation, and (E) because nothing in this excerpt has or could have proved anything conclusively. So what’s the difference between (B) and (D)? (D) hints at the idea that the “fashion designers and magazine editors” are skewing the nation’s preferences; the passage does not contain that particular negative connotation. Check the following for a visual: (B) (D) “make a point” “suggest” “a controlling body”

Passage OK? “as a result of this” (B), (D) “ultimate arbiters” (B)

“preferences are “taste in virtually NOT being skewed” every dimension” (D)

The advent of a technological singularity—that is, the period of dramatic and uncontrolled growth in technological progress—would have the effect of destabilising our guesses about the immediate and medium-term future by introducing into our set of preconceptions a host of novel and previously-impossible objects.

2. The author suggests that a technological singularity would be destabilising because (A) it would provide a new basis for scientific prediction (B) it would undermine a great deal of what we thought we knew (A)

(B)

Passage

OK?

(D) cannot be right because of the element of judgement which is not reflected in the passage. Try the following exercise for practice choosing between similar answer choices.

9.1.2. Exercise #4: Choosing between Hard Answer Choices Directions: Below are a series of Hard-rated Reading Comprehension questions and relevant excerpted passages. All the most obvious answer choices have been eliminated. Select the correct answer choice and describe the reason for that selection using the charts below. Despite the clear lack of artistic integrity, big-name draws like Damien Hirst are still invited to headline at major shows and openings while younger and less visible stimulating talents get marginalised by the critics, the press, and the remainder of the art community.

chap09.indd 2

The period between wars was, indeed, a fertile one for British poetry and prose. Evelyn Waugh, Graham Greene, John Betjeman, Stephen Spender, Anthony Powell—these great men of letters all went to school or university together. In fact, while a student at Eton, a young George Orwell was taught French by Aldous Huxley; that they both went on to write great works of eerie dystopian fiction strikes some as a coincidence as chilling as something you might find in their books.

3. The author suggests the relationship between George Orwell and Aldous Huxley is

5/23/2011 5:05:39 PM

9. 

Chapter 9    Critical Reading II: Reading Comprehension (A) a formative event representative of the strangeness of their adult works of fiction (B) a happenstance that evokes sensations that might be found in their literary work (A)

(B)

Passage

OK?

WORK [element of passage (line number)] A question usually contains three elements: work, passage and reference. The work is always a W-word: Why, What, or HoW and its inclusion turns the Reading Comprehension non-question into something more understandable. Once you’ve analysed the question, you know two things:

• • The difference, purists maintain, is one of practicality: weight-lifters train their bodies to be strong, to be hardy, to be durable; body-builders, on the other hand, chase perfection in symmetry, line and definition. The weightlifter could survive and possibly flourish in a Pleistocene forest; a body-builder would be knocked out by dehydration and joint-fatigue within a day.

What You Need to Know About: What’s the element of the passage you need to understand? Where is it located? What You Need to Do with It: What’s the work? Generally, questions that mention “the author” will go BEYOND the passage and will be Why or How questions; questions that stick WITHIN (ending with “because”, “suggests that”, or “refers to”) the passage will be What questions.

You can use this information to turn the SAT’s incomplete sentence into a proper question. People think in questions; if you can rephrase the question you’re given, you will have a better idea of what you’re looking for in the passage. Examples: i. The description of Mighty Joe Young in the passage primarily serves to

4. The author refers to a “Pleistocene forest” as a (A) determining situation for the viability of an athletic endeavor (B) testing ground for the evaluation of two differing physical enterprises (A)

(B)

Passage

OK?

9.1.3. Secondary Strategy #2: Two Questions Cracking a tough Reading Comprehension question will always require you to consider Two Questions: what is the question asking, and what is the passage saying? 1. What is the question asking? Your job is to turn the SAT Reading Comprehension question (which is usually an incomplete statement) into a question you understand by re-arranging the components of the question into the following framework:

chap09.indd 3

• • • •

Analysis: WHAT [description of Mighty Joe Young (passage)] What You Need to Know About: the description of Mighty Joe Young What You Need to Do with It: understand what it serves to do What the Question is Asking: what does the description of Mighty Joe Young serve to do?

ii. The author uses the terms “monks” and “beasts” (line 51) in order to • Analysis: WHY [use of “monks” and “beasts” (line 51)] • What You Need to Know About: the terms “monks” and “beasts” • What You Need to Do with It: understand why the author uses them • What the Question is Asking: why does the author use the terms “monks” and “beasts”? iii. Henry’s comment to Margaret in lines 58-69 emphasises his • Analysis: WHAT [Henry’s comment to Margaret (lines 58-59)] • What You Need to Know About: Henry’s comment to Margaret

5/23/2011 5:05:39 PM

9. 

• •

Part 2    SAT Preparation What You Need to Do with It: understand what it emphasizes What the Question is Asking: what does Henry’s comment to Margaret emphasise?

Your job here is to understand how to chop and change pieces of the question not only without losing any meaning, but actually adding functionality by turning the SAT non-question into a real question with a work-word and a question mark. But remember this: WHAT → HOW/WHY What always comes before Why or How; with questions of Reading Comprehension, you have to go through the What to get to the Why. So in example (ii) listed above, you’d first need to know what “monks” and “beasts” refer to— “what do these terms refer to?” —before you could start to crack why the author might have mentioned them. In order to get Beyond the passage, you first have to figure out what’s happening Within it. Try the following exercise for more practice with SAT question manipulation. Once you determine what the question is asking, you will in every instance be directed back to the passage. Your job, therefore, having identified What You Need to Know About and What You Need to Do with It, is to find out what the passage is saying. This implies the second Question, which is: 2. What is the passage saying? What is the thrust or general message of this particular part of the passage? How does it link with elements of the passage that come immediately before and after it? How does it fit in with the author’s overall position up until that point? The latter point is possible the most significant: you need to understand how the information you Locate using line numbers fits with the passage’s central argument; you must be aware of things like: • Counter Argument: Is the author building up an argument in order to knock it down? This is the most common kind of passage-based confusion. It can be easy to think that just because the author mentions something, he endorses it. Note: if the author is developing a counter-argument, then any Why/How questions about that area of the passage which ask what the author thinks will not at that point be reflected in what is said in the passage (because what will be said in the passage will be the opposite of what the author is actually trying to do). Counter arguments can be noticed by prompts such as:

chap09.indd 4





• •

“Their argument goes like this…” “Some would have you believe…” Types of Argument: Is the author presenting a single argument, or several smaller arguments? Is there a logical connection between the elements of the passage? Is the author supporting his assertions with evidence, or rhetoric? Is this an anecdote in the first-personal voice or a scholarly treatment of a sophisticated topic? Flow of Passage: Does the passage start with one topic and change to another aspect half-way? If so, what changes? Is there an introduction and then a segue into a seemingly different topic? (hint: this happens a lot)

If you do not understand What You Need to Know About and What You Need to Do with It when you refer to the passage, you will not be able to appreciate what the passage is saying; if you do not grasp what the passage is saying, you’ll never be able to link it to what the question is asking. In this way Reading Comprehension actually represents a delicate balancing act in which a clear understanding of the passage and question—and a vague understanding of what the answer could be based on the conjunction (SAT vocab!) of the passage and question in your own words—are among the best intermediate tools you have for dealing with Hard questions of Reading Comprehension. The hard part of Reading Comprehension is almost always the passage. The development of your intermediate vocabulary (see 6.1) will help with comprehension, but mostly it is just important to practice reading challenging passages. Try the following exercise to develop your sensitivity to What the Passage is Saying. Once you match up What the Question is Asking with What the Passage is Saying, you ought to formulate your own answer to the question. Note that this answer does not need to be in sentences: a feeling will suffice (SAT vocab!). Once you have formulated your Own Answer, you should cross-reference that against the answer choices as given. Almost all the time, more than one answer choice will sound correct; your job is to stick to your guns, ignore the correct-sounding-but-incorrect answer choices, and pick the answer most similar to what you came up with that Says the Same Thing a Different Way.

9.1.4. Exercise #5: What the Question is Asking Directions: below is a list of real Reading Comprehension questions. Your job is to Analyze the question into the WORK [element of passage (line number)], determine What You Need to Know About, state What You Need to Do with It, and state What the Question is Asking.

5/23/2011 5:05:39 PM

Chapter 9    Critical Reading II: Reading Comprehension 1. The reference to “the pedigree dogs” in line 22 is used to: a. Analysis: WHAT [pedigree dogs (line 22)] b. What You Need to Know About: the reference to “the pedigree dogs” c. What You Need to Do with It: understand how it is used d. What the Question is Asking: what is the reference to “the pedigree dogs” used to do? 2. In line 14, the reference to Seneca is used to emphasize the a. Analysis: b. What You Need to Know About: c. What You Need to Do with It: d. What the Question is Asking: 3. The phrase “unintentional hero” in line 78 refers to a. Analysis: b. What You Need to Know About: c. What You Need to Do with It: d. What the Question is Asking: 4. In lines 69-70, the author portrays scientists as a. Analysis: b. What You Need to Know About: c. What You Need to Do with It: d. What the Question is Asking: 5. In line 10, “zero hour” refers to the a. Analysis: b. What You Need to Know About: c. What You Need to Do with It: d. What the Question is Asking: 6. The author mentions “the perfidy of the masses” in line 18 in order to a. Analysis: b. What You Need to Know About: c. What You Need to Do with It: d. What the Question is Asking: 7. Fergus denies the implication that he is “sensitive” (line 44) because a. Analysis: b. What You Need to Know About: c. What You Need to Do with It: d. What the Question is Asking: 8. In the context of the second paragraph, the “vestibule” is best characterised as a. Analysis: b. What You Need to Know About: c. What You Need to Do with It: d. What the Question is Asking:

chap09.indd 5

9. 

9.1.5. Exercise #6: What is the Passage Saying? Directions. Read the following passage, excerpted from the preface of Five Hundred Mistakes of Daily Occurrence in Speaking, Pronouncing, and Writing the English Language, (1856). Then attend to the quoted lines below it, with each providing not only a summary but also a brief explanation how it relates to the passage as a whole (until that point) and whatever comes immediately before it. This book is offered to the public, not to be classed with elaborate or learned works, nor expected, like some of its more pretending companions among the offspring of the press, to run the gauntlet of literary criticism. It was prepared to meet the wants of persons—numbered by multitudes in even the most intelligent and refined communities—who from deficiency of education, or from carelessness of manner, are in the habit of misusing many of the most common words of the English language, distorting its grammatical forms, destroying its beauty, and corrupting its purity. The most thorough mode that could be adopted to correct such errors, would doubtless be to impart to the ignorant a practical knowledge of the principles of language, as embodied in treatises on grammar; but such a good work, however desirable its results, has, in time past, been too difficult for the promoters of education to complete, and is still too great to give promise of speedy accomplishment. A better expedient, bearing immediate fruits, has been adopted in the present volume, which, while it does not aim to produce a radical reform, cannot fail to render great service to those who need to improve their usual modes of expression, and to be more discriminating in their choice of words. The more frequent and less excusable mistakes that may be noticed in ordinary conversation or correspondence, are here taken up, one by one—exposed, explained, and corrected. They consist variously of abuses of grammar, misapplications of words and phrases, improprieties of metaphor and comparison, misstatements of meaning and faults of pronunciation. They are grouped miscellaneously, without classification, not so much because of the difficulty of devising an arrangement that would be systematic and intelligible, as from the evident fact that a division of subjects would render no assistance to those for whom the book is specially designed; for an appropriate classification would necessarily derive its features from the forms of grammar, and with these the readers of this book are supposed to be to a great extent unfamiliar. The volume is put forth with no flourish of trumpets, and makes no extravagant pretensions; yet the publishers believe it will be regarded as a timely and useful work. If the race of critics should not like it—and while books have their “faults,”

5/23/2011 5:05:39 PM

9.  critics have their “failings” —they are reminded that he who corrects an old error, may render no less service to his brethren, than he who discovers a new truth. If the work shall be the means of saving one sensitive man from a confusion of blushes, in the presence of a company before which he desired to preserve his equanimity, it will not have gone forth without a mission of benefit, which will merit at least one acknowledgment. “The most thorough mode”: “A better expedient, bearing immediate fruits”:

Part 2    SAT Preparation that repeat words and phrases from the passage! These are distracters, and should be ignored. Try to find the answer choice that Says the Same Thing a Different Way. 6. Use PoE: Use the Process of Elimination to disregard any answer choice that does not match what you came up with for your own answer. But: what is the process by which you can eliminate answer choices? On the grounds of Disqualifying Information. What is this? Disqualifying Information is anything contained in an answer choice that prevents it from being correct.

“those for whom the book is specially designed”: “the race of critics”: “one sensitive man”:

9.1.6. Secondary Strategy #3: Approach I It is now possible to consider the beginnings of an intermediate framework for your Specific Approach to questions. For each question you should be running through the following steps in the 64 second allotted: 1. Know what the Question is Asking: Consider the main idea of the question and your specific task: What Do You Need to Know About (element of passage) and What Do You Need to Do with It (i.e., the work)? 2. Locate Information: Use your general Reading Comprehension skills to Locate Information by line number, order, or keyword. 3. Know what the Passage is Saying: Read from the previous passage-area through the relevant passage-area. What is the thrust or main idea of this particular section of the passage? How does it link to what comes immediately before and after it? Most importantly: how does it fit with the author’s argument until that point? Watch for counter-arguments and Flow of Passage. 4. Formulate your own answer: Students very often hit upon something very nearly resembling the correct answer immediately, based on the intersection of what the question is asking and what the passage is saying. Even if it’s just a feeling, try to figure something out. 5. Read Every Answer choice: If you have understood the question and passage, your Own Answer will have every chance of resembling the correct answer. Do not allow yourself to be persuaded otherwise by answer choices

chap09.indd 6

9.1.7. Secondary Strategy #4: Identifying Disqualifying Information: Top-Down Method Disqualifying Information is anything that marks an answer choice as incorrect; another way to understand it is as anything in an answer choice that does not fall within the conjunction of what the question is asking and what the passage is saying. The aim is to identify Disqualifying Information in every answer choice is that not correct. But how do you identify it? Two ways: 1. Method #1: Top-Down: Find the Trigger 2. Method #2: Bottom-up: Find Inaccuracy Method #1: Top-Down: Find the Trigger: This method of Identifying Disqualifying Information is the most direct and is effectively an offshoot of Saying the Same Thing a Different Way. What is the Trigger? The Trigger is a word or phrase from the passage that can be Said a Different Way to get the correct answer More than half the questions of Specific Interpretation have some kind of Trigger; these roughly correspond to the Within questions whose work is “what”. All the passage excerpts provided in previous chapters and exercises were Triggers; within each excerpt was a clue that could take you directly to the correct answer choice when Said a Different Way. When Finding the Trigger, all you’re doing is using your Own Answer to find direct textual support for any similar answer choice that might get Said a Different Way (which would make it correct). Example:

5/23/2011 5:05:39 PM

Chapter 9    Critical Reading II: Reading Comprehension

Kevin had a glorious insensitivity to pain. “I don’t not feel it,” he’d sniff; “I just ignore it.” But we all knew he was different.You could see it in the way he ran track, speeding up just as all our bodies began to fail, apparently not cognizant of the burning of legs and lungs that normally attends a maximal effort. Our coach could never fault Kevin for a lack of effort because he won every race, but sometimes it seemed that Kevin would notice our looks and whispers and because of that, he’d maybe grimace and little stagger a bit at the end, just to seem human, like the rest of us. 1. In lines 2-3, the author refers to “the way he ran track” primarily in order to (A) demonstrate a marked suspicion on behalf of Kevin’s teammates (B) hint at the possibilities of foul-play (C) indicate that Kevin is better-developed than most athletes his age (D) suggest Kevin does not experience what most athletes struggle with (E) point out that Kevin is physiologically superior to his teammates So: what is the question asking? The question asks why the author refers to “the way [Kevin] ran track”. What You Need to Know About is how Kevin is described running track, and what You Need to Do with It is figure out why the author mentions this. Now: what is the passage saying? It is saying that Kevin is “apparently not cognizant of the burning of legs and lungs”. What does this mean? It means that in other words, he does not seem to be aware of the usual sensations that accompany hard running: burning legs and lungs. Put together the pieces to formulate your Own Answer: the author refers to “the way [Kevin] ran track” in order to make a point about Kevin being special, or different. Now, is there a similar answer choice that can be Said a Different Way? Answer choice (D) looks likely. The author refers to the way Kevin performed athletically to indicate that he “does not experience (from the passage: “not cognizant”) what most athletes struggle with (from the passage: “burning legs and lungs”). Thus, the likely answer choice is correct, and the answer is (D). The key to finding the Trigger is to ask the following question: Is there an answer choice that resembles your Own Answer that is Said a Different Way in the passage? Also, note that in some instances the Trigger is a hard word.

chap09.indd 7

9. 

This makes sense because the SAT doesn’t intend to make it easy to Say the Same Thing a Different Way. Try another from the same passage: 2. Kevin would apparently “grimace and stagger a little” (line 7) because (A) he wanted people to think he was working harder than he really was (B) he wanted to allay the coach’s suspicions (C) he wanted his teammates to think he was susceptible to the same effects (D) he wanted to put on a show for the crowd (E) he wanted to make his opponents feel better about their losses So: what is the question asking? The question asks why Kevin might “grimace and stagger”. What You Need to Know About is the context of his “grimacing and staggering”, and what you what You Need to Do with It is figure out what is meant to be a result of it. Now: what is the passage saying? The passage says he would notice the whispers of his teammates, and would therefore pretend to be feeling pain. Putting together the pieces, your Own Answer might be that Kevin “grimaces” in order to seem more like his teammates. This resembles answer choice (C); can this answer choice be Said a Different Way in the passage? Yes. The passage tells you that Kevin “just to seem human, like the rest of us.” This is the Trigger. In wanting “to seem human”, Kevin was indirectly expressing a desire to be perceived as similar to his teammates in their responses to hard exercise, or in other words, “susceptible to the same effects”. The Triggering phrase can be Said a Different Way to support answer choice (C). Your ability to find the Triggering word or phrase will be a great asset to you when taking the SAT. If you can use your Own Answer based on the conjunction of what the question is asking and what the passage is saying to find a similar answer choice that is directly supported by some Trigger, then you’ll be in a position to improve your accuracy rate on Reading Comprehension dramatically. Try the following exercise for more practice with finding the Trigger.

9.1.8. Exercise #7: Finding the Trigger Directions. Read the passage below and answer the questions. When determining What the Question is Asking, you should re-phrase the question in your own words. When determining What the Passage is Saying, you should Locate Information and describe the idea that passage is getting across. And when identifying the Trigger, you should quote words from the passage.

5/23/2011 5:05:39 PM

9.  In the introduction to a collection of essays, a well-known journalist explains his acquaintance with the phenomenon of social networking. It was only a year ago that my daughter asked me if I’d ever been friended. “I have been befriended by many,” I quipped; “and I believe I have befriended a few myself.” No, no, she replied patiently. Have I been friended? As in, facebookfriended? I scratched my head and she laughed at me. She was four years old, then. Since then I’ve done my homework and it has not been a particularly pleasant assignment. It turns out that “social networking” websites like MySpace, Bebo and most of all Facebook have become very popular in recent years. Launched in 2004 by Mark Zuckerberg, Facebook started out as a student website at Harvard College; it quickly swept to Princeton, Yale and many other universities in America. Shortly after that, it became international and started including school-aged kids like my daughter and post-college working professionals like myself. Now, anyone can join and the website boasts over 400 million members worldwide. These 400 million are all potential ‘friends’ —a connection is just a click away. And yet, with the rise of Facebook et al and the cultural obsession with social networking sites, it seems like real friendship has gotten harder and harder to find. A friend is a sympathetic companion with whom you have a history of shared experience; he is meant to be quite literally ‘there’ for you. How can someone you’ve never shaken hands with and who lives 2000 miles away be ‘there’ for you in any sense? The proliferation of Facebook and its cyber-cousins has degraded the quality of friendship to the extent that we might start needing another word for it as a means of differentiation from the computer-based variety; “face-to-face” friendship, perhaps, or maybe “real-world” friendship. I understand there to be arguments for the utility of such websites which go like this: if we consider our circumstances to be largely based on luck—where we were born, who our parents are, etc. —then Facebook and its ilk represent a way of escaping our circumstances and radically extending our circle of friends. A friend need no longer be a physical acquaintance and as such it is possible that friendship can now become something purer and more meaningful, based on shared interests and opinions. And yet, it seems in this case that a technological innovation which is meant to link people is actually having the opposite effect, slowly eroding interpersonal bonds and diffusing affections over a huge pool of strangers, making real friendship—with its object of uniting like-minded individuals—harder to achieve than ever. 1. In the context of the passage as a whole, the first paragraph is best described as

chap09.indd 8

Part 2    SAT Preparation (A) a partial transcription of a conversation (B) a vivid description of a surprising event (C) a personal recollection of a moment of discovery (D) a humorous digression that leads to a series of arguments (E) a reluctant admission of a humbling experience • What is this question asking? __________________ • What is the passage saying?____________________ • What is the Trigger? ­­­­­­­­­­­­­__________________________ 2. Lines 7-11 (“Launched in 2004…a click away”) refer to (A) the development of a particular enterprise (B) the stated mission of an organisation (C) the core philosophies of a business venture (D) the history of a technological innovation (E) the conceptual underpinnings of a company • What is this question asking? __________________ • What is the passage saying?____________________ • What is the Trigger? ­­­­­­­­­­­­­__________________________ 3. Lines 13-14 (“a friend is…for you”) are best characterized as (A) the critical reaction to a notion (B) the analysis of a set of observations (C) the accepted etiquette for a situation (D) the definition of a common term (E) the standard response to an action • What is this question asking? __________________ • What is the passage saying?____________________ • What is the Trigger? ­­­­­­­­­­­­­_________________________ 4. The author would most likely respond to the argument presented in lines 19-24 (“if we…interests and opinions”) by suggesting (A) that the product of a remote acquaintanceship would be indistinguishable from the old-fashioned variety (B) that social networking websites have increased the risk of identity theft (C) that the traditions of the past deserve respect and admiration (D) that whatever might result from such a connection could not be labelled friendship (E) that established ideas and conceptions are generally supported by a substantial body of thought • What is this question asking? __________________ • What is the passage saying?____________________ • What is the Trigger? ­­­­­­­­­­­­­_________________________ 5. Which of the following situations most closely resembles the author’s conclusion in lines 23-26 (“And yet…harder to achieve than ever”?

5/23/2011 5:05:39 PM

9. 

Chapter 9    Critical Reading II: Reading Comprehension

• • •

(A) A chain of bookstores begins selling e-novels which are riddled with typographical errors (B) A recently developed interactive shopping website sells poor-quality clothing (C) A new generic brand medication proves to be ineffective (D) A popular distance learning program results in a diminished standard of education (E) A hospital with live satellite up-links to the best medical centers in the world is started in a remote area What is this question asking? __________________ What is the passage saying?___________________ What is the Trigger? ­­­­­­­­­­­­­______________________

9.1.9. Secondary Strategy #5: Identifying Disqualifying Information It would be nice if every question had an obvious Trigger, but this is not the case. Sometimes, instead of Locating Information, formulating your Own Answer based on what the question is asking and passage is saying, determining a similar answer choice and supporting it with a Triggering phrase that gets Said a Different Way, you will have to use the BottomUp Method of Finding Inaccuracy. Using this method, you eliminate answer choices one-by-one based on Disqualifying Information they contain instead of by using a Trigger to select the correct answer in one step. However, the procedure is still the same: you first find out what the question is asking, determining What You Need to Know About and What You Need to Do with It; then you find what the passage is saying, considering the general thrust of the Located Information, how it links to the material directly before it, and how it fits with the author’s general point until then, watching carefully in the latter instance for Counter Arguments. You then formulate your Own Answer and consult the answer choices, looking for any similar options. The primary difference between Finding the Trigger and Finding Inaccuracy is at this point in the process, you will not be able to find a Trigger in the passage, because it won’t exist. This is generally the case for: 1. Comparative Questions: Short or long, there may be various Triggers that inform your decision-making but nothing that leads directly to the correct answer choice. 2. General Questions: The first sentence of a long passage can tip you off as to the primary purpose of the passage, but usually you cannot definitively assess a passage without eliminating all incorrect answer choices. 3. “Beyond” Questions: These are questions for which the work is “why” or “how” and are most often prompted by

chap09.indd 9

“in order to...”. With these, you need to identify what the passage is saying, and you need to go further in analysing it. So what are you looking for when you’re trying to Find Inaccuracy? Pay attention to answer choices that are Inaccurate because they are: • Irrelevant to what the question is asking. Answer choices that are Irrelevant make an inference that could be true but which simply are not of any use to you in terms of answering the question. Be wary of these…irrelevant answer choices can be very attractive because they appeal to subtle jumps in logic and reasoning, jumps that a person makes naturally in everyday conversation but which are not supported by the text. • Unimportant to what the question is asking. An unimportant answer choice might mention something that is briefly mentioned in the passage but which, for the purposes of the question, is not of any use. Again, these answer choices can be attractive because they are very literal, often repeating words from the passage in the hope that the test-taker will fall into the trap. A repeated word or phrase from the passage in an answer choice can usually be considered Disqualifying



Unrelated to what the passage is saying. These answer choices are the easiest to eliminate because they are not related to the passage or the question; you read these and think “huh?”

Finding Inaccuracy takes longer than Finding the Trigger because you have to consider each answer choice carefully, but it works just as well if you’re strict enough. So, once you’ve determined your Own Answer and located similar answer choices, how do you identify the Disqualifying Information in the answer choices? It helps to develop a hypothetical scenario for each answer choice in which you ask yourself: If this answer choice were correct, would the passage have to be different? When asking this question strictly, you are putting yourself in a position to be able to rigorously select or reject answer choices on the basis of what is contained in each answer choice, because you imagine what the passage would have to be like if the answer choice were correct. If the passage would have to be different than the way it currently is to accurately reflect that answer choice, then you know that the answer choice in question cannot be correct.

5/23/2011 5:05:39 PM

9. 10 Every word in an answer choice is selected by the SAT on purpose; there are not accidents or coincidences. Different words that sound similar can have radically different shades of meaning; count on this with the hard questions. Your job, once you’ve asked the question, is to determine those shades of meaning by remembering to: Take every word of the answer choices literally! When building your hypothetical scenario, it is not only important to take every word of the answer choices literally; you also have to have an idea of what the answer choices mean! To this end the development of your intermediate vocabulary becomes extremely important. Also, remember to: Use your imagination in building the hypothetical!

Your job is to think critically about what the passage would have to be like if a certain answer choice were correct, using your imagination and taking every word in the answer choice literally to build a hypothetical situation. Example: In 1896, at the tender age of 14, Pablo Picasso painted The First Communion, a large composition featuring his sister, Lola. He was studying at Spain’s most prestigious art academy by the following year, and inside a decade had caught the eye of Leo and Gertrude Stein, prominent American art collectors. Over the next four decades, he would move through his Blue Period, Rose Period, African-inspired Period, Analytic Cubism and Synthetic Cubism.The latter two styles—involving the two-dimensional “deconstruction” of objects and people—are largely considered to have been invented by Picasso, and represent a revolution in the 20th century visual arts. Now, Picasso’s work is studied around the world, and is in fact the most valuable of all time: in 2004, Garcon a la Pipe sold for 104 million USD.

1. This passage is primarily concerned with (A) explaining Picasso’s interest in Cubism (B) providing a character analysis of Picasso (C) summarising how and why Picasso is celebrated (D) tracing the development of the Cubist art movement (E) encouraging more students of modern painting to attend art school As with Finding the Trigger, Finding Inaccuracy means

chap09.indd 10

Part 2    SAT Preparation you must first understand what the question is asking. The question asks about the primary purpose of the passage: “What is the passage concerned with?” The passage is saying that Pablo Picasso was a child prodigy and international success who revolutionized 20th-century art. This is your Own Answer. Now, there doesn’t seem to be an obvious Trigger, and there probably won’t be: this is a General question. So, rereading the question, you’re in a position to Find Inaccuracy by asking, “If this answer choice were correct, would the passage have to be different?” 1. This passage is primarily concerned with (A) explaining Picasso’s interest in Cubism: in order for this answer choice to be correct the passage would have to go into Picasso’s interest in Cubism, probably stating why and when Picasso first got interested in Cubism, and probably relating what sustained his interest. The passage does not do this; therefore, it would have to be different and this answer choice is incorrect. (B) providing a character analysis of Picasso: in order for this answer choice to be correct, the passage would have to explore Picasso’s personality and the reasons he was the way he was, probably mentioning important events in his childhood and adolescence. The passage does not do this; therefore, it would have to be different and the answer choice is incorrect. (C) summarising how and why Picasso is celebrated: in order for this to be correct, the passage would have to state some of Picasso’s accomplishments and accolades, as well as possibly giving a brief history of his career and influence. The passage seems to do this; therefore, this answer choice might be correct. (D) tracing the development of the Cubist art movement: in order for this to be correct, the passage would have to discuss the origins of Cubism and the changes it has undergone. The passage does not do this; therefore, it would have to be different and the answer choice is incorrect. (E) encouraging more students of modern painting to attend art school: in order for this to be correct, the passage would have to discuss the benefits of formal art training and include a call-to-action for prospective young artists. The passage does not do this; therefore, it would have to be different and the answer choice is incorrect. The only possible answer choice is (C). (C) is correct. Now, try another: 2. The author mentions that Cubism involves “the twodimensional “deconstruction” of objects and people” in order to

5/23/2011 5:05:39 PM

9. 11

Chapter 9    Critical Reading II: Reading Comprehension (A) demystify an artistic riddle (B) clarify an academic issue (C) define an idiom (D) explain a technical style (E) address a common misconception The question is asking you to determine why the author mentions that Cubism involves “the two-dimensional “deconstruction” of objects and people”. You need to know about this “deconstruction”, and you need to figure why it is mentioned; what does it do for the passage? So what is the passage saying? The author mentions “deconstruction” in the context of listing Picasso’s art history; this information is given as a short description of the artistic style known as Cubism. This kind of “in order to…” question featuring short and non-specific answer choices is a great chance to explore the different shades of meaning between similar-sounding words. You need to ask “If this answer choice were correct, would the passage have to be different?” as you take every word of the answer choices literally and use your imagination to build up the hypothetical situations based on these answer choices. This is the process that will make it more difficult for you to get these kinds of questions wrong: 1. The author mentions that Cubism involves “the twodimensional “deconstruction” of objects and people” in order to (A) Demystify an artistic riddle: The effect of the mention of “deconstruction” might well be de-mystifying, but it is not addressing a riddle (though Cubism might seem riddling, but that is Irrelevant). Cubism is a style, not an enigma, and the passage does not discuss its demystification. Therefore, this answer choice is not correct. (B) Clarify an academic issue: The field of Cubism is not an “academic issue”; it is an artistic movement that is characterized by certain features. While people might study Cubism and there might be related academic issues pertaining to Cubism (again, Irrelevant), Cubism itself is not one. Therefore, this answer choice is not correct. (C) Define an idiom: The mention of “deconstruction” serves as a description or clarification—not a definition. A definition provides the meaning of a term; all the quote does is suggest a prime feature of Cubism. Additionally, an “idiom” is a commonlyused term or phrase, usually slang; Cubism refers to a style of painting and sculpting. Therefore, this answer choice is not correct. (D) Explain a technical style: The mention of “deconstruction” does, indeed, attempt to explain the essential characteristics of Cubism; and to call Cubism a technical style is accurate—it is a technique

chap09.indd 11

of painting used to express certain ideas. Therefore, this answer choice might be correct. (E) Address a common misconception: While there may be many misconceptions about Cubism, Cubism itself is not a misconception. Therefore, this answer choice is not correct. Try the following exercise for more practice with Finding Inaccuracy using the bottom-up method of Identifying Disqualifying Information.

9.1.10. Exercise #8: Finding Inaccuracy Directions: Read the following passage and answer the questions based on what it stated or implied in the passage. Clearly state your reasons for not choosing the incorrect answer choices, using the question “If this answer choice were correct, how would the passage have to be different?” and taking every word of the answer choices literally. I had never before attended the races. “The sport of kings” is not popular in Boston, my former home, but here in Chicago every one turns out on Derby Day, if at no other time. And so, catching something of the general enthusiasm, my friend Murray Jameson, who by the way is something of a sport, and I, who by the same token am not, found ourselves driving a very smart trap out Michigan avenue, amidst a throng of coaches, cabs, breaks and buggies, people and conveyances of every description—beautiful women beautifully costumed, young men, business men, toughs and wantons—all on their way to Washington Park, and all in a fever of excitement over the big race to be run that afternoon—the great American Derby. 1. This passage is best described as (A) a detailed summary of events: (B) a recollection of a novel experience: (C) a series of logical arguments: (D) a description of an inexplicable occurrence: (E) a comparison between unlike objects: 2. The narrator compares himself to Murray Jameson in order to (A) shed light on a disreputable characteristic: (B) demonstrate the perils of gambling addiction: (C) imply a deep and abiding like-mindedness: (D) indicate the possibility of friendship despite differing perspectives: (E) make a point about a personal tendency: 3. The reference of “people and conveyances of every description” serves to

5/23/2011 5:05:39 PM

9. 12 (A) underscore the nation’s passion for athletic events: (B) highlight the appeal for the children: (C) demonstrate the economic diversity of racing fans: (D) emphasize the popularity of an event: (E) imply the possibilities of theft and pick-pocketing:

9.1.11. Secondary Strategy #6: Find the Trigger before You Find Inaccuracy The material presented in this chapter shows how to crack a single hypothetical Specific or General Reading Comprehension question. Remember that Finding The Trigger tends to work better with “What” questions—if you can Locate Information, generate your Own Answer, and find a similar answer choice for which a direct word or phrase from the passage Says The Same Thing a Different Way, then you’ve cracked it. Likewise, Finding Inaccuracy tends to work better with General, Comparison, or “Why” questions, for which you take every word of the answer choices literally and develop a hypothetical situation by imaginatively asking yourself “If this answer choice were true, how would the passage have to be different?”. However, you should always try to Find the Trigger first with every question, regardless of which type it is; likewise, you should always resort to trying to Finding Inaccuracy if you cannot determine a Triggering word or phrase.

9.1.12. Secondary Strategy #7: Short Stories are better than Long Stories There can be a general tendency when Finding a Trigger or Finding Inaccuracy for people to convince themselves that an answer choice is correct, even when that involves telling themselves a very long story to make it fit. Example: The art of angling is ancient and venerable. Properly speaking, the term refers to fishing with an angle, or hook; the angle can be attached to a line and pole, but needn’t necessarily be.When you consider the varieties of rods, reels, hooks, baits, casting techniques, and geographical influences, it is easy to see how sophisticated angling can be. 1. The primary purpose of the passage is to (A) explain recent developments in sports technology (B) discuss the features of a pastime that provoke interest (C) explore the particularities of a popular hobby (D) reflect upon the motivations of passionate anglers (E) demonstrate the worthiness of an endeavour

chap09.indd 12

Part 2    SAT Preparation This question is asking you to determine the primary purpose of the passage. Using the processes of Finding Inaccuracy, you should be able to eliminate answer choices (A), (D), and (E) quickly. If you’re choosing between (B) and (C), a mistake would be to think this: “the passage mentions a bunch of different components (or “features”) of fishing (or “a pastime”), and people probably are interested in these elements, if they’re interested in fishing, so the passage must be discussing features of a pastime that make it popular. Therefore, (D).” You’re telling a Long Story! If you’d stopped at, “the passage mentions a bunch of different components (or “particularities”) of fishing (or “a popular hobby”)”, then Saying the Same Thing a Different Way, you’d get (C). Stick with the Short Story; it will always serve you better.

9.1.13. Secondary Strategy #8: (A) → (B); (A) A component of telling yourself a Long Story is often going through one answer choice to get to another. In the example above, someone who tells themselves a Long Story to get to answer choice (B) (“discuss the features of a pastime that provoke interest”) must go through answer choice (C) (“explore the particularities of a popular hobby”) in order to get there. In this and in all cases, when you have to use one answer choice to arrive at another, the answer choice you use will be a better selection.

9.1.14. Secondary Strategy #9: Literal vs Liberal Many students when faced with a difficult question of Reading Comprehension convince themselves that a word in the answer choice means something it does not. This will never be a good idea. It is usually OK if you can’t crack a tough word in the passage, because you can use the context in order to figure it out; you’re always looking for the main idea of the passage and you don’t normally want to get hung up on details. But in the answer choice, the reverse is true! Don’t consider any element of any answer choice unimportant or “fudgable”; while you can sometimes be liberal in your reading of passages, you always have to be literal when reading an answer choice. If intermediate-level vocabulary is an issue for you, then you will have a hard time doing this. Consult 6.1. for more practice with medium and medium-hard words that appears with frequency on the SAT.

9.1.15. Secondary Strategy #10: Approach II Combining the strategies of Finding the Trigger and Finding Inaccuracy (or, the top-down and bottom-up methods of Identifying Disqualifying Information), you are now in a

5/23/2011 5:05:40 PM

Chapter 9    Critical Reading II: Reading Comprehension position to consider a more complete approach to a question of General or Specific Interpretation: 1. Think: What is the question asking? (What Do You Need to Know About and What Do You Need to Do with It?) 2. Locate Information in passage using Line Numbers, Keywords, or the Order. 3. Think: What is the passage saying? (How does the element of the passage relate to what comes directly before it? How does it fit with the author’s main point thus far? Are there any Counter Arguments?) 4. Formulate your Own Answer based on the conjunction of what the questions asks and what the passage says.

chap09.indd 13

9. 13

5. Consult the answer choices and determine any that resemble your Own Answer. 6. Return to the passage and try to find textual support that can be Said a Different Way to translate directly into your similar answer choice. This support will be the Trigger. 7. No Trigger? Try to find Inaccuracy in the answer choices by asking the question “If this answer choice were true, would the passage have to be different?” to build a hypothetical situation in which you can compare the passage as it is to the passage as it would have to be, using your imagination and taking every single word of the answer choice literally in building the latter construction. 8. Use PoE to eliminate every answer choice but one.

5/23/2011 5:05:40 PM

9. 14

Part 2    SAT Preparation

9.2. Quiz #3: Reading Comprehension II 1. True or False: It is easier to think in questions than in incomplete statements

2. Rephrase this question: “The author mentions “conquistadores” in line 38 in order to”

3. What are three things to watch for when determining What the Passage is Saying?

4. What is Disqualifying Information?

5. What is a Trigger?

6. What is the connection between the Trigger and Saying the Same Thing a Different Way?

7. What is the question you ask to Find Inaccuracy?

8. True or False: it is not important to take every words of the answer choices literally; some are inserted for fun or by accident

9. What is better than a Long Story?

10. Explain (A) → (B); (A).

chap09.indd 14

5/23/2011 5:05:40 PM

Chapter

10

Critical Reading III: Reading Comprehension

I

n the previous chapter, we explored the beginnings of an ultimate approach to cracking a single Specific or General question of Reading Comprehension. We introduced the concept of Disqualifying Information and considered two different ways to Identify Disqualifying Information: from the Top-Down and from the Bottom-Up. These strategies represent the foundation of your advanced Reading Comprehension processes and are built on the intermediate processes of knowing what the question asks and what the passages says, which is in turn built on the basic processes of effective approaches to the different kinds of Reading Comprehension passages. In this chapter, we will continue to explore the same concept, but from a different perspective; here we are interested in developing the idea of what Disqualifying Information looks like: pinning down the words (verbs, adjective and nouns) that might indicate an answer choice that either contains potentially Disqualifying Information or, more importantly, lacks potentially Disqualifying Information. This element of Reading Comprehension  known as Disqualifying Language  represents the highest level of sophistication in terms of the application of skill and strategy. If you get good at thinking in terms of Disqualifying Language, then you put yourself in a position to crack Hard questions without even looking at the passage. We will, in addition to developing the skills and associated strategies of Disqualifying Language, look at a specific Tone, Style and Attitude procedural strategy and then attempt to ultimately put together your final Approach for a question of Reading Comprehension.

chap10.indd 1

10.1. More Secondary Strategies In theory, every question of General or Specific Interpretation can be cracked because of Disqualifying Information, either from the Top-Down by Finding the Trigger or from the Bottom-Up by Finding Inaccuracy. There is, however, a third way to consider Hard Reading Comprehension questions and that involves recognizing Disqualifying Language. This is a skill that will be important in coming to understand what an answer choice that lacks potentially Disqualifying Information (and which is probably a good answer choice) looks like. So how can you tell what it looks like? There is a chain of reasoning which is helpful here. Consider the following definition: Definition: Disqualifying Language indicates the presence of potentially Disqualifying Information Of course, it is impossible to tell exactly which information will be Disqualifying without the benefit of a passage in which to Identify Disqualifying Information using either the Top-Down or Bottom-Up methods, but Disqualifying Language can serve as a useful jumping-off point for these processes. So: how do you identify Disqualifying Language? Consider this identity: Identity: The answer choice that is the most right is the answer choice that is the least wrong

5/23/2011 5:03:50 PM

10.  But what does the “least wrong” answer choice look like? Deduction: The answer choice that has the least chance of being wrong will contain the least potentially Disqualifying Information But this begets another question: How can you tell which answer choice contains the least possible Disqualifying Information? Deduction: The answer choice that contains the least potentially Disqualifying Information when it contains the least information altogether So how do you tell which answer choices contain the least information altogether? You ask a question: The Question: Which answer choice could apply to the most possible situations and still not be incorrect? That’s what you’re looking for: answer choices with language that makes it so they can apply to the most possible situations. Example thought process: if your selected answer choice mentions chemistry, the passage better be about chemistry; if the passage is not about chemistry, then your answer choice is probably wrong. If your answer choice mentions science, then the passage can still be about chemistry and your answer choice will probably not be wrong; but the passage could be about physics, biology, etc (with the answer choice still not being wrong). If your answer choice mentions specialized knowledge, then the passage could talk about any science or other academic field and still not be wrong. See below for the visual expression of this reasoning: Here are three things to remember:

Part 2    SAT Preparation potentially Disqualifying Informationis not as good as being a good answer choice. So remember: try to find actually Disqualifying Information before you try to find potentially Disqualifying Information.

10.1.2. Secondary Strategy #12: “In Order To…” Disqualifying Language will most frequently crop up when you’re dealing with questions that end with • “in order to…” • “is used to…” • “primary purpose is to…” These are Why questions that ask after either the author’s intent or the effect of a quote or device. Other likely offenders are General questions and Comparison questions; Disqualifying Language comes into play when you have to go Beyond the passage.

10.1.3. Secondary Strategy #13: Say Something Saying Nothing The College Board is very good at creating correct answer choices that say something while saying nothingnothing too strong, nothing too specific, nothing too enduring. This fuzziness can be hard to recognize and, indeed, can actually be off-putting when considering correct answer choices; answer choices that Say Something Saying Nothing will not seem like they have very much to do with the passage as written: it will seem like they could apply to virtually any situation and still not be incorrect. But this, of course, is exactly what you have to train yourself to identify! Try the following exercise to develop your ability to Say Something Saying Nothing.

10.1.4. Exercise #9: Saying Something Saying Nothing Direction: below are a series of incomplete specific → less specific → least specific triples. The aim here is to develop your capacity to identify general words and phrases that can encompass other, more specific terms. Using the word bank at the bottom of the page, complete the groupings. Example: Television → Technology → Development _____

10.1.1. Secondary Strategy #11: Disqualifying Information before Disqualifying Language It will be a useful skill for you to know what a good answer choice looks like. And yet, looking like a good answer choicethat is, an answer choice that contains very little

chap10.indd 2

1. 2. 3. 4. 5.

Symphony → Classical Music → _________ _ → Left-wing ideology → ___________ Brain tissue → _ → _ ___ __ → → Consortium _ __ → _ Powerful Memory ____

5/23/2011 5:03:50 PM

Chapter 10    Critical Reading III: Reading Comprehension

Non-profit

Body Part

Political System

Communism

Organization

Organic Material Artistic Expression

Frightening Moment

Particular Experience

10.1.5. Secondary Strategy #14: The Laser Effect To grasp the effect of potentially Disqualifying Information, it helps to think of a laser. A laser is a tightly focused beam…but in order for a laser to hit its target it must be totally accurate. The more narrow the focus, the less likely it is to achieve its bull’s-eye. SAT questions are the same: the more specifically they describe an element of the passage, the more accurate they have to be in order not to be wrong. The right answer for many of these “in order to…” questions will be the one that says the leastwith the leastfocused laser. So what should you look for when trying to identify answer choices that do not contain Disqualifying Language? That is to say, what do answer choices that contain the least potentially Disqualifying Information look like? You’re after Non-Specified Nouns, Neutral Verbs and Adjectives and Deliberately Language.

10.1.6. Secondary Strategy #15: Non-Specified Nouns An answer choice uses Non-Specified Nouns when it mentions a thing without saying exactly what that thing is. Vague enough? This is the most significant kind of Disqualifying Language because while answer choices occasionally lack verbs and adjectives (though rarely will an answer choice lack both a verb and an adjective), there are always nouns involved. However, this is also the king of Disqualifying Language that is the hardest to identify. The rule: the less specified a noun, the less chance of that specification being inaccurate; thus, the greater chance of that answer choice being the correct answer choice. The ideal answer choice is the one that contains absolutely no information at all: there would be zero chance of it containing anything Disqualifying! See below for some common NonSpecified Nouns so you start to develop an idea of what an answer choice that lacks potentially Disqualifying Information looks like:

chap10.indd 3

10. 

condition, point of view, perspective, instance, viewpoint, approach, assertion, way, extent, individual, experience, practice, development, phenomenon When you see any of those words in an answer choice, it ought to be a clue as to a lack of potentially Disqualifying Information. Examples: 1. This passage is best described as: (A) a reflection on an individual’s actions in the past (B) an analysis of Keynes’s themes as they relate to economics (C) a criticism of primary education (D) a discussion of the theme of youthful abandon (E) an argument for pursuing a career as a professor of economics The big tip-off in this question is the use of the purposefully vague “an individual”. Which individual? And, furthermore, which “actions”? Another hint is the use of the word “reflection.” To reflect is very different than to analyze, criticize or argue; there can be many kinds of reflection that aren’t necessarily positive or negative. Thus, it is simple to deduce that every other answer choice is specifically saying more than answer choice (A); the others also refer specific information that must be mentioned in the passage in order for any of those answer choices to be correct (Keynes’s themes, education, youthful abandon, pursuing a career as a professor). For this reason, it is possible to identify answer choice (A) as a likely answer before you even read the passage. 2. In lines 53-57, the author primarily suggests that (A) the cost of a particular practice is too high (B) gerontology depletes resources that could be more productively used (C) people have frequently lamented their alienation from the natural world (D) new buildings should be designed with increased focus on aesthetics (E) utilitarian aspects of a construction should be factored into its blueprint The clue here: “a particular practice”. Which practice? The other answer choices specifically mention some subjects: the depletion of resources, the alienation from nature, design with an insufficient emphasis on aesthetics, non-utilitarian buildings. Answer choice (A) does not limit itself by making specifications t instead says “a particular practice” which, if you think about it, is the same as the subjects in answer choices (B), (C), (D) and (E)…those are all particular practices! By not stating which practice, (A) contains very little information which is likely to be potentially Disqualifying.

5/23/2011 5:03:50 PM

10.  3. The speculation about “past times” primarily implies that (A) an engineering breakthrough has had unfortunate consequences (B) a recording device has lowered the quality of mainstream news media reporting (C) a scientific development has made a certain experience more regular (D) a new piece of equipment has made television more accessible (E) an expensive technology has altered the public’s tastes This question is tricky because none of the answer choices say very muchbut (C) says the least by far. “A scientific development” this gives us the kind of development but not the specific development itself; is it a cure for a disease? A new technology? The discovery of a new element? It could be anything. We are also told that “a certain experience” has been made more regular. Which kind of experience? Describing an experience as “certain” literally discloses no information at all. By not stating which kind of experience, the answer choice says very little. The other answer choices disclose more information and thus are more likely to contain potentially Disqualifying Information and thus are less likely to be correct.

10.1.7. Secondary Strategy #16: Neutral Verbs and Adjectives Almost every answer choice has at least one a noun; sometimes these nouns are accompanied by verbs and adjectives. What you’re looking for when this is case are Neutral Verbs and Neutral Adjectives that reveal as little specific information as possible. This is the second kind of Disqualifying Language. Non-Neutral Verbs can be separated into two categories: + Verbs and –Verbs. +/- Verbs usually show up as the first word of an answer choice; the best thing to do with these is to Compare the Starting Verbs to see whether or not one is much more neutral and less limiting than the others. The neutral verb will probably be an answer choice that you want to consider seriously. Example: 1. The author’s list in lines 67-68 is meant to (A) illustrate a variety of perspectives (B) support an alternative teaching method (C) affirm the imaginative tendencies of children (D) encourage increased open-mindedness (E) disparage the concept of individual reality The key is in the first word: “Illustrate”, as opposed to “Support”, “Affirm”, “Encourage” and “Disparage”. All but “Illustrate” are +/- Verbs…they have positive and negative con-

chap10.indd 4

Part 2    SAT Preparation notations. Illustrate, however, is neutral. With questions like this, more often than not you can simply compare the first words and have a good idea right away about which answer choice might contain potentially Disqualifying Information. If an answer choice starts with “affirm” or “encourage” or “support”, then immediately you know that one-half of the available situations (the negative half ) have been rejected; those answer choices can no longer pertain to positive situations and still be right. Likewise, with “disparage” you know that What You Need to Know About must be negative or else incorrect. See below for a listing of common Neutral Verbs so you can further develop your idea of what a viable answer choice looks like. lend, suggest, illustrate, make, evoke, demonstrate, indicate, show, draw, place, express, highlight, emphasize, underscore Also: not only + Verbs and –Verbs can be tricky; “to prove” and “to define” are themselves specific without being either positive or negative. All the aforementioned verbs, however  lend, suggest, etc are safe bets. Sometimes, you will not be able to find an outlying NonNeutral starting verb. In that case, you will want to take a good look at the adjectives used to describe the nouns in the answer choice. Example: 4. The primary purpose of the passage is to: (A) challenge a traditional ethical position (B) describe a complex mechanical process (C) examine an childish impulse (D) urge a radical course of action (E) make a point about a particular practice Here, you are given five different adjectives: a traditional ethical position, a complex mechanical process, a childish impulse, a radical course of action and a particular practice. Which is the least qualified? Which is the least specified and therefore the most likely to be correct? (D) is possible, except that we know that any course of action that fits would have to be radical. (A), (B) and (C) are in this case too qualified. If you thought answer choice (E) says nothing saying something, you’d be right. An adjective is a word used to describe or modify a noun; as such, these can be substantial hints as to the potential for the inclusion of Disqualifying Information. Beware anything overly specific because then the passage is obliged to mention exactly that thing, if the answer choice is to be correct! See ahead for some useful Neutral Adjectives:

5/23/2011 5:03:50 PM

10. 

Chapter 10    Critical Reading III: Reading Comprehension

certain, unique, particular, few, some, distinct, common, specific

10.1.8. Secondary Strategy #17: Deliberately Weak Language The final dimension of recognizing potentially Disqualifying Information is understanding the importance of Deliberately Weak Language. Weak language serves to offer qualification and remove the “sting” or “strength” of a comment or assertion. Imagine you’re telling a friend you don’t like her hat: instead of saying, “I don’t like your hat” and risking the friendship and your physical safety, you might say “It could be that possibly, just maybe, you might want think about perhaps changing your hat.” See the difference? In answer choices try to find qualifications that weaken the strength of a statement. See below for some examples: may have, might be, could be, possibly, perhaps, potentially, maybe, apparently, somewhat, seeming, seemingly

10.1.9. Secondary Strategy #18: The Indefinite Article Effect In language, the ‘indefinite article’ is ‘a’ or ‘an’. Answer choices that include the indefinite article when none of the other answer choices do tend to be correct more often than not  just look at the examples offered so far (“an individual’s actions”, “a particular practice”, “a certain experience”, “a particular practice”). The SAT likes answer choices to be ‘indefinite’! Now, try an exercise designed to help you identifying Disqualifying Language.

10.1.10. Exercise #10: Identifying Disqualifying Language Directions. Attempt the following questions without the aid of a passage. With each, look for the different kinds of Disqualifying Language: Specified Nouns, +/- Verbs and Adjectives, Deliberately Strong Language. With each question, try to identify the answer choice that could pertain to the most possible situations and not be wrong by filling in the accompanying chart to get a sense of the potential for Disqualification amongst the verbs, adjectives and nouns. Note that some questions will not have verbs or adjectives; also note that some questions will have more than one noun.

chap10.indd 5

1. The metaphor in lines 97-98 is used to (A) summarize the challenges faced by minority groups (B) illustrate the way certain forces interact to produce a unique point of view (C) demonstrate the relation between historical and sociological perspectives (D) identify the causes of change in a turbulent social upheaval (E) compare the philosophical underpinnings of racism and sexism Verbs

Adjectives

Nouns

(A)

summarize

none

challenges faced by minority groups

(B)

illustrate

none

certain forces interact to produce a unique point of view

(C)

demonstrate

none

relation between historical and sociological perspectives

(D)

identify

none

causes of change in a turbulent social upheaval

(E)

compare

none

philosophical underpinnings of racism and sexism

2. The quotation in lines 24-25 primarily serves to (A) defend a technique (B) describe an effect (C) criticize a methodology (D) highlight an influence (E) acknowledge an accomplishment Verbs

Adjectives

Nouns

(A) (B) (C) (D) (E) 3. The purpose of lines 5-9 is to (A) explore competing views on gender and sexism (B) compare the critical theories of two groups (C) introduce a perspective on the role of the modern woman

5/23/2011 5:03:50 PM

10. 

Part 2    SAT Preparation

(D) indicate the historical interest in women throughout the ages (E) suggest the different components of a topic Verbs

Adjectives

(B) argue against the viewpoint of an individual (C) explore a certain perspective (D) undermine an established theory (E) develop a novel concept

Nouns

(A)

Verbs

(B)

(A)

(C)

(B)

(D)

(C)

(E)

(D)

4. The passage can be best described as (A) a discussion of the sociological roots of a tradition (B) a defense of an esoteric practice (C) a promotion of a particular group’s social history (D) an illustration of a unique approach (E) a demonstration of the authenticity of a culture

(E)

Verbs

Adjectives

Nouns

Adjectives

Nouns

7. The authors of both passages agree that John F. Kennedy’s ‘Berliner’ speech (A) led to a profound policy change (B) indicated a precocious wisdom (C) belied a belligerent position (D) implied a certain development (E) had a significant international impact

(A) Verbs

(B) (C)

(A)

(D)

(B)

(E)

(C)

5. The reference to Milton in line 44 serves to (A) highlight distinct literary theories (B) acknowledge a preeminent critic (C) deride the habits of most editors (D) explore the particularities of a commonly accepted practice (E) explain some technical idioms for an audience of laypeople Verbs

Adjectives

Nouns

Adjectives

Nouns

(D) (E) 8. Lines 24-29 serve primarily to (A) provide a contrast between radically different societies (B) illustrate a point with particular examples (C) defend a series of unlikely happenings (D) express sincere regret over past events (E) challenge the validity of a scientific conclusion

(A) (B)

Verbs

(C)

(A)

(D)

(B)

(E)

(C)

6. The primary purpose of the passage is to (A) present a controversial theory

chap10.indd 6

Adjectives

Nouns

(D) (E)

5/23/2011 5:03:51 PM

10. 

Chapter 10    Critical Reading III: Reading Comprehension

10.2. Reading Comprehension: TSA TSA (“Tone, Style and Attitude”) questions tend to offer a range of answer choices in which a word or phrase is meant to characterize the feel of a passage: “unabashed admiration” on down through “utter contempt”. In most cases, the way to do this is to pay attention to the wording of the passage; what descriptorsadjectives and adverbsare used? Tone is not about what is being said, but how it is being said. Note the difference between the following sentences. • John walked the dog • All in a huff, John dragged the dog quickly ‘round the block. The first sentence is pretty hum-drum; it’s purely descriptive and doesn’t tell us much of anything about the tone, style or attitude of the author (unless “factual” is an option). The second sentence indicates impatience: impatience from John, who just wants to get the walk over with and even impatience on behalf of the author: notice how the word “around” is shortened to “’round”. Impatience is so pervasive, the author won’t even write out the whole sentence. When trying to determine tone, think about the following criteria: 1. Word Choice: any strongly positive/negative words? 2. Sentence Length: short sentences indicate a more punctuated, snappy tone; long, multi-clause sentences suggest a more deliberate, cerebral (SAT vocab!) tone. 3. Punctuation: watch for exclamation marks! Big warning! Also, “quotation marks” and italics. 4. What is left Unsaid: what is left out be more influential than what is said; for instance, not saying anything bad is just as telling as only saying good things. Try the following TSA exercise to get a better idea of identifying tone. Generally, the correct answers to TSA questions tend to follow certain forms:

10.2.1. Exercise #11: Tone Directions. The following passages discuss similar issues with different tones. What is the tone? Note the word choice, the punctuation and what is left unsaid. Passage 1: Fantasy literature typically consists of two subgenres: that which is good and that which is great. In the latter category we have the works of J.R. Tolkien, C.S. Lewis, George R.R. Martin, Gene Wolfe, Jack Vance and Tad Williams. Perhaps Ursula le Guin, as well, but on that one it’s too close to call. These authors create a new universe in which characters we identify with stomp around, having fights and romances, indulging in espionage and wizardry and gener-

chap10.indd 7

ally having a great deal of fun. These characters are plausible despite the fantastical nature of their exploits and this is what the best fantasy literature helps up us to understand: ordinary people have it in them to do extraordinary things.

• • • •

Tone Word Choice: Punctuation: Left unsaid:

Passage 2: As a “genre” if it were worthy of such an accoladefantasy literature combines the escapism of computer games with the intellectual rigour of a colouring book. The ubiquity of the output and the strange readiness for most publishers to proffer event the paltriest offering does nothing to honor the giants that have come and gone. It used to be that fantasy book were books, first, with complex themes and subtexts; now, sadly, that distinction seems to have vanished as we are all but overwhelmed with products that are badly written, thinly plotted and generally of poor quality all round.

• • • •

Tone Word Choice: Punctuation: Left unsaid:

10.2.2. Secondary Strategy #19: Critical, not Resentful The SAT often features correct TSA answer choices whose meanings are fundamentally non-emotional. The SAT is much more likely to go with critical over resentful; “resentment” is a feeling, while “criticism” is a belief. A good way to think of this is to ask the following question: Can you feel this way about a person? You can feel resentment towards a person, but you cannot feel criticism. Nor can you be critical of a personproperly speaking you can only be critical of a person’s actions, beliefs or attitudes. Thus, words like critical “sceptical”, “disapproving”, “analytical” are much more likely to be correct answers than those that are emotive and personal.

10.2.3. Secondary Strategy #20: The Appreciation Effect A curious feature of TSA questions is the preponderance of correct answer choices that contain the word “appreciation”. You will always be better off selecting “appreciation” as your answer choice. This effect encapsulates the bias towards non-

5/23/2011 5:03:51 PM

10.  emotive, impersonal language: “appreciation” implies respect and acknowledgement, but not passion or emotion.



“I appreciate you”: You don’t usually tell a person you appreciate them; mostly you tell someone you appreciate something about them. “I appreciate your honesty”. Sounds better, right? This is the main point about Tone questions on the SAT...answer choices will usually be words that apply to things, not people.

10.3. Secondary Strategy #21: Ultimate Approach Now, it’s time to combine what you know about actual and potential Disqualifying Information to create your ultimate Eleven-Step Approach to answering questions of Specific and General Interpretation. 1. Think: What is the question asking? What Do You Need to Know About and What Do You Need to Do with It? Turn the question-stump into a real question. 2. Locate Information in passage using Line Numbers, Order and Keywords. 3. Think: What is the passage saying? What’s the thrust or main idea? How does it fit with what comes immediately before it? How does it fit with the author’s argument until that point? Be on the lookout for Counter Arguments. 4. Formulate your Own Answer based on the conjunction of what the question asks and what the passage says. This answer might be more feeling than thought. 5. With your Own Answer, return to the question and read each answer choice. 6. Identify any similar answer choices that you think may be correct. 7. Return to the passage and try to find words or phrases that can be Said a Different Way in a direct translation to the answer choice. This would be the Trigger and if

chap10.indd 8

Part 2    SAT Preparation you can find it, make your answer and move to the next question. 8. No Trigger? Find Inaccuracy in the answer choices by asking “If this answer choice were correct, would the passage have to be different?” In building these hypothetical scenarios, use your imagination while taking every answer choice literally and be sure to avoid telling a Long Story to make an answer choice fit. 9. Still stuck? Try to find that answer choice that looks like it contains the least potentially Disqualifying Information. Start with Non-Specified Nouns (condition, point of view, perspective, instance, viewpoint, approach, kind, certain, assertion, way, extent, individual, experience, practice), followed by Neutral Verbs (lend, suggest, illustrate, make, evoke, demonstrate, indicate, show, draw, place, express, highlight, emphasize, underscore) and Neutral Adjectives, (certain, unique, particular, few, some, distinct, common, specific) and finishing off with Deliberately Weak Language (may have, might be, could be, possibly, perhaps, potentially, maybe, apparently, somewhat, seeming, seemingly) 10. Eliminated two answer choices? Look for the Indefinite Article Effect and guess. 11. Can’t eliminate any answer choices? Skip it. Final words: Recall that you have, on average, 1 minute and 4 seconds for each Reading Comprehension question. Most students breeze through Sentence Completions (we’ll get there soon) and so give themselves a bit more time for Reading Comprehension questions, but that won’t solve all your problems. It is important to practice these strategies until they become habits; it’s no use applying the framework if it takes you four minutes for each question! These approaches and processes for Reading Comprehension are not something to be memorized and then not used; you will only see maximum success with repeated application and habituation.

5/23/2011 5:03:51 PM

Chapter 10    Critical Reading III: Reading Comprehension

10. 

10.4. Quiz #4: Reading Comprehension III 1. True or False: Disqualifying Language tends to appears in questions that ask “in order to…”, “serves to…”, “primary purpose is to…”, etc?

2. True or False: You should apply the principles of Disqualifying Language to all questions before looking for Triggers or Disqualifying Information.

3. What is the question you should ask to determine which answer choice contains the least potentially Disqualifying Information?

4. What is a Non-specified Noun?

5. Give five examples of Non-Specified Nouns

6. What is the Indefinite Article Effect?

7. Give five examples of Neutral Verbs

8. Give five examples of Neutral Adjectives

9. What is the Appreciation Effect?

10. What are the two characteristics of most correct TSA answer choices?

chap10.indd 9

5/23/2011 5:03:51 PM

Chapter

11

Critical Reading IV: Sentence Completion

S

entence Completion questions constitute a not-insignificant chunk of the Critical Reading portion of the SAT. This is good news, because these questions are for the most part straightforward, easily prepared for and subject to an effective procedural strategy. There’s no getting around it: knowing words matters, and unlike with questions of Reading Comprehension (where the premium is on intermediate range vocabulary), with Sentence Completions your vocabulary needs to be both advanced and wide-ranging. And yet, there are ways around this. In this chapter, we will start by exploring the basic skills of Sentence Completions, building in complexity and sophistication through the intermediate concepts and then discussing one of the most advanced and innovative elements of this program of SAT preparation, the technique of Guessing Meaning, though you can actually deduce the meaning of a hard word you’re not already familiar with. On top of all that, we will formulate two inclusive approachesmaking use of all the developed skills set forth in this chapterthat will leave you totally prepared for Sentence Completions on test day.

11.1. Formal Distribution See below for a chart detailing the number of Sentence Completion question you will see, and the proportion constituted in comparison to the other elements.

chap11.indd 1

Formal Sub-Sections

Avg. # Questions % SAT per SAT

Reading Comprehension (CR) 48

28.24

Math (multiple-choice)

44

25.88

Improving Sentence (Writing) 25

14.71

Sentence Completion (CR)

19

11.18

Error ID (Writing)

18

10.59

Math (grid-in)

10

5.88

Improving Paragraphs (Writing) 6

3.53

11.2. Directions Sentence Completion questions are always preceded by the same set of directions. Consult the box below for the exact directions you’ll see on the SAT; familiarise yourself with these now so you don’t waste any time reading them on test day: Each sentence below has one or two blanks, each blank indicating that something has been omitted. Beneath the sentence are five words or sets of words labelled A through E. Choose the word or set of words that, when inserted in the sentence, best fits the meaning of the sentence as a whole.

5/23/2011 5:06:30 PM

11. 

Part 2    SAT Preparation

Note that the instructions ask you to find the word or set of words that best completes the sentence; the fit may not be perfect and it may not be what you think represents the best fit. However, unlike Reading Comprehension, the correct answer choice usually is straightforward.

11.3. Difficulty Distribution The Difficulty Distribution of Sentence Completion is as follows: Difficulty Easy Medium Hard

Frequency 30.41 41.52 28.07

11.5. Sentence Completions: Function Questions of Sentence Completions test your ability to find the word that best fits a gap in a sentence. The questions either have one blank or two.

11.5.1. Single-Blank Questions

Remember that Sentence Completion questions are Ordered by Difficulty; as such, you know that the final two (or so) questions in any section of Sentence Completion will be the hardest.

11.4. Frequency Distribution Within the set of Sentence Completion questions, there exists a sectional breakdown based on the # blanks to be completed in each sentence. The 19 Sentence Completion questions on the SAT split fairly evenly into Single-Blank and Double-Blank. Sent. Comp. section

% of Sent. Comp.

Average # per test

Single Blank

52.63

10

Double Blank

47.37

9

The fact of this means that you will be seeing on average approximately 140 vocabulary words per text (5 words for each SingleBlank question, 10 words for each Double-Blank question). Most people would think that the Single-Blank Sentence Completion questions would be easier; most people are wrong in this case. Note that despite presenting at face value twice as much work, Double-Blank questions are significantly easier than Single-Blank questions. See below for a Difficulty Distribution by question-type: Single Blank

Double Blank

Easy

27.78

33.33

Medium

41.11

41.98

Hard

31.11

24.69

The difficulty of a question is usually based on the difficulty of the vocabulary. This means that you will probably see 15-

chap11.indd 2

18 hard vocabulary words for the Single-Blank questions and 22-25 hard vocabulary words for the Double-Blank questions: around 40 truly challenging words in all.

Sentence Completion questions in which only one term is missing ask you to determine the missing term in the context of the remainder of the sentence using the other words and any other elements of grammar or punctuation. Example:



Richard, and avid mountaineer,  the heights of Mt Kilimanjaro on his 30th birthday. (A) ascended (B) considered (C) sprinted (D) estimated (E) enchanted

11.5.2. Double-Blank Questions Sentence Completion questions in which two terms are missing ask you to determine the missing terms using in the context of the remainder of the sentence using the other words and any other elements of grammar or punctuation. These seem more difficult but are actually easier than Single-Blank questions because you can eliminate pairs based on the strength of the inaptness (SAT-word!) of one word in the pair; therefore, at the minimum you need to be able to determine that four words are inappropriate, and then you’ve cracked the question. Example: • It can be a mistake to think that just because a student is  she is ; sometimes the brightest students say very little in class. (A) outgoing . . incompetent (B) diffident . . unintelligent (C) taciturn . . disingenuous (D) gregarious . . malevolent (E) eloquent . . affable 4/10: Because you only need to know four out of 10 words (40%) to successfully engage the Process of Elimination, Double-Blank questions require comparatively less knowledge than Single-Blank questions, for which you must be able to recognize four words out of five (80%) to run the Process of Elimination.

5/23/2011 5:06:30 PM

11. 

Chapter 11    Critical Reading IV: Sentence Completion

11.6. Sentence Completions: Skills The skills required to maximise your score with Sentence Completions are simple. Some of these steps you may already take instinctively; but if not, it is in your interest to make them habitual, so that you apply them instinctively on test day.

11.6.1. Skill #1: Finding the Clue The Clue is the word or words that suggest the meaning of the blank; without the Clue, the missing word could be anything. Example: Clueless Sentence: John and Len ran through the  . How could you possibly complete that sentence? There isn’t any Clue! As such, it wouldn’t be an SAT question. However, the addition of a Clue-word changes the picture: Clue-ed Sentence: John and Len ran through the  ; after exiting the aviary, they went for ice cream. What’s the clue? “Aviary”, meaning roughly a place with lots of birds. Do not be surprised if the Clues are words that seem difficult: this is what makes the questions hard! See below for some examples; the clues are in bold. Example: The  of the novel was based on real-life events that occurred during the reign of King Henry VIII. • Clue: “real life events” • Answer: plot Example: Usually Katrina was  but , happy to make new acquaintances but wary of who she trusted. • Clues: “happy to make new acquaintances” and “wary” • Answer: friendly . . cautious Example: John found himself  on the matter, caught as he was between two hard decisions. • Clue: “caught as he was between two hard decisions” • Answer: equivocating The last clue is a straightforward definition of a hard word. If you don’t know the word then you won’t be able to answer the question, no matter how simple the clue is. Finding the Clue is not always easy but it is always important. The following are all varieties of Clue:

• • •

chap11.indd 3

Synonyms: meaning the same thing as the missing word. Occurs quite frequently with Double-Blank questions. Antonyms: meaning the opposite of the missing word; often used with “but” or “although”. Definitions: definition of the missing word. Usually found with Hard questions, following a comma or a colon.

Comma/Colon Continuation: About 1/3 of the time, the clue will be a definition of the missing word; when this happens, the Clue usually follows the missing word, set off by a comma or a colon.Whenever you see a comma/colon following a blank, you know that what comes after will be a definition or a description.

Try the following exercise to develop your ability to identify the Clue.

11.6.2. Exercise #12: Finding the Clue Directions. What follows is a series of answered Sentence Completion questions, with the replaced words underlined. Indicate the Clue for each sentence and the type of Clue (Synonym, Antonym, or Definition). With double-blank questions write the clues in order of the blanks. 1. Although the weather report had suggested cold rain, the day was sunny and temperate. • Clue(s): “sunny” • Type(s): synonym 2. Everyone needs to earn a living somehow, but even the most unscrupulous individuals abide by certain professional standards of what they are and aren’t willing to do. • Clue(s): • Type(s): 3. The complexity of the device baffled scientists: they could not understand how an ancient civilization could produce something so intricate. • Clue(s): • Type(s): 4. John has been described as mercurial, subject to sudden changes of mood and feeling. • Clue(s): • Type(s): 5. The physiology of even a single-celled organism is surprisingly sophisticated, containing internal structures carefully honed over millions of years of evolution. • Clue(s): • Type(s): 6. The application of statistical analytics to sports science has resulted in some innovation in professional athletics; the cross-disciplinary approach has been particularly

5/23/2011 5:06:31 PM

11.  useful in developing new metrics for efficiency and effectiveness. • Clue(s): • Type(s): 7. A part-time singer in a garage band, Jonathon hid his musical tendencies from his co-workers at the hospital. • Clue(s): • Type(s): 8. Despite the overwhelming evidence against him, the accused maintained an expression of serenity throughout the trial, apparently unfazed by strength of the prosecutor’s case. • Clue(s): • Type(s): 9. It was unclear to the students whether they had a test on Monday: the teacher’s comments seemed almost deliberately ambiguous. • Clue(s): • Type: 10.  Although the scholarly article was intended for a sophisticated audience, it was written in such a way that a child could find enlightenment within it. • Clue(s): • Type(s):

Part 2    SAT Preparation

Continuation-Cues: and, so, because, therefore, since, comma, colon   Note: amplification is a kind of continuation that suggests a relationship of increasing intensity between the clue and the missing word, or, with a Double-Blank question, between the missing words. When the relationship is amplification, you know the missing word will be similar to the clue, only more intense. • Example: “John implied that he was not just  but , willing to eat a horse if his mother would let him” Continuation-Cue: “not just…but”. The relation of the elements of this Direction-Cue implies an expression of increase; in this case, you’re looking for two words that mean similar things, only the second has to mean more of it than the first. Answer: “hungry . . ravenous” Reverse: a question-stump Reverses itself if it goes for a word that is opposite in meaning to the Clue(s). • Example: “Though calling himself a gentlemen, Emilio acted in a most  manner” • Reverse-Cue: “Though”. You are expected to provide a word meaning the opposite of “gentlemanly”. Answer: “boorish”.

11.6.3. Skill #2: Identifying Direction-Cues Direction-Cues are words within the question-stump that, with the Clues, suggest a possible meaning for the blanks. These are not definitions or synonyms, but conjunctions and preposition that indicate the direction of the sentence. There are two directions:

Important: The Reverse-cue can signify a reverse from the Clue (Single-Blank), or a reverse between blanks (Double-Blank) in the relation between the missing words



Continuation: a question-stump Continues in the direction of its Clues if the ultimate answer is similar in meaning to words in the question (definition, synonym).



Example: “After his long run, Mike was  he wanted to eat until he could eat no more”

• •

Continuation-Cue: the colon. You are provided a definition of the missing word; your job is to supply the word. Answer: “famished”.

Example: “English novelist Evelyn Waugh writes in a distinctly - manner, achieving his effect in layers of dense, lugubrious prose”

Continuation-Cue: the comma. The use of the comma indicates that the missing word is similar in meaning to “layers of dense, lugubrious prose”. Answer: “melancholy”.

chap11.indd 4

Example: “Rico’s speech was  but surprisingly  : despite the obscurity of his reference his listeners could generally understand him” • Reverse-Cue: “But”. You are provided definitional Clues but are expected to determine a relation of Reverse between the missing words. Answer: “esoteric . . intelligible” Important: Watch for the Double Negative effect. Two Reverse cues function together like a continuation clue.



Example: “The pedigree of the dog, while , could not  for its missing ear: it lost the show” • Reverse-Cue: “while” and “not”. In tandem, (SAT vocab!) the Cues cancel each other and you’re left

5/23/2011 5:06:31 PM

11. 

Chapter 11    Critical Reading IV: Sentence Completion with a basic Continuation. Answer: impressive . . compensate Reverse-Cues: While, but, despite, though, although, seemingly, apparently, not, even, contrast, differing, though, yet Alone, the Direction-Cues cannot help you crack a question; when used in combination with the Clue(s), however, you are able to develop a better idea of the missing word(s).

11.6.4. Skill #3: Generating Own Solution If you combine your abilities to identify Clues and identify the Direction of a sentence based on punctuations and conjunctions (the Cues), then you put yourself in a position to Generate your Own Solution. This is an important skill and will play a significant part in your overall strategy. If the goal is to make it so that the only limiting factor in your Sentence Completion competency is a lack of advanced vocabulary, Generating your Own Solution the only job is to translate that solution into an answer choice. Example: 1. While the town’s population maintained a position of satisfaction with the local government, the State comptroller was  at the actions of the city council. • Clue: “satisfaction” • Direction-Cue: “while” • Own Solution: unpleased • Answer: aggrieved In this case the limiting factor is recognizing the synonymous relationship between “unpleased” and “aggrieved.” Your Own-Solution can be simple or it can be complex; you can use a phrase, if necessary, or a general description. Example: 2. The child played gently with the  toy, making sure not to damages its delicate workings and mechanisms.

• • •

Clue: “Gently”, “delicate workings and mechanisms” Direction-Cue: comma Own Solution: “thing with delicate workings and mechanisms” • Answer: fragile A general Own Solution can be valuable; the harder the question, typically the less precise your Own Solution will be. That is perfectly OK. Note, however, that unlike your Own Answer in Reading Comprehension, your Own Solution has to be articulated (even if that articulation is just a general description).

chap11.indd 5

Try an exercise designed to help develop your ability to Generate your Own-Solution.

11.6.5. Exercise #13: Generating Own Solution Directions. Below is a series of incomplete sentences. Identify the Clue, and Direction-Cues, and determine your Own Solution in the spaces provided. Remember that your Own Solution can be word, a phrase, or a general description. On double-blank questions, write the Clues, Direction-Cues, and Own Solutions in order of the blank. 1. Janus wrote with a  style, carefully selecting words for maximum effect.



Clue(s): “carefully selecting words for maximum effect” • Direction-Cue(s): comma • Own Solution: deliberate 2. The  of the displaced people of Pandora is a tragic one, but their burden is no greater than that experienced by  populations the world over.

• • •

Clue(s): Direction-Cue(s): Own Solution: 3. The professor, a  man, enjoyed company of all sorts and was known for his , never having turned a guest away without food and drink. • Clue(s): • Direction-Cue(s): • Own Solution: 4. The excitement surrounding the Megasaurus has reached almost  levels: it is difficult to exaggerate the interest in this one possibly-fictional species.

• • •

Clue(s): Direction-Cue(s): Own Solution: 5. Because the shopkeeper treated his customers with honesty and respect, he earned a reputation for .

• • •

Clue(s): Direction-Cue(s): Own Solution: 6. Unlike the abstract art of contemporary times, work from earlier  was life-like and placed a great deal of emphasis on faithful . • Clue(s): • Direction-Cue(s): • Own Solution:

5/23/2011 5:06:31 PM

11.  7. Despite a reputation for fierceness, the Tutsi were not a  tribe, preferring instead to tend their cattle and crops without  or tumult.

• • •

Clue(s): Direction-Cue(s): Own Solution: 8. As an extension of his philosophy of , Marco made a point to welcome people of all ages and races into his home.

• • •

Clue(s): Direction-Cue(s): Own Solution:

9. The man was known for his  dress and loud, attention-grabbing behavior.

• • •

Clue(s): Direction-Cue(s): Own Solution:

11.6.6. Skill #4: Using the Word-Parts If you use Clues and Direction-Cues to Generate your Own Solution (either a word, phrase or general description), then your only limitation in selecting correct answers will be your level of vocabulary. The main way the SAT test-makers create hard questions of Sentence Completions is in the use of challenging, unfamiliar words in the question and answer choices: this is one of the reasons why it is important to study high-frequency vocabulary (with the other reason being to develop your ability to recognise shades of literal meaning in Reading Comprehension answer choices). The good news, however, is that a lot of complicated English words can be broken down into parts and understood more easily that way. Some words are atomistic and their meaning can’t be determined by using the parts of the word; others, though, can be analyzed this way, and usually the longer (and harder) the word, the more easily it can be broken down. Such a word can typically be split into two parts: Prefix: Attached to the word root; can provide indications as to +/- polarity or outright meaning of a word. Examples:



Bene, meaning “good” or “well”. Beneficial, benevolent, beneficiary. • Demo, meaning “people”. Democracy, demographic, epidemic. • Sanct, meaning “holy”. Sanctify, sanctuary, sanctimonious. Word Root : The root is the primary unit of a word, contain-

chap11.indd 6

Part 2    SAT Preparation ing its most significant aspects and cannot be reduced into smaller parts. Examples: • Vita, meaning “life”. Revitalize. • Spect, meaning “to look”. Inspect, retrospect. • Duct, meaning “to lead” or “to pull”. Product, deduct, induct. Sometimes you can put two Word-Parts together to make words. Example: Telephone. Tele meaning “from afar” and phone meaning “to hear”. Using this technique, you can approximate the definition (into good/bad, positive/negative, etc) of words for which you don’t know the exact definition and that’s a very useful skill on Sentence Completions. This is the reason it is important to study Word Parts (see 6.3: Word Part List). So what can you do when you find yourself facing a word that sounds vaguely familiar, but which you simply do not know? You can try to Guess the Meaning using its Word Parts. Occasionally, the hard words in Sentence Completions and Reading Comprehension can be puzzled out by using the following method: 1. Attempt to isolate Word Parts of unknown SAT word: This can be apparent; the prefixes and roots you learn are partly to prepare you for this. Break the word into a Prefix and a Root; ignore any Suffixes (as these tend not to be important). 2. Identify Known Word with similar Word Parts: You have to use your imagination and think back to all the words you have heard that either contain or sound like they contain the Word Part. The spellings of words change over time; as such you will not be able to count 100% on seeing a Root or Prefix spelled the way you’re familiar with, but you probably will have heard it. Think back to books you’ve read, films you’ve seen or conversations you’ve had in which you have come across words which might include the Word Part in question. 3. Apply meaning of Known Word to unknown Word Part: Using what you know of the word you selected in Step 2, try to puzzle out the meaning of the Word Part by working backwardsstart with the definition of the known word and figure out what the Word Part must mean. 4. Apply meaning of learned-Word Part to the unknown SAT word: The result of this will be a group of words that combine to form something that resembles the definition of the missing word. This works for both prefixes and word roots. Easy example: Unequivocal = Un + Equi + Vocal You probably already know the meaning of ‘Un’, but let’s try it anyway.

5/23/2011 5:06:31 PM

11. 

Chapter 11    Critical Reading IV: Sentence Completion 1. Identify Known Word with similar Word Part: “unknown” 2. Apply meaning of Known Word to Unknown Word Part: “unknown” means not known, so “un-” must mean not. 3. Apply meaning of learned-Word Part to unknown SAT word: since “un-” means not, then “unequivocal” means not equivocal. Now apply this technique to the other identified word parts: Equi: 1. Identify Known Word with similar Word Part: “equidistant” 2. Apply meaning of Known Word to unknown Word Part: “equidistant” means having the same distance, so “equi-” probably means same. 3. Apply meaning of learned-Word Part to unknown SAT word: since “equi-” means same or equal, then “equivocal” has something to do with sameness or equality. Vocal: 1. Identify Known Word with similar Word Part: “vocalise” 2. Apply meaning of Known Word to unknown Word Part: “vocalise” means speaking, so “vocal” probably means speak. 3. Apply meaning of learned-Word Part to unknown SAT word: since “vocal” means speaking, “equivocal” has something to do with speaking. Now, the final step in the process is to put all the word parts back together to try to find the word’s meaning. You have: Un + equi + vocal = not + same + speaking The tricky part is turning this Literal Definition into the Actual Definition; a fast-and-ready etymology like this requires a bit of creativity. If you are “not same speaking”, then you are not speaking similarly on different issues; you’re not wavering. Your mind is made up. You have a strong opinions, one way or the other. And...that’s the exact definition of the word. Now try an exercise designed to help you develop your ability with this advanced skill.

11.6.7. Exercise #14: Guessing Meaning Directions. What follows is a series of hard words composed of prefix + root. Apply your strategy of Guessing Meaning to the following words by 1) Isolating Word Parts; 2) Identifying Similar Known Word with Similar Word Parts; 3) Using Known Word to Define Unknown Word Parts; and 4) Using Learned Word Parts to Define Unknown Word. When engaged in step (2), cast your net as widely as possible for Similar Words: words that simply sound like they contain the unknown-root can be of use. When turning the literal definition into the actual definition, see if you can put

chap11.indd 7

the meanings of the Word Parts together in a way that make sense. This too takes creativity. 1. Ambidextrous: a. Ambi: i. Identify Known Word with Similar Word Part: “ambiguous” ii. Apply meaning of Known Word to Unknown Word part: “ambiguous” means being inclined both ways, so “ambi-” means both. b. Dexter: i. Identify Known Word with Similar Word Part: “dexterity” ii. Apply meaning of Known Word to Unknown Word Part: “dexterity” means skillful use of hands, so “dexter” probably means hands or good hands c. Literal Definition: “both good hands” d. Actual Definition: able to use both hands equally well. 2. Endogamy: a. Endo: i. Identify Known Word with Similar Word Part: ii. Apply meaning of Known Word to Unknown Word Part: b. Gamy: i. Identify Known Word with Similar Word Part: ii. Apply meaning of Known Word to Unknown Word Part: c. Literal Definition: d. Actual Definition: 3. Polyglot: a. Poly: i. Identify Known Word with Similar Word Part: ii. Apply meaning of Known Word to Unknown Word Part: b. Glot: i. Identify Known Word with Similar Word Part: ii. Apply meaning of Known Word to Unknown Word Part: c. Literal Definition: d. Actual Definition: 4. Subcutaneous: a. Sub: i. Identify Known Word with Similar Word Part: ii. Apply meaning of Known Word to Unknown Word Part: b. Cuti:

5/23/2011 5:06:31 PM

11.  i. Identify Known Word with Similar Word Part: ii. Apply meaning of Known Word to Unknown Word Part: c. Literal Definition: d. Actual Definition: 5. Retrograde: a. Retro: i. Identify Known Word with Similar Word Part: ii. Apply meaning of Known Word to Unknown Word Part: b. Grade: i. Identify Known Word with Similar Word Part: ii. Apply meaning of Known Word to Unknown Word Part: c. Literal Definition: d. Actual Definition: 6. Grandiloquent: a. Grand: i. Identify Known Word with Similar Word Part: ii. Apply meaning of Known Word to Unknown Word Part: b. Loqu i. Identify Known Word with Similar Word Part: ii. Apply meaning of Known Word to Unknown Word Part: c. Literal Definition: d. Actual Definition: 7. Verisimilitude: a. Veri/verus: i. Identify Known Word with Similar Word Part: ii. Apply meaning of Known Word to Unknown Word Part: b. Similis: i. Identify Known Word with Similar Word Part: ii. Apply meaning of Known Word to Unknown Word Part: c. Literal Definition: d. Actual Definition: 8. Ignominious: a. In: i. Identify Known Word with Similar Word Part: ii. Apply meaning of Known Word to Unknown Word Part: b. Nomen: i. Identify Known Word with Similar Word Part:

chap11.indd 8

Part 2    SAT Preparation ii. Apply meaning of Known Word to Unknown Word Part: c. Literal Definition: d. Actual Definition: 9. Patronymic: a. Patro: i. Identify Known Word with Similar Word Part: ii. Apply meaning of Known Word to Unknown Word Part: b. Nonym: i. Identify Known Word with Similar Word Part: ii. Apply meaning of Known Word to Unknown Word Part: c. Literal Definition: d. Actual Definition: 10. Agnostic: a. A: i. Identify Known Word with Similar Word Part: ii. Apply meaning of Known Word to Unknown Word Part: b. Gnosos: i. Identify Known Word with Similar Word Part: ii. Apply meaning of Known Word to Unknown Word Part: c. Literal Definition: d. Actual Definition:

11.7. Sentence Completions: Strategies Having attained familiarity with the basic, intermediate, and advanced skills needed for success on Sentence CompletionsFinding Clues, Direction-Cues, Generating Own Solutions, and using Word Partsyou’re now ready to take on the Strategies.

11.7.1. Strategy #1: Clues before Direction-Cues before Word Parts In answering Sentence Completion questions, always try to Find the Clue before you resort to checking for DirectionCues and Guessing Meaning with Word Parts. The Clue is your best and most important resource.

11.7.2. Strategy #2: +/- Polarity In a pinch, if you’re stuck on translating a word, you can sometimes use prefixes to get a sense of whether the unknown SAT

5/23/2011 5:06:31 PM

11. 

Chapter 11    Critical Reading IV: Sentence Completion word is broadly “positive” or “negative”. When used in conjunction with an Own-Solution that is a +/- Statement, you can usually eliminate some answer choices on the really Hard questions. Memorize the following prefixes to help with this: Negative: anti- (against), bell- (war), dys- (ill, bad), a-, il-, im-, in-, ir-, non-, un- (not), male- (evil), mis- (hatred), pseudo- (false, untrue)

Positive: -ami (friend), bene- (good), eu- (good), phil(love)

11.7.3. Strategy #3: Approach for Single-Blank Questions Using the skills of Finding Clues, Direction-Cues, Generating your Own Solution and using Word Parts, the strategy for Single-Blank questions is very straightforward. 1. What is the sentence saying? You may want to rephrase the question-stump in order to maximise your comprehension. If you do not understand the words in the questionor if you do not understanding the formation of the question-stumpthen you may do well to consider skipping the question. 2. Find the Clue(s), be they synonyms, antonyms or definitions. 3. Identify Direction-Cues, watching for Continuation (usually indicated by commas or colons) and Reverse (usually indicated by words like “but”, “although”, “despite”). Recall that amplification is a kind of Continuation. 4. Generate Own Solution. Fill in the blank with a word, phrase, or general description that captures the meaning of the missing word. 5. Translate into an Answer Choice. The answer choice words may be straight-forward or they may be complicated (if it’s a later question then the vocabulary almost certainly be challenging). 6. Use Word Parts with Process of Elimination. Depending on the difficulty of the question and the extent of your vocabulary, if the translation isn’t obvious, use the Process of Elimination with what you know about Word Parts to disqualify answer choices that you know cannot be your Own Solution. In a pinch, use +/- Polarity to help with this (a-, il-, im-, in-, ir-, non-, un- all mean not; ami-, bene-, eu-, phil- all mean good). 7. Rule of One. If you can’t eliminate at least one answer choice, skip it.

chap11.indd 9

11.7.4. Strategy #4: Approach for Double-Blank questions The strategy for Double-Blank questions looks very similar to that for Single-Blank questions, except you have the chance to choose the blank you want to solve, and you only need to know 4/10 words. 1. What is the sentence saying? You may want to rephrase the question-stump in order to maximise your comprehension. If you do not understand the words in the questionor if you do not understand the formation of the question-stumpthen you may do well to consider skipping the question. 2. Which blank is easier? This is determined by which blank has the stronger Clue; one blank will always be easier to fill than the other. 3. Apply Single-Blank Strategy to Easier Blank. Having selected the easier of the two blanks, apply the approach for Single-Blank questions, stopping with your Own Solution. 4. Apply Single-Blank Strategy to Harder Blank. Having determined an Own Solution for one of the blanks, you can now apply the approach for Single-Blank questions to what was the harder blank: by virtue of your Own Solution (be it a word, a phrase or a general description), the Double-Blank question is now a Single-Blank question. It is ok if you’re not sure about what this word should be, but you should have an advantage by having generated your Own Solution for the previous blank. Watch for Contrast between blanks. 5. Translate into SAT-speak. The word-pairs in the answer choices must fit both blanks in order to be correct. You can use your Own Solution word-pair to eliminate answer choice word-pairs on the basis of the un-fittingness of the first word (even if you don’t know the meaning of the second answer choice word), or the second word (even if you don’t know the meaning of the first answer choice word) or the relation between the two (if you don’t know the meaning of either answer choice word). The relation might be opposites, synonyms or amplification. 6. Use Word Parts with Process of Elimination. You have twice as many opportunities to use Word Parts to help you for double-blank questions. Since you have to select a pair of words in combination, you can try to Guess the Meaning of one word in an answer-choice pair using Word Parts; if you can get a good idea of one of the words in a combination, then you can either eliminate it right away because it doesn’t fit, or try to crack the second word using Word Parts.

5/23/2011 5:06:31 PM

11. 10 7. Rule of One: if you can’t eliminate at least one answer choice, skip it.

11.7.5. Strategy #5: Save the Worst for Last Remember that Sentence Completion questions, unlike Reading Comprehension questions, are always Ordered by Difficulty. So question 7 will be harder than question 4count on it! Therefore, you have a better idea of whether to skip it or not. The SAT does you the favor of Saving you the Worst For Last, so work with what is given and apply intelligent test-taking strategy.

11.7.6. Strategy #6: Cut your Losses You must get through the Sentence Completion questions quickly; the more time you spend on these, the less time you’ll

chap11.indd 10

Part 2    SAT Preparation have for the Reading Comprehension that follows directly after. So, if you’re stuck on the last question, and really don’t know the answer, Cut Your Losses and move directly to the Reading Comprehension.

Golden rule: Spend no more than an average 30 seconds per question. An eight-question series should be finished in four minutes; a six-qeustion series should be finished in three minutes.

11.7.7. Strategy #7: The Deal-Breaker If you cannot determine the meaningful vocabulary in either the question-stump or any of the answer choices, then you have reached your Deal-Breaker. Skip it.

5/23/2011 5:06:31 PM

Chapter 11    Critical Reading IV: Sentence Completion

11. 11

11.8. Quiz #5: Sentence Completions 1. True or False: Sentence Completion questions are not Ordered by Difficulty

2. True or False: In a Sentence Completion question, the sentence will always contain a clue as to what the blank ought to be

3. Approximately how many advanced vocabulary words can you expect to see on a full test of Sentence Completions?

4. In most cases, what kind of Clue is used in conjunction with a comma/colon?

5. Name three Direction-Cues that indicate a relation of Contrast.

6. What does it mean to Generate your Own-Solution?

7. What are the two kinds of Word Parts?

8. Name the four steps in the process of Guessing Meaning.

9. Name five prefixes that mean “not”.

10. What are two primary differences in strategy between Single-Blank questions and Double-Blank questions?

chap11.indd 11

5/23/2011 5:06:31 PM

Chapter

12 T

Writing I: Agreement

he SAT added its official Writing Section in 2005; before then, Writing had been a SAT Subject Test (though most colleges required you to take it). This was just one of the changes the College Board made to the SAT at that time; it also altered the Critical Reading section and made Math a bit easier. But too much change can be a bad thing. Some colleges claim they don’t look at Writing scores when considering an applicant’s SAT because they argue the Writing section (and particularly the Essay) are too coach-able; some colleges go so far as to suggest that they only consider your score out of 1600points on the old two section Math-Verbal scale. But wouldn’t they have counted your Writing when it was a SAT Subject Test? However, recent finding suggests that of all the SAT sections (Critical Reading, Math, and Writing), your Writing score is the most predictable indicator of first-year success at university. The critics are right, however, the Writing section is coachable. The following chapters contain everything that is required for success on Writing. We will comprehensively review every element of grammar you might be tested on, undertake exercises designed to reinforce your skills with these and consider some practical strategies and approaches that will minimize the “element of surprise”. Writing is only Hard when you get caught off guard by an error you didn’t see; the way to prepare for this situation is to start to try to see everything. Knowledge is the key, here. Many SAT preparation guides will tell you to “use your ear” when it comes to cracking Writing; we will forego such dubious advice—as you should rarely ever use your ear—and instead tell you all about every kind of error you’ll see on the SAT. Then the only job is to familiarise yourself with the Error Checklist so you can run through for every problem in the average time allotted (43 seconds). Note: if you learned English as a second language, then you’re actually at an advantage: most native English speakers will never have never truly

chap12.indd 1

revised the rules of grammar and usage! All through the next five chapters, we will explore the properties of the correct usage of the English language, with a focus on increase complexity and sophistication—from clauses to sentences; from sentences to paragraphs and beyond—as we ultimately build towards preparing you to write an excellent Essay.

12.1. Formal Distribution The Writing section presents as a mixture of three different types of question: Improving Sentences, Error Identification and Improving Paragraphs. See the chart below for a breakdown: Formal Sub-Sections

Avg. # Questions % per SAT SAT

Reading Comprehension (CR) 48

28.24

Math (multiple-choice)

44

25.88

Improving Sentences (Writing)

25

14.71

Sentence Completion (CR)

19

11.18

Error ID (Writing)

18

10.59

Math (grid-in)

10

5.88

Improving Paragraphs (Writing)

6

3.53

12.2. Directions Writing questions are always preceded by the same sectional sets of directions. Consult the box below for the exact directions you’ll see on the SAT for Improving Sentences (questions 1 – 11 in the long Writing section and 1 – 14 in the short Writing section):

5/23/2011 5:06:41 PM

12. 

The following sentences test correctness and effectiveness of expression. Part of each sentence or the whole sentence is underlined; beneath the sentence are five ways of phrasing the underlined material. Choice A repeats the original phrasing; the other four choices are different. If you think the original phrasing produces a better sentence than any of the alternatives, select choice A; if not, select one of the other choices. In making your selection, follow the requirements of standard written English; that is, pay attention to grammar, choice of words, sentence construction and punctuation. Your selection should result in the most effective sentence—clear and precise, without awkwardness or ambiguity.

Consult the box below for the exact directions you’ll see on the SAT for Error ID (questions 12 – 29 in the long Writing section): The following sentences test your ability to recognize grammar and usage errors. Each sentence contains either a single error or no error at all. No sentence contains more than one error. The error, if there is one, is underlined and lettered. If the sentence contains an error, select the one underlined part that must be changed to make the sentence correct. If the sentence is correct, select choice E. In choosing answers, follow the requirements of standard written English. Consult the box below for the exact instructions you’ll see on the SAT for Improving Paragraphs (questions 30-35 in the long Writing section): Read the passage and select the best answers for the questions that follow. Some questions are about particular sentences or parts of sentences and ask you to improve sentence structure or word choice. Other questions ask you to consider organization and development. In choosing answers, follow the requirements of standard written English. Note that in all instances the directions tell you to “follow the requirements of standard written English.” These requirements differ significantly from those of standard spoken English in terms of idiomatic expression and degree of precision. This is one reason why it’s not good to use your grammar “Ear”: the

chap12.indd 2

Part 2    SAT Preparation test is written specifically to trap you on the Hard questions by making you think you hear an error when actually none exists.

12.3. Difficulty Distribution The Difficulty Distribution of Writing is as follows: Difficulty

Frequency

Easy

41.95

Medium

50.79

Hard

7.26

Of all three sections on the SAT, Writing has by far the lowest proportion of Hard questions and the highest proportion of Easy questions. Remember that Writing questions—except for Improving Paragraphs—are Ordered by Difficulty; this means that the questions near the end of each Writing section are rated most difficult and, therefore, subject to General Strategy #8: Skip the Hard Questions. Also, each Writing subsection has its own Difficulty Distribution; see the following chart for information: Imp. Sentences

Error ID

Imp. Paragraphs

Easy

44.44

44.44

24.07

Medium

50.67

44.44

70.37

Hard

4.89

11.11

5.56

Error Identification presents as the Hardest component of the Writing section; as it is Ordered by Difficulty this means that you can count on questions 26, 27, 28 and 29 of the long Writing section being the Hardest you’ll see. This suggests an identity: The Difficulty of a Writing section is a function of the ease with which you miss an error or the haste with which you assign an error where none exists

12.4. Agreement: Frequency Distribution Questions of Improving Sentences and Error Identification, questions explore five standard English grammatical relations—six, if you count the option “No Error”. These relations are: Agreement, Parallelism, Modifiers, Wordiness and Idiomatic Expression. Each of these material sections has its own set of

5/23/2011 5:06:41 PM

12. 

Chapter 12    Writing I: Agreement sub-sections and every Writing question falls under the header of one of these grammatical relations. These sub-sections are the building blocks on which the Writing section is based. Writing sub-section Agreement Parallelism Modifiers Wordiness Idiomatic Expression No Error

% of Writing

Average # per test

33.24 27.99 10.20 6.12 4.66 17.78

14 12 4 3 2 8

Note that Agreement and Parallelism are the primary Writing sub-sections, followed by the “No Error” category ((A) for Improving Sentences and (E) for Error Identification). Agreement

Parallelism

Modifiers

Wordiness

Idiomatic

No Error

Easy

45.24

50.42

44.12

29.41

25.00

40.00

Medium

49.21

42.86

50.00

47.06

50.00

49.23

Hard

5.56

6.72

5.88

2.94

25.00

10.77

The outliers are “Idiomatic Expression” and “No Error”. This means that you’re more likely to see those questions later in the section: 9, 10, 11, 26, 27, 28, and 29 on the long Writing section and 12, 13, and 14 on the short Writing section.

Consult the following chart for information on the Frequency of questions of Agreement in Improving Sentences: Material subsection

% of Improving Sentences

Average # per test

Agreement

25.41

6

Consult the following chart for information on the Frequency of questions of Agreement in Error Identification: Material sub-section % of Error ID Average # per test Agreement

41.98

26, 27, 28 and 29 will always contain at least one Hard Agreement question

12.6. Agreement: Function

12.5. Agreement: Sectional Frequency

8

Questions of Agreement are more likely to appear in Error Identification than in Improving Sentences. This means that for the most part you don’t need to correct the grammatical inaccuracy; you just need to be able to spot it.

chap12.indd 3

The specific proportion of material sub-types on the Writing sections rarely varies. You can count on seeing a certain number of Agreement questions, a certain number of Parallelism questions, etc; within those sets you can count on seeing a certain number of verb tense questions, a certain number of comma splice questions, etc. This means that if you apply yourself to a rigorous program of preparation, earning the section from the ground up, sub-type by sub-type, there need be no surprises on the day of the SAT. This differs from Reading Comprehension, in which there is always an “unknowability factor” —difficulty of passage, vocabulary, etc. There is no “unknow-ability” factor in Writing: you know exactly what you’ll see. The issue is how well you prepare yourself to deal with it Each of these material sub-sections has its own DifficultyDistribution.

Agreement is the most important sub-section of Writing. There are questions that pertain to the matching-up of subjects and verbs, nouns and pronouns, and nouns and numbers within a clause. Remember: Clause Definition: a clause is a group of words that includes a subject and a predicate (usually just a verb) Consult the following chart for the Frequency Distribution of Agreement questions on the SAT: Agreement subsection % of Agreement Average # per test Subject-verb

49.56

7

Noun-pronoun

39.82

5

Noun-number

10.62

2

5/23/2011 5:06:41 PM

12. 

Part 2    SAT Preparation

Each of these Agreement subsections has its own Difficulty Distribution. These will help inform your guessing and skipping:

S-V Agreement type % of S-V Agreement Average # per test

Subject-Verb Noun-Pronoun Noun-Number

Number

66.04

5

53.97

38.60

33.33

Formation

22.64

2

Medium 42.86

52.63

66.67

Tense

11.32

1

Hard

8.77

0.00

Easy

3.17

12.6.1. Subject-Verb Agreement Subject-Verb agreement is the most significant grammatical relation you will find among SAT questions. Some definitions: Subject: The subject of a sentence is the person, place or thing that does the verb. It generally (but not always) comes first in the sentence and is always a noun. • Example: I went to the store. Subject: I. • Example: The dog bit me. Subject: The dog. • Example: Fred and Sally watched a film. Subject: Fred and Sally. Note that the latter example offers a Compound Subject. A Compound Subject is a subject where two or more things are linked by “and”. Compound Subject Golden Rule: a Compound Subject always takes the plural verb. Verb: The verb is the action-word in the sentence; it gives the sentence its direction and meaning. • Example: I drove the car. Verb: drove. • Example: John has run a marathon. Verb: has run. • Example: A computer was bought by Fred. Verb: was bought. Note that the latter example offers a verb in the Passive Voice. Verbs come in two voices—active and passive. You tell the difference between the two by the way they interact with the Subject. Passive Voice Golden Rule: The Subject does the active verb; the Subject has the passive verb happen to it. The passive voice is formed by adding is/was + past participle. The SAT usually doesn’t like the passive voice (more on this to come). Subjects must agree with their verbs in number, formation and (sometimes) tense. Consult the following chart for more information on the kinds of Subject-Verb Agreement questions you will find on the SAT:

chap12.indd 4

What do these categories mean? S-V Agreement: Number: Singular subjects take singular verbs; plural subjects take plural verbs. Always! It is your job to make sure you don’t get confused and assign a plural verb to a single subject. This sounds easy but the SAT makes it tricky by using Distracters (more on those later) with come between Subject and Verb and collective subjects which sound plural but take the singular verb. Consider it a good idea to always keep track of the subject. i. Example: The car drive very smoothly—no problems at all. • Subject: “car” • Better: “drives” ii. Example: The number of students have increased each year until they all can no longer fit in the building. • Subject: “number” • Better: “has” iii. Example: John and I knew for a long time that we wanted to be doctors, but only recently has the implications of this ambition become known, in terms of commitment. • Subject: “implications” • Better: “have” S-V Agreement: Formation: These questions tend to come early in the section and involve a mis-formulation of the verb in the context of its subject. The verb will be mis-conjugated, most often turned into a gerund (any verb that ends with “ing”); other times, the verb will be missing altogether. These errors can resemble errors of Modifiers and, like questions of Subject-Verb agreement, are liable to contain Distracting inserted phrases. i. Example: Astrophysicists hope one day explaining the universe. • Distracter: “one day” • Better: “to explain” ii. Example: William Burroughs, the famous Beat writer, being known best for exploring the seedy under-life of post-war America. • Distracter: “the famous Beat writer” • Better: “is”

5/23/2011 5:06:41 PM

12. 

Chapter 12    Writing I: Agreement iii. Example: The Hercules beetle, so called because it can lift fifty times its body weight, found only in the Amazonian rain-forest. • Distracter: “so called because it can lift fifty times its body weight” • Better: “is found” S-V Agreement: Tense: Normally, tense does not matter between a subject and a verb; the tense of a sentence is generally established by the other verbs, not subjects. However, when the subject is specifically from the past or the future, then the tense of the verb must match. i. Example: By the year 2500, scientists predict that we have the power to live twice as long as we already do. • Better: “we will have” ii. Example: Yesterday Ricky drives the car into the pond. • Better: “drove” iii. Example: By 1958, every house in America has its own television. • Better: “had” Try the following exercise for practice with subjects and their verbs.

4. The likelihood of gaining admission to top universities increase when you study hard during high school. • Subject:



5. The important of safety when dealing with chemicals being impossible to overstate. • Subject:



1. The girl’s directions was both inaccurate and potentially dangerous. • Subject: “the girl’s directions”







Re-write:

3. The Guatemalan tree frog, known for its bright colors and bulging eyes, is actually one of the most poisonous species on Earth. • Subject:



chap12.indd 5

Re-write:

Re-write:

7. My brothers, four and eight years older than me, respectively, tells me that I must learn to ride a bicycle if I’m to take the first steps toward independence. • Subject:



Re-write:

8. The difficulty with cold fission lying in the instability of atomic reactions at super-cool temperatures. • Subject:



Re-write:

9. The words of the priest falls on deaf ears. • Subject:

Re-write: The girl’s directions were both inaccurate and potentially dangerous.

2. The sandwiches our mother made for us yesterday are delicious. • Subject:

Re-write:

6. The trajectory of the space shuttle take it right past my house. • Subject:

12.6.2. Exercise #15: Subject-Verb Agreement Directions. The following sentences contain errors of Subject-Verb agreement. Some sentences may contain no errors. Identify the subject and re-write the grammatically correct sentence.

Re-write:



Re-write:

12.6.3. Noun-Pronoun Agreement Just like subjects and their verbs, nouns and their pronouns must agree in number, case, and consistency; the noun sets the standard and the pronoun must be made to agree with it. Some definitions:



Noun: A noun is a person, place or thing, and is the building block of a sentence. i. Example: The car is red. Noun: car, red ii. Example: John and Fred bought new jackets in preparation for the wedding. Nouns: John, Fred, jackets, wedding, preparation

5/23/2011 5:06:41 PM

12. 



Part 2    SAT Preparation

Pronoun: The pronoun is the noun’s representative in the sentence; it replaces the noun but “speaks with the noun’s voice” —as such it takes the same verb number, conjugation, and tense. i. Example: Nate loaned the car to me. Pronoun: me ii. Example: the sisters wished they more time to shop. Pronoun: they

The SAT will ask questions that involve four different types of pronoun. Consult the chart for more information about the types of pronouns and their forms:

Type of Pronoun

Forms

Personal Pronouns: stand Singular: I/me, you, he/him, she/her, it in place for the names Plural: We/us, they/them of people and things. Examples: He and John went to the store. Possessive Pronoun: used Singular: mine, yours, his, hers, its to indicate possession, Plural: ours, theirs or ownership. Example: Those shoes are hers, so give them back! Demonstrative Pronoun: Singular: this, that used to indicate which Plural: these, those thing the speaker is referring to, and distinguishes that thing from other things. Example: You tried your best, and you can’t do any better than that. Relative Pronoun: refers back to people or things previously mentioned. Example: People who drink beer should quit before they get fat.

Who, Which, That

Note that the Forms of the pronouns are a complete list—there are only seven Personal pronouns, seven Possessive pronouns, four Demonstrative pronouns and three Relative pronouns (though Personal pronouns have a lot of variation, as we will see). See the following chart for more information about the Frequency of question-types of Noun-Pronoun Agreement:

chap12.indd 6

N-P Agreement type

% of N-P Agreement

Average # per test

Number

47.73

3

Ambiguity

20.45

1

Case

11.36

1

Relative Pronoun

11.36

99, what is one possible value of x?

Go to the next page chap24.indd 23

5/30/2011 1:17:39 PM

24.24

Part 3    Four Full Practice Tests

4  

   

4    4 

BOXCO’S OCTOBER BOX PRODUCTION uncorrugated

corrugated

black

2250

450

brown

4000

gray total

total

   4

  wz 2 +



y y = 2w3 * x z

14. What is the largest value of y with x = 1, z =2 and w = 3?

3000 7000

10000

11. Boxco manufactures boxes with both corrugated and uncorrugated cardboard, both of which are available in black, brown and grey. On the basis of the information presented in the table above, how many gray corrugated boxes were made in October?



x° z°

Note:: Figure Note Figurenot not drawn drawnto toscale scale 15. In the figure above, l || n and 35 < x < 45 and 45 70, what is the lowest possible value of q? (A) 70 (B) 73 (C) 75 (D) 77 (E) 84

2.

c

b a

c2 = a2 + b2

60° x s 45° s 2 45° 30° s x 3 Special Right Triangles 2x

If 3 + 6x = 15, what is 1 + 2x? (A) 5 (B) 10 (C) 15 (D) 20 (E) 25

Go to the next page chap24.indd 30

5/30/2011 1:17:43 PM

24.31

Chapter 24     SAT Practice Test #2

7   ,

   

,

,

7   7  ,

3. What will the 6th term of the above sequence be?

(A)

(D)

(B)

(E)

   7



5. Half of the students who took Mr Jones’ science class liked it and half of those who liked it plan to take another class with him. What is the probability that a randomly selected student in Mr Jones’ class will take it again next year?

1 9 1 (B) 6 (A)



(C)

1 4



(D)

1 2



(E)

2 3

(C)

4. Ricky receives $3 for every meal he serves, plus a weekly wage of $20. If he serves n meals in a week, which of the following expressions represents the total dollar amount Ricky receives in a week? (A) (20 + 3)*n (B) (n-1)*3 + 20 (C) 20*n (D) 3 + 20*n (E) 20 + 3*n



6. A swimming pool is 10 ft deep, 20 ft long and 20 ft wide. It is completely full of water. A cold—snap freezes the water and the ice expands to occupy 10% more volume than the water. What is the volume of the ice? (A) 4000 ft3 (B) 4400 ft3 (C) 4440 ft3 (D) 4444 ft3 (E) 2200 ft3

Go to the next page chap24.indd 31

5/30/2011 1:17:43 PM

24.32

Part 3    Four Full Practice Tests

7  

   

7   7 

VOTES FOR SCHOOL PRESIDENT A F 20% E 22%

B 15% C D

9%

y

(A) 90 (B) 150 (C) 220 (D) 240 (E) 310

B

C

8. Mike, Al and Larry play a game. Al asks Larry to whisper to him an integer that Mike isn’t allowed to know. If the integer is odd, Al announces twice the the number. If it is even, Al announces the number itself. If the number announced is 34, which of the following could have been the original number Larry whispered to Al? I. 3.4 II. 17 III. 34 (A) I only (B) II only (C) III only (D) II and III only (E) I, II and III



31%

A



   7

9. How many minutes are there in d days, h hours and m minutes? (A) 24d + 60h + m (B) 30(d + h) + 60m (C) 24*60d + 60h + m (D) 60*(d + h + m) (E) 60d + 24m + 24h

7. The graph above represents the results of the election for Edgewood High’s student body president. If half of the 2000 member senior class voted, how many votes did the 2nd place candidate receive?



x

10. In the xy coordinate system above, determine the sum of slopes of line segments AB , BC and CA .

19 16



(A)



(B) 29



(C)



(D)



16

13 6

16 6 19 (E) 6

Go to the next page chap24.indd 32

5/30/2011 1:17:44 PM

24.33

Chapter 24     SAT Practice Test #2

7  

   

7   7 

11. Jan’s new locker combination has 4 digits, each from 0 to 20. The rule for the combination is that the first digit must be prime, the second digit must be odd and the third digit must be the product of the first three digits. Which of the following could be the combination to the lock? (A) 1 – 3 – 3 – 9 (B) 4 – 3 – 12 – 19 (C) 5 – 1 – 5 – 11 (D) 5 – 2 – 6 – 11 (E) 3 – 1 – 3 – 7



   7



13. The base of a pyramid is a square with side length 2 and the height of each triangular face of the pyramid is 4. What is the height of the pyramid, as measured by the distance from the base to the apex?

(A) 2 7



(B) 3 3



(C) 13



(D) 15



(E) 17

5=|x+3| 3y = 2x

14. How many integers in the set of all integers from 1 to 200, inclusive, are multiples of 19?

z=y+6



12. For the system of equations listed above, what is one possible value of z?

1 3



(A)



(B) 2



(C)

10 3



(D)

20 3



(E)

23 3

(A) 9 (B) 10 (C) 11 (D) 18 (E) 19

3

Go to the next page chap24.indd 33

5/30/2011 1:17:44 PM

24.34

Part 3    Four Full Practice Tests

7  

   

7   7 



   7



17. In the xy coordinate plane, what is the minimum value of the function f(x) = 3x2 + 5x – 9 ? (A) –12 (B) –9 (C) –1 (D) 0 (E) 9

y

A

x l 15. The figure above shows line l and a point, A, not on that line. What is the equation of the line that contains point A and which is perpendicular to line l ?

2 3



(A) y = − x − 3



(B) y =



2 5 x − 3 2 3 5 x − (C) y = − 2 2



3 (D) y = x+5 2



(E)

16. If y =

y=

3 5 x − 2 2

18. The sum of a set of consecutive integers is 30. There are two possible sets for which this condition holds. How many elements are there, total, in both sets?

(A) 31 (B) 33 (C) 60 (D) 62 (E) 64

7x 2 what happens to the value of y when both x z3

and z are tripled?

1 9



(A) y is not changed



(B) y is multipled by 1 (C) y is multiplied by



(D) y is tripled (E) y is multiplied by 9

3

Go to the next page chap24.indd 34

5/30/2011 1:17:45 PM

24.35

Chapter 24     SAT Practice Test #2

7  

   

7   7 



   7



19. The cost of h hats is c dollars. If Jane wishes to spend Y% of X dollars on hats, which of the following expressions represents how many hats she will buy?

(A)

100y c * x h



(B)

100y h * x+y c



(C)

xy h * 100 c



(D)

x h * y c



x+y c (E) * y h

20. The figure above is a cube with surface area 96 square inches. How long is the shortest possible diagonal between any two corners in the cube?

(A) 4 2



(B) 6 6



(C) 6 2



(D) 4 6



(E) 6 6

STOP If you finish your work before time is called, you may check your work on this section only. Do not turn to any other section in the test.

chap24.indd 35

5/30/2011 1:17:46 PM

24.36

Part 3    Four Full Practice Tests

8  

   

8   8 

   8





Section 8 Time — 20 Minutes 19 Questions Turn to Section 8 of your answer sheet to answer the questions in this section. Directions. For each question in this section, select the best answer from among the choices given and fill in the corresponding circle on the answer sheet.

Each sentence below has one or two blanks, each blank indicating that something has been omitted. Beneath the sentence are five words or sets of words labeled A through E. Choose the word or set of words that, when inserted in the sentence, best fits the meaning of the sentence as a whole. Example: The volcano, long considered —— , has recently become ——: local townspeople have been evacuated from the area, out of range of ash and magma. (A) (B) (C) (D) (E)

threatening . . docile viable . . unlikely quiescent . . lethargic extinct . . endangered dormant . . active (A) (B) (C) (D) (E)

1. Higher than expected rainfall resulted in —— at harvest-time, filling the granaries entirely. (A) a mixture (B) a scarcity (C) a melange (D) a n assortment (E) a surplus 2. As the extent of the company’s corruption —— became known, the board of directors grew increasingly —— . (A) suddenly . . devious (B) eventually . . apathetic (C) gradually . . incensed (D) incrementally . . affable (E) p  recipitately . . perverse

3. The —— of the government’s new missile defense system is said to make the country safe from even the most —— foe.



(A) incoherence . . implacable (B) disjunction . . resolute (C) e ffectiveness . . innocuous (D) laxness . . energetic (E) e fficacy . . belligerent

4. As a boy he was considered ——, even ——: no one expected him in later life to become such a worldly, calculating businessman. (A) eccentric . . mundane (B) ingenuous . . naïve (C) t imorous . . obdurate (D) a ltruistic . . atavistic (E) d  evious . . innocent 5. Mickey was  hiker: he complained often and with great pathos. (A) an avid (B) a novice (C) a querulous (D) an intrepid (E) a genteel 6. The athletes complained that their coach’s pre-race speeches, filled with empty —— and —— sentiments, were not sufficiently motivating. (A) platitudes . . clichéd (B) aphorisms . . stirring (C) b analities . . exuberant (D) invective . . milquetoast (E) n  iceties . . churlish

Go to the next page chap24.indd 36

5/30/2011 1:17:46 PM

24.37

Chapter 24     SAT Practice Test #2

8  

   

8   8 



   8



Directions. The passages below are followed by questions based on their content; questions following a pair of related passages may also be based on the relationship between the paired passages. Answer the questions on the basis of what is stated or implied in the passages and in any introductory material that may be provided.

Questions 7-19 are based on the following passages.

Passage 2

The passages below are excerpted from a 2006 book of essays on the history of art. Passage 1

5

10

15

20

25

30

35

The fullest flowering of art in the past two centuries no doubt occurred under the watchful eye of the Pre-Raphaelite Brotherhood. The Brotherhood, a formal organization with a stated mission and a coherent if loose four-point ideology, 40 devoted itself to reacting against what it perceived to be the stilted products of generations of post-Renaissance imitators. In particular, they endeavored to return to the vivid coloration, intense detail and compositional complexity of 16th century Flemish and Italian art. And as a consequence of this 45 commitment they created some extraordinary art. To compare the Pre-Raphaelite Brotherhood to the anything that masquerades under the aegis of Modern Art is laughable. How can Rossetti’s Proserpine, with its evident mastery of texture, Classical posture and dynamic motion, be 50 evaluated by the same standards with which we judge Duchamp’s shameless Fountain? The reflexive commercialization of art in contemporary times, which is a natural and regrettable extension of shockvalue progenitors like Duchamp, is a symptom of a fundamental illness that is rotting the spirit of free and creative expression from the inside. Why else would Damien Hirst create a diamond-encrusted sculpture and then with the help of some rich friends, finance the purchase himself? In days gone by, art was supported by patronage: somebody paid you for a painting that was essentially worth it. If today through complicated back-room dealings artists can now invest successfully in their own work, how can it be said that what is being produced has any value? And if we take as a definition that art is what has value for its own sake but not only for its own sake, we’re left with a logical if stark question: can the high-profile pieces of today be called art in any meaningful sense of the word?

There is a strong reactionary tendency amongst modern critics who wish to carve themselves a little corner of territory in the landscape of cultural academia to deride that which they do not comprehend. That good-faith ignorance is lessened, however, by a deliberate and willful misunderstanding that is in its own right as “self-serving” and “empty” as the modern or post-modern pieces these critics affect to despise. Modern art is technically if vaguely defined as any piece or production that exists as a result of an abrogation of the artistic traditions of the past. As such, this classification must subsume not just the slightly more out-there Dadaists and Surrealists, but the Cubists, Fauvists, Impressionists and even the Romantics. Some would be shocked to know that the Pre-Raphaelites themselves are considered the first example of avant-garde art, reactionary in intent and purpose. So much for conservatism. The purpose of art is to extend and include. Pieces that meet those criteria can be a life-like and scrupulously detailed painting; but they can just as easily be light installations or field paintings or graffiti tags. Anyone who so narrowly restricts their definition of art to exclude objects that interest and arouse but which do not meet an arbitrary application of technical criteria should seriously re-evaluate their purpose in the art world full stop. Maybe they should be accountants, instead. 7. The tone of the first sentence of Passage 1 (“the fullest flowering…Brotherhood) is best described as



(A) equivocal (B) emphatic (C) moderate (D) obdurate (E) intemperate

8. The references to the “mission” (line 5) and “ideology” (line 6) serve to



(A) point out differing perspectives (B) indicate a political philosophy (C) distract from the essence of a view (D) represent aspects of a definition (E) suggest a response

Go to the next page chap24.indd 37

5/30/2011 1:17:46 PM

24.38

8  

Part 3    Four Full Practice Tests

   

8   8 

9. The “vivid coloration, intense detail and compositional complexity” (lines 9-10) are best described as (A) foundational elements of a theory of art history (B) superficial details of successful portraiture (C) important components of a little-known school of art (D) cosmetic aspects of oil painting (E) essential characteristics of art from an earlier time

10. In line 15, “aegis” most nearly means (A) protection (B) guise (C) label (D) style (E) ruse 11. How would the author of Passage 2 most likely respond to the assertion in lines 14-16 (“To compare… laughable”)? (A) By directing attention to the post-modern technical aspects of the later Pre-Raphaelites (B) By defending the artist’s right of compositional freedom (C) B  y sowing doubt about the academic credentials of the author (D) B  y pointing out that the Pre-Raphaelites embodied a vital feature of modernism (E) B  y discrediting the Pre-Raphaelites as derivative and unskilled 12. The author of Passage 1 mentions Damien Hirst in order to (A) undermine an opponent (B) buttress a moderate argument (C) temper a radical hypothetical question (D) support a strong claim (E) deduce a tentative conclusion



   8



13. Lines 29-32 (“in days gone by…worth it”), primarily serve to (A) imply that Damien Hirst has established a model that will be used in future (B) suggest a connection between the legitimacy of art and the existence of an external buyer (C) h  int at the profitability of the contemporary art market (D) d  raw a parallel between the price of a piece of artwork and its aesthetic value (E)  express appreciation over an unexpected development 14. In the first paragraph of Passage 2, the author makes the point that (A) many art critics are ignorant about their fields of study

(B) most art critics reject modernism out of hand (C) s ome critics embody the qualities they reject in modern art (D) m  odern art is enjoying a critical resurgence (E) a rt criticism as a field is reactionary and narrowminded

15. In line 52, “abrogation” most nearly means

(A) annulment (B) obligation (C) derivation



(D) extension (E) rejection

16. The artists mentioned in lines 55-56 (“Cubists, Fauvists, Impressionists and even the Romantics”) are offered as examples of (A) artists whose work does not give the appearance of what is normally thought of as modernism (B) artists whose work is generally considered too controversial to exhibit (C) a rtists who use modern technology to explore new dimensions of expression (D) a rtists who despite eccentric methods have gained mainstream recognition (E) a rtists whose work bear the hallmarks of fin de siècle modernism

Go to the next page chap24.indd 38

5/30/2011 1:17:47 PM

24.39

Chapter 24     SAT Practice Test #2

8  

   

8   8 

17. The final sentence of Passage 2 is characterized by (A) pathos (B) comparison (C) anecdote (D) metaphor (E) irony 18. The author of Passage 2 would most likely determine which of the following to be an example of “an arbitrary application of technical criteria” (line 67)?





   8



19. The authors of both passages develop their points through the use of (A) direct quotation (B) empirical observation (C) specific allusions (D) vivid imagery (E) appeal to emotion

(A) “loose four-point ideology” (lines 5-6) (B) “stilted products” (line 7) (C) “ the reflexive commercialization of art” (line 21) (D) “ vivid coloration…compositional complexity” (lines 9-10) (E) “ free and creative expression” (lines 25-26)

STOP If you finish your work before time is called, you may check your work on this section only. Do not turn to any other section in the test.

chap24.indd 39

5/30/2011 1:17:47 PM

24.40

Part 3    Four Full Practice Tests

9  

   

9   9 

   9



SECTION 9 Time – 20 minutes 16 Questions Turn to Section 9 of your answer sheet to answer the questions in this section.

Directions. For this section, solve each problem and decide which is the best of the choices given. Fill in the corresponding circle on the answer sheet.You may use any available space for scratchwork.

Reference Information

Notes 1.  The use of a calculator is permitted. 2.  All numbers used are real numbers. 3.  Figures that accompany problems in this test are intended to provide information useful in solving the problems.   They are drawn as accurately as possible EXCEPT when it is stated in a specific problem that the figure is not drawn to scale. All figures lie in a plane unless otherwise indicated. 4. Unless otherwise specified, the domain of any function f is assumed to be the set of all real numbers x for which f(x) is a real number.

r

l w

h

h



h

w l b A = πr2 1 V = lwh A = bh V = πr2h A = lw C = 2πr 2 The number of degrees of arc in a circle is 360. The sum of the measures in degree of the angles of a triangle is 180.

1. If 16 jellybeans in a jar of 20 are blue, then what is the probability that a randomly chosen bean will NOT be blue?

r

1 20 1 (B) 10 1 (C) 5 1 (D) 4 4 (E) 5 (A)

c

b a

c2 = a2 + b2

60° x s 45° s 2 45° 30° s x 3 Special Right Triangles 2x

2. For which of the following values of k will 4k + 3 be less than 20? (A) 4 (B) 5 (C) 6 (D) 7 (E) 8

Go to the next page chap24.indd 40

5/30/2011 1:17:47 PM

24.41

Chapter 24     SAT Practice Test #2

9  

   

9   9 

3. Lou has two more peanuts than walnuts and eight nuts total. How many walnuts does Lou have?

(A) 7 (B) 6 (C) 5 (D) 4 (E) 3



   9



5. If n is any odd number, which of the following must be even? (A) 2n + 1 (B) 2n - 3 (C) 3n + 2 (D) 3n + 3 (E) n + 4

B

A

C D Note : Figure not drawn to scale Note: Figure not drawn to scale

4. In the figure above, ABC is a triangle and AB = BC. If AC is one-half of the length of BC and point D bisects AC, what is the ratio of the length of AD to the length of AB?

1 6 1 (B) 5

(A)



(C)

1 4



(D)

1 3



(E)

1 2

6. For a certain kind of bicycle tire, the attainable speed, s, varies in direct proportion with the tread count, t. If a rider can achieve a speed of 18 mph on a tire with 6 treads, what speed will she attain on a tire with 12 treads?

(A) 48 mph (B) 36 mph (C) 30 mph (D) 24 mph (E) 12 mph

Go to the next page chap24.indd 41

5/30/2011 1:17:48 PM

24.42

Part 3    Four Full Practice Tests

9   D

   

9   9 

C

a

a

H

G

B

E

c d 0 9. If points a, b, c and d lie on a number line in the order shown above, which of the following must be true? I.

F

(A) 2 (B) 3 (C) 4 (D) 5 (E) 6

8. Let m  n be defined as mn + m +2n. If 4  x is 13, what is x?

(A) (B)

1 3 2 3

c+d >0 2 II.

7. In the figure above, ABCD and EFGH are both squares. If a = 6 and b = 3, how many squares the size of EFGH can fit inside square ABCD?



b

b A

   9



b >0 c

III. b + c > 0

(A) I only (B) II only (C) III only (D) I and II only (E) I, II and III

10. A positive integer, n, is tripled, increased by six, tripled again and then squared. Which of the following is a representation of the negative reciprocal of that number?

(A)



(B)



(C) 1 (D)

3 2





(E)

5 2







−1 (18 + 9n)2

−1 (18 + 3n)2 −1 (C) (3 + n)2 1 (D) (3n + 18n 2 ) 1 (E) (3n + 18)2

Go to the next page chap24.indd 42

5/30/2011 1:17:48 PM

24.43

Chapter 24     SAT Practice Test #2

9  

   

9   9 

11. In the xy coordinate plane, line m is the reflection of line n about the y-axis. If the slope of line n is 1 what is the 3 slope of line m?

(A) -3



(B) −



(C)



(D) 1



(E) 3



   9



13. If the function f is defined by f(x) = ax2 + bx + c, where a and c are negative constants, which of the following could be the graph of f?

1 3

1 3

(A)

(D)

(B)

(E)

(C)

B

A 12. The figure above is a grid and you can move along the grid from left to right and up and down (no diagonals). Each side of each square in the grid has length l. If backward moves are not allowed, how many paths length 7 are there from A to B? (A) 16 (B) 12 (C) 8 (D) 4 (E) 0

14. 160% of x is 300% of y. What is the ratio of x to y?

(A)



(B)



15 16

15 8 15 (C) 4 15 (D) 2 15 (E) 1

Go to the next page chap24.indd 43

5/30/2011 1:17:49 PM

24.44

Part 3    Four Full Practice Tests

9  

   

9   9 

   9





y B B D

A

A

(A) 1 (B) 2 (C) 3 (D) 4 (E) 6

D

E

F

x C

15. In the figure above, ABC is equilateral and DEF is formed by connecting the mid-points of AB, BC and CA. If the area of ABC is 12, what is the area of DEF?

C

16. In the xy coordinate plane above, the curve described by f(x) = x2 – 3 intersects with rectangle ABCD at two points. If the area of rectangle ABCD is 10 and BA = 1, then at what points do the curve and rectangle ABCD intersect?

(A) (–3, 15) and (3, 15) (B) (–4, 14) and (4, 14) (C) (–5, 22) and (5, 22) (D) (–6, 25) and (6, 25) (E) (–7, 28) and (7, 28)

STOP If you finish your work before time is called, you may check your work on this section only. Do not turn to any other section in the test.

chap24.indd 44

5/30/2011 1:17:50 PM

24.45

Chapter 24     SAT Practice Test #2

10  

   

10  

10 





10

SECTION 10 Time – 10 minutes 14 Questions Turn to Section 10 of your answer sheet to answer the questions in this section.

Directions. For each question in this section, select the best answer from among the choices given and fill in the corresponding circle on the answer sheet.

The following sentences test correctness and effectiveness of expression. Part of each sentence or the entire sentence is underlined; beneath each sentence are five ways of phrasing the underlined material. Choice A repeats the original phrasing; the other four choices are different. If you think the original phrasing produces a better sentence than any of the alternatives, select choice A; if not, select one of the other choices. In making your selection, follow the requirements of standard written English; that is, pay attention to grammar, choice of words, sentence construction and punctuation. Your selection should result in the most effective sentence—clear and precise, without awkwardness or ambiguity. EXAMPLE: In 1947, Jackie Robinson becoming the first AfricanAmerican to play Major League Baseball. (A) becoming the first African-American (B) became the first African-American (C) will becomes the first African-American (D) become the first African-American (E) is becoming the first African-American (A) (B) (C) (D) (E)

1. Jefferson endeavors to find a seat at the back of the movie theater so that his family do not notice him. (A) do not (B) did not (C) does not (D) would not (E) would have not 2. Of all the mythological beasts, the Loch Ness monster is the more dangerous because it is not only amphibious but temperamental too. (A) is the more dangerous because it is not only amphibious but temperamental too (B) is the most dangerous because it is not only amphibious but also temperamental (C) is the most dangerous because it is not only amphibious it is also temperamental (D) is the more dangerous being that it is amphibious and temperamental (E) a re the most dangerous because it is not only temperamentally amphibious 3. Alistair is a full-fledged member of the NRA, he has a carefully maintained gun cabinet in his home. (A) Alistair is a fully-fledged member of the NRA, he has a carefully maintained gun cabinet in his home (B) Alistair is a fully-fledged member of the NRA and a carefully maintained gun cabinet in his home (C) A  listair, a fully-fledged member of the NRA, has a carefully maintained gun cabinet in his home (D) Alistair, who has a carefully maintained gun cabinet in his home and is a fully-fledged member of the NRA (E) Having a carefully-maintained gun cabinet in his home, the NRA includes Alistair as a fully-fledged member

Go to the next page chap24.indd 45

5/30/2011 1:17:50 PM

24.46

10  

Part 3    Four Full Practice Tests

   

10   10  

4. Many businessmen disagree on how best to structure and what is the measure of effectiveness for an organisation. (A) how best to structure and what is the measure of effectiveness (B) how best to structure and measuring the effectiveness (C) how best to structure and be measuring the effectiveness (D) t he best structuring and how to measure of effectiveness (E) t he best structure and measure of effectiveness 5. Walking briskly through the antechamber, the draperies in the castle smelled of mold to the tourists. (A) the draperies in the castle smelled of mold to the tourists (B) t he draperies in the castle smelling of mold to the tourists (C) the castles’ draperies smelled of mold to the tourists (D) t he tourists noticed that the draperies in the castle smelled of mold (E) t he smell of the castle draperies was mold 6. Whether Wilberforce can manage to prevent his little brother from breaking household rules depends on his constant attention. (A) breaking household rules depends on (B) breaking household rules depending on (C) t o break household rules depends on (D) t o break household rules which is depending on (E) t o break household rules because of a dependence on 7. The boy takes his birthday very seriously and as such demanded it be celebrated with an event like jumping on trampolines, scuba diving and elephant rides. (A) celebrated with an event like (B) celebrated with an event such as (C) c elebrated with events being (D) c elebrated with events like (E) c elebrated with events and



  10

8. Although John studied this morning, since he does not feel confident about this afternoon’s test. (A) Although John studied this morning, since he does not feel confident about this afternoon’s test (B) Although John studied this morning, he does not feel confident about this afternoon’s test (C) A  lthough John studied this morning; he does not feel confident about this afternoon’s test (D) J ohn studied this morning, since he is not feeling confident about this afternoon’s test (E) J ohn studied this morning, he does not feel confident about this afternoon’s test 9. The sheer expressiveness and emotion of Robert Burns’ poetry cannot compare to Auden and Eliot. (A) to Auden and Eliot (B) with Auden and Eliot (C) t o that of Auden and Eliot (D) like Auden and Eliot (E) b etween Auden and Eliot 10. Harold Pinter, the recently deceased Nobel-prize winning playwright, is known for depicting marriage as combative and family being dysfunctional. (A) family being dysfunctional (B) family is dysfunctional (C) f amily was dysfunctional (D) family having been dysfunctional (E) f amily as dysfunctional 11. Although social injustice in many forms and varieties occur around the world, a recent burgeoning of non-profit organizations has emerged to fight this trend. (A) Although social injustice in many forms and varieties occur around the world (B) Although social injustice in many forms and varieties occurring around the world (C) A  lthough social injustice in many forms and varieties occurred around the world (D) A  lthough occurring around the world in many forms and varieties, social injustice (E) A  lthough social injustice in many forms and varieties occurs around the world

Go to the next page chap24.indd 46

5/30/2011 1:17:50 PM

24.47

Chapter 24     SAT Practice Test #2

10  

   

10   10  

12. The teacher believes that classroom-based testing which necessitates a great deal of rote memorization and that is beyond the scope of all but the most industrious student. (A) which necessitates a great deal of rote memorization and that is (B) w  hich necessitates a great deal of rote memorization being (C) which necessitates a great deal of rote memorization is (D) n  ecessitating a great deal of rote memorization and that is (E) w  ith the necessity of a great deal of rote memorization and that is



  10

14. Exploring the outer reaches of deep space, the diary of the cosmonaut reveals a mind that is at once tormented and hopeful. (A) deep space, the diary of the cosmonaut reveals (B) deep space, the diary of the cosmonaut reveal (C) d  eep space, the cosmonaut reveals in his diary (D) d  eep space, the cosmonaut revealing in his diary (E) d  eep space, with the diary of the cosmonaut revealing

13. The philosopher, Karl Popper, argued that everything we think of as magic or miracles is merely unexplained science. (A) we think of as magic or miracles is merely unexplained science (B) we think of as magic or miracles are merely unexplained science (C) w  e think of as magic and miracles being merely unexplained science (D) w  e think of magically or miracles is merely unexplained science (E) w  e think of as magic or miracles having been merely unexplained science

STOP If you finish your work before time is called, you may check your work on this section only. Do not turn to any other section in the test.

chap24.indd 47

5/30/2011 1:17:50 PM

Critical Reading Section 2 1 2 3 4 5 6 7 8 9 10 11 12

B E D C A B C D C C E C

Section 5 13 14 15 16 17 18 19 20 21 22 23 24

A C B E D B E C C A A C

1 2 3 4 5 6 7 8 9 10 11 12

Section 8

B C E A E B B A D A C A

13 14 15 16 17 18 19 20 21 22 23 24

B C A D D B D B C E B A

1 2 3 4 5 6 7 8 9 10

E C E B C A B D E C

11 12 13 14 15 16 17 18 19

Number Correct

Number Correct

Number Correct

Number Incorrect

Number Incorrect

Number Incorrect

D D B C E A E D C

Mathematics Section 4 9 40 10 4, 5, 6, 7, 8, 9 11 2250 12 4 13 300 14 1.5, 3/2 15 80-100 16 1243 17 365 18 36

B D B E C A A A

1 2 3 4 5 6 7 8

Section 7 1 2 3 4 5 6 7 8 9 10

D A E E C B C D C E

Section 9 11 12 13 14 15 16 17 18 19 20

C B D B E C B D C A

1 2 3 4 5 6 7 8

C A E C D B C D

9 10 11 12 13 14 15 16

Number Correct

Number Correct

Number Correct

Number Incorrect

Number Incorrect

Number Incorrect

A A B E D B C C

Writing Section 10

Section 3 1 2 3 4 5 6 7 8 9

CR-24.indd 48

C D B B A D E C C

10 11 12 13 14 15 16 17 18

E A B A D C D E B

19 20 21 22 23 24 25 26 27

C C D D E A A B E

28 29 30 31 32 33 34 35

C D C C C B D A

1 2 3 4 5 6 7

C B C E D A D

8 9 10 11 12 13 14

Number Correct

Number Correct

Number Correct

Number Incorrect

Number Incorrect

Number Incorrect

B C E E C A C

5/9/2011 5:40:55 PM

Chapter 24    SAT Practice Test #2

24. 49

The SAT Scoring Process Scoring. Each properly filled circle is compared by a computer with the correct answer. A correct response earns one point; an omitted response earns zero points. With each incorrect response to a multiple-choice choice question, one-quarter of a point is subtracted to adjust for the effect of random guessing. The SAT Writing section has 49 questions. If, for instance, a student had 38 correct responses, 8 incorrect responses, and 3 omitted, the ultimate raw score is determined like so: 38 right – 8 wrong = 38 – 2 = 36 Raw Points 4 The calculation of raw scores often involves rounding fractions to the nearest whole number. For example, a raw score of 36.75 is rounded to 37; a raw score of 38.5 is rounded to 39. Note: for the Writing section, the essay raw score counts for roughly 30% of your scaled score; your multiple-choice raw score counts for approximately 70%.

How to Score Practice Test #2 Critical Reading (Sections 2, 5, and 8)

• • • •

Step 1. Count the number of correct and incorrect responses for Section 2 and record the number in the space provided on the Scoring Sheet. Do not count the omitted responses. Step 2. Count the number of correct and incorrect responses for Section 5 and record the number in the space provided on the Scoring Sheet. Do not count the omitted responses. Step 3. Count the number of correct and incorrect responses for Section 8 and record the number in the space provided on the Scoring Sheet. Do not count the omitted responses. Step 4. Add up the number of correct and incorrect responses. Enter the totals into the Scoring Sheet. To calculate A, use the formula: Number Correct – Number Incorrect = A 4

• •

Step 5. To determine B, your rounded Critical Reading Raw Score, round A to the nearest whole number. Step 6. To determine your Critical Reading Score Range, find the Total Rounded Raw Score you obtained in the Critical Reading Conversion Table (Table 1). Enter this range in the appropriate box on the Scoring Sheet.

Maths (Section 4, 7, and 9)

• • • • •

SP-24.indd 49

Step 1. Count the number of correct and incorrect responses for the multiple-choice questions (1-8) in Section 4 and record the number in the space provided on the Scoring Sheet. Do not count the omitted responses. Step 2. Count the number of correct responses for the student-produced response questions (9-18) in Section 4 and record the number in the space provided on the Scoring Sheet. Step 3. Count the number of correct and incorrect responses in Section 7 and record the number in the space provided on the Scoring Sheet. Do not count the omitted responses. Step 4. Count the number of correct and incorrect responses in Section 9 and record the number in the space provided on the Scoring Sheet. Do not count the omitted responses. Step 5. Add up the number of correct and incorrect responses. Enter the totals into the Scoring Sheet. To calculate A, use the formula:

5/23/2011 5:13:07 PM

24. 50

Part 3    Four Full Practice Tests

Number Correct – Number Incorrect = A 4

• •

Step 6. To determine B, your Rounded Math Raw Score, round A to the nearest whole number. Step 7. To determine your Math Score Range, find the Total Rounded Raw Score you obtained in the Math Conversion Table (Table 2). Enter this range in the appropriate box on the Scoring Sheet.

Writing (Sections 3 and 10)

• • • •

Step 1. Enter your Essay Score for Section 1 in the appropriate box on the Scoring Sheet. Your score should range from 0 to 6. Step 2. Count the number of correct and incorrect responses in Section 3 and record the number in the space provided on the Scoring Sheet. Do not count the omitted responses. Step 3. Count the number of correct and incorrect responses in Section 10 and record the number in the space provided on the Scoring Sheet. Do not count the omitted responses. Step 4. Add up the number of correct and incorrect responses. Enter the totals into the Scoring Sheet. To calculate A, use the formula: Number Correct – Number Incorrect = A 4

• • •

SP-24.indd 50

Step 5. To determine B, your Rounded Writing Multiple-Choice Raw Score, round A to the nearest whole number. Step 6. To determine your Writing Scaled Score Range, use Table 3. Find the Total Writing Multiple-Choice Raw Score in the left column of Table 3 and the Essay Score across the top row of the table. Enter this range in the appropriate box on the Scoring Sheet. Step 7. To determine your Writing Multiple-Choice Subscore Range, find the Total Writing Multiple-Choice Raw Score on the Writing Multiple-Choice Conversion Table (Table 4). Enter this range into the appropriate box on the Scoring Sheet.

5/23/2011 5:13:07 PM

24. 51

Chapter 24    SAT Practice Test #2

SAT Practice Test #2 Scoring Sheet Critical Reading Section A. Section 2:

__________ # correct

___________ # incorrect

__________ # correct

___________ # incorrect

__________ # correct

___________ # incorrect

+

B. Section 5: +

C. Section 8: = D. Total Unrounded Raw Score:

__________ # correct



( __________ / 4) = __________ # incorrect A

E. Total Rounded Raw Score (nearest whole number):

_________ B

F. Critical Reading Scaled Score Range (see Table 1):

Maths Section G. Section 4: (questions 1-8)

__________ # correct

___________ # incorrect

+

H. Section 4: (question 9-18)

__________ # correct +

I. Section 7:

__________ # correct

___________ # incorrect

__________ # correct

___________ # incorrect

+

J. Section 9: =

SP-24.indd 51

5/23/2011 5:13:07 PM

24. 52

Part 3    Four Full Practice Tests

K. Total Unrounded Raw Score:

__________ # correct



( __________ / 4) = __________ # incorrect A

L. Total Rounded Raw Score (rounded to the nearest whole number):

_________ B

M. Math Scaled Score Range (see Table 2):

Writing Section N. Section 1:



Essay Score

O. Section 3:

__________ # correct

___________ # incorrect

__________ # correct

___________ # incorrect

+

P. Section 10: = Q. Total MC Unrounded Raw Score:

__________ # correct

R. Total MC Rounded Raw Score (nearest whole number):



( __________ / 4) = _________ # incorrect A

_________ B

S. Writing Scaled Score Range (see Table 3):

T. Writing MC Subscore Range (see Table 4):

SP-24.indd 52

5/23/2011 5:13:07 PM

24. 53

Chapter 24    SAT Practice Test #2

Table 1. Critical Reading Conversion Table Raw Score 67 66 65 64 63 62 61 60 59 58 57 56 55 54 53 52 51 50 49 48 47 46 45 44 43 42 41 40 39 38 37 36 35 34 33 32 31

SP-24.indd 53

Scaled Score 800 780-800 760-800 720-800 700-800 690-790 680-780 660-760 650-750 650-730 640-720 630-710 620-700 610-690 600-680 610-670 600-660 590-650 580-640 570-630 570-630 560-620 560-620 550-610 540-600 530-590 530-590 520-580 520-580 510-570 510-570 500-560 500-560 490-550 490-550 480-540 480-540

Raw Score 30 29 28 27 26 25 24 23 22 21 20 19 18 17 16 15 14 13 12 11 10 9 8 7 6 5 4 3 2 1 0 -1 -2 -3 -4 and below

Scaled Score 470-530 470-530 460-520 460-520 450-510 450-510 440-500 440-500 430-490 430-490 420-480 420-480 410-470 400-460 400-460 390-450 380-440 380-440 370-430 360-420 340-420 330-410 320-400 300-380 280-380 270-370 260-360 250-350 220-320 210-310 200-290 200-280 200-240 200-210 200

Table 2. Mathematics Conversion Table Raw Score 54 53 52 51 50 49 48 47 46 45 44 43 42 41 40 39 38 37 36 35 34 33 32 31 30 29 28 27 26 25

Scaled Score 800 760-800 720-800 700-780 690-770 680-740 670-730 650-710 640-700 630-690 620-680 600-680 590-670 580-660 570-660 560-650 560-640 550-630 540-620 540-620 530-610 530-610 520-600 510-590 510-570 500-560 490-550 490-550 480-540 470-530

Raw Score 24 23 22 21 20 19 18 17 16 15 14 13 12 11 10 9 8 7 6 5 4 3 2 1 0 -1 -2 -3 -4 and below

Scaled Score 450-520 450-510 450-510 440-500 430-490 430-490 420-480 410-470 400-460 390-450 370-450 360-440 350-430 340-420 330-410 320-420 310-410 300-400 290-390 270-370 260-360 250-350 230-330 210-310 200-300 200-280 200-260 200-240 200

5/23/2011 5:13:08 PM

24. 54 MC Raw Score 49 48 47 46 45 44 43 42 41 40 39 38 37 36 35 34 33 32 31 30 29 28 27 26 25 24 23 22 21 20 19 18 17 16 15 14 13 12 11 10 9 8 7 6 5 4 3 2 1 0 -1 -2 -3 -4 -5

SP-24.indd 54

Part 3    Four Full Practice Tests

0 650-690 620-690 600-690 580-680 570-670 550-670 530-660 520-640 520-630 500-620 500-620 490-610 480-600 480-590 470-590 460-580 460-580 450-570 440-560 430-550 430-550 420-540 410-530 400-520 390-510 380-500 380-490 370-480 370-480 360-470 350-460 340-460 330-450 320-440 310-430 300-420 300-420 290-400 280-400 280-400 260-380 250-370 240-360 230-350 220-340 220-340 210-330 200-320 200-300 200-290 200-270 200-250 200-230 200-230 200

1 660-730 640-720 620-720 600-720 580-710 570-710 570-700 560-690 550-680 640-670 540-670 530-660 520-650 510-640 500-630 490-620 490-620 480-610 470-600 460-590 450-580 440-570 430-560 430-560 420-550 410-540 400-530 390-520 380-510 370-500 370-500 360-490 350-480 340-470 340-460 330-450 320-450 310-430 300-430 290-420 280-410 270-400 260-390 250-380 240-370 230-370 220-360 210-340 200-330 200-310 200-300 200-280 200-270 200-260 200-230

Table 3. Writing Conversion Table Essay Score 2 3 680-740 700-760 660-740 680-760 640-740 670-760 620-740 640-760 600-730 630-760 590-720 610-740 580-710 600-730 580-710 600-720 570-700 590-710 570-700 580-710 560-690 580-700 560-690 570-700 550-680 570-700 540-670 560-690 530-660 550-680 520-660 550-670 520-650 540-660 510-640 530-660 500-620 510-650 490-600 510-630 480-590 500-620 470-590 490-620 460-580 490-610 450-580 480-600 440-580 470-600 430-560 460-590 420-550 450-490 420-550 450-580 410-540 440-570 400-530 430-560 390-520 420-550 380-510 410-540 370-500 400-530 360-490 390-520 360-480 390-510 360-480 380-500 350-470 370-490 340-460 360-480 320-460 350-480 310-450 340-470 300-430 330-460 290-420 320-450 290-410 310-440 270-410 300-430 260-390 290-430 250-380 290-420 240-380 270-410 240-370 270-400 220-350 250-380 210-340 240-370 200-330 230-350 200-310 230-340 200-290 210-310 200-280 200-300 200-250 200-280

4 740-790 720-790 700-780 680-780 670-780 640-760 630-750 620-750 620-750 610-740 600-730 590-730 590-720 580-710 570-710 570-700 560-690 550-680 540-670 530-660 530-650 520-650 520-640 510-630 500-620 490-620 480-610 470-600 460-590 460-590 450-580 440-570 430-560 420-550 410-540 400-530 390-520 380-510 370-500 360-490 360-480 350-470 340-460 330-460 330-450 320-450 310-440 300-430 280-410 270-410 260-390 260-370 240-360 230-330 220-300

5 780-800 760-800 730-800 720-800 700-800 670-800 660-790 660-790 650-780 640-770 630-760 620-760 620-750 610-740 600-740 600-730 590-720 580-710 570-700 560-690 550-680 540-670 530-670 520-660 520-650 510-640 500-630 490-620 480-610 480-610 480-610 470-600 460-590 450-580 440-470 430-560 420-550 410-540 400-530 390-520 380-510 370-500 360-490 350-480 350-480 340-470 330-450 320-440 310-440 300-430 290-420 290-410 280-390 270-360 260-340

6 800 780-800 760-800 750-800 730-800 720-800 690-800 680-800 670-800 670-800 660-790 650-780 650-780 640-770 630-760 620-750 610-740 600-730 590-720 580-710 580-700 570-690 560-690 550-670 540-660 530-650 520-650 520-650 510-640 510-640 500-630 490-620 480-610 470-600 470-600 460-590 450-580 440-570 430-560 420-550 410-540 400-530 390-520 380-510 370-500 360-490 350-480 340-470 330-460 330-460 320-450 320-420 310-410 300-400 300-380

5/23/2011 5:13:08 PM

24. 55

Chapter 24    SAT Practice Test #2

Table 4. Writing Multiple-Choice Conversion Table Raw Score 49 48 47 46 45 44 43 42 41 40 39 38 37 36 35 34 33 32 31 30 29 28 27 26 25 24

SP-24.indd 55

Scaled Score 78-80 77-80 74-80 72-78 70-78 69-77 68-76 67-75 66-74 64-74 63-73 62-72 61-71 60-70 59-69 58-68 57-67 56-66 55-65 54-64 53-63 52-62 51-61 50-60 49-59 48-58

Raw Score 23 22 21 20 19 18 17 16 15 14 13 12 11 10 9 8 7 6 5 4 3 2 1 0 -1 and below

Scaled Score 47-57 46-56 45-55 44-54 43-53 42-52 41-51 40-50 39-49 38-48 37-47 36-46 35-45 34-44 33-43 32-42 31-41 30-40 29-39 27-37 25-35 24-34 22-32 20-26 20

5/23/2011 5:13:08 PM

chapter

25 SAT Practice Test #3 SAT Reasoning Test—General Directions Timing

•• You will have 3 hours and 45 minutes to work on this test. •• There are nine separately timed sections: •• One 25-minute essay •• Five other 25-minute sections •• Two 20-minute sections •• One 10-minute section •• You may only work on one section at a time. •• If you finish a section before time is called, check your work on that section. You may NOT turn to any other section. Marking Answers

•• Be sure to mark your answer sheet properly. •• You must use a no. 2 pencil. •• Mark only one answer for each question. •• Make sure you color/shade the entire circle darkly and completely. •• Do not make any stray marks on your answer sheet. •• If you erase, do so completely. Incomplete erasures may be scored as intended answers. •• Use only the answer spaces that correspond to the question numbers. Scoring

•• For each correct answer, you receive one point. •• For questions you omit, you receive no points. •• For a wrong answer to a multiple-choice question, you lose one-fourth of a point. •• If you can eliminate one or more of the answer choices as wrong, you increase your chances of choosing the correct answer and earning one point.

•• If you can’t eliminate any choice, move on. You can return to the question later if there is time.

•• For a wrong answer to a student-produced response (“grid-in”) math question, you don’t lose any points. •• The essay is scored on a 1 to 6 scale by two different readers. The total essay score is the sum of the two readers’ scores. •• Off-topic essays, blank essays and essays written in ink will receive a score of zero.

chap25.indd 1

5/30/2011 1:25:00 PM

25. 2

Part 3    Four Full Practice Tests

2 3

1 2 3 4 5 6 7 8 9 10

A

1 2 3 4 5 6 7 8 9 10

B

C

D E

A

B

C

D E

A

B

C

D E

A

B

C

D E

A

B

C

D E

A

B

C

D E

A

B

C

D E

A

B

C

D E

A

B

C

D E

A

B

C

D E

A

B

C

D E

A

B

C

D E

A

B

C

D E

A

B

C

D E

A

B

C

D E

A

B

C

D E

A

B

C

D E

A

B

C

D E

A

B

C

D E

A

B

C

D E

11 12 13 14 15 16 17 18 19 20

A

11 12 13 14 15 16 17 18 19 20

B

D E

C

A

B

C

D E

A

B

C

D E

A

B

C

D E

A

B

C

D E

A

B

C

D E

A

B

C

D E

A

B

C

D E

A

B

C

D E

A

B

C

D E

A

B

C

D E

A

B

C

D E

A

B

C

D E

A

B

C

D E

A

B

C

D E

A

B

C

D E

A

B

C

D E

A

B

C

D E

A

B

C

D E

A

B

C

D E

21 22 23 24 25 26 27 28 29 30

A

21 22 23 24 25 26 27 28 29 30

B

D E

C

A

B

C

D E

A

B

C

D E

A

B

C

D E

A

B

C

D E

A

B

C

D E

A

B

C

D E

A

B

C

D E

A

B

C

D E

A

B

C

D E

A

B

C

D E

A

B

C

D E

A

B

C

D E

A

B

C

D E

A

B

C

D E

A

B

C

D E

A

B

C

D E

A

B

C

D E

A

B

C

D E

A

B

C

D E

31 32 33 34 35 36 37 38 39 40

A

B

C

D E

A

B

C

D E

A

B

C

D E

A

B

C

D E

A

B

C

D E

A

B

C

D E

A

B

C

D E

A

B

C

D E

A

B

C

D E

A

B

C

D E

31 32 33 34 35 36 37 38 39 40

A

B

C

D E

A

B

C

D E

A

B

C

D E

A

B

C

D E

A

B

C

D E

A

B

C

D E

A

B

C

D E

A

B

C

D E

A

B

C

D E

A

B

C

D E

. 0 1 2 3 4 5 6 7 8 9

. 0 1 2 3 4 5 6 7 8 9

. 0 1 2 3 4 5 6 7 8 9

. 0 1 2 3 4 5 6 7 8 9

. 0 1 2 3 4 5 6 7 8 9

. 0 1 2 3 4 5 6 7 8 9

Student Produced Responses 9

10 . 1 2 3 4 5 6 7 8 9

. 0 1 2 3 4 5 6 7 8 9

. 0 1 2 3 4 5 6 7 8 9

. 0 1 2 3 4 5 6 7 8 9

14

. 1 2 3 4 5 6 7 8 9

. 0 1 2 3 4 5 6 7 8 9

. 0 1 2 3 4 5 6 7 8 9

. 0 1 2 3 4 5 6 7 8 9

15 . 1 2 3 4 5 6 7 8 9

chap25.indd 2

11

. 0 1 2 3 4 5 6 7 8 9

. 0 1 2 3 4 5 6 7 8 9

. 0 1 2 3 4 5 6 7 8 9

12 . 1 2 3 4 5 6 7 8 9

. 0 1 2 3 4 5 6 7 8 9

. 0 1 2 3 4 5 6 7 8 9

. 0 1 2 3 4 5 6 7 8 9

16 . 1 2 3 4 5 6 7 8 9

. 0 1 2 3 4 5 6 7 8 9

. 0 1 2 3 4 5 6 7 8 9

. 0 1 2 3 4 5 6 7 8 9

13 . 1 2 3 4 5 6 7 8 9

. 0 1 2 3 4 5 6 7 8 9

. 0 1 2 3 4 5 6 7 8 9

. 0 1 2 3 4 5 6 7 8 9

17 . 1 2 3 4 5 6 7 8 9

. 0 1 2 3 4 5 6 7 8 9

. 0 1 2 3 4 5 6 7 8 9

. 0 1 2 3 4 5 6 7 8 9

. 1 2 3 4 5 6 7 8 9

18 . 1 2 3 4 5 6 7 8 9

. 0 1 2 3 4 5 6 7 8 9

. 0 1 2 3 4 5 6 7 8 9

. 0 1 2 3 4 5 6 7 8 9

. 1 2 3 4 5 6 7 8 9

5/30/2011 1:25:02 PM

25. 3

Chapter 25     SAT Practice Test #3

4 5

1 2 3 4 5 6 7 8 9 10

A

B

C

D E

A

B

C

D E

A

B

C

D E

A

B

C

D E

A

B

C

D E

A

B

C

D E

1 2 3 4 5 6 7 8 9 10

A

B

C

D E

A

B

C

D E

A

B

C

D E

A

B

C

D E

A

B

C

D E

A

B

C

D E

A

B

C

D E

A

B

C

D E

A

B

C

D E

A

B

C

D E

A

B

C

D E

A

B

C

D E

A

B

C

D E

A

B

C

D E

11 12 13 14 15 16 17 18 19 20

A

B

C

D E

A

B

C

D E

A

B

C

D E

A

B

C

D E

A

B

C

D E

A

B

C

D E

11 12 13 14 15 16 17 18 19 20

A

B

C

D E

A

B

C

D E

A

B

C

D E

A

B

C

D E

A

B

C

D E

A

B

C

D E

A

B

C

D E

A

B

C

D E

A

B

C

D E

A

B

C

D E

A

B

C

D E

A

B

C

D E

A

B

C

D E

A

B

D E

C

21 22 23 24 25 26 27 28 29 30

A

B

C

D E

A

B

C

D E

A

B

C

D E

A

B

C

D E

A

B

C

D E

A

B

C

D E

21 22 23 24 25 26 27 28 29 30

A

B

C

D E

A

B

C

D E

A

B

C

D E

A

B

C

D E

A

B

C

D E

A

B

C

D E

A

B

C

D E

A

B

C

D E

A

B

C

D E

A

B

C

D E

A

B

C

D E

A

B

C

D E

A

B

C

D E

A

B

D E

C

31 32 33 34 35 36 37 38 39 40

A

B

C

D E

A

B

C

D E

A

B

C

D E

A

B

C

D E

A

B

C

D E

A

B

C

D E

A

B

C

D E

A

B

C

D E

A

B

C

D E

A

B

C

D E

31 32 33 34 35 36 37 38 39 40

A

B

C

D E

A

B

C

D E

A

B

C

D E

A

B

C

D E

A

B

C

D E

A

B

C

D E

A

B

C

D E

A

B

C

D E

A

B

C

D E

A

B

C

D E

. 0 1 2 3 4 5 6 7 8 9

. 0 1 2 3 4 5 6 7 8 9

. 0 1 2 3 4 5 6 7 8 9

. 0 1 2 3 4 5 6 7 8 9

. 0 1 2 3 4 5 6 7 8 9

. 0 1 2 3 4 5 6 7 8 9

Student Produced Responses 9

10 . 1 2 3 4 5 6 7 8 9

. 0 1 2 3 4 5 6 7 8 9

. 0 1 2 3 4 5 6 7 8 9

. 0 1 2 3 4 5 6 7 8 9

14

. 1 2 3 4 5 6 7 8 9

. 0 1 2 3 4 5 6 7 8 9

. 0 1 2 3 4 5 6 7 8 9

. 0 1 2 3 4 5 6 7 8 9

15 . 1 2 3 4 5 6 7 8 9

chap25.indd 3

11

. 0 1 2 3 4 5 6 7 8 9

. 0 1 2 3 4 5 6 7 8 9

. 0 1 2 3 4 5 6 7 8 9

12 . 1 2 3 4 5 6 7 8 9

. 0 1 2 3 4 5 6 7 8 9

. 0 1 2 3 4 5 6 7 8 9

. 0 1 2 3 4 5 6 7 8 9

16 . 1 2 3 4 5 6 7 8 9

. 0 1 2 3 4 5 6 7 8 9

. 0 1 2 3 4 5 6 7 8 9

. 0 1 2 3 4 5 6 7 8 9

13 . 1 2 3 4 5 6 7 8 9

. 0 1 2 3 4 5 6 7 8 9

. 0 1 2 3 4 5 6 7 8 9

. 0 1 2 3 4 5 6 7 8 9

17 . 1 2 3 4 5 6 7 8 9

. 0 1 2 3 4 5 6 7 8 9

. 0 1 2 3 4 5 6 7 8 9

. 0 1 2 3 4 5 6 7 8 9

. 1 2 3 4 5 6 7 8 9

18 . 1 2 3 4 5 6 7 8 9

. 0 1 2 3 4 5 6 7 8 9

. 0 1 2 3 4 5 6 7 8 9

. 0 1 2 3 4 5 6 7 8 9

. 1 2 3 4 5 6 7 8 9

5/30/2011 1:25:03 PM

25. 4

Part 3    Four Full Practice Tests

6 7

1 2 3 4 5 6 7 8 9 10

A

B

C

D E

A

B

C

D E

A

B

C

D E

A

B

C

D E

A

B

C

D E

A

B

C

D E

1 2 3 4 5 6 7 8 9 10

A

B

C

D E

A

B

C

D E

A

B

C

D E

A

B

C

D E

A

B

C

D E

A

B

C

D E

A

B

C

D E

A

B

C

D E

A

B

C

D E

A

B

C

D E

A

B

C

D E

A

B

C

D E

A

B

C

D E

A

B

C

D E

11 12 13 14 15 16 17 18 19 20

A

B

C

D E

A

B

C

D E

A

B

C

D E

A

B

C

D E

A

B

C

D E

A

B

C

D E

11 12 13 14 15 16 17 18 19 20

A

B

C

D E

A

B

C

D E

A

B

C

D E

A

B

C

D E

A

B

C

D E

A

B

C

D E

A

B

C

D E

A

B

C

D E

A

B

C

D E

A

B

C

D E

A

B

C

D E

A

B

C

D E

A

B

C

D E

A

B

C

D E

21 22 23 24 25 26 27 28 29 30

A

B

C

D E

A

B

C

D E

A

B

C

D E

A

B

C

D E

A

B

C

D E

A

B

C

D E

21 22 23 24 25 26 27 28 29 30

A

B

C

D E

A

B

C

D E

A

B

C

D E

A

B

C

D E

A

B

C

D E

A

B

C

D E

A

B

C

D E

A

B

C

D E

A

B

C

D E

A

B

C

D E

A

B

C

D E

A

B

C

D E

A

B

C

D E

A

B

C

D E

31 32 33 34 35 36 37 38 39 40

A

B

C

D E

A

B

C

D E

A

B

C

D E

A

B

C

D E

A

B

C

D E

A

B

C

D E

A

B

C

D E

A

B

C

D E

A

B

C

D E

A

B

C

D E

31 32 33 34 35 36 37 38 39 40

A

B

C

D E

A

B

C

D E

A

B

C

D E

A

B

C

D E

A

B

C

D E

A

B

C

D E

A

B

C

D E

A

B

C

D E

A

B

C

D E

A

B

C

D E

.

. 0 1 2 3 4 5 6 7 8 9

. 0 1 2 3 4 5 6 7 8 9

. 0 1 2 3 4 5 6 7 8 9

. 0 1 2 3 4 5 6 7 8 9

. 0 1 2 3 4 5 6 7 8 9

. 0 1 2 3 4 5 6 7 8 9

Student Produced Responses 9

10 . 1 2 3 4 5 6 7 8 9

. 0 1 2 3 4 5 6 7 8 9

. 0 1 2 3 4 5 6 7 8 9

. 0 1 2 3 4 5 6 7 8 9

14

. 1 2 3 4 5 6 7 8 9

. 0 1 2 3 4 5 6 7 8 9

. 0 1 2 3 4 5 6 7 8 9

. 0 1 2 3 4 5 6 7 8 9

15 . 1 2 3 4 5 6 7 8 9

chap25.indd 4

11

. 0 1 2 3 4 5 6 7 8 9

. 0 1 2 3 4 5 6 7 8 9

. 0 1 2 3 4 5 6 7 8 9

12 . 1 2 3 4 5 6 7 8 9

. 0 1 2 3 4 5 6 7 8 9

. 0 1 2 3 4 5 6 7 8 9

. 0 1 2 3 4 5 6 7 8 9

16 . 1 2 3 4 5 6 7 8 9

. 0 1 2 3 4 5 6 7 8 9

. 0 1 2 3 4 5 6 7 8 9

. 0 1 2 3 4 5 6 7 8 9

13 . 1 2 3 4 5 6 7 8 9

. 0 1 2 3 4 5 6 7 8 9

. 0 1 2 3 4 5 6 7 8 9

. 0 1 2 3 4 5 6 7 8 9

17 . 1 2 3 4 5 6 7 8 9

. 0 1 2 3 4 5 6 7 8 9

. 0 1 2 3 4 5 6 7 8 9

. 0 1 2 3 4 5 6 7 8 9

1 2 3 4 5 6 7 8 9

18

. 1 2 3 4 5 6 7 8 9

. 0 1 2 3 4 5 6 7 8 9

. 0 1 2 3 4 5 6 7 8 9

. 0 1 2 3 4 5 6 7 8 9

. 1 2 3 4 5 6 7 8 9

5/30/2011 1:25:05 PM

25. 5

Chapter 25     SAT Practice Test #3

8 9 10

chap25.indd 5

1 2 3 4 5 6 7 8 9 10

A

B

C

D E

A

B

C

D E

A

B

C

D E

A

B

C

D E

A

B

C

D E

A

B

C

D E

A

B

C

D E

A

B

C

D E

A

B

C

D E

A

B

C

D E

1 2 3 4 5 6 7 8 9 10

A

B

C

D E

A

B

C

D E

A

B

C

D E

A

B

C

D E

A

B

C

D E

A

B

C

D E

A

B

C

D E

A

B

C

D E

1 2 3 4 5 6 7 8 9 10

A

B

C

D E

A

B

C

D E

A

B

C

D E

A

B

C

D E

A

B

C

D E

A

B

C

D E

A

B

C

D E

A

B

C

D E

A

B

C

D E

A

B

C

D E

A

B

C

D E

A

B

C

D E

11 12 13 14 15 16 17 18 19 20

A

B

C

D E

A

B

C

D E

A

B

C

D E

A

B

C

D E

A

B

C

D E

A

B

C

D E

A

B

C

D E

A

B

C

D E

A

B

C

D E

A

B

C

D E

11 12 13 14 15 16 17 18 19 20

A

B

C

D E

A

B

C

D E

A

B

C

D E

A

B

C

D E

A

B

C

D E

A

B

C

D E

A

B

C

D E

A

B

C

D E

11 12 13 14 15 16 17 18 19 20

A

B

C

D E

A

B

C

D E

A

B

C

D E

A

B

C

D E

A

B

C

D E

A

B

C

D E

A

B

C

D E

A

B

C

D E

A

B

C

D E

A

B

C

D E

A

B

C

D E

A

B

C

D E

21 22 23 24 25 26 27 28 29 30

A

B

C

D E

A

B

C

D E

A

B

C

D E

A

B

C

D E

A

B

C

D E

A

B

C

D E

A

B

C

D E

A

B

C

D E

A

B

C

D E

A

B

C

D E

21 22 23 24 25 26 27 28 29 30

A

B

C

D E

A

B

C

D E

A

B

C

D E

A

B

C

D E

A

B

C

D E

A

B

C

D E

A

B

C

D E

A

B

C

D E

21 22 23 24 25 26 27 28 29 30

A

B

C

D E

A

B

C

D E

A

B

C

D E

A

B

C

D E

A

B

C

D E

A

B

C

D E

A

B

C

D E

A

B

C

D E

A

B

C

D E

A

B

C

D E

A

B

C

D E

A

B

C

D E

31 32 33 34 35 36 37 38 39 40

A

B

C

D E

A

B

C

D E

A

B

C

D E

A

B

C

D E

A

B

C

D E

A

B

C

D E

A

B

C

D E

A

B

C

D E

A

B

C

D E

A

B

C

D E

31 32 33 34 35 36 37 38 39 40

A

B

C

D E

A

B

C

D E

A

B

C

D E

A

B

C

D E

A

B

C

D E

A

B

C

D E

A

B

C

D E

A

B

C

D E

A

B

C

D E

A

B

C

D E

31 32 33 34 35 36 37 38 39 40

A

B

C

D E

A

B

C

D E

A

B

C

D E

A

B

C

D E

A

B

C

D E

A

B

C

D E

A

B

C

D E

A

B

C

D E

A

B

C

D E

A

B

C

D E

5/30/2011 1:25:06 PM

25. 6

1 

Part 3    Four Full Practice Tests

  

ESSAY

ESSAY 





1



ESSAY Time—25 Minutes Turn to page 2 of your answer sheet to write your ESSAY. The essay gives you an opportunity to show how effectively you can develop and express ideas. You should, therefore, take care to develop your point of view, present your ideas logically and clearly and use language precisely. Your essay must be written on the lines provided on your answer sheet—you will receive no other paper on which to write. You will have enough space if you write on every line, avoid wide margins, and keep your handwriting to a reasonable size. Remember that people who are not familiar with your handwriting will read what you write. Try to write or print so that what you are writing is legible to those readers.

Important Reminders:

•• A pencil is required for the essay. An essay written in ink will receive a score of zero. •• Do not write your essay in your test book. You will receive credit only for what you write on your answer sheet. •• An off-topic essay will receive a score of zero. •• If your essay does not reflect your original and individual work, your test scores may be canceled. You have twenty-five minutes to write an essay on the topic assigned below.

Think carefully about the issue presented in the following excerpt and the assignment below. Consider the following paradox: “the one thing I can’t tolerate is intolerance”. This reflexive contradiction applies to moral imperatives and other immutable laws. Can a rule absolutely and unequivocally suggest that there might be times when it is not right to follow any rule? If so, when would we know not to follow that one? Assignment: Is there an exception to every rule? Plan and write an essay in which you develop your point of view on this issue. Support your position with reasoning and examples taken from your reading, studies, experience, or observations.

BEGIN WRITING YOUR ESSAY ON PAGE 2 OF THE ANSWER SHEET.

If you finish before time is called, you may check work on this section only. Do not turn to any other section in the test.

chap25.indd 6

5/30/2011 1:25:08 PM

25. 7

Chapter 25     SAT Practice Test #3

Section

1

IMPORTANT: Use a No. 2 PENCIL. Do NOT write outside the border! Words written outside the essay box or written in ink WILL NOT APPEAR in the copy sent to be scored and your score will be affected.

Begin your essay on this page. If you need more space, continue on the next page.

chap25.indd 7

5/30/2011 1:25:08 PM

25. 8

Part 3    Four Full Practice Tests

Continuation of ESSAY Section 1 from previous page. Write below only if you need more space. IMPORTANT: DO NOT START on this page—if you do, your essay may appear blank and your score may be affected.

chap25.indd 8

5/30/2011 1:25:08 PM

25. 9

Chapter 25     SAT Practice Test #3

2  

   

2    2 



   2



Section 2 Time—25 Minutes 24 Questions Turn to Section 2 of your answer sheet to answer the questions in this section.

Directions. For each question in this section, select the best answer from among the choices given and fill in the corresponding circle on the answer sheet.

Each sentence below has one or two blanks, each blank indicating that something has been omitted. Beneath the sentence are five words or sets of words labeled A through E. Choose the word or set of words that, when inserted in the sentence, best fits the meaning of the sentence as a whole. Example: The volcano, long considered ——, has recently become ——: local townspeople have been evacuated from the area, out of ränge of ash and magma. (A)  (B)  (C)  (D)  (E) 

threatening . . docile viable . . unlikely quiescent . . lethargic extinct . . endangered dormant . . active

(A) (B) (C) (D) (E)

1. The boy claimed to feel no ——, but it was clear he harbored some lingering resentment. (A) enthusiasm (B) charity (C) neutrality (D) animosity (E) etiquette

3. Dr Salter’s —— was complemented by his ——: his patients felt respected and fully informed. (A) benevolence . . megalomania (B) aversion . . elegance (C) vociferousness . . lucidity (D) eloquence . . erudition (E) cynicism . . coyness 4. The group’s goals were —— and ill-defined from the start, which perhaps accounted for their —— failure. (A) amorphous .. penultimate (B) distinct . . ineluctable (C) nebulous . . eventual (D) opaque . . subordinate (E) judicious . . subjective 5. Beneath her —— exterior, Wendy is actually —— in her daily habits and routines. (A) sagacious . . lackadaisical (B) indolent . . meticulous (C) nondescript . . palliative (D) picayune . . scrupulous (E) ascetic . . punctilious

2. The flight crew was —— but ——, doing their jobs thoroughly but with attention to their passengers’ comfort. (A) efficient . . conscientious (B) derelict . . convivial (C) fastidious . . brusque (D) impetuous . . careful (E) indolent . . amusing

Go to the next page chap25.indd 9

5/30/2011 1:25:08 PM

25. 10

2  

Part 3    Four Full Practice Tests

   

2    2 



   2



Directions. The passages below are followed by questions based on their content. Questions following a pair of related passages may also be based on the relationship between the paired passages. Answer the questions on the basis of what is stated or implied in the passages and in any introductory material that may be provided.

Questions 6-9 are based on the following passages. Passage 1

5

10

Malcolm X has been called a racist and a rabble-rouser, and is largely remembered for his controversial anti-white invective. However, it would be unwise and perhaps irresponsible to dismiss his legacy and accomplishments as a major figure in the Civil Rights movement. In ten years he grew the Nation of Islam from a regional organization with 400 members to a national political movement of almost 40,000; he parleyed with heads of state; he made a pilgrimage to Mecca and upon returning to America showed the courage and foresight to rescind most his previously-held hate-driven doctrines. His Autobiography paints a most compelling picture of a man who brought himself up from nothing to a position of power and influence, whose oratory and administrative capacities helped grow a small movement into a social revolution.

Passage 2

5

10

Malcolm X was in many respects the antithesis of Dr Martin Luther King Jr The latter’s moderate and intellectual philosophies of peace certainly contrasted with the screeds and diatribes of “X”. But you only had to watch Malcolm X talk publically to understand the man’s extraordinary charisma and political acumen. He later revealed that he took it upon himself with Dr King’s blessing to harangue and frighten the public with the militancy of his convictions, all in order to make the beliefs of MLK Jr a more viable and acceptable alternative. Did it work? Forty years later America voted in its first African-American President. You bet it worked.

6. 7.

Unlike the author of Passage 2, the author of Passage 1 (A) disproves the counterfactual (B) makes use of direct quotation (C) describes an individual (D) reference a literary work (E) provides a personal anecdote The main point of Passage 2 is primarily developed through the use of (A) comparison (B) metaphor (C) analogy (D) imagery (E) dialogue 8. Dr Martin Luther King Jr’s attitude as described in Passage 2 could best be described as (A) combative (B) analytical (C) pragmatic (D) peaceful (E) idealistic 9. The “hate-driven doctrines” mentioned in line 13 of passage 1 are most similar to (A) “screeds and diatribes” (line 23) (B) “extraordinary charisma” (line 25) (C) “political acumen” ( line 26) (D) “militancy of his convictions” (lines 28-29) (E) “a more viable and acceptable alternative” (line 30)

Go to the next page chap25.indd 10

5/30/2011 1:25:08 PM

25. 11

Chapter 25     SAT Practice Test #3

2  

   

2    2 

Questions 10-15 are based on the following passage. The following passage is excerpted from an 1857 novel in which a young boy prepares to go to boarding school.

5

10

15

20

25

30

35

Tom was carried off by the chambermaid in a brown study, from which he was roused in a clean little attic by that buxom person calling him a little darling and kissing him as she left the room which indignity he was too surprised at to resent. And still thinking of his father’s last words and the look with which they were spoken, he knelt down and prayed that, come what might, he might never bring shame or sorrow on the dear folks at home. Indeed, the Squire’s last words deserved to have their effect, for they had been the result of much anxious thought. All the way up to London he pondered what he should say to Tom by way of parting advice, something that the boy could keep in his head ready for use. By way of assisting meditation, he had even gone the length of taking out his flint and steel and tinder and hammering away for a quarter of an hour till he had manufactured a light for a long Trinchipoli cheroot, which he silently puffed; to the no small wonder of the coach driver, who was an old friend and an institution on the Bath road; and who always expected a talk on the prospects and doings, agricultural and social, of the whole country when he carried the Squire. To condense the Squire’s meditation, it was somewhat as follows: “I won’t tell him to read his Bible and love and serve God; if he won’t do that for his mother’s sake and teaching, he won’t for mine. Shall I go into the sort of temptations he’ll meet with? No, I can’t do that. Never do for an old fellow to go into such things with a boy. He won’t understand me. Do him more harm than good, ten to one. Shall I tell him to mind his work and say he’s sent to school to make himself a good scholar? Well, but he isn’t sent to school for that ‑ at any rate, not for that mainly. I don’t care a straw for Greek particles, or the digamma, no more does his mother. What is he sent to school for? Well, partly because he wanted to go. If only he’ll turn out a brave, helpful, truth-telling Englishman and a gentleman and a Christian, that’s all I want,” thought the Squire; and upon this view of the case framed his last words of advice to Tom, which were well enough suited for his purpose.



   2



10. The organization of the passage is best described as (A) a dialogue followed by an argument (B) a discussion followed by an explanation (C) a depiction followed by an analysis (D) an event followed by a remembrance (E) a description followed by a hypothetical scenario 11. The Squire goes to “the length of taking out f lint and steel and tinder” (lines 16-17) because (A) he wanted to pretend to ignore the coach driver’s questions (B) he required distraction from the tedious scenery (C) h  e thought it would help him find a solution to a problem (D) he found the atmosphere in the coach pleasant (E) h  e sought stress relief at the prospect of his son leaving home 12. In line 22, “institution” most nearly means (A) respected person (B) organization (C) corporation (D) charity (E) local church 13. The Squire considers “Greek particles” and “the diagamma” (line 38) as (A) elements of a Classical education (B) evidence of a strong language department at school (C) fundamental elements of scholarship (D) n  ot representative of what he wants his son to get out of his education (E) requirements to receive a scholarship to university 14. The Squire considers all the following pieces of advice EXCEPT (A) an exhortation to remain faithful to God (B) an injunction to study (C) a warning against potential dangers (D) a recommendation of honesty (E) a suggestion to play hard at games 15. The author suggests that the Squire’s final words to Tom (A) went unheeded in the main (B) were tentative and hesitant (C) covered a variety of subjects (D) were attended very closely (E) were sharp and imperative

Go to the next page chap25.indd 11

5/30/2011 1:25:08 PM

25. 12

2  

Part 3    Four Full Practice Tests

   

2    2 

Questions 16-24 are based on the following passage. This passage is excerpted from a 1994 book review of a historical study of an intellectual group in England in the 1700s.

5

10

15

20

25

30

35

The Lunar Society, so-called because its meetings took place in the light of the full moon - less for occult purposes than to mitigate the difficulties of night-time travel - was comprised of learned gentleman who variously occupied themselves as natural philosophers, or industrialists, or intellectuals. The society met regularly between the years of 1765 and 1813 in Birmingham, England, and included such luminaries as Erasmus Darwin (Charles’ father), inventor James Watt, and Josiah Wedgewood, teacup-maker extraordinaire. What do we know about this society? Little enough. The members collaborated on scientific experiments in the fields of electricity and geology; some of the fruits of these enquiries would turn out to have extensive industrial applications. The Lunar Society also maintained correspondence with other intellectual groups, most notably that of Benjamin Franklin; in this respect they enjoyed a measure of inf luence in affairs of State. The Lunar Society played a direct role in the development of the Midlands Enlightenment, upon which the Industrial Revolution of the 19th Century was predicated. It may be counter-productive to attempt to assess directly the utility of the Lunar Society: they were, in one sense, a group of likeminded individuals, with similar backgrounds and hobbies, who enjoyed each other’s company and conversation, not engaged in light-hearted pursuits exactly but certainly pursuing their interests at their leisure. But to view the Lunar Society in something of a broader context, as a salon whose members were scientists and businessmen, knowing that Britain was at that time standing at the brink of a technological and economic quantum leap, is to get the sense that open discussion and collaborative exploration amongst talented individuals can be the catalyst for remarkable happenings. And to venture even further afield, quite apart from the unique time and position these men occupied, any honest pledge to abide by the life of the mind has value in its own right, as a restorative from the pressures of everyday life and as a tonic to the qualities of curiosity and frankness that always attend progress made in any direction.



   2



16. The author suggests that the Lunar Society derived its name from concerns that were largely (A) secretive (B) practical (C) nocturnal (D) religious (E) mystical 17. In line 4, “learned” most nearly means (A) skilled (B) educated (C) wealthy (D) happy (E) successful 18. The purpose of the question at the start of the second paragraph is to (A) highlight a pressing concern (B) demonstrate mastery of a subject (C) identify the causes of a problem (D) challenge the aims of a group (E) lead into further explanation ofa topic 19. The author suggests that it might “be counter- productive to attempt to assess directly the utility of the Lunar Society” (lines 24-26) because (A) it was in the main a group of enthusiastic hobbyists (B) the group’s manifesto was essentially seditious (C) o n the surface it wouldn’t appear that the group had much value (D) t he scientific experiments of the members were largely unsuccessful (E) it is impossible to determine the impact of an individual within a group 20. In the description of the Lunar Society in lines 26- 31 (“they were...their leisure”), the actions of the group could best be characterised as (A) serious (B) scholarly (C) undignified (D) intangible (E) elective

Go to the next page chap25.indd 12

5/30/2011 1:25:08 PM

25. 13

Chapter 25     SAT Practice Test #3

2  

   

2    2 



   2

21. The author references “a broader context” in line 32 in order to (A) provide a different perspective (B) defend a serious charge (C) refute a logical fallacy (D) encourage a chain of reasoning (E) reinforce a prior claim

23. The third paragraph is best described as

22. The author suggests that the Lunar Society was the “catalyst” (line 37) for which event?







(A) The Industrial Revolution (B) The war of Independence (C) The Midlands Enlightenment (D) The invention of electricity (E) The development of the theory of evolution





(A) a comparison between unlike objects (B) a progression of decreasingly specific explanations (C) a logical deduction starting from self-evident premises (D) a n extended digression into the benefits of the scientific method (E) a series of interpretations

24. The primary purpose of the passage is to



(A) discuss a complicated argument (B) examine the motivations of a political body (C) d  iscuss the background and some activities of a group (D) determine the overall Utility of leisure time (E) encourage amateur scientists to make a contribution

STOP If you finish your work before time is called, you may check your work on this section only. Do not turn to any other section in the test.

chap25.indd 13

5/30/2011 1:25:08 PM

25. 14

Part 3    Four Full Practice Tests

3  

   

3    3 



   3



Section 3 Time—25 Minutes 35 Questions Turn to Section 3 of your answer sheet to answer the questions in this section.

Directions. For each question in this section, select the best answer from among the choices given and fill in the corresponding circle on the answer sheet.

The following sentences test correctness and effectiveness of expression. Part of each sentence or the entire sentence is underlined; beneath each sentence are five ways of phrasing the underlined material. Choice A repeats the original phrasing; the other four choices are different. If you think the original phrasing produces a better sentence than any of the alternatives, select choice A; if not, select one of the other choices. In making your selection, follow the requirements of standard written English; that is, pay attention to grammar, choice of words, sentence construction and punctuation. Your selection should result in the most effective sentence—clear and precise, without awkwardness or ambiguity. EXAMPLE: In 1947, Jackie Robinson becoming the first AfricanAmerican to play Major League Baseball. (A) becoming the first African-American (B) became the first African-American (C) will become the first African-American (D) becomes the first African-American (E) is becoming the first African-American (A) (B) (C) (D) (E)

1. Due to the heavy snowfall, Julius finds that he can only get to the store for the supplies his family needed if he will ride his snowmobile. (A) his family needed if he will ride (B) his family are needing if he will ride (C) his family need if he will ride (D) his family need if he is riding (E) his family needs if he rides 2. In the early 1940s, inexpensive comic books and mystery novels being known as pulp fiction were very easy to come by; lately, however, these have become collectors items are worth many hundreds of dollars each. (A) being known as pulp fiction were (B) known as pulp fiction were (C) known as pulp fiction was (D) being as they were known as pulp fiction were (E) otherwise known as pulp fiction were 3. In 1962, the mailing list of the first direct-order retail business had just over 500 names; today that list is grown to nearly six million households and Sears Roebuck is the largest retailer in the Midwest.

(A) is grown (B) is growing (C) grew (D) has grown (E) had grown

Go to the next page chap25.indd 14

5/30/2011 1:25:08 PM

25. 15

Chapter 25     SAT Practice Test #3

3  

   

3    3 

4. The teacher said that Henry’s short story was both clever because of being original and surprising because of its unexpected ending; this started him on a long and successful career as a writer. (A) was both clever because of being original (B) was both clever because of its originality (C) was clever because it was original (D) is cleverly original (E) because of its originality is clever 5. When preparing a team for a race, the coach must consider factors of physical fitness, psychological toughness and anxiety about having to perform in front of crowds. (A) anxiety about having to perform in front of crowds (B) anxiety about performing in front of crowds (C) a nxiety about being in front of crowds and performing (D) performance anxiety in front of crowds (E) a nxiety about having to be performing in front of crowds 6. It can be difficult to say whether a species on the verge of extinction has a strong chance of long-term survival. (A) whether a species on the verge of extinction has (B) whether a species on the verge of extinction have (C) w  hether a species on the verge of extinction is having (D) if a species on the verge of extinction having had (E) if a species on the verge of extinction were to have 7. A great public school which has had princes and Prime Ministers as its student, Eton is now recognized for providing financial aid to nearly 80% of its student body. (A) its student, Eton is now recognized (B) its student, Eton now being recognized for (C) its students, Eton is now recognized (D) their student, Eton are now recognized (E) their students, Eton are now recognizing



   3



8. As the daughter of two college professors, Jenny’s essays are almost always well-written and meticulously researched. (A) Jenny’s essays are almost always well-written and meticulously researched (B) the essays of Jenny were almost always well-written and meticulously researched (C) J enny writes essays that are almost always well-written and meticulously researched (D) J enny, being well-written and meticulously esearched, like her essays (E) a nd the well-written essays of Jenny are almost always meticulously researched 9. Rupert, stranded at the New York City airport for three straight days, realizing it was unwise of him not to have booked the travel insurance. (A) realizing it was unwise (B) realizing it is unwise (C) realizing it would be unwise (D) realized it was unwise (E) realized it is unwise 10. The local government, a collection of swindlers and confidence men, has been allowed to ransack the city treasury. (A) a collection of swindlers and confidence men, has been (B) a collection of swindlers and confidence men, are (C) a collection of swindlers and confidence men, have been (D) a collection of swindlers and confidence men and being (E) a collection of swindlers and confidence men, and 11. Because Brideshead Revisited has as its subject an aristocratic family and writer Evelyn Waugh is sometimes called a snob. (A) family and writer Evelyn Waugh is (B) family; and writer Evelyn Waugh is (C) family, writer Evelyn Waugh being (D) family, writer Evelyn Waugh is (E) family, with writer Evelyn Waugh being

Go to the next page chap25.indd 15

5/30/2011 1:25:09 PM

25. 16

3  

Part 3    Four Full Practice Tests

   

3    3 

Directions. The following sentences test your ability to recognize grammar and usage errors. Each sentence contains either a single error or no error at all. No sentence contains more than one error.The error, if there is one, is underlined and lettered. If the sentence contains an error, select the one underlined part that must be changed to make the sentence correct. If the sentence is correct, select choice E. In choosing answers, follow the requirements of standard written English. EXAMPLE: The other students and me decided to A B confront our teacher about the test. C D No error. E

(A) (B) (C) (D) (E)

12. The paleontologist who was examining the alleged ruins A B of the tower of Babel destroying the delicate operation C D with an ill-timed sneeze. No Error E 13. When Professor Carey learned that his partner and him A B C had won a MacArthur Foundation grant, they celebrated D in grand style. No Error E 14. One of the best collections of modern sports parapherna A B lia can be found at the Rowing Museum, located on the C D River Thames. No Error E



   3



15. Once Luis buys a fishing rod of his own, he would no A B C D longer be dependent on the generosity of his friends.

No Error E

16. It is argued that the Hercules beetle is proportionally A B more stronger than any other animal in existence: it can C D lift fifty times its body weight. No Error E 17. Numerologists have debated for many years the A significance of certain patterns in the Abrahamic holy B texts, in particular focusing on what do the chapter C D head ings mean. No Error E 18. The 1919 World Series of baseball has been studied by A B sociologists as the most publicized incident of fraud in



C the history of professional athletics. No Error D E

19. On the inscription of the book, Tom dedicated the novel A B to his mother, who managed to raise him without any C D help from family or friends. No Error E 20. Meteorologists predicted that the heavy snowfall A forecasted for the weekend could cause as much damage B C than a fully-formed tornado. No Error D E

Go to the next page chap25.indd 16

5/30/2011 1:25:09 PM

25. 17

Chapter 25     SAT Practice Test #3

3  

   

3    3 

21. Unless the outsourcing industry can tighten regulations A and quality control, they will be hard pressed to sustain B C D existing rates of growth through 2020. No Error E 22. Seven years after leaving the Holy Roman Empire in A dubious circumstances, Johann Kepler returned back from B C Austria in order to collect a debt from Charles V. D No Error E 23. The author’s central idea was that everyday happenings A B can be portrayed in a manner just as scary and C frighteningly as even the most gratuitously violent crimes. D No Error E 24. In Ayn Rand’s Fountainhead, Howard Roark was A regarded to be not only radical and unconventional,



B but also a threat to the established order of the guild of C D architects. No Error E

   3





25. The availability of skilled labor and the local operating A costs are always factors when considering whether to B C expand a business. No Error D E 26. The central tenet of modern democracy consists in the A notion that everyone has a right to their opinions.



B

C

D

No Error E

27 The foreign disputation, recently arrived and settled in A B the embassy compound, found the strangeness of the



C climate as disagreeable as the blandness of the food. No Error D E



28. The vice-president of the company weighed strongly A B against the proposed buyout, arguing that the number C D possible advantages far exceeds the increased overheads.

No Error E

29. Among the guiding principles of the political A philosophy of libertarianism are a total commitment to B C D free discourse and free market. No Error E

Go to the next page chap25.indd 17

5/30/2011 1:25:09 PM

25. 18

3  

Part 3    Four Full Practice Tests

   

3    3 

Directions. The following passage is an early draft of an essay. Some parts of the passage need to be rewritten. Read the passage and select the best answers for the questions that follow. Some questions are about particular sentences or parts of sentences and ask you to improve sentence structure or word choice. Other questions ask you to consider organization and development. In choosing answers, follow the requirements of standard written English.

Questions 30-35 refer to the following passage.

5

10

15

20

(1) Song writing isn’t as easy as people make it out to be. (2) It takes effort to match that perfect melody with thought-provoking lyrics. (3) It famously took John Lennon two hours to write ‘Imagine’ (4) I don’t think anyone but he could have done it in less than two years, it takes me at least three months to write a composition I am proud of. (5) The great debate is whether lyrics or melody comes first. (6) I favor the latter, as I find it easier to hit on a tune than to write words. (7) But some people think that it’s not just a question of easiness. (8) That it reflects a fundamental and underlying judgment on the importance of words versus notes. (9) It can get pretty heated. (10) It might just be best to divide the labor and assign one person the duties of writing lyrics and another the job of coming up with a good tune. (11) The only problem with that of course is when you get two people with two different concepts. (12) But even then the result can be interesting and worth exploring. (13) Some people say it is impossible to judge the real worth of a song. 30. Which is the best sentence to place after sentence 2? (A) To be fair, for some people it comes naturally. (B) Not every problem has a quick solution. (C) Cartoonists often have the same problem. (D) Harvard did a study on this very topic. (E) You must always follow your artistic instinct.



   3



31. Which is the best version of the underlined portion of sentence 4 (reproduced below)?

I don’t think anyone but he could have done it in less than two years, it takes me three months to write a composition I am proud of.



(A) (As it is) (B) less than two years, taking me sometimes as long as (C) less than two years, I am taking (D) less than two years; for instance, it takes me (E) fewer than two years, with me it is taking

32. In context, which of the following would be the best way to combine sentence 7 and sentence 8 (reproduced below)?

But some people think that it’s not just a question of easiness. That it reflects a fundamental and underlying judgment on the importance of words versus notes.



(A) But some people think that it reflects a fundamental and underlying judgment on the importance of words versus notes and it’s not just a question of easiness. (B) But some people think that the choices made during the process of songwriting reflect a fundamental and underlying judgment on the importance of words versus notes. (C) S ome people think that it’s not just a question of easiness and that it reflects a fundamental and underlying judgment on the importance of words versus notes. (D) M  oreover, some people think that it’s not just a question of easiness; nevertheless, it reflects a fundamental and underlying judgment on the importance of words versus notes. (E) R  egardless, some people think that the matter should not be considered a question of easiness, it reflects a fundamental and underlying judgment on the importance of words versus notes.









33. In context, what would be a more precise replacement for the word “It” in sentence 9 (reproduced below)? It can get pretty heated. (A) The songwriting process (B) The debate between wordsmiths and melody-makers (C) The tension between critics and artists (D) The relationship between consumers and singers (E) The professional reluctance of songwriters

Go to the next page chap25.indd 18

5/30/2011 1:25:09 PM

25. 19

Chapter 25     SAT Practice Test #3

3  

   

3    3 

34. Which sentence should be deleted from the passage?

(A) Sentence 2 (B) Sentence 5 (C) Sentence 9 (D) Sentence 11 (E) Sentence 13



   3



35. Which is the best version of the underlined portion of sentence 12 (reproduced below)?

But even then the result can be interesting and worth exploring.



(A) (As it is) (B) is worth it to explore (C) can be worth exploring (D) is worth exploring (E) worthy of being explored

STOP If you finish your work before time is called, you may check your work on this section only. Do not turn to any other section in the test.

chap25.indd 19

5/30/2011 1:25:09 PM

25. 20

Part 3    Four Full Practice Tests

4  

   

4    4 

   4



SECTION 4 Time – 25 minutes 18 Questions Turn to Section 4 of your answer sheet to answer the questions in this section.

Directions. For this section, solve each problem and decide which is the best of the choices given. Fill in the corresponding circle on the answer sheet.You may use any available space for scratchwork.

Notes

Reference Information

1.  The use of a calculator is permitted. 2.  All numbers used are real numbers. 3. Figures that accompany problems in this test are intended to provide information useful in solving the problems.   They are drawn as accurately as possible EXCEPT when it is stated in a specific problem that the figure is not drawn to scale. All figures lie in a plane unless otherwise indicated. 4. Unless otherwise specified, the domain of any function f is assumed to be the set of all real numbers x for which f(x) is a real number.

1.

r

l w

h

h

r

h

w l b A = πr2 1 V = lwh A = bh V = πr2h A = lw C = 2πr 2 The number of degrees of arc in a circle is 360. The sum of the measures in degree of the angles of a triangle is 180. If s + q + t = 8 and s + q = 4, what is t? (A) -8 (B) -4 (C) 2 (D) 4 (E) 8

c

b a

c = a2 + b2 2

60° x s 45° s 2 45° 30° s x 3 Special Right Triangles 2x

2. Maria needs two eggs and three sticks of butter to make a cake. If she wants to make three cakes, how many eggs and sticks of butter will she need in total? (A) 10 (B) 12 (C) 15 (D) 18 (E) 21

Go to the next page chap25.indd 20

5/30/2011 1:25:10 PM

25. 21

Chapter 25     SAT Practice Test #3

   

4    4 

3. If the total area of three identical squares is 7 square inches, what is the area, in square inches, of two of the squares?



14 (D) 3



(E) 17 3

(B)

Frequency





2 1 4 5 6 7 8 9 Shoe Sizes 3 2 1 4 5 6 7 8 9 Shoe Sizes

(C)

4. A perfect number is an integer whose factors sum to form that integer, for example, 6 = 3 + 2 + 1. Which of the following is a perfect number? (A) 28 (B) 24 (C) 20 (D) 12 (E) 10

Frequency



3 7 6 (B) 7 7 (C) 3

   4

3

(A)

3 2 1 4 5 6 7 8 9 Shoe Sizes

(D)

Frequency



(A)



Frequency

4  

3 2 1

Students

a

b

c

d

e

f

g

h

i

Shoe Size

5

8

9

4

7

4

8

8

7

5. Which of the following graphs represents the information on students’ shoe sizes presented in the table above? The first letter of each student’s name is in the top row and his or her shoe size underneath.

(E)

Frequency

4 5 6 7 8 9 Shoe Sizes

3 2 1 4 5 6 7 8 9 Shoe Sizes

Go to the next page chap25.indd 21

5/30/2011 1:25:11 PM

25. 22

Part 3    Four Full Practice Tests

4  

   

4    4 

O



   4



8. If a and b are positive integers and 4a2 = 16b4, which of the following could equal a + b? I. 0 II. 2 III. 3 (A) I only (B) II only (C) III only (D) I and III only (E) I, II and III

6. In the circle in the figure above, the circumference is 2π. How many points are 1 unit from point O? (A) 4 (B) 8 (C) 16 (D) 32 (E) More than 32

7. A farmer has to make a feed for his pigs. He mixes grain, seeds and meat for the feed and he has available four kinds of grain, three kinds of seed and two kinds of meat. If he wants to serve a different mixture every day, for how many full weeks could he serve a different mixture each day? (A) 1 (B) 2 (C) 3 (D) 4 (E) 5

Go to the next page chap25.indd 22

5/30/2011 1:25:12 PM

25. 23

Chapter 25     SAT Practice Test #3

4  

   

4    4 

   4





Directions. For Sudent-Produced Response questions 9-18, use the grids at the bottom of the answer sheet page on which you have answered questions 1-8. Each of the remaining 10 questions requires you to solve the problem and enter your answer by marking the circles in the special grids, as shown in the examples below. You may use any available space for scratchwork. Answer: Write answer in boxes.

0

0

0

1

1

1

•• •• ••

1

0

0

0

1

1

1

not

3 1 / 2

1

0

0

0

1

1

1

1

0

0

0

1

1

1

2

2

2

2

2

2

2

2

2

2

2

2

2

2

2

2

3

3

3

3

3

3

3

3

3

3

3

3

3

3

3

4

4

4

4

4

4

4

4

4

4

4

4

4

4

4

4

5

5

5

5

5

5

5

5

5

5

5

5

6

6

6

6

6

6

6

6

7

7

7

7

7

7

7

7

8

8

8

8

8

8

8

8

9

9

9

9

9

9

9

9

2

1 3 2

Decimal point

3

will receive credit only if the circles are filled in correctly. Although not required, it is suggested that you write your answer in the boxes at the top of the columns to help you fill in the circles accurately. Some problems may have more than one correct answer. In such cases, grid only one answer. No question has a negative answer. 1 Mixed numbers such as 3 must be gridded as 3.5 or

7/2. (If

2 0 1 2 0 1

2 . 5

Fraction line

•• Mark no more than one circle in any column. •• Because the answer sheet will be machine-scored, you ••

Answer: 201 Either position is correct.

Answer: 2.5

7 / 1 2 1

Grid in result.

7 12

is gridded, it will be interpreted

31 as 2

.).

•• Decimal Answers: If you obtain a decimal answer with

more digits than the grid can accommodate, it may be either rounded or truncated, but it must fill the entire

9. If Rochelle can catch three fish per day, at that rate how many fish will she catch in two weeks?

Note : You may start your answers in any column, space permitting. Columns not needed should be left blank.

grid. For example, if you obtain an answer such as 0.6666..., you should record your result as .666 or .667. A less accurate value such as .66 or .67 will be scored as incorrect. Acceptable ways to grid

2 / 3 1

0

0

0

1

1

1

2 are: 3

. 6 66 1

0

0

0

1

1

1

. 6 67 1

0

0

0

1

1

1

2

2

2

2

2

2

2

2

2

2

2

2

3

3

3

3

3

3

3

3

3

3

3

3

4

4

4

4

4

4

4

4

4

4

4

4

5

5

5

5

5

5

5

5

5

5

5

5

6

6

6

6

6

6

6

6

6

6

6

6

7

7

7

7

7

7

7

7

7

7

7

7

10. What is the greatest 4-digit integer that has 11 as a factor?

Go to the next page chap25.indd 23

5/30/2011 1:25:13 PM

25. 24

Part 3    Four Full Practice Tests

4  

   

4    4 

   4





14. If x is a positive even integer and x < 8, what is one possible value of 2x-1 ? y° x°

z° z°

y° x°

m 11. In the figure above, if m is a line, what is x + y + z ?

12. The area of a rectangular plot of land is 8000 sq ft. If the length of one side of the plot is 40 ft, what is the perimeter of the plot?

h (p) = p2 + 3 g (p) = p + 5 13. What is the different between h(g(3)) and g(h(3))?

15. In the xy coordinate plane, circle A has radius three and is tangent to circle B, which has radius four. Circle B is centered at the origin. If the center of circle A is (0, t) and t > 0, what is t?

x 0

2

16. On the number line above there are 5 equal intervals between 0 and 2. If x = 1 + p, what is p?

Go to the next page chap25.indd 24

5/30/2011 1:25:13 PM

25. 25

Chapter 25     SAT Practice Test #3

4  

   

4    4 

17. If Jane does not study for her final exam, she will pass with probability p and fail with probability f. If she studies, she will pass with probability q and fail with probability r. If studying doubles Jane’s chances of passing and if she has a 70% chance of failing if she doesn’t study, what is the percent chance that she will fail even if she studies?



   4



18. The terms of two sequences are added to make a third sequence. If the first sequence is defined as multiples of 7, and the second sequence is defined as multiples of 9, what will be the 100th term of the third sequence?

STOP If you finish your work before time is called, you may check your work on this section only. Do not turn to any other section in the test.

chap25.indd 25

5/30/2011 1:25:13 PM

25. 26

Part 3    Four Full Practice Tests

5  

   

5    5 



   5

SECTION 5 Time – 25 minutes 24 Questions Turn to Section 5 of your answer sheet to answer the questions in this section.

Directions. For each question in this section, select the best answer from among the choices given and fill in the corresponding circle on the answer sheet.

Each sentence below has one or two blanks, each blank indicating that something has been omitted. Beneath the sentence are five words or sets of words labeled A through E. Choose the word or set of words that, when inserted in the sentence, best fits the meaning of the sentence as a whole. Example: The volcano, long considered ——, has recently become ——: local townspeople have been evacuated from the area, out of ränge of ash and magma. (A) threatening . . docile (B) viable . . unlikely (C) quiescent . . lethargic (D) extinet . . endangered (E) dormant . . active

(A) (B) (C) (D) (E)

1. The bat is —— predator, leaving its nest to hunt its —— during the nighttime hours. (A) an ambivalent.. repast (B) a nocturnal . . prey (C) an irregular . . victims (D) an adroit . . companions (E) a sophisticated . . rivals 2. Even the opposing fans —— the athlete’s performance for its bravery. (A) decried (B) rejected (C) merited (D) lauded (E) assisted

3. The —— of the customer was appalling: he sneered at every produet in the störe.

(A) insolence

(B) euphoria



(D) imprudence

(E) ingenuity

(C) vitality

4. The author said the cover art —— the novel, capturing its essence perfectly. (A) authorized (B) deified (C) epitomized (D) legitimized (E) nullified 5. The —— of Monaco gained independence nine years ago and has since ratified a Constitution and established a parliament. (A) principality (B) lacuna (C) amalgam (D) legality (E) conglomeration 6. The anesthesiologist —— spoiled the mixture when he misread the label and added too much nitrogen. (A) irascibly (B) deftly (C) ambiguously (D) ubiquitously (E) inadvertently 7. The scientist‘s argument was ——, using as a premise what it intended to conclude. (A) circular (B) tautological (C) entropic (D) reduetive (E) auspicious 8. Isla’s putative charm actually consisted of ostentatious groveling and flattery. (A) surreptitious (B) obsequious (C) muted (D) apposite (E) sporadic

Go to the next page chap25.indd 26

5/30/2011 1:25:13 PM

25. 27

Chapter 25     SAT Practice Test #3

5  

   

5    5 



   5



The passages below are followed by questions based on their content; questions following a pair of related passages may also be based on the relationship between the paired passages. Answer the questions on the basis of what is stated or implied in the passages and in any introductory material that may be provided.

5

10

Questions 9-10 are based on the following passage.

Questions 11-12 are based on the following passage.

Emergency government bailout checks with 10 zeroes. A system of securitized commodities so complex that the head of the US Federal bank had to have someone explain it to him. And the mightiest and proudest masters of the universe— Lehman, AIG, Bear Sterns—shutting their doors for good. The financial crisis of 2008 may have seemed like a novelty— “nothing this bad has ever happened before!” —but that is true only in scope, not species. Investment banks have been underwriting risky commoditized debt for almost two hundred years: indeed, all you need to do is crack the spine of Anthony Trollope’s The Way We Live Now to enter a Victorian world where the only difference between now and then is that back then speculators bet on railroads, not mortgages.

The traditions of Congolese sapeurs—modern dandies who wear expensive designer clothing despite themselves living in conditions of ultra-poveity—are provocative, to say the least. Picture Papa Griffe, well-known sapeur, striding down a Kinshasa avenue in flamboyant Versace and Prada top-shelf items. Behind him run crowds of barefoot children to whom he is a hero, a legend, a star. They inhabit the same environs but live in different worlds. This juxtaposition provides a fertile jumping-off point for a consideration of the ideas and concepts behind fashion. Is it art? Is it frippery? Is it negligent for a man to wear a suit of clothes whose cost exceeds his neighbors’ average yearly household income? Or does this willingness itself display the best and most resilient tendencies of the human spirit ?

9. The author uses incomplete sentences in the first paragraph primarily to (A) express disappointment with the financial advisers who allowed the collapse to occur (B) heighten the dramatic tension of a depiction (C) c all attention to the irresponsibility of the US Federal Reserve (D) establish an academic basis for dispute (E) undermine a set of preconceptipns 10. In lines 10-11, “scope, not species” refers to (A) the magnitude of the financial crisis (B) the demonstrated human cost of the financial crisis (C) the unparalleled avarice of certain money managers (D) the depressions of previous epochs (E) Trollope’s depiction of Victorian speculators

5

10

11. The organisation of the passage is best characterised by which of the following ? (A) A historical overview followed by a series of predictions (B) An analysis of recent happens followed by a general waming (C) A  depiction of a situation followed by a philosophical line of inquiry (D) A  recollection followed by the development of a theory (E) A parable followed by a firm conclusion 12. In line 13, “frippery” most nearly means (A) dry humor (B) vain pretense (C) failed attempt (D) belligerent posturing (E) bland gesture

Go to the next page chap25.indd 27

5/30/2011 1:25:13 PM

25. 28

5  

Part 3    Four Full Practice Tests

   

5    5 

Questions 13-24 are based on the following passages. In the following passages, two authors evaluate the literary and artisüc merits of graphic novels. The first passage is excerpted’from a 1987 critical essay; the second passage is excerpted from a 2006 introduction to a graphic novel.

5

10

15

20

25

30

35



Passage 2

40

Passage 1 Gone are the brightly-colored and vividly-inked BIFFs and POWs and ZAMs of your grandfather’s comic books; gone, in fact, are comic books altogether. Today, the idiom is “graphic novel” and the new aesthetic is dark, gritty, untamed and real. The artists and writers at the vanguard of this movement— Alan Moore, Frank Miller, David Mazzuchelli and many others—strive after realism in every respect: plotting, dialogue, illustration. Coloring. Pacing. Inking. Realism is the watchword. It’s got to be real. Really, really real. But what’s so real about a grown man fighting criminals in his underwear? The application of a postmodern perspective to the field of comic books creates a unique juxtaposition. On the one hand, we retain at least nominally the heroes of our childhood, figures we are used to seeing do childish things—flying, dodging bullets, etc—in the clean and well maintained futuristic city-scapes in which there are no homeless and no taxes. On the other hand, we are now imagining these old heroes—heroes from the same childhood, mind you, still associated with all the childhood needs and expectations of security and warmth—in blighted urban environments, with corrupt politicians, unjust wars and social ills of every kind, adults with adult problems in very adult world. This is the ultimate dislocation, the ultimate disjunction and it is in this cross-hatching between dissimilar subjects - childish unreality and adult hyper-reality—that the authors are trying to make us think. And that’s what’s really unforgivable. Comic books—pardon me, graphic novels—are about escapism. And imagination. They don’t function like novels or films in which the believing suspension of disbelief is a necessary component of the artistic experience; with graphic novels—pardon you, comic books—the disbelief is part of the fun and the magic. The superheroes are dying, one by one, as we try to turn them into human beings.

   5



45

50

55

60

In 2005 TIME magazine created a list of what its reviewers considered the 100 best novels written since 1923. And smack in the middle of that list was The Watchmen, an epic 1986 graphic novel by Alan Moore, illustrated by Dave Gibbon. Now this really is something. TIME has long been considered the bulwark of American middle-class taste, a kind of centrifuged reduction of essential Americana. For its critics to include The Watchmen with The Great Gatsby, The Invisible Man and Catch-22, and, more importantly, for its editors to permit that selection, heralds the dawning of a new age, an age in which prejudicial critical imperatives no longer automatically trump considerations of valid artistic expression. It was not so long ago that many serialized novels—novels which are today considered Literature—included illustrations as chapter headers. And it couldn‘t be more appropriate that now, with graphic novels achieving a new dimension of reality—both in story-telling and visual artistry—the wheel spins and we end up where we started, with a legitimate and authentic artistic medium that makes use of different elements of perception in order to realize its purpose of provoking thought and emotion. This collapsing of boundaries has been happening in film and music for decades; now it is time for writers and illustrators to explore the limits of expression within their own certain zero-sum parameters: there must be pages bound into a book. A publisher must be willing to market it. A reader must be willing to purchase it. But after that, anything goes. 13. The authors of both passages would agree that graphic novels

(A) have a negative effect on younger generations (B) have experienced a comparatively recent reinvention (C) remain true to their roots as pulp comics (D) are a valid art form (E) a re superior to much literature that is currently being produced

14. Lines 10-11 are characterised by (A) alliteration (B) personification (C) repetition (D) allegory (E) understatement

Go to the next page chap25.indd 28

5/30/2011 1:25:14 PM

25. 29

Chapter 25     SAT Practice Test #3

5  

   

5    5 



   5



15. The question in lines 11-12 serves to (A) introduce a dimension of levity (B) suggest the direction of an argument (C) point out a technicality (D) construe a metaphor (E) hint at an alternative interpretation

20. In line 49, “reduction” most nearly means (A) distillation (B) diminishment (C) augmentation (D) simpliflcation (E) summary

16. Which of the following pairing best captures the “unique juxtaposition” (lines 15-16)? (A) bad and good (B) amoral and pious (C) static and dynamic (D) child-like and adult (E) irreverent and serious

21. The assumption operating beneath the author’s qualification in line 52 (“more importantly...that selection”) is (A) editors usually retain the ultimate discretionary power with respect to publication (B) the editors of TIME are important arbiters of cultural taste (C) r eviewers almost always get input on most literary features (D) t he editorial board of a magazine is generally compelled to follow executive command decisions (E) editors and reviewers usually agree on most matters

17. The author of Passage 2 would most likely respond to line 33 (“and that’s...unforgivable”) by (A) suggesting that the purpose of any artistic endeavor is to stimulate thought (B) proving that the written word and detailed illustration can work in powerful synergy (C) r ejecting the notion that modern graphic novels are superior to their antecedents (D) e ncouraging new graphic artists to explore the limits of social acceptability (E) identifying graphic novels vvith other kinds of emerging media 18. The tone of Passage 1 is (A) resentful (B) moderate (C) awestruck (D) equivocal (E) opinionated

22. The author of Passage 1 would most likely respond to lines 53-55 (“heralds...artistic expression”) by (A) implying that the most artistically legitimate graphic novels have already been written (B) asserting the primacy of graphic novels in the modern canon (C) intimating an affmity for all types of Visual art (D) questioning the artistic value of the graphic novel (E) s upporting the notion of a new and more substantial appreciation

19. The phrase “now this is really something” (line 46) serves to (A) delineate a timeline (B) summarise a sequence of events (C) underscore the significance of an event (D) perpetuate an old-fashioned idea (E) defend a novel conception

Go to the next page chap25.indd 29

5/30/2011 1:25:14 PM

25. 30

5  

Part 3    Four Full Practice Tests

   

5    5 

23. The author of Passage 2 mentions “Literature” in line 57 order to (A) suggest that what is now appreciated as art was once dismissed (B) a rgue that all well-written novels include illustrations (C) make a point about historical novels (D) defend a much-debated literary style (E) emphasize an apparent contradiction



   5



24. It can be inferred from Passage 1 that the author would most likely consider The Watchmen’s inclusion in the TIME Top 100 list as (A) a well-deserved honorific for a talented writer (B) a miscarriage of justice for the originators of the graphic novel genre (C) a n instance of poor judgment with respect to the purpose and effect of graphic novels (D) a misinterpretation of a canonical text (E) a condescending award that ought to be rejected

STOP If you finish your work before time is called, you may check your work on this section only. Do not turn to any other section in the test.

chap25.indd 30

5/30/2011 1:25:14 PM

25. 31

Chapter 25     SAT Practice Test #3

7  

   

7    7 

   7



SECTION 7 Time – 25 minutes 20 Questions Turn to Section 7 of your answer sheet to answer the questions in this section.

Directions For this section, solve each problem and decide which is the best of the choices given. Fill in the corresponding circle on the answer sheet.You may use any available space for scratchwork.

Reference Information

Notes 1.  The use of a calculator is permitted. 2.  All numbers used are real numbers. 3. Figures that accompany problems in this test are intended to provide information useful in solving the problems.   They are drawn as accurately as possible EXCEPT when it is stated in a specific problem that the figure is not drawn to scale. All figures lie in a plane unless otherwise indicated. 4. Unless otherwise specified, the domain of any function f is assumed to be the set of all real numbers x for which f(x) is a real number.

r

l w

h

h

r

h

w l b A = πr2 1 V = lwh A = bh V = πr2h A = lw C = 2πr 2 The number of degrees of arc in a circle is 360. The sum of the measures in degree of the angles of a triangle is 180.

1. If the recipe for a garden salad calls for 10 tomatoes and Richard plans to make three such salads, how many tomatoes will he need? (A) 10 (B) 15 (C) 30 (D) 45 (E) 50

c

b a

c = a2 + b2 2

60° x s 45° s 2 45° 30° s x 3 Special Right Triangles 2x

2. 6, 16, 26,36… The first term in the above sequence is 6 and each term after the first is determined by adding x to the preceding term. What is the value of x? (A) 10 (B) 6 (C) 5 (D) 2 (E) 1

Go to the next page chap25.indd 31

5/30/2011 1:25:14 PM

25. 32

   

7    7 

3. A, B, C, D and E are points on a line in that order. If AB = 10, AC is three times AB , DE = 5 and AE

= 60, what is CD ?



(A) 5 (B) 20 (C) 15 (D) 20 (E) 25

   7





Questions 5-6 refer to the following graph. DISTANCE FROM SCHOOL Number of Students

7  

Part 3    Four Full Practice Tests

1

2

3

4

5

6

Distance in Miles 5. What is the median distance that students live from school in miles?

4. Steve is making his breakfast and he wants to eat eggs and apples. If he will have three items total and twice as many eggs as apples, how many eggs will he eat for breakfast?

(A) 2 (B) 3 (C) 4 (D) 5 (E) 6



(A) 3.0 (B) 3.5 (C) 4.0 (D) 4.5 (E) 5.0

6. What fraction of students lives at least 5 miles from the school?

(A) 1

9

(B) 2



(C)

3 9



(D)

7 18



(E)

1 2

9

Go to the next page chap25.indd 32

5/30/2011 1:25:15 PM

25. 33

Chapter 25     SAT Practice Test #3

7  

   

7    7 

7. If point Q is the midpoint of line segment RS and point P is the midpoint of line segment RQ , which of the following must be true? I.

QR = 2 RS

II.

RP =



S -1

T UV 0

1

2

9. Points P, Q, R, S, T, U, and V are points on the number line, as presented above. Which of the following is closest in value to |R * S| ? (A) Q (B) R (C) S (D) U (E) V

1 QS 2

III. RS = 4 PQ

P QR -2

   7



(A) I only (B) II only (C) III only (D) I and II only (E) II and III only

4x2 + y < 4x + y

x f(x)

-2 -7

-1 0

0 1

1 2

2 9

3 28

10. For what value of x is the above inequality true? (A) –2.0 (B) –1.0 (C) 0.0 (D) 0.5 (E) 1.0

8. The table above displays input and output values for a certain function f. Which of the following could be f? (A) x3 + 1 (B) - x3 + 1 (C) 2x3 + 2 (D) - 2x3 - 2 (E) 3x3 + 2 11. A list consists of s multiples of 6 and f multiples of 5. Within the set of integers from 1 to 25, inclusive, what is s + f? (A) 4 (B) 5 (C) 7 (D) 9 (E) 11

Go to the next page chap25.indd 33

5/30/2011 1:25:16 PM

25. 34

Part 3    Four Full Practice Tests

7  

   

7    7 

12. How many rectangular shoe boxes, with length 12 inches, width 6 inches, and height 6 inches, would fit into a cubical room 10 ft wide, 10 ft long and 10 ft high? (A) 1000 (B) 2000 (C) 2500 (D) 3333 (E) 4000

   7





y A F E B C

D x

13. Acme Box Company’s cost function for producing x boxes is given by the function:

c(x) =

30x - 20 + m, x

where m is a constant. If 10 boxes are produced at a total cost of $50, what is m? (A) 22 (B) 27 (C) 33 (D) 41 (E) 49

14. In the figure above, what is the average of the slopes of line segments AB , BC , and CD ?

(A) -2



(B) 0



(C)



(D) 1



(E) 2

1 2

15. For how many ordered pairs of positive integers (x, y) is 3x + 4y 11? (A) 0 (B) 1 (C) 2 (D) 3 (E) 4

Go to the next page chap25.indd 34

5/30/2011 1:25:18 PM

25. 35

Chapter 25     SAT Practice Test #3

7  

   

7    7 

B

O A

C



   7



18. Six people travel in a six-seated car, but only three are qualified drivers. If only qualified drivers can drive, how many different seating arrangements are there for the car? (A) 6 (B) 36 (C) 60 (D) 120 (E) 360

Note : Figure not drawn to scale. 16. In the figure above, ABC is equilateral and AB = BC = CA = 8. If the radius of the circle with center O is 5, what is the area of, ABC? (A) 12 (B) 15 (C) 24 (D) 36 (E) 48

17. If the average of x and y is 4, and the average of p and q is 5 and x + y +2p + 2q + z = 32, what is z? (A) -2 (B) 2 (C) 4 (D) 8 (E) 12

Go to the next page chap25.indd 35

5/30/2011 1:25:18 PM

25. 36

Part 3    Four Full Practice Tests

7  

   

7    7 

19. In the xy plane, line m passes through the point (1, 2) and is perpendicular to the line y – 3x = k, where k is a constant. If the two lines intersect at the point (n, n + 2), what is the value of n? (A)



(B) 1



of the following must be true? I.

xy >1 z

II. x + y +z > 1

3

   7

20. If x, y and z are all greater than 0 but less than 1, which

1 4





(C) 2

3 1 4



(D) 1



(E) 2 1



III. (xyz) > 1 (A) I only (B) II only (C) III only (D) I And III only (E) I, II and III

3

STOP If you finish your work before time is called, you may check your work on this section only. Do not turn to any other section in the test.

chap25.indd 36

5/30/2011 1:25:18 PM

25. 37

Chapter 25     SAT Practice Test #3

8  

   

8    8 



   8



Section 8 Time—20 Minutes 19 Questions Turn to Section 8 of your answer sheet to answer the questions in this section.

Directions. For each question in this section, select the best answer from among the choices given and fill in the corresponding circle on the answer sheet.

Each sentence below has one or two blanks, each blank indicating that something has been omitted. Beneath the sentence are five words or sets of words labeled A through E. Choose the word or set of words that, when inserted in the sentence, best fits the meaning of the sentence as a whole. Example: The volcano, long considered ——, has recently become ——: local townspeople have been evacuated from the area, out of ränge of ash and magma. (A)  (B)  (C)  (D)  (E) 

threatening . . docile viable . . unlikely quiescent . . lethargic extinct . . endangered dormant . . active

(A) (B) (C) (D) (E)

1. Often —— products are as good as name-brand items and less expensive too. (A) generic (B) expired (C) insipid (D) arbitrary (E) miniature 2. Some psychiatrists claim that instead of being a normal result of introversion, —— can be a sign of —— repression. (A) diffidence . . healthy (B) reticence . . pathological (C) timidity . . acceptable (D) truculence . . muted (E) affluence . . atypical

3. Rex’s tendency to —— his anecdotes earned him a reputation for —— , if not outright dishonesty. (A) embellish . . deception (B) exaggerate . . arrogance (C) underscore . . perfidy (D) highlight . . tedium (E) alternate . . profundity 4. The choir’s performance was —— : it was clear the singers were just going through the motions, with no real energy or ——. (A) rote . . lethargy (B) inspired . . exuberance (C) mechanical . . hostility (D) perfunctory . .vitality (E) unexceptionable . . stimulus 5. The sloth is a —— animal, dull and slow-moving. (A) avaricious (B) lugubrious (C) chary (D) rambunctious (E) pusillanimous 6. The teacher was known for his uprightness and moral —— ; therefore, people were shocked when rumors of —— behavior started to circulate. (A) austerity. . propitious (B) felicity . . sedentary (C) antipathy . . risque (D) deviance . . malevolent (E) rectitude . . licentious

Go to the next page chap25.indd 37

5/30/2011 1:25:18 PM

25. 38

8  

Part 3    Four Full Practice Tests

   

8    8 



   8



Directions. The passage below is followed by questions based on its content. Answer the questions on the basis of what is stated or implied in the passage and in any introductory material that may be provided.

Questions 7-19 are based on the following passages.

35

The passage below is excerpted from a 2007 essay on the accessibility of academia. The “Sokal Affair” could be described thus: a college professor, sick of the sloppy and abstruse language—and by extension, thought—of contemporary academic discourse decides to play a little joke. He finds a respectable journal of postmod5 ern cultural studies and submits the silliest-sounding article he can think of. The article is itself a pastiche of grandiose assertion and fawning quotes from prominent post-structuralists; the argument is an attempt to prove that the laws of physics are not eternal and universal but are instead prod10 ucts of convention and society. The article gets published and then three months later the professor admits it was all a joke, a hoax, an egg-on-your-face gag that leaves a lot of people embarrassed, angry and wondering about the direction of contemporary scholarship. The article was “Transgressing the Boundaries: Towards 15 a Transformative Hermeneutics of Quantum Gravity”; the journal was Social Text; the professor was Alan Sokal, a physicist at New York University who specializes in statistical mechanics and combinatorics. Sokal’s hoax, as it came to be known, was at the most 20 superficial level an experiment of vernacular. Many defend the complexity of academic discourse by suggesting that nuanced and precise ideas must necessarily find their expression in technical and perhaps complex language. Sokal set out 25 to test the reverse: is it possible to disguise dubious scholarship with esoteric jargon? If so, is it possible to differentiate between a subtle and carefully-expressed argument and deliberately obfuscated nonsense? The editors of Social Text failed this assessment. On a deeper level, Sokal was seeking to confirm his suspi30 cions that intentional pandering to editorial preconceptions can and does a published author make. Social Text was a wellknown journal specializing in deconstruction, which at the time was a very popular topic in post-modern studies.

40

45

50

55

60

Sokal theorized that any argument that suggested that physical laws might themselves be products of societal convention—a suggestion to which he later famously responded by inviting people to test the power of that construction by jumping out the window of his tenth-floor apartment— would be ‘trendy’ enough to merit publication, regardless of its soundness. And again, he was right. The final and perhaps most pernicious dimension of Sokal’s experiment was the political angle. Social Text was generally considered a progressive periodical; Sokal himself was known as something of a left-winger. Sokal suspected that his work would be subjected to only the most cursory scrutiny because of his apparent alignment with the political views of the editorial staff. The fact that he was right in this is indeed alarming. Polysyllabic argot is one thing; even editorial narcissism can be explained. But a demonstrable political bias in the gatekeepers of scholarship is truly a frightening prospect. If the purpose of academia is to seek gradual but tangible improvements and refinements in our edifice of knowledge and if as the genetic fallacy states the status or validity of information obtains irrespective of the source, no matter how wrong-headed or sloppy that source might be, then to accept a paper designed to be both wrong-headed and sloppy on the strength of the academic credentials and political leanings of the author indeed seems the height not just of academic arrogance but of an indication that the system of publication and peer-review, on which the future of knowledge and scholarship depends, is on murky ground indeed. 7. As indicated in lines 2-3 (“sloppy…extension, thought”) Sokal apparent believes that

(A) contemporary academic language is intentionally hard to understand (B) the language of scholarship prohibits contribution from laymen (C) imprecise language begets imprecise thinking (D) s cholarly expression need not be exact to be effective (E) e soteric terminology need never be used under any circumstances

Go to the next page chap25.indd 38

5/30/2011 1:25:18 PM

25. 39

Chapter 25     SAT Practice Test #3

8  

   

8    8 



   8



8.

In line 8, “pastiche” most nearly means (A) mixture (B) organisation (C) illustration (D) analysis (E) argument

13. “Pandering to editorial preconceptions” (lines 37-38) refers to

9.

The second paragraph is best characterized as (A) a trifling digression (B) an analogous scenario (C) a biased conclusion (D) a factual summary (E) a transitional premise



10. The author mentions Sokal’s academic specialties in line 23 in order to (A) provide his personal background information (B) establish his scientific authority (C) indicate the relevance of his interests (D) acknowledge his accomplishments (E) segue to a related topic 11. The argument in lines 26-30 (“many defend…complex language”) is most analogous to (A) an amateur musician who requires a very expensive musical instrument (B) a clinical researcher who only hires accomplished assistants (C) a novelist who uses complex descriptive language (D) a sportsman who trains his body very seriously (E) a teacher who expects the best from her students 12. In line 34, “obfuscated” most nearly means

(A) invalid (B) annoying (C) pedantic (D) ambitious (E) confusing





(A) espousing popularly accepted opinions for the sake of publication (B) rejecting widely held beliefs (C) a dopting specific theories regardless of their credibility (D) a general practice of evaluating the rigor of academic journals (E) exploring even the most radical approaches

14. Sokal’s response in lines 44-47 (“he famously… tenthfloor apartment) suggests (A) he is not particularly hospitable (B) he enjoys the company of physicists (C) h  e does not believe that physical laws are social inventions (D) he has a broad appreciation for the physical sciences (E) he does not believe the law of gravity is immutable 15. Lines 24-76 are best described as (A) a number of scientific claims (B) three approaches to academia (C) a logical deduction followed by a conclusion (D) a series of considerations of the consequences of an experiment (E) a comparison between valid and invalid forms of scholarship 16. The author’s mention of the genetic fallacy in lines 65-68 serves to (A) help make a point about the future of academia (B) suggest a valid standard for the disqualification of an argument (C) rebut the Social Text’s defense (D) defend Sokal’s central argument (E) r einforce the important of balance and impartiality in academic research 17. Sokal’s attitude towards Social Text is (A) resigned (B) skeptical (C) hostile (D) neutral (E) appreciative

Go to the next page chap25.indd 39

5/30/2011 1:25:18 PM

25. 40

8  

Part 3    Four Full Practice Tests

   

8    8 

18. The author would most likely disagree with which of the following statements? (A) The precise articulation of sophisticated ideas sometimes requires expression in complicated language. (B) Inadequate citations should be the primary worry of scholarly publications. (C) The continuing success of scholarship relies in part on rigorous academic and editorial scrutiny. (D) The political dimension of the Sokal affair is perhaps the most serious. (E) The purpose of academia involves a process of slow and meticulous refinement.



   8



19. The passage as a whole is best described as (A) a cautionary tale of scholarly hubris (B) a story with several possible interpretations (C) an exploration of the meaning of an episode (D) a personal anecdote with a basis in reality (E) an extended allegory concerning the nature of science

STOP If you finish your work before time is called, you may check your work on this section only. Do not turn to any other section in the test.

chap25.indd 40

5/30/2011 1:25:18 PM

25. 41

Chapter 25     SAT Practice Test #3

9  

   

9    9 

   9



SECTION 9 Time – 20 minutes 16 Questions Turn to Section 9 of your answer sheet to answer the questions in this section.

Directions. For this section, solve each problem and decide which is the best of the choices given. Fill in the corresponding circle on the answer sheet. You may use any available space for scratchwork.

Reference Information

Notes 1.  The use of a calculator is permitted. 2.  All numbers used are real numbers. 3.  Figures that accompany problems in this test are intended to provide information useful in solving the problems.   They are drawn as accurately as possible EXCEPT when it is stated in a specific problem that the figure is not drawn to scale. All figures lie in a plane unless otherwise indicated. 4. Unless otherwise specified, the domain of any function f is assumed to be the set of all real numbers x for which f(x) is a real number.

r

l w

h

r

h

h

w l b A = πr2 1 V = lwh A = bh V = πr2h A = lw C = 2πr 2 The number of degrees of arc in a circle is 360. The sum of the measures in degree of the angles of a triangle is 180.

1. 3x2 = k*x, and k = 6. If x is a positive integer, which of the following could be the value of x? (A) 1 (B) 1.8 (C) 2 (D) 4 (E) 6

c

b a

c2 = a2 + b2

60° x s 45° s 2 45° 30° s x 3 Special Right Triangles 2x

 sock drawer contains 7 pairs of socks in either white or 2  Sasha’s black. If 3 of the pairs are white, then what is the probability that Sasha randomly selects from the drawer a black pair?

1 7 2 7 3 7



(A)



(B)



(C)



(D)



(E) 5

4 7 7

Go to the next page chap25.indd 41

5/30/2011 1:25:19 PM

25. 42

Part 3    Four Full Practice Tests

9  

   

9    9 

3. In ABC, AB = 4 and BC = 3. Which of the following could NOT be a possible value of AC ? (A) 2 (B) 3 (C) 4 (D) 6 (E) 7

A

B

y° x° D



   9



y

x

6. The graph of y = h(x) is shown above. For what value of x does h(x) = h(-2)? (A) -1 (B) 0 (C) 1 (D) 2 (E) 3

E

Note : Figure not drawn to scale.

4. In the figure above, AB || DE. If x = 60 and y = 40, what is z? (A) 30 (B) 50 (C) 70 (D) 80 (E) 90

5. Mike and Martin leave home in separate directions. Mike walks due north at 8 miles per hour and Martin walks due east at 6 miles per hour. At the end of half an hour, how far apart are Mike and Martin, in miles? (A) 5 (B) 6 (C) 8 (D) 10 (E) 11

5

7. If 3p3 = q 2 , then which of the following values could equal p?

(A)

2



(B)

3



(C)

5



(D)

2 2



(E)

2 3

Go to the next page chap25.indd 42

5/30/2011 1:25:19 PM

25. 43

Chapter 25     SAT Practice Test #3

9  

   

9    9 

8. Wallace bakes a cake using a recipe that calls for the weights of flour, sugar and milk in a 2 : 3 : 4 ratio, respectively. If Wallace uses 4 ounces of flour in the recipe, how much flour, sugar and milk will he altogether to make the cake? (A) 18 ounces (B) 14 ounces (C) 12 ounces (D) 9 ounces (E) 7 ounces



   9

10. A machine produces wingnuts with a desired width, w, of 25 mm. All wingnuts whose widths differ from the targeted 25 mm by more than 3 mm are discarded. Which of the following inequalities describes those wingnuts that will NOT be discarded? (A) | w – 3 | ≤ 25 (B) | w – 5 | ≤ 20 (C) | w – 10| ≤ 15 (D) | w – 15| ≤ 10 (E) | w – 25| ≤ 3

y 25 20 15 10 5 x 5 10

15 20 25

9. The graph above shows data collected on a certain topic. Which of the functions below best describes the data? (A) y = 5x (B) y = x + 5 (C) y = 5x + 5 (D) y = 5 – x (E) y = 55x

11. The figure above is formed by the removal of an equilateral triangle from a rectangle. If the length and width of the rectangle are 4 and 5 inches, respectively, then what is the area of the existing figure, in square inches? (A) 10 (B) 12 (C) 16 (D) 20 (E) 36

Go to the next page chap25.indd 43

5/30/2011 1:25:20 PM

25. 44

9  

Part 3    Four Full Practice Tests

   

9    9 

12. If 30% of x equals 60% of y and if 20% of y is 3, then what is x? (A) 12 (B) 15 (C) 18 (D) 24 (E) 30

13. If 0  p  6 and -2  q  4, then which of the following expressions gives the range of possible values for pq? (A) 0  pq  6 (B) -2  pq  6 (C) -12  pq  4 (D) -12  pq  24 (E) -12  pq  48

|x + y| < |x| + |y| 14. For what values of x and y does the above expression hold? I. x > 0, y < 0 II. x < 0, y < 0 III. x > 0, 0 < y < 1

(A) I only (B) II only (C) III only (D) I and II only (E) I, II and III



   9

15. If (x + 3)2 = x2 + cx + m, what is c * m? (A) 36 (B) 45 (C) 54 (D) 63 (E) 72

16. After the second term, each term in a sequence is 4 1 of the preceding term. If n is the first greater than 4 term and p is the second term, what is the ratio of the third term to the second term?

(A)

p+4 p



(B)

4p + 1 p



(C) p + 16

4p

(D) p + 4

16 + p

(E)

p 4-p

STOP If you finish your work before time is called, you may check your work on this section only. Do not turn to any other section in the test.

chap25.indd 44

5/30/2011 1:25:20 PM

25. 45

Chapter 25     SAT Practice Test #3

10 



10   10  

  10

SECTION 10 Time – 10 minutes 14 Questions Turn to Section 10 of your answer sheet to answer the questions in this section.

Directions. For each question in this section, select the best answer from among the choices given and fill in the corresponding circle on the answer sheet.

The following sentences test correctness and effectiveness of expression. Part of each sentence or the entire sentence is underlined; beneath each sentence are five ways of phrasing the underlined material. Choice A repeats the original phrasing; the other four choices are different. If you think the original phrasing produces a better sentence than any of the alternatives, select choice A; if not, select one of the other choices. In making your selection, follow the requirements of standard written English; that is, pay attention to grammar, choice of words, sentence construction and punctuation. Your selection should result in the most effective sentence—clear and precise, without awkwardness or ambiguity. EXAMPLE: In 1947, Jackie Robinson becoming the first AfricanAmerican to play Major League Baseball. (A) becoming the first African-American (B) became the first African-American (C) will become the first African-American (D) becomes the first African-American (E) is becoming the first African-American (A) (B) (C) (D) (E)

1. Dr Johnson’s Dictionary of the English Language, the first of its kind in any language, is written between 1744 and 1752. (A) is written (B) was written (C) wrote (D) being written (E) having been written 2. Tolstoy’s War and Peace not only should make the reader ponder issues of mortality and morality, but as well as exciting the interest and imagination. (A) as well as exciting (B) as well as to excite (C) also should excite (D) also exciting (E) also being exciting 3. A gourmand is someone with a penchant for food and drink, also takes an academic interest in the subject. (A) A gourmand is someone with a penchant for food and drink, also takes an academic interest in the subject (B) A gourmand, taking an academic interest in the subject, is someone with a penchant for food and drink (C) S omeone who has a penchant for food and drink, a gourmand’s interest is also academic (D) A gourmand is someone with a penchant for food and drink, he takes an academic interest in the subject (E) A gourmand is someone with a penchant for food and drink who takes an academic interest in the subject

Go to the next page chap25.indd 45

5/30/2011 1:25:21 PM

25. 46

10 

Part 3    Four Full Practice Tests



10   10  

4. Emily told the teacher, which being known for her compassion, all about the problems she was having at home. (A) which being known for her compassion (B) that is known for her compassion (C) who was known for her compassion (D) who was known compassionately (E) who is known for having compassion 5. The long-term financial effects of the dot-com bubble are only now becoming known. (A) are only now becoming known (B) is only now becoming known (C) has become known only now (D) only now becoming known (E) and becoming known only now 6. The largest of the canine family, the Rhodesian Ridgeback being over four feet tall at the shoulders and is capable of running at speeds of up to 30 miles per hours. (A) Rhodesian Ridgeback being over four feet tall (B) Rhodesian Ridgeback were over four feet tall (C) Rhodesian Ridgeback having been over four feet tall (D) Rhodesian Ridgeback is over four feet tall (E) Rhodesian Ridgeback, over four feet tall 7. A vigorous defense of human rights and to guarantee against future atrocities is the only safeguards against systemic tyranny. (A) to guarantee against future atrocities is (B) guaranteeing against future atrocities is (C) guarantees against future atrocities is (D) a guarantee against future atrocities is (E) a guarantee against future atrocities are 8. The Iron Pillar of Delhi and the Baghdad battery are two examples of out-of-place artifacts (OOPArts), which it is the belief of some conspiracy theorists suggests past visits from intelligent alien life. (A) which it is the belief of some conspiracy theorists suggests (B) which it is the belief of some conspiracy theorists suggesting (C) w  hich some conspiracy theorists are believing suggests (D) which some conspiracy theorists believe suggest (E) which some conspiracy theorists believe suggests

  10

9. Not recognizing that you need to carry proof of license and registration when driving, Larry was given an extra ticket in addition to the warning for speeding. (A) Not recognizing that you will need to carry proof of license and registration when driving (B) Not recognizing that he will need to carry proof of license and registration when driving (C) N  ot recognizing that he needed to carry proof of license and registration when driving (D) N  ot recognizing that he would have needed to carry proof of license and registration when driving (E) H  e needed to carry proof of license and registration when driving 10. Roberto Clemente, playing baseball for the Pittsburgh Pirates for 20 years, he was born in Puerto Rico but felt like a true American. (A) Roberto Clemente, playing baseball for the Pittsburgh Pirates for 20 years, he was born in Puerto Rico but felt like a true American (B) Roberto Clemente played baseball for the Pittsburgh Pirates for 20 years, he was born in Puerto Rico but felt like a true American (C) R  oberto Clemente, who played baseball for the Pittsburgh Pirates for 20 years, was born in Puerto Rico but felt like a true American (D) R  oberto Clemente, playing baseball for the Pittsburgh Pirates for 20 years, although born in Puerto Rico but felt like a true American (E) S ince he was playing baseball for the Pittsburgh Pirates for 20 years, Roberto Clemente was born in Puerto Rico but felt like a true American 11. It cannot be said that many people today know of Hugh Astor, nevertheless he deserves attention and fame for having invented what we know as modern real estate practices. (A) Hugh Astor, nevertheless he deserves (B) Hugh Astor; nevertheless he deserves (C) Hugh Astor, who nevertheless he deserves (D) Hugh Astor; deserving (E) Hugh Astor, for the reason that he is deserving

Go to the next page chap25.indd 46

5/30/2011 1:25:21 PM

25. 47

Chapter 25     SAT Practice Test #3

10 



10   10  

12. The campaign posters informed us that anyone who desires strong leadership and the chance to actively participate in the government should vote for Councilman Green. (A) who desires strong leadership and the chance to actively participate in the government should (B) who desire strong leadership and the chance to actively participate in the government should (C) w  ho is desirous of strong leadership and the chance for active participation in governmental should (D) w  ho desires strong leadership and the chance for active participation in governmental are (E) w  ho desires strong leadership and the chance for active participation in governmental having been



10

14. Having been established in 1884, the US Postal Service are known for fast, reliable service and an unfailing commitment to honesty and integrity. (A) the US Postal Service are known for (B) the US Postal Service will be known for (C) the US Postal Service would be known for (D) the US Postal Service being known for (E) the US Postal Service is known for

13. The United States’ one-dollar bill actually contains obscure Masonic references, as do the inscription at the base of the Jefferson memorial. (A) as do the inscription at the base of the Jefferson memorial (B) a s does the inscription at the base of the Jefferson memorial (C) a nd the inscription at the base of the Jefferson memorial (D) w  hile the inscription at the base of the Jefferson memorial having (E) w  hether do the inscription at the base of the Jefferson memorial

STOP If you finish your work before time is called, you may check your work on this section only. Do not turn to any other section in the test.

chap25.indd 47

5/30/2011 1:25:21 PM

Critical Reading Section 2 D A D C B D A C A D C A

1 2 3 4 5 6 7 8 9 10 11 12

Section 5 13 14 15 16 17 18 19 20 21 22 23 24

D E D B B E C E A C B C

1 2 3 4 5 6 7 8 9 10 11 12

Section 8

B D A C A E A B B A C B

13 14 15 16 17 18 19 20 21 22 23 24

B C B D A E C A A D E C

1 2 3 4 5 6 7 8 9 10

A B A D B E C A D B

11 12 13 14 15 16 17 18 19

Number Correct

Number Correct

Number Correct

Number Incorrect

Number Incorrect

Number Incorrect

C E A C D A B B C

Mathematics Section 4 9 10 11 12 13 14 15 16 17 18

D C D A B E C D

1 2 3 4 5 6 7 8

Section 7 42 9999 90 480 50 1, 1/2, 1/3 7 .2, 2/10 40 1600

1 2 3 4 5 6 7 8 9 10

C A E A C B E A D D

Section 9 11 12 13 14 15 16 17 18 19 20

D E A B D D C E A C

1 2 3 4 5 6 7 8

C D E D A D B D

9 10 11 12 13 14 15 16

Number Correct

Number Correct

Number Correct

Number Incorrect

Number Incorrect

Number Incorrect

B E A E D A C C

Writing Section 10

Section 3 1 2 3 4 5 6 7 8 9

CR-25.indd 48

E B D B D A C C D

10 11 12 13 14 15 16 17 18

A D D C E C C D E

19 20 21 22 23 24 25 26 27

A D B C C B E D E

28 29 30 31 32 33 34 35

D B A D B B E A

1 2 3 4 5 6 7

B C E C A D E

8 9 10 11 12 13 14

Number Correct

Number Correct

Number Correct

Number Incorrect

Number Incorrect

Number Incorrect

D C C B A B E

5/9/2011 5:41:13 PM

Chapter 25    SAT Practice Test #3

25. 49

The SAT Scoring Process Scoring. Each properly filled circle is compared by a computer with the correct answer. A correct response earns one point; an omitted response earns zero points. With each incorrect response to a multiple-choice choice question, one-quarter of a point is subtracted to adjust for the effect of random guessing. The SAT Writing section has 49 questions. If, for instance, a student had 38 correct responses, 8 incorrect responses, and 3 omitted, the ultimate raw score is determined like so: 38 right – 8 wrong = 38 – 2 = 36 Raw Points 4 The calculation of raw scores often involves rounding fractions to the nearest whole number. For example, a raw score of 36.75 is rounded to 37; a raw score of 38.5 is rounded to 39. Note: for the Writing section, the essay raw score counts for roughly 30% of your scaled score; your multiple-choice raw score counts for approximately 70%.

How to Score Practice Test #3 Critical Reading (Sections 2, 5, and 8)

• • • •

Step 1. Count the number of correct and incorrect responses for Section 2 and record the number in the space provided on the Scoring Sheet. Do not count the omitted responses. Step 2. Count the number of correct and incorrect responses for Section 5 and record the number in the space provided on the Scoring Sheet. Do not count the omitted responses. Step 3. Count the number of correct and incorrect responses for Section 8 and record the number in the space provided on the Scoring Sheet. Do not count the omitted responses. Step 4. Add up the number of correct and incorrect responses. Enter the totals into the Scoring Sheet. To calculate A, use the formula: Number Correct – Number Incorrect = A 4

• •

Step 5. To determine B, your rounded Critical Reading Raw Score, round A to the nearest whole number. Step 6. To determine your Critical Reading Score Range, find the Total Rounded Raw Score you obtained in the Critical Reading Conversion Table (Table 1). Enter this range in the appropriate box on the Scoring Sheet.

Maths (Section 4, 7, and 9)

• • • • •

SP-25.indd 49

Step 1. Count the number of correct and incorrect responses for the multiple-choice questions (1-8) in Section 4 and record the number in the space provided on the Scoring Sheet. Do not count the omitted responses. Step 2. Count the number of correct responses for the student-produced response questions (9-18) in Section 4 and record the number in the space provided on the Scoring Sheet. Step 3. Count the number of correct and incorrect responses in Section 7 and record the number in the space provided on the Scoring Sheet. Do not count the omitted responses. Step 4. Count the number of correct and incorrect responses in Section 9 and record the number in the space provided on the Scoring Sheet. Do not count the omitted responses. Step 5. Add up the number of correct and incorrect responses. Enter the totals into the Scoring Sheet. To calculate A, use the formula:

5/23/2011 5:13:18 PM

25. 50

Part 3    Four Full Practice Tests

Number Correct – Number Incorrect = A 4

• •

Step 6. To determine B, your Rounded Math Raw Score, round A to the nearest whole number. Step 7. To determine your Math Score Range, find the Total Rounded Raw Score you obtained in the Math Conversion Table (Table 2). Enter this range in the appropriate box on the Scoring Sheet.

Writing (Sections 3 and 10)

• • • •

Step 1. Enter your Essay Score for Section 1 in the appropriate box on the Scoring Sheet. Your score should range from 0 to 6. Step 2. Count the number of correct and incorrect responses in Section 3 and record the number in the space provided on the Scoring Sheet. Do not count the omitted responses. Step 3. Count the number of correct and incorrect responses in Section 10 and record the number in the space provided on the Scoring Sheet. Do not count the omitted responses. Step 4. Add up the number of correct and incorrect responses. Enter the totals into the Scoring Sheet. To calculate A, use the formula: Number Correct – Number Incorrect = A 4

• • •

SP-25.indd 50

Step 5. To determine B, your Rounded Writing Multiple-Choice Raw Score, round A to the nearest whole number. Step 6. To determine your Writing Scaled Score Range, use Table 3. Find the Total Writing Multiple-Choice Raw Score in the left column of Table 3 and the Essay Score across the top row of the table. Enter this range in the appropriate box on the Scoring Sheet. Step 7. To determine your Writing Multiple-Choice Subscore Range, find the Total Writing Multiple-Choice Raw Score on the Writing Multiple-Choice Conversion Table (Table 4). Enter this range into the appropriate box on the Scoring Sheet.

5/23/2011 5:13:18 PM

25. 51

Chapter 25    SAT Practice Test #3

SAT Practice Test #3 Scoring Sheet Critical Reading Section A. Section 2:

__________ # correct

___________ # incorrect

__________ # correct

___________ # incorrect

__________ # correct

___________ # incorrect

+

B. Section 5: +

C. Section 8: = D. Total Unrounded Raw Score:

__________ # correct



( __________ / 4) = __________ # incorrect A

E. Total Rounded Raw Score (nearest whole number):

_________ B

F. Critical Reading Scaled Score Range (see Table 1):

Maths Section G. Section 4: (questions 1-8)

__________ # correct

___________ # incorrect

+

H. Section 4: (question 9-18)

__________ # correct +

I. Section 7:

__________ # correct

___________ # incorrect

__________ # correct

___________ # incorrect

+

J. Section 9: =

SP-25.indd 51

5/23/2011 5:13:18 PM

25. 52

Part 3    Four Full Practice Tests

K. Total Unrounded Raw Score:

__________ # correct



( __________ / 4) = __________ # incorrect A

L. Total Rounded Raw Score (rounded to the nearest whole number):

_________ B

M. Math Scaled Score Range (see Table 2):

Writing Section N. Section 1:



Essay Score

O. Section 3:

__________ # correct

___________ # incorrect

__________ # correct

___________ # incorrect

+

P. Section 10: = Q. Total MC Unrounded Raw Score:

__________ # correct

R. Total MC Rounded Raw Score (nearest whole number):



( __________ / 4) = _________ # incorrect A

_________ B

S. Writing Scaled Score Range (see Table 3):

T. Writing MC Subscore Range (see Table 4):

SP-25.indd 52

5/23/2011 5:13:18 PM

25. 53

Chapter 25    SAT Practice Test #3

Table 1. Critical Reading Conversion Table Raw Score 67 66 65 64 63 62 61 60 59 58 57 56 55 54 53 52 51 50 49 48 47 46 45 44 43 42 41 40 39 38 37 36 35 34 33 32 31

SP-25.indd 53

Scaled Score 800 780-800 760-800 720-800 700-800 690-790 680-780 660-760 650-750 650-730 640-720 630-710 620-700 610-690 600-680 610-670 600-660 590-650 580-640 570-630 570-630 560-620 560-620 550-610 540-600 530-590 530-590 520-580 520-580 510-570 510-570 500-560 500-560 490-550 490-550 480-540 480-540

Raw Score 30 29 28 27 26 25 24 23 22 21 20 19 18 17 16 15 14 13 12 11 10 9 8 7 6 5 4 3 2 1 0 -1 -2 -3 -4 and below

Scaled Score 470-530 470-530 460-520 460-520 450-510 450-510 440-500 440-500 430-490 430-490 420-480 420-480 410-470 400-460 400-460 390-450 380-440 380-440 370-430 360-420 340-420 330-410 320-400 300-380 280-380 270-370 260-360 250-350 220-320 210-310 200-290 200-280 200-240 200-210 200

Table 2. Mathematics Conversion Table Raw Score 54 53 52 51 50 49 48 47 46 45 44 43 42 41 40 39 38 37 36 35 34 33 32 31 30 29 28 27 26 25

Scaled Score 800 760-800 720-800 700-780 690-770 680-740 670-730 650-710 640-700 630-690 620-680 600-680 590-670 580-660 570-660 560-650 560-640 550-630 540-620 540-620 530-610 530-610 520-600 510-590 510-570 500-560 490-550 490-550 480-540 470-530

Raw Score 24 23 22 21 20 19 18 17 16 15 14 13 12 11 10 9 8 7 6 5 4 3 2 1 0 -1 -2 -3 -4 and below

Scaled Score 450-520 450-510 450-510 440-500 430-490 430-490 420-480 410-470 400-460 390-450 370-450 360-440 350-430 340-420 330-410 320-420 310-410 300-400 290-390 270-370 260-360 250-350 230-330 210-310 200-300 200-280 200-260 200-240 200

5/23/2011 5:13:18 PM

25. 54 MC Raw Score 49 48 47 46 45 44 43 42 41 40 39 38 37 36 35 34 33 32 31 30 29 28 27 26 25 24 23 22 21 20 19 18 17 16 15 14 13 12 11 10 9 8 7 6 5 4 3 2 1 0 -1 -2 -3 -4 -5

SP-25.indd 54

Part 3    Four Full Practice Tests

0 650-690 620-690 600-690 580-680 570-670 550-670 530-660 520-640 520-630 500-620 500-620 490-610 480-600 480-590 470-590 460-580 460-580 450-570 440-560 430-550 430-550 420-540 410-530 400-520 390-510 380-500 380-490 370-480 370-480 360-470 350-460 340-460 330-450 320-440 310-430 300-420 300-420 290-400 280-400 280-400 260-380 250-370 240-360 230-350 220-340 220-340 210-330 200-320 200-300 200-290 200-270 200-250 200-230 200-230 200

1 660-730 640-720 620-720 600-720 580-710 570-710 570-700 560-690 550-680 640-670 540-670 530-660 520-650 510-640 500-630 490-620 490-620 480-610 470-600 460-590 450-580 440-570 430-560 430-560 420-550 410-540 400-530 390-520 380-510 370-500 370-500 360-490 350-480 340-470 340-460 330-450 320-450 310-430 300-430 290-420 280-410 270-400 260-390 250-380 240-370 230-370 220-360 210-340 200-330 200-310 200-300 200-280 200-270 200-260 200-230

Table 3. Writing Conversion Table Essay Score 2 3 680-740 700-760 660-740 680-760 640-740 670-760 620-740 640-760 600-730 630-760 590-720 610-740 580-710 600-730 580-710 600-720 570-700 590-710 570-700 580-710 560-690 580-700 560-690 570-700 550-680 570-700 540-670 560-690 530-660 550-680 520-660 550-670 520-650 540-660 510-640 530-660 500-620 510-650 490-600 510-630 480-590 500-620 470-590 490-620 460-580 490-610 450-580 480-600 440-580 470-600 430-560 460-590 420-550 450-490 420-550 450-580 410-540 440-570 400-530 430-560 390-520 420-550 380-510 410-540 370-500 400-530 360-490 390-520 360-480 390-510 360-480 380-500 350-470 370-490 340-460 360-480 320-460 350-480 310-450 340-470 300-430 330-460 290-420 320-450 290-410 310-440 270-410 300-430 260-390 290-430 250-380 290-420 240-380 270-410 240-370 270-400 220-350 250-380 210-340 240-370 200-330 230-350 200-310 230-340 200-290 210-310 200-280 200-300 200-250 200-280

4 740-790 720-790 700-780 680-780 670-780 640-760 630-750 620-750 620-750 610-740 600-730 590-730 590-720 580-710 570-710 570-700 560-690 550-680 540-670 530-660 530-650 520-650 520-640 510-630 500-620 490-620 480-610 470-600 460-590 460-590 450-580 440-570 430-560 420-550 410-540 400-530 390-520 380-510 370-500 360-490 360-480 350-470 340-460 330-460 330-450 320-450 310-440 300-430 280-410 270-410 260-390 260-370 240-360 230-330 220-300

5 780-800 760-800 730-800 720-800 700-800 670-800 660-790 660-790 650-780 640-770 630-760 620-760 620-750 610-740 600-740 600-730 590-720 580-710 570-700 560-690 550-680 540-670 530-670 520-660 520-650 510-640 500-630 490-620 480-610 480-610 480-610 470-600 460-590 450-580 440-470 430-560 420-550 410-540 400-530 390-520 380-510 370-500 360-490 350-480 350-480 340-470 330-450 320-440 310-440 300-430 290-420 290-410 280-390 270-360 260-340

6 800 780-800 760-800 750-800 730-800 720-800 690-800 680-800 670-800 670-800 660-790 650-780 650-780 640-770 630-760 620-750 610-740 600-730 590-720 580-710 580-700 570-690 560-690 550-670 540-660 530-650 520-650 520-650 510-640 510-640 500-630 490-620 480-610 470-600 470-600 460-590 450-580 440-570 430-560 420-550 410-540 400-530 390-520 380-510 370-500 360-490 350-480 340-470 330-460 330-460 320-450 320-420 310-410 300-400 300-380

5/23/2011 5:13:18 PM

25. 55

Chapter 25    SAT Practice Test #3

Table 4. Writing Multiple-Choice Conversion Table Raw Score 49 48 47 46 45 44 43 42 41 40 39 38 37 36 35 34 33 32 31 30 29 28 27 26 25 24

SP-25.indd 55

Scaled Score 78-80 77-80 74-80 72-78 70-78 69-77 68-76 67-75 66-74 64-74 63-73 62-72 61-71 60-70 59-69 58-68 57-67 56-66 55-65 54-64 53-63 52-62 51-61 50-60 49-59 48-58

Raw Score 23 22 21 20 19 18 17 16 15 14 13 12 11 10 9 8 7 6 5 4 3 2 1 0 -1 and below

Scaled Score 47-57 46-56 45-55 44-54 43-53 42-52 41-51 40-50 39-49 38-48 37-47 36-46 35-45 34-44 33-43 32-42 31-41 30-40 29-39 27-37 25-35 24-34 22-32 20-26 20

5/23/2011 5:13:18 PM

chapter

26 SAT Practice Test #4 SAT Reasoning Test—General Directions Timing

• •

• •

You will have 3 hours and 45 minutes to work on this test. There are nine separately timed sections: • One 25-minute essay • Five other 25-minute sections • Two 20-minute sections • One 10-minute section You may only work on one section at a time. If you finish a section before time is called, check your work on that section. You may NOT turn to any other section.

Marking Answers

• • • • • • •

Be sure to mark your answer sheet properly. You must use a no. 2 pencil. Mark only one answer for each question. Make sure you color/shade the entire circle darkly and completely. Do not make any stray marks on your answer sheet. If you erase, do so completely. Incomplete erasures may be scored as intended answers. Use only the answer spaces that correspond to the question numbers.

Scoring

• • • • • •

chap26.indd 1

For each correct answer, you receive one point. For questions you omit, you receive no points. For a wrong answer to a multiple-choice question, you lose one-fourth of a point. • If you can eliminate one or more of the answer choices as wrong, you increase your chances of choosing the correct answer and earning one point. • If you can’t eliminate any choice, move on. You can return to the question later if there is time. For a wrong answer to a student-produced response (“grid-in”) math question, you don’t lose any points. The essay is scored on a 1 to 6 scale by two different readers. The total essay score is the sum of the two readers’ scores. Off-topic essays, blank essays and essays written in ink will receive a score of zero.

5/23/2011 5:12:21 PM

26.

Part 3    Four Full Practice Tests

2 3

1 2 3 4 5 6 7 8 9 10

A

1 2 3 4 5 6 7 8 9 10

B

C

D E

A

B

C

D E

A

B

C

D E

A

B

C

D E

A

B

C

D E

A

B

C

D E

A

B

C

D E

A

B

C

D E

A

B

C

D E

A

B

C

D E

A

B

C

D E

A

B

C

D E

A

B

C

D E

A

B

C

D E

A

B

C

D E

A

B

C

D E

A

B

C

D E

A

B

C

D E

A

B

C

D E

A

B

C

D E

11 12 13 14 15 16 17 18 19 20

A

11 12 13 14 15 16 17 18 19 20

B

21 22 23 24 25 26 27 28 29 30

D E

C

A

B

C

D E

A

B

C

D E

A

B

C

D E

A

B

C

D E

A

B

C

D E

A

B

C

D E

A

B

C

D E

A

B

C

D E

A

B

C

D E

A

B

C

D E

A

B

C

D E

A

B

C

D E

A

B

C

D E

A

B

C

D E

A

B

C

D E

A

B

C

D E

A

B

C

D E

A

B

C

D E

A

B

C

D E

21 22 23 24 25 26 27 28 29 30

A

B

D E

C

A

B

C

D E

A

B

C

D E

A

B

C

D E

A

B

C

D E

A

B

C

D E

A

B

C

D E

A

B

C

D E

A

B

C

D E

A

B

C

D E

A

B

C

D E

A

B

C

D E

A

B

C

D E

A

B

C

D E

A

B

C

D E

A

B

C

D E

A

B

C

D E

A

B

C

D E

A

B

C

D E

A

B

C

D E

31 32 33 34 35 36 37 38 39 40 31 32 33 34 35 36 37 38 39 40

A

B

C

D E

A

B

C

D E

A

B

C

D E

A

B

C

D E

A

B

C

D E

A

B

C

D E

A

B

C

D E

A

B

C

D E

A

B

C

D E

A

B

C

D E

A

B

C

D E

A

B

C

D E

A

B

C

D E

A

B

C

D E

A

B

C

D E

A

B

C

D E

A

B

C

D E

A

B

C

D E

A

B

C

D E

A

B

C

D E

. 0 1 2 3 4 5 6 7 8 9

. 0 1 2 3 4 5 6 7 8 9

. 0 1 2 3 4 5 6 7 8 9

. 0 1 2 3 4 5 6 7 8 9

. 0 1 2 3 4 5 6 7 8 9

. 0 1 2 3 4 5 6 7 8 9

Student Produced Responses 9

10 . 1 2 3 4 5 6 7 8 9

. 0 1 2 3 4 5 6 7 8 9

. 0 1 2 3 4 5 6 7 8 9

. 0 1 2 3 4 5 6 7 8 9

14

. 1 2 3 4 5 6 7 8 9

. 0 1 2 3 4 5 6 7 8 9

. 0 1 2 3 4 5 6 7 8 9

. 0 1 2 3 4 5 6 7 8 9

15 . 1 2 3 4 5 6 7 8 9

chap26.indd 2

11

. 0 1 2 3 4 5 6 7 8 9

. 0 1 2 3 4 5 6 7 8 9

. 0 1 2 3 4 5 6 7 8 9

12 . 1 2 3 4 5 6 7 8 9

. 0 1 2 3 4 5 6 7 8 9

. 0 1 2 3 4 5 6 7 8 9

. 0 1 2 3 4 5 6 7 8 9

16 . 1 2 3 4 5 6 7 8 9

. 0 1 2 3 4 5 6 7 8 9

. 0 1 2 3 4 5 6 7 8 9

. 0 1 2 3 4 5 6 7 8 9

13

. 1 2 3 4 5 6 7 8 9

. 0 1 2 3 4 5 6 7 8 9

. 0 1 2 3 4 5 6 7 8 9

. 0 1 2 3 4 5 6 7 8 9

17 . 1 2 3 4 5 6 7 8 9

. 0 1 2 3 4 5 6 7 8 9

. 0 1 2 3 4 5 6 7 8 9

. 0 1 2 3 4 5 6 7 8 9

. 1 2 3 4 5 6 7 8 9

18

. 1 2 3 4 5 6 7 8 9

. 0 1 2 3 4 5 6 7 8 9

. 0 1 2 3 4 5 6 7 8 9

. 0 1 2 3 4 5 6 7 8 9

. 1 2 3 4 5 6 7 8 9

5/23/2011 5:12:23 PM

26.

Chapter 26     SAT Practice Test #4

4 5

1 2 3 4 5 6 7 8 9 10

A

B

C

D E

A

B

C

D E

A

B

C

D E

A

B

C

D E

A

B

C

D E

A

B

C

D E

1 2 3 4 5 6 7 8 9 10

A

B

C

D E

A

B

C

D E

A

B

C

D E

A

B

C

D E

A

B

C

D E

A

B

C

D E

A

B

C

D E

A

B

C

D E

A

B

C

D E

A

B

C

D E

A

B

C

D E

A

B

C

D E

A

B

C

D E

A

B

C

D E

11 12 13 14 15 16 17 18 19 20

A

B

C

D E

A

B

C

D E

A

B

C

D E

A

B

C

D E

A

B

C

D E

A

B

C

D E

11 12 13 14 15 16 17 18 19 20

A

B

C

D E

A

B

C

D E

A

B

C

D E

A

B

C

D E

A

B

C

D E

A

B

C

D E

A

B

C

D E

A

B

C

D E

A

B

C

D E

A

B

C

D E

A

B

C

D E

A

B

C

D E

A

B

C

D E

A

B

D E

C

21 22 23 24 25 26 27 28 29 30

A

B

C

D E

A

B

C

D E

A

B

C

D E

A

B

C

D E

A

B

C

D E

A

B

C

D E

21 22 23 24 25 26 27 28 29 30

A

B

C

D E

A

B

C

D E

A

B

C

D E

A

B

C

D E

A

B

C

D E

A

B

C

D E

A

B

C

D E

A

B

C

D E

A

B

C

D E

A

B

C

D E

A

B

C

D E

A

B

C

D E

A

B

C

D E

A

B

D E

C

31 32 33 34 35 36 37 38 39 40

A

B

C

D E

A

B

C

D E

A

B

C

D E

A

B

C

D E

A

B

C

D E

A

B

C

D E

A

B

C

D E

A

B

C

D E

A

B

C

D E

A

B

C

D E

31 32 33 34 35 36 37 38 39 40

A

B

C

D E

A

B

C

D E

A

B

C

D E

A

B

C

D E

A

B

C

D E

A

B

C

D E

A

B

C

D E

A

B

C

D E

A

B

C

D E

A

B

C

D E

. 0 1 2 3 4 5 6 7 8 9

. 0 1 2 3 4 5 6 7 8 9

. 0 1 2 3 4 5 6 7 8 9

. 0 1 2 3 4 5 6 7 8 9

. 0 1 2 3 4 5 6 7 8 9

. 0 1 2 3 4 5 6 7 8 9

Student Produced Responses 9

10 . 1 2 3 4 5 6 7 8 9

. 0 1 2 3 4 5 6 7 8 9

. 0 1 2 3 4 5 6 7 8 9

. 0 1 2 3 4 5 6 7 8 9

14

. 1 2 3 4 5 6 7 8 9

. 0 1 2 3 4 5 6 7 8 9

. 0 1 2 3 4 5 6 7 8 9

. 0 1 2 3 4 5 6 7 8 9

15 . 1 2 3 4 5 6 7 8 9

chap26.indd 3

11

. 0 1 2 3 4 5 6 7 8 9

. 0 1 2 3 4 5 6 7 8 9

. 0 1 2 3 4 5 6 7 8 9

12 . 1 2 3 4 5 6 7 8 9

. 0 1 2 3 4 5 6 7 8 9

. 0 1 2 3 4 5 6 7 8 9

. 0 1 2 3 4 5 6 7 8 9

16 . 1 2 3 4 5 6 7 8 9

. 0 1 2 3 4 5 6 7 8 9

. 0 1 2 3 4 5 6 7 8 9

. 0 1 2 3 4 5 6 7 8 9

13

. 1 2 3 4 5 6 7 8 9

. 0 1 2 3 4 5 6 7 8 9

. 0 1 2 3 4 5 6 7 8 9

. 0 1 2 3 4 5 6 7 8 9

17 . 1 2 3 4 5 6 7 8 9

. 0 1 2 3 4 5 6 7 8 9

. 0 1 2 3 4 5 6 7 8 9

. 0 1 2 3 4 5 6 7 8 9

. 1 2 3 4 5 6 7 8 9

18

. 1 2 3 4 5 6 7 8 9

. 0 1 2 3 4 5 6 7 8 9

. 0 1 2 3 4 5 6 7 8 9

. 0 1 2 3 4 5 6 7 8 9

. 1 2 3 4 5 6 7 8 9

5/23/2011 5:12:25 PM

26.

Part 3    Four Full Practice Tests

6 7

1 2 3 4 5 6 7 8 9 10

A

B

C

D E

A

B

C

D E

A

B

C

D E

A

B

C

D E

A

B

C

D E

A

B

C

D E

A

B

C

D E

A

B

C

D E

A

B

C

D E

A

B

C

D E

1 2 3 4 5 6 7 8 9 10

A

B

C

D E

A

B

C

D E

A

B

C

D E

A

B

C

D E

A

B

C

D E

A

B

C

D E

A

B

C

D E

A

B

C

D E

A

B

C

D E

A

B

C

D E

11 12 13 14 15 16 17 18 19 20 11 12 13 14 15 16 17 18 19 20

A

B

C

D E

A

B

C

D E

A

B

C

D E

A

B

C

D E

A

B

C

D E

A

B

C

D E

A

B

C

D E

A

B

C

D E

A

B

C

D E

A

B

C

D E

A

B

C

D E

A

B

C

D E

A

B

C

D E

A

B

C

D E

A

B

C

D E

A

B

C

D E

A

B

C

D E

A

B

C

D E

A

B

C

D E

A

B

C

D E

21 22 23 24 25 26 27 28 29 30 21 22 23 24 25 26 27 28 29 30

A

B

C

D E

A

B

C

D E

A

B

C

D E

A

B

C

D E

A

B

C

D E

A

B

C

D E

A

B

C

D E

A

B

C

D E

A

B

C

D E

A

B

C

D E

A

B

C

D E

A

B

C

D E

A

B

C

D E

A

B

C

D E

A

B

C

D E

A

B

C

D E

A

B

C

D E

A

B

C

D E

A

B

C

D E

A

B

C

D E

31 32 33 34 35 36 37 38 39 40 31 32 33 34 35 36 37 38 39 40

A

B

C

D E

A

B

C

D E

A

B

C

D E

A

B

C

D E

A

B

C

D E

A

B

C

D E

A

B

C

D E

A

B

C

D E

A

B

C

D E

A

B

C

D E

A

B

C

D E

A

B

C

D E

A

B

C

D E

A

B

C

D E

A

B

C

D E

A

B

C

D E

A

B

C

D E

A

B

C

D E

A

B

C

D E

A

B

C

D E

.

. 0 1 2 3 4 5 6 7 8 9

. 0 1 2 3 4 5 6 7 8 9

. 0 1 2 3 4 5 6 7 8 9

. 0 1 2 3 4 5 6 7 8 9

. 0 1 2 3 4 5 6 7 8 9

. 0 1 2 3 4 5 6 7 8 9

Student Produced Responses 9

10 . 1 2 3 4 5 6 7 8 9

. 0 1 2 3 4 5 6 7 8 9

. 0 1 2 3 4 5 6 7 8 9

. 0 1 2 3 4 5 6 7 8 9

14

. 1 2 3 4 5 6 7 8 9

. 0 1 2 3 4 5 6 7 8 9

. 0 1 2 3 4 5 6 7 8 9

. 0 1 2 3 4 5 6 7 8 9

15 . 1 2 3 4 5 6 7 8 9

chap26.indd 4

11

. 0 1 2 3 4 5 6 7 8 9

. 0 1 2 3 4 5 6 7 8 9

. 0 1 2 3 4 5 6 7 8 9

12 . 1 2 3 4 5 6 7 8 9

. 0 1 2 3 4 5 6 7 8 9

. 0 1 2 3 4 5 6 7 8 9

. 0 1 2 3 4 5 6 7 8 9

16 . 1 2 3 4 5 6 7 8 9

. 0 1 2 3 4 5 6 7 8 9

. 0 1 2 3 4 5 6 7 8 9

. 0 1 2 3 4 5 6 7 8 9

13

. 1 2 3 4 5 6 7 8 9

. 0 1 2 3 4 5 6 7 8 9

. 0 1 2 3 4 5 6 7 8 9

. 0 1 2 3 4 5 6 7 8 9

17 . 1 2 3 4 5 6 7 8 9

. 0 1 2 3 4 5 6 7 8 9

. 0 1 2 3 4 5 6 7 8 9

. 0 1 2 3 4 5 6 7 8 9

1 2 3 4 5 6 7 8 9

18

. 1 2 3 4 5 6 7 8 9

. 0 1 2 3 4 5 6 7 8 9

. 0 1 2 3 4 5 6 7 8 9

. 0 1 2 3 4 5 6 7 8 9

. 1 2 3 4 5 6 7 8 9

5/23/2011 5:12:27 PM

26.

Chapter 26     SAT Practice Test #4

8 9 10

chap26.indd 5

1 2 3 4 5 6 7 8 9 10

A

B

C

D E

A

B

C

D E

A

B

C

D E

A

B

C

D E

A

B

C

D E

A

B

C

D E

A

B

C

D E

A

B

C

D E

A

B

C

D E

A

B

C

D E

1 2 3 4 5 6 7 8 9 10

A

B

C

D E

A

B

C

D E

A

B

C

D E

A

B

C

D E

A

B

C

D E

A

B

C

D E

A

B

C

D E

A

B

C

D E

1 2 3 4 5 6 7 8 9 10

A

B

C

D E

A

B

C

D E

A

B

C

D E

A

B

C

D E

A

B

C

D E

A

B

C

D E

A

B

C

D E

A

B

C

D E

A

B

C

D E

A

B

C

D E

A

B

C

D E

A

B

C

D E

11 12 13 14 15 16 17 18 19 20

A

B

C

D E

A

B

C

D E

A

B

C

D E

A

B

C

D E

A

B

C

D E

A

B

C

D E

A

B

C

D E

A

B

C

D E

A

B

C

D E

A

B

C

D E

11 12 13 14 15 16 17 18 19 20

A

B

C

D E

A

B

C

D E

A

B

C

D E

A

B

C

D E

A

B

C

D E

A

B

C

D E

A

B

C

D E

A

B

C

D E

11 12 13 14 15 16 17 18 19 20

A

B

C

D E

A

B

C

D E

A

B

C

D E

A

B

C

D E

A

B

C

D E

A

B

C

D E

A

B

C

D E

A

B

C

D E

A

B

C

D E

A

B

C

D E

A

B

C

D E

A

B

C

D E

21 22 23 24 25 26 27 28 29 30

A

B

C

D E

A

B

C

D E

A

B

C

D E

A

B

C

D E

A

B

C

D E

A

B

C

D E

A

B

C

D E

A

B

C

D E

A

B

C

D E

A

B

C

D E

21 22 23 24 25 26 27 28 29 30

A

B

C

D E

A

B

C

D E

A

B

C

D E

A

B

C

D E

A

B

C

D E

A

B

C

D E

A

B

C

D E

A

B

C

D E

21 22 23 24 25 26 27 28 29 30

A

B

C

D E

A

B

C

D E

A

B

C

D E

A

B

C

D E

A

B

C

D E

A

B

C

D E

A

B

C

D E

A

B

C

D E

A

B

C

D E

A

B

C

D E

A

B

C

D E

A

B

C

D E

31 32 33 34 35 36 37 38 39 40

A

B

C

D E

A

B

C

D E

A

B

C

D E

A

B

C

D E

A

B

C

D E

A

B

C

D E

A

B

C

D E

A

B

C

D E

A

B

C

D E

A

B

C

D E

31 32 33 34 35 36 37 38 39 40

A

B

C

D E

A

B

C

D E

A

B

C

D E

A

B

C

D E

A

B

C

D E

A

B

C

D E

A

B

C

D E

A

B

C

D E

A

B

C

D E

A

B

C

D E

31 32 33 34 35 36 37 38 39 40

A

B

C

D E

A

B

C

D E

A

B

C

D E

A

B

C

D E

A

B

C

D E

A

B

C

D E

A

B

C

D E

A

B

C

D E

A

B

C

D E

A

B

C

D E

5/23/2011 5:12:28 PM

26.

1 

Part 3    Four Full Practice Tests

  

ESSAY

ESSAY 





1



ESSAY Time – 25 Minutes Turn to page 2 of your answer sheet to write your ESSAY. The Essay gives you an opportunity to show how effectively you can develop and express ideas. You should, therefore, take care to develop your point of view, present your ideas logically and clearly and use language precisely. Your essay must be written on the lines provided on your answer sheet—you will receive no other paper on which to write. You will have enough space if you write on every line, avoid wide margins and keep your handwriting to a reasonable size. Remember that people who are not familiar with your handwriting will read what you write. Try to write or print so that what you are writing is legible to those readers.

Important Reminders:

• • • •

A pencil is required for the essay. An essay written in ink will receive a score of zero. Do not write your essay in your test book. You will receive credit only for what you write on your answer sheet. An off-topic essay will receive a score of zero. If your essay does not reflect your original and individual work, your test scores may be canceled. You have twenty-five minutes to write an essay on the topic assigned below. Think carefully about the issue presented in the following excerpt and the assignment below. Science is the application of the scientific method—hypothesis, experiment, conclusion—to questions of the natural world. Art also seeks to answer questions, but mostly these are inner: how is it that we feel? That we see? That we experience? Art and science share a common ground of exploration and application of principles and methods, but some would say their respective realms are mutually exclusive.

Assignment: Is it important to try to find similarities between things that seem different? Plan and write an essay in which you develop your point of view on this issue. Support your position with reasoning and examples taken from your reading, studies, experience, or observations.

BEGIN WRITING YOUR ESSAY ON PAGE 2 OF THE ANSWER SHEET.

If you finish before time is called, you may check work on this section only. Do not turn to any other section in the test.

chap26.indd 6

5/23/2011 5:12:28 PM

26.

Chapter 26     SAT Practice Test #4

Section

1

IMPORTANT: Use a No. 2 PENCIL. Do NOT write outside the border! Words written outside the essay box or written in ink WILL NOT APPEAR in the copy sent to be scored and your score will be affected.

Begin your essay on this page. If you need more space, continue on the next page.

chap26.indd 7

5/23/2011 5:12:29 PM

26.

Part 3    Four Full Practice Tests

Continuation of ESSAY Section 1 from previous page. Write below only if you need more space. IMPORTANT: DO NOT START on this page—if you do, your essay may appear blank and your score may be affected.

chap26.indd 8

5/23/2011 5:12:29 PM

26.

Chapter 26     SAT Practice Test #4

2  

   

2    2 



   2



Section 2 Time—25 Minutes 24 Questions Turn to Section 2 of your answer sheet to answer the questions in this section.

Directions. For each question in this section, select the best answer from among the choices given and fill in the corresponding circle on the answer sheet.

Each sentence below has one or two blanks, each blank indicating that something has been omitted. Beneath the sentence are five words or sets of words labeled A through E. Choose the word or set of words that, when inserted in the sentence, best fits the meaning of the sentence as a whole. Example: The volcano, long considered ——, has recently become ——: local townspeople have been evacuated from the area, out of range of ash and magma. (A) threatening .. docile (B) viable . . unlikely (C) quiescent . . lethargic (D) extinct . . endangered (E) dormant.. active

(A) (B) (C) (D) (E)

1. The townspeople were adamant in their demands for ———: they wanted self-rule without any interference. (A) necessity (B) infinity (C) hostility (E) obloquy (D) autonomy 2. The snow stopped and started at unpredictable intervals; this ——— made it impossible to go on the hiking trip as planned. (B) permanence (C) paucity (A) plasticity (D) intangibility (E) intermittence

3. The movie villain’s suaveness and —— made it very difficult to dislike him. (A) urbanity (B) truculence (C) desperation (D) affinity (E) euphony 4. It would be a mistake to equate professional —— with —— : a gloomy outlook doesn’t necessarily entail a sloppy job performance. (A) ubiquity . . disinterest (B) hostility . . harmony (C) pessimism . . negligence (D) congeniality . . efficaey (E) phlegm .. antipathy 5. The book of short stories was criticized for a lack of diversity because it was perceived as —— in tone and content. (A) homogenous (B) incredulous (C) dubious (E) perfidious (D) anachronous 6. The physician’s departure was distinctly —— after it became known that he had been embezzling medical supplies. (A) austere (B) ignominious (C) sedulous (D) magnanimous (E) intrepid 7. The pressure of the situation only served to —— the already-anxious players. (B) ameliorate (C) enervate (A) propitiate (D) consolidate (E) expectorate 8. Julius, not trusting the —— of fate, left nothing to chance with his life insurance policy. (A) epochs (B) escutcheons (C) vicissitudes (D) susurrations (E) insurrections

Go to the next page chap26.indd 9

5/23/2011 5:12:29 PM

26.10

2  

Part 3    Four Full Practice Tests

   

2    2 



   2



Directions. The passages below are followed by questions based on their content; questions following a pair of related passages may also be based on the relationship between the paired passages. Answer the questions on the basis of what is stated or implied in the passages and in any introductory material that may be provided.

Questions 9-12 are based on the following passages.

5

10

15

20

25

Passage 1 As the world’s most popular beverage, coffee has become a serious business in recent years. And with so much money to be made, why shouldn’t it be? Yet the ubiquity of perceived “cookie-cutter” coffee-house chains has provoked a most unusual reaction amongst connoisseurs and professional baristas: get out of our shoppes! 1t is now not unusual to be greeted with a sneer or a derisive snort when asking for a plain old cappuccino at any venue that claims to sell high-end beans. The staff are surly and the goateed clientele finger-tappingly impatient. As part of a broad cultural reaction, the message is clear: if you want the freshest coffee, you better realize that the customer isn’t always right and that money doesn’t necessarily talk. The things we’ll put up with for a cup of Joe. Passage 2 The java industry—once small and regional—has grown into something of a multi-national leviathan. As a product, coffee cannot be beat: it’s addictive, relatively harmless and tasty. This might account for why so many large companies are now funneling cash into a search for the holy grail of coffee. Small cafes in Seattle cannot afford to send scouts and coffee-experts all over Central America; they cannot afford to analyze beans in a lab; they cannot afford to consult with soil specialists on growing techniques. While it is valid to criticize the modern coffee business as an essentially profit-driven enterprise, it is important to realize that with this professionalism comes the pressure to beat “the other guy” by sourcing and selling truly excellent coffee. 9. The author of Passage 1 apparently believes (A) the sale of coffee in large chains is a legitimate enterprise (B) the populariry of coffee has dwindled (C) t he quality of coffee sold in large chains is just as good as that sold in boutiques (D) c offee connoisseurs are on the whole welcoming of novices (E) t he mainstream sale of coffee will negatively impact the industry as a whole

10. The author of Passage 2 would most likely respond to the “connoisseurs and professional baristas” (lines 6-7) by (A) agreeing that the tenets of capitalism are incompatible with the philosophies of most coffee drinkers (B) implying that specialist cafes are reliant on the charity of large corporate chains (C) a rguing for the right of the consumer (D) p  ointing out that the profitability of the industry makes it possible to develop better produets (E) s uggesting they form a cooperative farm and grow their own beans 11. Compared to Passage 1, the tone of Passage 2 could be best described as (A) pessimistic (B) appreciative (C) p  edantic (D) incorrigible (E) a mbivalent 12. Both authors would agree that (A) technology is vitally important in the produetion of coffee (B) most coffee-drinkers are malevolent (C) the market reaction to large retail coffee chains has been remarkably inclusive (D) g ood coffee is worth compromising for (E) large corporate organizations will inevitably suppress creativity

Go to the next page chap26.indd 10

5/23/2011 5:12:29 PM

26.11

Chapter 26     SAT Practice Test #4

2  

5

10

15

20

25

30

35

40

   

2    2 

Questions 13-24 are based on the following passage. The following passage is excerpled from Charles Darwin’s Origin of the Species, published in 1859. 45 When on board the H.M.S Beagle, as naturalist, I was much struck with certain facts in the distribution of the organic beings inhabiting South America and in the geological regions of the present to the past inhabitants of that continent. These facts, as will be seen in the latter chapters of this 50 volume, seemed to throw light on the origin of species - that mystery of mysteries, as it has been called by some of our greatest philosophers. On my return home, it occurred to me, in 1837, that something might perhaps be made out on this question by patiently accumulating and reflecting on all sorts 55 of facts which could possibly have any bearing on it. After five years’ work I allowed myself to speculate on the subject and drew up some short notes; these I enlarged in 1844 into a sketch of conclusions, which then seemed to me probable: from that period to the present day I have steadily pursued the same object. This abstract, which I now publish, must necessarily be imperfect. I cannot here give references and authorities for my several Statements; and I must trust to the reader reposing some confidence in my accuracy. No doubt errors may have crept in, though I hope I have always been cautious in trusting to good authorities alone. I can here only give the general conclusion at which I have arrived, with a few facts in illustration, but which, I hope, in most cases will suffice. No one can feel more sensible than I do of the necessity of hereafter Publishing in detail all the facts, with reference, on which my conclusions have been grounded; and I hope in a future work to do this. For I am well aware that scarcely a single point is discussed in this volume on which facts cannot be adduced, often apparently leading to conclusions directly opposite to those at which they have arrived. A fair result can be obtained only by fully stating and balancing the facts and arguments on both sides of each question; and this here is impossible. In considering the origin of the species, it is quite conceivable that a naturalist, reflecting on the mutual affinities of organic beings, on their embryological relations, their geographical distribution, geological succession and other such facts, might come to the conclusion that species had not been independently created, but had descended, like varieties, from other species. Nevertheless, such a conclusion, even if well founded, would be unsatisfactory, until it could be shown how the innumerable species inhabiting this world have been modified, so as



   2



to acquire that perfection of structure and co-adaptation which justly excites our admiration. Naturalists continually refer to external conditions, such as climate, food, etc, as the only possible cause of Variation. In one limited sense, we shall hereafter see, this may be true; but it is preposterous to attribute to mere external conditions, the structure, for instance, of the woodpecker, with its feet, tail, beak and tongue, so admirably adapted to catch insects under the bark of trees. In the case of the mistletoe, which has seeds that must be transported by certain birds and which has flowers with separate sexes absolutely requiring the agency of certain insects to bring pollen from one flower to the other, it is equally preposterous to account for the structure of this parasite, with its relations to several distinct organic beings, by the effect of external conditions, or of habit, or of the volition of the plant itself. 13. The primary purpose of the passage is (A) to explore the mechanics of biological evolution in detail (B) to offer a series of disclaimers about the validity of published biological work (C) t o address some issues before launching into a substantial scientific treatise (D) t o support a scientific movement with rhetoric (E) t o prove beyond question a particular theory 14. The “facts’’ (line 6) serve to (A) clarify a worrisome issue (B) demystify a scientific enigma (C) simplify a biological process (D) r einforce an accepted belief (E) reconfigure a previously held tenet 15. Lines 11-14 (“something might be...bearing on it”) primarily serve to (A) reference a potential plan (B) conclude a line of reasoning (C) s uggest a likely result (D) d  efend a controversial theory (E) m  ention an idea which is later rejected 16. In context, the second paragraph is characterized as (A) a strong warning (B) a hypothetical scenario (C) a n extended qualification (D) a statistical analysis (E) a scientific history

Go to the next page chap26.indd 11

5/23/2011 5:12:29 PM

26.12

2  

Part 3    Four Full Practice Tests

   

2    2 

17. The author’s suggestion that the “abstract...necessarily be imperfect” (lines 20- 21) implies that (A) in any significant work errors are bound to be made (B) a mateur naturalists tend to make invalid logical deductions (C) t he academic sources of a valid scientific treatise must be irreproachable (D) peer review is a vital component of academic inquiry (E) t he work in question was written in haste and with little attention to accuracy 18. The author mentions mutual “affinities... such facts” in lines 44-47 in order to (A) demonstrate the complexity of organic beings (B) gesture at disparate and unconnected biological features (C) w  arn about aspects of biological science that can precipitate invalid reasoning (D) indicate criteria that might lead to a conclusion (E) s uggest the importance of an enterprise 19. In lines 50-51, the author suggests the “conclusion” is “unsatisfactory” because

(A) it is derived from improperly attributed sources (B) it is the result of a school of perfunctory and nonrigorous naturalism (C) it is vital to be aware of the dimension of adaptation as pertaining to the speciation across continents (D) it is necessary to know the processes of adaptation in addition to the modifications themselves (E) it is difficult to generate a valid conclusion without the benefit of a substantial and differentiated edifice of observation

20. In line 55, “excites” most nearly means (A) assumes (B) accepts (C) engages (D) d  irects (E) e nthuses



   2



21. In lines 59-63, the author reveals his dissatisfaction with which behavior of most naturalists? (A) The willingness with which they reject a theory that does not accord with their preconceptions (B) The readiness with which they abandon a valid evolutionary supposition (C) The persistence with which they maintain certain pseudo-scientific dogma (D) The implacability of their reasoning even when faced with new evidence (E) The alacrity with which they attribute certain organic characteristics to an invalid source 22. In line 66, “agency” most nearly means (A) actions (B) organization (C) company (D) d  esign (E) c ollective 23. Lines 70-72 (“by the effect... plant itself ”) chiefly serve to emphasize (A) the primacy of will in evolutionary theory (B) the author’s opinions regarding the unsuitability of certain explanations (C) t he combination of elements that accounts for certain organic structures (D) t he complexity of a hitherto-unaccepted scientific theory (E) t he methodological validity of an experiment 24. The author’s tone in lines 60-72 is best described as (A) deflnite (B) placatory (C) a mbivalent (D) c ompromising (E) u  nderstated

STOP If you finish your work before time is called, you may check your work on this section only. Do not turn to any other section in the test.

chap26.indd 12

5/23/2011 5:12:29 PM

26.13

Chapter 26     SAT Practice Test #4

3  

   

3   3 



   3



Section 3 Time–25 Minutes 35 Questions Turn to Section 3 of your answer sheet to answer the questions in this section. Directions. For each question in this section, select the best answer from among the choices given and fill in the corresponding circle on the answer sheet.

The following sentences test correctness and effectiveness of expression. Part of each sentence or the entire sentence is underlined; beneath each sentence are five ways of phrasing the underlined material. Choice A repeats the original phrasing; the other four choices are different. If you think the original phrasing produces a better sentence than any of the alternatives, select choice A; if not, select one of the other choices. In making your selection, follow the requirements of standard written English; that is, pay attention to grammar, choice of words, sentence construction and punctuation. Your selection should result in the most effective sentence—clear and precise, without awkwardness or ambiguity. EXAMPLE: In 1947, Jackie Robinson becoming the first AfricanAmerican to play Major League Baseball. (A) becoming the first African-American (B) became the first African-American (C) will become the first African-American (D) becomes the first African-American (E) is becoming the first African-American (A) (B) (C) (D) (E)

1. The problem of illegal downloading, only occasionally lessened by media awareness campaigns. (A) downloading, only occasionally lessened by (B) downloading, being only occasionally lessened by (C) d  ownloading is only occasionally lessened by (D) d  ownloading, only on occasion lessened by (E) d  ownloading, by occasionally lessening 2. Burdened with a squalling infant and narcoleptic canine, Marlo’s dash through the airport to catch his connection was challenging. (A) Marlo’s dash through the airport to catch his connection was challenging (B) dashing through the airport was something Marlo found challenging (C) the connection was challengingly dashed for by Marlo (D) M  arlo met his challenge and dashed through the airport to catch his connection (E) M  arlo dashed through the airport to catch his connection and it was a challenge 3. History tells us that ancient Pictish barbarians using the skulls of their vanquished opponents frightening the Romans in battle. (A) using the skulls of their vanquished opponents frightening (B) using the skulls of their vanquished opponents frightened (C) u  sing the skulls of their vanquished opponents to frighten (D) u  sed the skulls of their vanquished opponents for the purpose of frightening (E) u  sed the skulls of their vanquished opponents to frighten

Go to the next page chap26.indd 13

5/23/2011 5:12:30 PM

26.14

3  

Part 3    Four Full Practice Tests

   

3   3 



   3



4. Nutritionists working to reduce body mass index for their clients utilizing techniques of calorie-reduction and food-hypnosis. (A) utilizing techniques of calorie-reduction and foodhypnosis (B) utilize techniques of calorie-reduction and foodhypnosis (C) h  aving been utilizing techniques of calorie-reduction and food-hypnosis (D) w  ho utilize techniques of calorie-reduction and food hypnosis (E) w  ho are utilizing techniques of calorie-reduction and food-hypnosis

8. The zookeeper bequeathed a substantial sum of money to the Baltimore Petting Zoo, he had spent most of his professional life working there. (A) Zoo, he had spent most of his professional life working there (B) Zoo; where he had spent most of his professional life working there (C) Z  oo and he had spent most of his professional life working there (D) Z  oo, for the reason being that he had spent most of his professional life working there (E) Z  oo as he had spent most of his professional life working there

5. The purpose of the novel was not only to raise awareness but also to evoke a situation with humor. (A) not only to raise awareness but also to evoke (B) not only to raise awareness as well as to evoke (C) n  ot only to raise awareness as well as evoking (D) n  ot only to raise awareness and to evoke (E) r aising awareness and to evoke

9. An important mineral in the smelting industry, zinc is more often used pure than turned into an alloy. (A) zinc is more often used pure than turned (B) zinc is most often used pure than turned (C) z inc is more often used pure than turns (D) z inc is more often used pure than is turned (E) z inc is more often used pure than turning

6. The key to effective teaching involves identifying and supporting a student’s strengths rather than recognize weaknesses. (A) supporting a student’s strengths rather than recognize weaknesses (B) supporting a student’s strengths rather than to recognize weaknesses (C) s upporting a student’s strengths rather than recognizing weaknesses (D) t o support a student’s strengths rather than to recognize weaknesses (E) t o support a student’s strengths rather than recognize weaknesses

10. Best known as a society photography, Patrick McMullan’s recent exhibition on Sudanese refugees startles some critics. (A) Patrick McMullan’s recent exhibition on Sudanese refugees startles some critics (B) Patrick McMullan’s recent exhibition on Sudanese refugees startling some critics (C) Patrick McMullan’s recent exhibition on Sudanese refugees had the effect of startling some critics (D) a recent exhibition by Patrick McMullan on Sudanese refugees startled some critics (E) P  atrick McMullan startled some critics with his recent exhibition on Sudanese refugees 11. We had never experienced anything like it, we were stunned by the IMAX experience. (A) We had never experienced (B) We weren’t expecting anything like it (C) Never having experienced (D) Never having been experienced by us (E) Not experiencing

7. The applications for business strategy that fill the Art of War is helpful for managers in planning tactics about engaging the competition. (A) The Art of War is helpful for managers in planning tactics (B) The Art of War is helping managers in planning tactics (C) T  he Art of War has been helpful for managers in planning tactics (D) T  he Art of War are helping managers in planning tactics (E) T  he Art of War are helpful for managers in planning tactics

Go to the next page chap26.indd 14

5/23/2011 5:12:30 PM

26.15

Chapter 26     SAT Practice Test #4

3  

   

3   3 

Directions. The following sentences test your ability to recognize grammar and usage errors. Each sentence contains either a single error or no error at all. No sentence contains more than one error. The error, if there is one, is underlined and lettered. If the sentence contains an error, select the one underlined part that must be changed to make the sentence correct. If the sentence is correct, select choice E. In choosing answers, follow the requirements of standard written English. EXAMPLE:



The other students and me decided to A B confront our teacher about the test. C D No error. E (A) (B) (C) (D) (E)

12. The moment we heard the alarm we began to A file out the classroom slowly and carefully, B kept close together to prevent separation in the C D hallway. No Error E 13. The Wildlife Preserve of Tanzania, home to nearly fourteen hundred species of animal and consisting of over 2000 square miles, were A B founded by an oil tycoon who wished to give C D back to society. No Error E 14. Lured to the kitchen by the mouth-watering A scent of freshly baked bread, the children were B C crushed to find that the pastries did not taste as freshly as they smelled. No Error D E



   3



15. The Arabian striped cobra, who has been A called the most poisonous snake in the Eastern Hemisphere, is currently on the endangered B species list owing to high demand for purses C made of its diamond-patterned skin. No Error D E 16. The instant Stringer saw the whiskery two-foot A bass, he unreeled his fishing line and hooked B his juiciest bait before casting far into the C D rushing stream. No Error E 17. Bunk and Herc hotly debated the common but A mistaken maxim that when two people enter B into a business partnership they are usually not C capable to be friends. No Error D E 18. As a valued member of the Chinese Emperor’s A retinue, Marco Polo assumed a political B independence that made him unpopular C with his fellow courtiers. No Error D E 19. The Ministry of Education has introduced a A policy on the mandatory distribution of second-language coloring books, because it B believes children will respond to this better C D than other language-learning media. No Error E

Go to the next page chap26.indd 15

5/23/2011 5:12:30 PM

26.16

3  

Part 3    Four Full Practice Tests

   

3   3 

20. Disgusted by the result of the season, it was A agreed by the coaches that the entire starting B lineup would be scrapped and replaced with C D younger players. No Error E 21. It is unfortunate that the opening of the new opera house, which had been scheduled for A B June, would now be delayed several months C because of workers’ strikes at the loading D dock. No Error E 22. Many organizations, including the Red Cross, A has declared against animal rights violations B C occurring in Central America. No Error D E 23. When Nick spoke to Lester and I, he asked A B about whether we would be interested in C joining his stage crew for a production of Bye D Bye Birdie. No Error E



   3



24. Running as strongly as she could, Keema was A determined to beat her roommate’s best time in the 800m event; coming through the tape, B however, she found she had missed her goal C D by less than a second. No Error E 25. The general’s insistence on cleanliness and A order were not, as most people thought, B C arbitrary imperatives but rather the direct D result of much experience engaging the enemy. No Error E 26. When a pilot first starts flying a plane, it takes time for one to adapt to the quirks and specs of A its manufacture; the consensus is that it requires B C six months for the symbiosis to be complete. D No Error E 27. The extended family, composed not just of A siblings but grandparents, cousins and second-cousins as well, decided collectively B C where to take their annual holiday trip. D No Error E

Go to the next page chap26.indd 16

5/23/2011 5:12:30 PM

26.17

Chapter 26     SAT Practice Test #4

3  

   

3   3 

28. In terms of the canon of Regency literature, A B Jane Austen’s Pride and Prejudice is better loved and more studied than Elizabeth Gaskell. C D No error E 29. In the modern world, the professional journalist must make a choice between freelance A investigative reporting, which is often risky B and poorly-compensated, or staffing on a C major newspaper, which can be restrictive and D uninspiring. No Error E



   3



Directions. The following passage is an early draft of an essay. Some parts of the passage need to be re-written. Read the passage and select the best answers for the questions that follow. Some questions are about particular sentences or parts of sentences and ask you to improve sentence structure or word choice. Other questions ask you to consider organization and development. In choosing answers, follow the requirements of standard written English.

Questions 30-35 refer to the following passage.

5

10

15

20

(1) Bird-watching is not for the faint-hearted. (2) Imagine being forty feet up in a swaying tree with a pair of binoculars slung about your neck and the wind nipping at your heels. (3) There is a rare spotted spread-tail swallow roosting in the adjacent one and you simply have to get a look at it. (4) Suddenly, the branch no longer bears your weight as you thought it would, you tumble with your gear down a long way to the bottom. (5) When you’re lying there, on the ground, dazed and possibly hurt, is your only thought to get back up and find that nest? (6) If it isn’t you aren’t a real bird-watcher. (7) I’ve been watching birds for about as long as I can remember. (8) The funny thing is that it wasn’t my mother or father who got me interested in it. (9) I just looked up one day and saw a bird and thought to myself, “I’d like to know more about that thing.” (10) I loved their elegance, their beauty and their efficiency. (11) Bird-watching may not be the sport of kings, but take it from someone who knows. (12) You can’t beat it. (13) There are few things better than hiking on a dark morning with the certain knowledge that today you will see something you have never seen before.

Go to the next page chap26.indd 17

5/23/2011 5:12:30 PM

26.18

3  

Part 3    Four Full Practice Tests

   

3   3 



   3



30. In context, which of the following most logically replaces “one” in sentence 3 (reproduced below)?

34. In context, which is the best way to combine sentence 11 and sentence 12 (reproduced below)?







There is a rare spotted spread-tail swallow roosting in the adjacent one and you simply have to get a look at it. (A) branch just overhead (B) gust of wind (C) carefully lined nest (D) old-growth oak tree (E) rare bird

31. In context, which is the best version of the underlined portion of sentence 4 (reproduced below)?

Suddenly, the branch no longer bears your weight as you thought it would, you tumble with your gear down a long way to the bottom



(A) (As it is) (B) you thought it would, tumbling with your gear (C) you thought it would and you tumble with your gear (D) tumbling with your gear (E) you thought it would, you and your gear tumble

32. Sentence 5 is best described as

(A) an exploration of the meaning of a term (B) the conclusion of a personal anecdote (C) a response to a life-threatening situation (D) a criterion for determining the legitimacy of a desire (E) a recollection of a successful expedition

Bird-watching may not be the sport of kings, but take it from someone who knows. You can’t beat it.



(A) Bird-watching may not be the sport of kings, but take it from someone who knows, you can’t beat it. (B) Bird-watching may not be the sport of kings, but take it from someone who knows: bird-watching cannot be beat by you. (C) Bird-watching may not be the sport of kings, but take it from someone who knows, it is impossible to beat. (D) You can’t beat bird-watching, even though it may not be the sport of kings. (E) Bird-watching may not be able to be beat and nor is it the sport of kings. 35. Which sentence is best to add after sentence 13? (A) When you watch birds, there’s always something to learn. (B) It is impossible for a man to know everything. (C) In conclusion, bird-watching is a great pastime. (D) Bird-watching is also called ornithology. (E) I cannot remember the last time I did not enjoy a bird-watching session.

33. In context, which of the following should be inserted at the beginning of sentence 10?

(A) As a result, (B) A natural consequence, (C) It only made sense that, (D) In retrospect, (E) It didn’t long before I knew that

STOP If you finish your work before time is called, you may check your work on this section only. Do not turn to any other section in the test.

chap26.indd 18

5/23/2011 5:12:30 PM

26.19

Chapter 26     SAT Practice Test #4

4  

   

4    4 

   4



SECTION 4 Time – 25 minutes 18 Questions Turn to Section 4 of your answer sheet to answer the questions in this section.

Directions. For this section, solve each problem and decide which is the best of the choices given. Fill in the corresponding circle on the answer sheet.You may use any available space for scratchwork.

Reference Information

Notes 1.  The use of a calculator is permitted. 2.  All numbers used are real numbers. 3.  Figures that accompany problems in this test are intended to provide information useful in solving the problems.   They are drawn as accurately as possible EXCEPT when it is stated in a specific problem that the figure is not drawn to scale. All figures lie in a plane unless otherwise indicated. 4. Unless otherwise specified, the domain of any function f is assumed to be the set of all real numbers x for which f(x) is a real number.

r

l w

h

h w

r

h

l b 1 V = lwh A = bh V = πr2h A = lw 2 The number of degrees of arc in a circle is 360. The sum of the measures in degree of the angles of a triangle is 180. A = πr2 C = 2πr

1. If 3 + x = 3 + 4 , then which of the following could x 4 equal x? (A)

(C) 1 (D) 3 (E) 4

a c = a2 + b2 2

k

3 4

(B) 1

60° x s 45° s 2 45° 30° s x 3 Special Right Triangles 2x

c

b

j O x°

3 4





l

2. In the figure above, j, k and l are lines that intersect at point O. What is the value of 2x +2y +2z? (A) 90 (B) 180 (C) 270 (D) 360 (E) 450

Go to the next page chap26.indd 19

5/23/2011 5:12:31 PM

26.20

Part 3    Four Full Practice Tests

4  

   

4    4 

3. Let the operation # be defined as a # b = a + 2b. What is 3 # 2? (A) 7 (B) 8 (C) 9 (D) 12 (E) 15

A



A

B

C

D

E

F

-2

-1

0

1

2

3

5. On the number line above, what is the equivalent of AB + CE? (A) BD (B) AE (C) AF (D) BF (E) CF

B

O

D

   4



C

4. In the figure, AB || DC, AD || BC, AB = 6 and the circle with center O is inscribed in figure ABCD. If the area of ABCD is 36, what is the area of the circle with center O? (A) 3π (B) 9π (C) 13π (D) 21π (E) 24π

6. If 15 5 = q p , where q and p are positive integers and p > q, which of the following could be the value of p? (A) 50 (B) 60 (C) 75 (D) 90 (E) 105

Go to the next page chap26.indd 20

5/23/2011 5:12:32 PM

26.21

Chapter 26     SAT Practice Test #4

4  

   

4    4 

7. Relative to the graph of f(x) = kx2 , where k is a constant, where in the xy coordinate plane would be the graph of f(x + 1)?

(A) Shifted one unit to the left (B) Shifted one unit to the right (C) Shifted two units up (D) Shifted two units down (E) Shifted one unit up, one unit right

   4





a

8. A six-sided die is rolled twice and the fraction b is formed, where a is the value shown after the first throw and b is the value shown after the second throw. There is an equal probability of each side “showing” on each throw. What is the probability that the fraction a is equal to or less than 1? 12 (A) 36 (B)

b

15 36

21 36 24 (D) 36 (C)

(E)

27 36

Go to the next page chap26.indd 21

5/23/2011 5:12:32 PM

26.22

Part 3    Four Full Practice Tests

4  

   

4    4 

   4



Directions. For Sudent-Produced Response questions 9-18, use the grids at the bottom of the answer sheet page on which you have answered questions 1-8. Each of the remaining 10 questions requires you to solve the problem and enter your answer by marking the circles in the special grids, as shown in the examples below. You may use any available space for scratchwork. Answer: Write answer in boxes.

Grid in result.

• •

7 12

7 / 1 2

• • •

not



1 3 2

3 1 / 2

Decimal point

0

0

0

0

0

0

0

0

0

1

1

1

1

1

1

1

1

1

1

1

1

2

2

2

2

2

2

2

2

2

2

2

2

2

3

3

3

3

3

3

3

3

3

3

3

3

3

3

4

4

4

4

4

4

4

4

4

4

4

4

4

4

4

5

5

5

5

5

5

5

5

5

5

5

5

6

6

6

6

6

6

6

6

7

7

7

7

7

7

7

7

8

8

8

8

8

8

8

8

9

9

9

9

9

9

9

9

0

0

0

1

1

1

1

2

2

2

3

3

4

2

7/2. (If

2 0 1 2 0 1

2 . 5

Fraction line

Mark no more than one circle in any column. Because the answer sheet will be machine-scored, you will receive credit only if the circles are filled in correctly. Although not required, it is suggested that you write your answer in the boxes at the top of the columns to help you fill in the circles accurately. Some problems may have more than one correct answer. In such cases, grid only one answer. No question has a negative answer. 1 Mixed numbers such as 3 must be gridded as 3.5 or



Answer: 201 Either position is correct.

Answer: 2.5

is gridded, it will be interpreted

31 as 2

.)

Decimal Answers: If you obtain a decimal answer with more digits than the grid can accommodate, it may be either rounded or truncated, but it must fill the entire

9. If 2x2 = 98 and x > 0, what is the value of x?

Note : You may start your answers in any column, space permitting. Columns not needed should be left blank.

grid. For example, if you obtain an answer such as 0.6666..., you should record your result as .666 or .667. A less accurate value such as .66 or .67 will be scored as incorrect. Acceptable ways to grid

2 / 3

2 are: 3

. 6 66

. 6 67

0

0

0

0

0

0

0

0

0

1

1

1

1

1

1

1

1

1

1

1

1

2

2

2

2

2

2

2

2

2

2

2

2

3

3

3

3

3

3

3

3

3

3

3

3

4

4

4

4

4

4

4

4

4

4

4

4

5

5

5

5

5

5

5

5

5

5

5

5

6

6

6

6

6

6

6

6

6

6

6

6

7

7

7

7

7

7

7

7

7

7

7

7

10. It takes a pastry chef 20 minutes to make 2 pies. How long will it take that chef to make 12 pies at that same rate, in hours ?

Go to the next page chap26.indd 22

5/23/2011 5:12:33 PM

26.23

Chapter 26     SAT Practice Test #4

4  

   

4    4 

11. A certain rectangle has area 36 and side length 2. What is the perimeter of the rectangle?

12. On her birthday, Sandra received a total of $120 worth of presents. If the value of each present ranged from $15 to $30, what is one possible number of presents that Sandra received?

13. What is the least possible integer divisible by 2, 3, 4, 6 and 8?



   4



as old as his sister. His sister is currently twice as old as the family dog, Fido and Fido is now six years old. How old is Alex’s father now?

y

l

x 15. In the xy coordinate plane pictured above, line l passes through points (0 , 2) and (4 , 6). If line m (not pictured) shares no points with line l , what is the slope of line m?

14. In 10 years, Alex will be half as old as his father and twice

Go to the next page chap26.indd 23

5/23/2011 5:12:34 PM

26.24

4  

Part 3    Four Full Practice Tests

   

4    4 

16. The sum of 7 consecutive integers is 700. What is the sum of the smallest and the largest of those integers?



   4



18. Egdar makes and sells cookies. He has to pay $100 per month for oven rent and he has to pay $1 per cookie in dough and labour costs. He charges $2 per cookie. How many cookies does Edgar need to sell in one month in order to fully cover his costs?

17. If the six circles shown above are placed in a row so that and are always on the ends, how many different arrangements are possible?

STOP If you finish your work before time is called, you may check your work on this section only. Do not turn to any other section in the test.

chap26.indd 24

5/23/2011 5:12:34 PM

26.25

Chapter 26     SAT Practice Test #4

5  

   

5   5 



   5



Section 5 Time—25 Minutes 24 Questions Turn to Section 5 of your answer sheet to answer the questions in this section. Directions. For each question in this section, select the best answer from among the choices given and fill in the corresponding circle on the answer sheet.

Example:

2. It was felt that the —— of the comedian was inappropriate given the unmistakably —— predisposition of the audience in the conservative town. (A) jocularity.. droll (B) exuberance .. ebullient (C) introversion .. lugubrious (D) c atastrophe .. serene (E) irreverence .. sober

The volcano, long considered —— , has recently become ——: local townspeople have been evacuated from the area, out of range of ash and magma. (A) threatening .. docile (B) viable .. unlikely (C) quiescent.. lethargic (D) extinct.. endangered (E) dormant.. active (A) (B) (C) (D) (E)

3. D’Angelo was —— to discover that what he considered aristocratic characteristics like openness and sincerity were in the Middle Ages thought of as —— tendencies. (A) dismayed .. plebian (B) motivated .. elitist (C) e ntitled . . moderate (D) a ggrieved .. virtuous (E) c hagrined . . chivalrous

Each sentence below has one or two blanks, each blank indicating that something has been omitted. Beneath the sentence are flve words or sets of words iabeled A through E. Choose the word or set of words that, when inserted in the sentence, best fits the meaning of the sentence as a whole.

1. The fine weather and other —— omens at the outset of the camping trip did little to —— the growing tension of the hikers. (A) favorable .. admonish (B) auspicious .. diminish (C) positive .. develop (D) a ppreciable .. navigate (E) d  istinguishing .. lessen

4. Just as it is the duty of the strong to care for the weak, so Omar feels that it is the responsibility of extroverts to socialize with —— individuals. (A) prehensile (B) exuberant (C) diffident (D) indigent (E) cavalier 5. Recent anthropological findings would have us believe that the primitive —— of modern hamo sapiens were not merely capable of speech but positively ——. (A) interlocutors .. mellifluous (B) descendents . . garrulous (C) progenitors . . nefarious (D) p  redecessors .. taciturn (E) a ntecedents .. loquacious

Go to the next page chap26.indd 25

5/23/2011 5:12:34 PM

26.26

5  

Part 3    Four Full Practice Tests

   

5   5 



   5



The passages below are followed by questions based on their content; questions following a pair of related passages may also be based on the relationship between the paired passages. Answer the questions on the basis of what is stated or implied in the passages and in any introductory material that may be provided.

5

10

15

Questions 6-7 are based on the following passage.

Questions 8-9 are based on the following passage.

Post-feminism, also known as Third-wave feminism, describes a variety of beliefs and doctrines that either serve as reactions to the tenets of Second-wave feminism or assume critical positions with respect to previous feminist ideologies. The term was first coined in 1982 as part of an article pub- 5 lished in the New York Times Magazine; that article explored the perspectives of many women who claimed to agree with feminist goals and policies, but who did not self-identify as feminists. Since then, post-feminism has been debated with some controversy: more traditional supporters of the women’s 10 liberation movement label it a counterproductive and counterintuitive position, some going so far as to suggest that it represents a total abnegation of the aims of feminism and signals a return to the repression of yore on behalf of the very women who need liberating the most.

Avon was a tough little kid. Imagine a ten-year old with home-made tattoos and a good sized muscle in his arm. Imagine eyes that glint. We never knew where Avon went while we played our games on the street. He would disappear around a coner, kicking a can with his hands stuffed in his pockets. I followed him, once, just to see what would happen. He walked for miles. I did not know where we were. When he found the spot he was looking for—under a large tree; there weren’t any trees in the Bronx, back then—he pulled a thick book out of his back pocket, settled down and read for hours. Imagine my surprise.

6.

The primary purpose of the passage is to (A) examine a socioeconomic change (B) explore the meaning and perception of a term (C) d  ramatize a political movement (D) d  iscuss the ramifications of a unanimously popular change in policy (E) argue against the likelihood of a social trend

7. Critics suggest that post-feminism is “counterproductive and counterintuitive” (line 15) because (A) it implies a philosophy of equality (B) it agitates for continued improvement for the condition of women (C) it defends practices which some consider radical (D) it synthesizes the best elements of two belief systems (E) it suggests a regressive ideological tendency

8. The author’s references to “home-made tattoos” (line 2) and “a good sized muscle in his arm” (Iines 2-3) suggest that (A) as an only child Avon was allowed to do things his friends were not (B) as a hobby Avon enjoyed casual weight-lifting (C) A  von had a troubled personal history (D) a mongst his peers Avon was generally treated with respect (E) Avon was known as a trouble-maker 9. In line 14, the author’s “surprise” is most Iikely accounted for in that (A) Avon was better known for his affinity for math and science (B) Avon was not thought of as literary-minded (C) it was widely acknowledged that Avon could not read (D) A  von was out of place in a natural setting (E) it was incongruous to consider Avon without his tattoo

Go to the next page chap26.indd 26

5/23/2011 5:12:35 PM

26.27

Chapter 26     SAT Practice Test #4

5  

   

5   5 

Questions 10-17 are based on the following passage.

5

10

15

20

25

30

35

The following passage is an excerpt from a 2004 article by a prominent cultural critic. “Super-size me.” The McDonald’s Corporation, a cultural phenomena and cheap-eats goliath visited by over 47 million customers daily, was started in 1940 by Richard and Maurice McDonald in San Bernardino, California. In 1948, the brothers McDonald implemented the “Speedee Service System”, which would become the basic model of the modern fast-food delivery system. In 1955, the organization sold its first franchise, establishing its business plan and opening the door for enormously rapid expansion, growth and profitability. And by 1965, the company had become listed and traded on the public stock exchange. Do you want fries with that? Now with over 30,000 locations in more than 100 countries, McDonald’s is renowned for its emphasis on uniformity: a restaurant in Moscow will provide the same level of hygienic service and dining as a restaurant in Alabama. There are no surprises at McDonald’s. The stores look the same. The food tastes the same. Everything is the same. And this is what the critics can’t stand. Many argue that McDonald’s is the archetypal example of Western cultural imperialism, an aesthetic globalization that seeks to annex tastes, not goods and chattels. They consider the ubiquity of the Big Mac a portent of the apocalypse; never mind the fact that it represents a calorie-rich dining experience with little investment of time or money for the consumer. What these critics choose not to see are the slight variations in menu and venue that suggest if anything an assimilation of McDonald’s into local cultures instead of what is usually supposed to be the case. You can’t find a hamburger in India: there you’ll be eating turkey quarter-pounders. And you won’t see bacon in Israel, either: please enjoy tofu strips on your breakfast sandwich. Many outlets now serve more salad than they do French fries, as a result of negative publicity concerning obesity. Far from being a corporate juggernaut that pays scant attention to local traditions and customs of a place except when trying to understand how to best exploit and therefore profit from them, McDonald’s is the prime exemplar of how local and regional tastes can work with the forces of capitalism to create a more free and comprehensive global market.



   5



10. In the context of the passage as a whole, the first paragraph is best characterized as (A) a detailed exploration of a discovery (B) a colorful depiction of an occurrcnce (C) a brief description of a concept (D) a short history of an organization (E) a thorough explanation of a term 11. The comparison in Iines 17-19 (“a restaurant...in Alabama”) serves to (A) reconcile venues with seemingly disparate geographical features (B) establish premises for an argument against cultural imperialism (C) g esture at the global footprint of a company (D) u  nderscore the diversity of McDonald’s business plan (E) m  ake a point about the standards of McDonald’s 12. In Iine 22, “this” refers to (A) the crassness of McDonald’s logo (B) the uniformity of McDonald’s restaurants (C) t he aggression of McDonald’s expansion (D) M  cDonald’s apparent lack of environmental concern (E) t he dearth of opportunities for professional advancement within the McDonald’s corporation 13. In line 23, “archetypal” most nearly means (A) fundamental (B) peripheral (C) despicable (D) c ontroversial (E) s uperlative 14. In line 26, “ubiquity” most nearly means (A) sanctimony (B) obloquy (C) omnipresence (D) s carcity (E) u  rgency

Go to the next page chap26.indd 27

5/23/2011 5:12:35 PM

26.28

5  

Part 3    Four Full Practice Tests

   

5   5 



   5



Questions 18-24 are based on the following passage.

15. The author mentions that McDonald’s “represents a calorie-rich dining experience with Iittle investment of time and money” (lines 28-29) in order to (A) counter a nutritional fad (B) support an assertion with a fact (C) r ebut an argument with a practical point (D) s egue into the topic of obesity (E) p  recipitate a conclusion 16. In line 33, the author refers to “what is usually supposed to be the case” to suggest that 5 (A) many critics make assumptions about the cultural effect of a popular restaurant (B) it is unusual to find errors in a McDonald’s menu (C) p  art of McDonald’s business plan allows franchise owners a certain amount of discretionary power 10 (D) m  any McDonald’s locations have different interior designs (E) McDonald’s product offerings have evolved over time 17. Lines 34-39 are best regarded as (A) occasions where regional preferences have been 15 subsumed by corporate policy (B) a celebration of multi-cultural cuisine (C) instances of dogmatically held beliefs (D) e xamples of differentiation according to local customs and requirements 20 (E) t ypes of organizational inefficiency McDonald’s is seeking to rectify

25

30

35

The following is excerpted from a 1998 novel about life in preWWII Jamaica. Naimond said that everything he does, he does for love. He likes to compare himself to Edward VIII, I think it was, who gave away the throne of England to be with an American woman, Mrs. Simpson. Me, I don’t think I’d have given that throne away. We walk down the beach in the evenings, before the bugs come out, the big ones, I mean, with nasty stickers and we talk about the places we’d like to go and the things we’d like to do. Naimond always squints out over the ocean, like he’s trying to see across to Spain, or wherever is directly across from us. He puts his hand right over his eyes and makes out like he can almost see it. He’s been squinting so long that his eyes are starting to get stuck like that. “Don’t you want to get off the island? Go somewhere new? Be someone new?” But I figure you have to know who you are first, before you can want to be someone different. Naimond is what people who read too many novels might call a “born romantic”. He sees romance everywhere, which on our island, isn’t easy. He’ll see romance in a buggy thicket on the hottest day of the year, grabbing my hand and running into it like some crazed fool, shouting about princesses and pirates and 25 gold doubloons. He’ll see it in the meanest stall of the cheapest market, riffling through the junk for hours to find what he calls antiques. I think he even sees it in me, sometimes, in the looks I occasionally catch him sending my way. And that’s the strangest thing of all. Naimond says he’s going to leave the island and go to America, or England, or Russia. Somewhere cold. Somewhere new. He says he’s going to seek his fortune and come back and fix things, build better roads, more schools, a bigger hospital with lots of different kinds of medicines. He says he’s going to do that for this love of Jamaica, for his love of the people, for his love of his homeland. Just like Edward VIII. But I don’t think I’d mind if he never left at all.

Go to the next page chap26.indd 28

5/23/2011 5:12:35 PM

26.29

Chapter 26     SAT Practice Test #4

5  

   

5   5 

18. The passage is best described as (A) a reflection on the living conditions of a particular place in time (B) the exploration of a character through the viewpoint of another (C) a sociological study of two young people in relation to the govemment of the epoch (D) a contemplative digression on youth and innocence (E) a discussion of the practical concerns of island-living 19. The Statement “He’s been squinting...stuck like that” in lines 13-14 suggests (A) the extent of Naimond’s myopia (B) the depth of the narrator’s feelings for Naimond (C) t he confusion felt by the narrator at the prospect of leaving home (D) t he great distance between Jamaica and Spain (E) t he intensity with which Naimond maintains his goals 20. In lines 17-18 (“But I figure...someone different”), the narrator’s aside serves to (A) respond to a line of questioning (B) encourage speculation on possible courses of action (C) a rgue against an assessment (D) defend the integrity of a particular perspective (E) s uggest the utility of a new point of view



   5



22. The goals mentioned in lines 33-35 (“build better... medicines”) are best considered as (A) changes that would improve the quality of life for the island’s residents (B) hints of the depth of Naimond’s resentment for the lack of existing infrastructure (C) a n indication of the lengths to which Naimond will go to win the love of the narrator (D) e vidence of Naimond’s professional ambitions (E) e xamples of Naimond’s ennui 23. In the context of the passage as a whole, the references to King Edward VIII represent (A) a distraction to the tedium of everyday living (B) a long-held historical fascination (C) a rejection of monarchy in all forms (D) a metaphor for the fortunes of an island nation (E) a personification of an individual’s strongly-held belief 24. The narrator’s tone could best be described as (A) uninspired (B) contemptuous (C) appreciative (D) a nalytic (E) d  isingenuous

21. The narrator refers to Naimond as a “born romantic” (line 20) because (A) Naimond has an appreciation for wildlife and botany (B) N  aimond behaviors are unconventional and actuated by fantasy (C) N  aimond is motivated in his search for fine art (D) N  aimond appearance suggests inattention to detail (E) N  aimond espoused ideas and beliefs are reminiscent of fiction of the Romantic era

STOP If you finish your work before time is called, you may check your work on this section only. Do not turn to any other section in the test.

chap26.indd 29

5/23/2011 5:12:35 PM

26.30

Part 3    Four Full Practice Tests

7  

   

7   7 

   7



SECTION 7 Time – 25 minutes 20 Questions Turn to Section 7 of your answer sheet to answer the questions in this section.

Directions. For this section, solve each problem and decide which is the best of the choices given. Fill in the corresponding circle on the answer sheet.You may use any available space for scratchwork.

Reference Information

Notes 1.  The use of a calculator is permitted. 2.  All numbers used are real numbers. 3.  Figures that accompany problems in this test are intended to provide information useful in solving the problems.   They are drawn as accurately as possible EXCEPT when it is stated in a specific problem that the figure is not drawn to scale. All figures lie in a plane unless otherwise indicated. 4. Unless otherwise specified, the domain of any function f is assumed to be the set of all real numbers x for which f(x) is a real number.

r

l w

h

h

w l b A = πr2 1 V = lwh A = bh V = πr2h A = lw C = 2πr 2 The number of degrees of arc in a circle is 360. The sum of the measures in degree of the angles of a triangle is 180.

1. If 4y = x + 8 and y = 3, then what is x?

r

h

(A) –4 (B) –3 (C) –2 (D) 0 (E) 4

c

b a

c2 = a2 + b2

60° x s 45° s 2 45° 30° s x 3 Special Right Triangles 2x

2. A basket of fruit is 1 apples and 1 peaches and contains 4 3 only apples, peaches and plums. If there are 12 pieces of fruits in the basket, then how many plums are in the basket?

(A) 3 (B) 4 (C) 5 (D) 6 (E) 7

Go to the next page chap26.indd 30

5/23/2011 5:12:37 PM

26.31

Chapter 26     SAT Practice Test #4

7  

   

7   7 

3. There are nine classes in a school and each class contains at least 20 students and at most 25 students. What is the range of the possible total number of students in the school?

5. If f(x) =

(A) 45 (B) 55 (C) 65 (D) 75 (E) 85



4 + 3x 2 2x

   7





for all x > 0, then what is f(3)?

1 6 1 (B) 5 6 2 (C) 5 3 (D) 6 1 2 (E) 7

(A) 4

CALORIES CONSUMED BY CHRIS 3000

y

2500

C

2000

B

1500 E A

D

(A) (B) (C) (D) (E)

AB BC CD DE EA

500 Mon

x

4. Based on the figure above, which of the following line segments is longest?

1000

Tues

Wed

Thu

Fri

Mon

6. The figure above shows six days of calorie consumption for Chris. Chris normally consumes more than 2,500 calories per day, except when he is sick. Chris fell sick during course of the six-day period. For how many days was he sick?

(A) 2 (B) 3 (C) 4 (D) 5 (E) 6

Go to the next page chap26.indd 31

5/23/2011 5:12:37 PM

26.32

7  

Part 3    Four Full Practice Tests

   

7   7 

7. The function f(x) is defined as the sum of g(x) and h(x). If g(3) = 4 and h(3) = 5 and g(6) = 7 and h(6) = 8, what is f(6)?

(A) 4 (B) 5 (C) 9 (D) 12 (E) 15

   7





10. If twice a certain three-digit number is a four-digit number, and one-sixth of that three-digit number is a two-digit number, which of the following inequalities describes that three-digit number? (A) 100 < x < 300 (B) 256 < x < 410 (C) 375 < x < 781 (D) 500 < x < 594 (E) 635 < x < 1240

5, 9, 15, 23, 33… 8. Each term in the sequence above can be described as 3 plus the product of n and (n + 1), where n is the term number. What is the value of the 10th term?

(A) 75 (B) 87 (C) 93 (D) 113 (E) 134

9. If three sides of a four-sided figure are 2, 3 and 4, all of the following could be lengths of the fourth side EXCEPT: (A) 1 (B) 3 (C) 5 (D) 8 (E) 10

11. If the average of positive integers x, y and z is 3 and xy = 4, which of the following could be the value of z? (A) 1 (B) 3 (C) 5 (D) 7 (E) 9

12. If p and q are negative integers, which of the following must also be a negative integer? I. pq II. (p/q) III. 2(p+q)

(A) I only (B) II only (C) III only (D) I and II only (E) I, II and III

Go to the next page chap26.indd 32

5/23/2011 5:12:38 PM

26.33

Chapter 26     SAT Practice Test #4

7  

   

7   7 

   7





JOHN’S WEIGHT

y Weight in pounds

56

13. In the figure above, a small equilateral triangle with side length x is inside a larger equilateral triangle with side length 4x. What is the area, in terms of x, of the shaded region?

(A) x 2

3 3 4



(B)

7 3 4



(C) x 2



(D) x 2 13



(E)

x2

x2

3

15 3 4

14. If k is the greatest prime factor of 51 and j is the greatest prime factor of 64, what is k + j?

(A) 21 (B) 19 (C) 14 (D) 12 (E) 10

40 32 24 16 8 x .5 1 1.5 2 2.5 3 3.5 4 Age in years

11 3 4 4

48

15. The graph above shoes John’s weight in pounds from his birth to age 4. John’s weight at age 1.5 is approximately what per cent of his weight at age 4? (A) 82 (B) 73 (C) 60 (D) 52 (E) 40

16. Let the operation

be defined by p



numbers p and q. If 3



(A) 6 (B) 10 (C) 12 (D) 14 (E) 18

4=2

q = p + q for all 2pq

x, what is the value of x?

Go to the next page chap26.indd 33

5/23/2011 5:12:39 PM

26.34

Part 3    Four Full Practice Tests

7  

   

7   7 

17. Cube A has side length 3 and cube B has side length 6. What is the ratio of the volume of cube A to the volume of cube B?

1 8 (B) 1 3 111 (C) 216 (D) 111 216 163 (E) 216

A

   7



B

(A)

C O · 19. In the figure above, AC is the arc of a circle with center · O and radius 6. If arc AC is one third of the circumference of the circle and BO bisects angle